NCSBN

Pataasin ang iyong marka sa homework at exams ngayon gamit ang Quizwiz!

Name the respiratory disorder based on findings. Cough Exertion diagnosis dyspnea fatigue fainting swelling of feet or ankles

Cor pulmonale Cor pulmonale is an alteration in the structure and function of the right ventricle caused by a primary disorder of the respiratory system.

Chronic inflammation extending deep into the lining of the affected part of the GI tract results in abdominal pain, cramping and diarrhea. Cirrhosis GERD Cholecystitis Crohns Diverticulitis Hepatitis Ulcerative colitis Pancreatitis Intestinal obstruction Peptic Ulcer

Crohn's disease is an inflammatory bowel disease. Unlike ulcerative colitis, inflammation affects the entire thickness of the bowel wall and can affect any part of the digestive tract (although it most commonly affects the ilium and beginning of the colon).

acne buffalo hump hirsutism moon-shaped face upper body obesity with thin arms and legs

Cushing syndrome Cushing syndrome is caused when the adrenal gland secretes too much cortisol or when someone takes too much corticosteroid medication. The endocrine gland secrete hormones which regulate various functions throughout the body

thick, sticky mucus wheezing exercise intolerance repeated lung infections

Cystic Fibrosis Cystic fibrosis is a hereditary disorder that causes severe damage to the lungs and digestive systems. It is a disorder that affects the exocrine glands, causing them to produce thick and sticky mucus.

herniation of bladder into vaginal canal

Cystocele This hernia-like disorder occurs when the urinary bladder protrudes through the wall of the vagina. Treatments include estrogen, surgery, or mechanical support by pessary.

This disorder includes Alzheimer's disease, traumatic brain injury and Huntington's disease.

Neurocognitive disorders This group of disorders was formerly referred to as "dementia, delirium, amnestic and other cognitive disorders."

What is the drug classification for: celecoxib

Non-steroidal Anti-inflammatory Drugs (NSAIDs) block the cyclooxygenase (COX-1 & COX-2) enzymes and reduce prostaglandins throughout the body, thereby reducing inflammation, pain, and fever. They are used to control mild-to-moderate pain, reduce fever, and to treat various inflammatory conditions, such as osteoarthritis.

What is the drug classification for: ketorolac

Non-steroidal Anti-inflammatory Drugs (NSAIDs) block the cyclooxygenase (COX-1 & COX-2) enzymes and reduce prostaglandins throughout the body, thereby reducing inflammation, pain, and fever. They are used to control mild-to-moderate pain, reduce fever, and to treat various inflammatory conditions, such as osteoarthritis.

What is the drug classification for: meloxicam

Nonopioid Analgesics target and block the chemical substances released by the brain in response to injury (particularly prostaglandin) that facilitate the transmission of the pain stimuli to the brain. These drugs are used to control mild-to-moderate pain and/or fever.

What is the drug classification for: fentanyl

Opioid Analgesics interact with opioid receptors in the central nervous system, acting as agonists of endogenously occurring opioid peptides (eukephalins and endorphins); this action alters perception and response to pain. They can be categorized as long-acting, short-acting, or rapid-onset agents. Fentanyl is a rapid-acting and long-acting Opioid Analgesic approved for cancer breakthrough pain. They are all Schedule II drugs. They are used in the management of moderate-to-severe pain.

Lithium levels and overdose symptoms

0.8-1.2 nausea vomiting diarrhea drowsiness muscle weakness tremor lack of coordination blurred vision tinnitus

Order: Acetaminophen elixir 100 mg by mouth every 4 hours as needed for pain. Available concentration of acetaminophen is 80 mg/0.8 mL. How much acetaminophen elixir will the nurse administer?

1 mL

The nurse has received a physician's order that reads: Administer fentanyl 50 mcg IV every 1 to 2 hours, as needed, for pain. Fentanyl is packaged as 100 mcg/2 mL ampules. How many milliliters of fentanyl will the nurse draw up to administer to the client?mL.

100 mcg/2 mL = 50 mcg/x mL 100 x = 50 X 2 x = 100/100 = 1 mL

The client receives 300 mg phenytoin by mouth daily for seizures and the pharmacy sent phenytoin 125 mg/5 mL suspension. How many mL of suspension will the nurse administer?

12 mL/day

The nurse prepares vancomycin 500 mg IV in 250 mL of normal saline to infuse over 2 hours. What is the administration rate in mL/minute?

2 mL/min for 2 hours

The client receives furosemide 40 mg by mouth daily. The pharmacy stocks furosemide 20 mg tablets. How many tablets does the nurse administer for a 40 mg dose?

2 tablets

The child weighs 68.2 pounds. The nurse must administer amoxicillin by mouth at 30 mg/kg/day in divided doses every 6 hours. How much amoxicillin does the nurse administer each day? How many milligrams of amoxicillin does the nurse administer for each dose?

233 mg of amoxicillin for each dose

The client receives epinephrine 0.25 mcg/min IV via infusion pump. The pharmacy sends epinephrine 0.1 mg in 250 mL of normal saline. What rate in mL/hr will the nurse use to program the infusion pump?

37.5 mL/hour

Lactated Ringer's (LR) solution is a hypotonic fluid. True or False

False Lactated Ringer's is an isotonic fluid solution used in many different clinical situations, including fluid resuscitation. An example of a hypotonic fluid is 0.45% sodium chloride.

Breast cancer is the most common cause of death in women. True False

False Lung cancer is the leading cause of cancer deaths in women in the U.S. and Canada.

During a 12-hour shift a patient who underwent a transurethral resection of the prostrate (TURP), had an IV intake of 1200 mL, oral intake of 400 mL, continuous bladder irrigation of 2400 mL, 2 syringe flushes of 50mL each and indwelling urinary catheter output of about 3000 mL. What is the end of shift fluid intake

4100

The nurse is given and order that reads: administer ephedrine 5 mg slow IV push for a systolic blood pressure finding less than 90 mm Hg. The client's vital sign findings: temperature 98.2 F (36.7 C); pulse 120; respirations 20; blood pressure 88/54. Ephedrine comes packaged as 50 mg/mL and must be diluted with normal saline to a total volume of 10 milliliters. How much ephedrine will the nurse administer? (Write the answer using whole numbers.) mL.

50 mg diluted into 10 milliliters equals 50 mg/10 mL or 5 mg/1 mL. The nurse will administer 1 milliliter (after dilution) of ephedrine.

The nurse is reviewing the list of new client admissions. For which of these clients should contact precautions be implemented? A 45 year-old diagnosed with pneumonia A 3 year-old diagnosed with scarlet fever A 6 year-old diagnosed with mononucleosis A 60 year-old diagnosed with herpes simplex

A 60 year-old diagnosed with herpes simplex In addition to standard precautions, clients with herpes simplex infections must have contact precautions implemented due to the associated, potentially weeping, skin lesions. The child with scarlet fever would be on droplet precautions. Precautions other than standard may need to be implemented for pneumonia, but the causative organism would need to be identified. There are no special precautions for mononucleosis.

A nurse is assessing a 12 year-old child who has been diagnosed with hemophilia A. Which lab result would the nurse expect? An excess of white blood cells A deficiency of clotting factor VIII An excess of red blood cells A deficiency of clotting factors VIII and IX

A deficiency of clotting factor VIII Hemophilia A is characterized by an absence or deficiency of Factor VIII.

Eating a high-fiber diet can help reduce symptoms of this disease. Cirrhosis GERD Cholecystitis Crohns Diverticulitis Hepatitis Ulcerative colitis Pancreatitis Intestinal obstruction Peptic Ulcer

A low-fiber diet is the most common cause of diverticular disease. The disease is made up of two conditions: diverticulosis and diverticulitis.

Infection by the bacterium Helicobacter pylori is the most common cause of this condition. Cirrhosis GERD Cholecystitis Crohns Diverticulitis Hepatitis Ulcerative colitis Pancreatitis Intestinal obstruction Peptic Ulcer

A peptic ulcer is a sore on the lining of the stomach or duodenum. The bacterium called Helicobacter pylori is a major cause of peptic ulcers; NSAIDs are another common cause of peptic ulcers.

Several clients are admitted to an adult medical unit. For which client condition(s) would the nurse institute airborne precautions? A positive purified protein derivative (PPD) test with an abnormal chest x-ray A tentative diagnosis of viral pneumonia with productive brown sputum Autoimmune deficiency syndrome (AIDS) with cytomegalovirus (CMV) Advanced carcinoma of the lung with hemoptysis mixed with a yellow tinge

A positive purified protein derivative (PPD) test with an abnormal chest x-ray The client who must be placed in airborne precautions is the client with the findings that suggest a suspicious tuberculin lesion. A sputum smear for acid-fast bacillus would be done next. CMV usually causes no signs or symptoms in children and adults with healthy immune systems. Good handwashing is recommended for CMV. When findings do occur, they are often similar to those of mononucleosis, including sore throat, fever, muscle aches and fatigue.

There has been a disaster at a child day care center. Which of these children at the site would the triage nurse put in the "treat last" category? A preschooler with a lower leg fracture on one side and an upper leg fracture on the other A toddler with severe and deep abrasions over 98% of the body A school-age child with singed eyebrows and hair near the forehead An infant with intermittent bulging anterior fontanelle during crying episodes

A toddler with severe and deep abrasions over 98% of the body In a disaster situation, the child with severe deep abrasions, over 98% of the body is at great risk for developing shock and infection. Because this child is least likely to survive the wounds, the toddler with severe abrasions would be treated last. There is usually a color-coded triage system to identify victims. The "black" category are those victims who are given no priority for treatment - they are either already dead or have such extensive injuries that they would not be expected to survive. Although, it would seem that the least injured or uninjured person (such as the child with bulging fontanelles when crying) should be treated last, this is not the case in disaster situations.

Drug classification for benazepril

ACE Inhibitors slow the activity of the enzyme angiotensin converting enzyme (ACE), which decreases the production of angiotensin II. As a result, blood vessels relax and dilate, blood pressure is lowered, and more oxygen-rich blood can reach the heart. The spelling of many angiotensin-converting enzyme (ACE) inhibitors end with "pril". They are primarily used to control blood pressure, treat heart failure, help prevent strokes.

"Drug holidays" are sometimes used in the management of this disorder.

ADHD A drug holiday refers to the deliberate interruption of pharmacotherapy for a defined period and for a specific clinical purpose. Sometimes a clinician will give a child with attention deficit hyperactivity disorder (ADHD) a "vacation" from medications on weekends or during summer break from school.

A 42 yo breathes 30 times per minute. Abnormal or expected findings

Abnormal Normal respiratory rate in adolescents and adults is 12-20 breaths per minute.

A 5 month old has a sunken anterior fontanel Abnormal or expected findings

Abnormal The fontanel should be flat; a sunken fontanel indicates possible dehydration.

When you examine the mouth, you see the soft palate is moist and pink with whitish spots Abnormal or expected findings

Abnormal The soft palate should be reddish pink; white spots are a sign of possible infection

What is the best way for a nurse to obtain the health history of a 14 year-old client? Have the mother present to verify information Focus the discussion of risk factors in the peer group Allow an opportunity for the teen to express feelings Use the same type of language as the adolescent

Allow an opportunity for the teen to express feelings Adolescents need to express their feelings. Generally, they talk freely when given an opportunity and the privacy to do so.

A nurse is assigned to a newly hospitalized adolescent. What should be the major threat experienced by this hospitalized adolescent? Restricted physical activity Pain management Altered body image Separation from family

Altered body image The hospitalized adolescent may see each of these as a threat. However, the major threat felt when hospitalized for this age group is the fear of an altered body image. There is great emphasis on physical appearance during this developmental stage.

Where should the nurse administer the annual purified protein derivative (PPD) to the client with a left arm Permcath™?

Always avoid using the arm with a shunt so as to prevent restriction of blood flow and possible clotting or rupture of the fistula. Using the opposite forearm for the PPD administration also reduces the chance for infection.

Which is the appropriate injection site to give an influenza vaccine to an adult?

An influenza (or flu) vaccine is less than 1 mL of fluid, so it is acceptable to administer this injection in the deltoid muscle. The deltoid muscle of the upper arm is a site that is easily accessible in public settings where mass vaccinations are administered.

The nurse is assessing the heart sounds of a patient admitted to the telemetry unit with a diagnosis of mitral stenosis. Indicate where the nurse should place the stethoscope to best assess the mitral valve.

Auscultation of heart sounds is a key component of the physical assessment. It is important that the nurse is able to identify the area on the chest that corresponds to each of the four valves. The mitral area or apex of the heart is located at the fifth intercostal space, left midclavicular space.

Fluttering or "thumping" sensation in the chest. Due to: Atrial fibrillation Myocardial infarction Cardiac tamponade Occlusive arterial disease hemophilia Raynaud's phenomenon Heart Failure Thrombophlebitis DIC Sickle cell disease

Atrial fibrillation (AFib) is the most common type of irregular heartbeat. Many clients have no symptoms. A client with AFib is five times more likely to have a stroke than someone without AFib.

A nurse is teaching a client with Raynaud's phenomenon about lifestyle and behavioral changes that will improve the quality of life. Besides smoking cessation, what would be the next most important thing this client should do? Avoid cold temperatures Avoid spicy foods Keep feet dry Reduce stress

Avoid cold temperatures Raynaud's phenomenon is a condition where cold temperatures or strong emotions cause blood vessel spasms, preventing blood flow to the fingers, toes, ears, and nose. Besides not smoking, the most important teaching would be to avoid cold temperatures. Both cold and nicotine cause arterial vasoconstriction and will aggrevate this phenomenon. The question is asking what is the most important teaching. The other approaches tend to be needed less frequently and so are a lower priority.

Barrett's esophagus is a complication of this condition. Cirrhosis GERD Cholecystitis Crohns Diverticulitis Hepatitis Ulcerative colitis Pancreatitis Intestinal obstruction Peptic Ulcer

Complications of GERD include bronchospasm, chronic cough or hoarseness, dental problems, esophageal ulcer and Barrett's esophagus (a change in the lining of the esophagus that can increase the risk of cancer).

What is the drug classification for: nifedipine

Calcium Channel Blockers (CCBs) slow the rate at which calcium passes into the heart muscle and into the vessel walls; this relaxes the vessels and allows blood to flow more easily through them, thereby lowering blood pressure. They are used to treat hypertension, angina, and abnormal heart rhythms (atrial fibrillation, paroxysmal supraventricular tachycardia). They are also used for client post-MI who cannot tolerate beta-blockers.

Drug classification for verapamil

Calcium Channel Blockers slow the rate at which calcium passes into the heart muscle and into the vessel walls; this relaxes the vessels and allows blood to flow more easily through them, thereby lowering blood pressure. They are used to treat hypertension, angina, and abnormal heart rhythms (atrial fibrillation, paroxysmal supraventricular tachycardia). They are also used for client post-MI who cannot tolerate beta-blockers.

Ecchymoses hyper pigmentation pruritus xerosis uremic frost half-and-half nails

Chronic Kidney Disease In addition to pruritus, pigmentary disorders, and ecchymosis, clients with chronic kidney disease may also have xerosis (dry skin), uremic frost, half-and-half nails.

Lactulose has been prescribed for a client with advanced liver disease. Which finding should the nurse use to evaluate the effectiveness of this treatment? Less jaundice Increased appetite Decreased lethargy Less edema

Decreased lethargy Lactulose is a synthetic sugar used to treat constipation and reduce the amount of ammonia in the blood of clients with liver disease. It works by drawing ammonia from the blood into the colon, where it is removed by the body. Hepatic encephalopathy (HE) occurs in people with end-stage liver disease. People with HE may experience problems with memory, concentration and may experience drowsiness and lethargy; lactulose is used to help manage these symptoms. Lactulose is not used to treat edema or jaundice.

An infant weighed 7 pounds 8 ounces at birth. If growth occurs at a normal rate, what would be the expected weight change at six months of age? Double the birth weight Triple the birth weight Gain six ounces each week Add two pounds each month

Double the birth weight Although growth rates vary, infants normally double their birth weight by six months. At 12 months weight is tripled.

The nurse is caring for a 14 year-old child in the postanesthesia care unit (PACU) following corrective surgery for scoliosis. Which action should receive priority in the plan? Assist to stand up at bedside within the first few hours Initiate the antibiotic therapy prescribed for 10 days Evaluate the movement and sensation of extremities Teach client isometric exercises for the legs

Evaluate the movement and sensation of extremities Following corrective surgery for scoliosis, the neurological status of the extremities requires priority attention in the PACU, as well as on the postop surgical unit. The other options may be done after the neurological status.

The patient is able to stand on one foot, with eyes shut, for five seconds. Abnormal or expected findings

Expected Balancing on one foot, with eyes shut is one sign of normal cerebellar function.

Clients diagnosed with hyperosmolar hyperglycemic state (HHS) experience severe ketoacidosis. True False

False This rare but deadly metabolic state is more common in the elderly with type 2 diabetes mellitus. HHS is characterized by hyperglycemia and severe dehydration without ketoacidosis.

Cardiac murmurs related to endocarditis. Etiology Finding

Finding Endocarditis is inflammation of the inside lining of the heart chambers (endocardium) and heart valves. Heart murmurs are heard in a majority of clients with endocarditis.

A nurse is teaching a client with atrial fibrillation about the use of warfarin at home. The nurse should reinforce the need to avoid which of the following? Exposure to sunlight Foods rich in vitamin K Large indoor gatherings Active physical exercise

Foods rich in vitamin K Vitamin K acts as an antidote to the pharmacologic action of warfarin therapy and will decrease its effectiveness. Foods high in vitamin K include dark green leafy vegetables, tomatoes, bananas, cheese and fish.

Anxiety exophthalmos heat intolerance restlessness weakness

Graves' disease Graves' disease is an autoimmune disorder that leads to overactivity of the thyroid gland. Common cause of hyperthyroidism, an over-production of thyroid hormones, which causes an enlargement of the thyroid and other symptoms such as exophthalmos, heat intolerance, and anxiety.

A nurse is caring for a client several days after a cerebral vascular accident (CVA). Warfarin has been prescribed. Today's prothrombin level is 40 seconds (normal range 10 to 14 seconds). Which finding requires priority follow-up? Gum bleeding Generalized weakness Homan's sign Lung sounds

Gum bleeding The prothrombin time is high and indicates an elevated risk for bleeding. Neurological assessments remain important for post-CVA clients.

A nurse is assessing a healthy child at the two-year check up. Which finding should the nurse report immediately to the health care provider? Growth pattern appears to have slowed Height and weight percentiles vary widely Short-term weight changes are uneven Recumbent and standing height are different

Height and weight percentiles vary widely On the growth curve, height and weight should be close in percentiles at this age. A wide difference may indicate a problem.

dry, scaly skin muscle cramps tingling of lips, fingers, toes

Hypoparathyroidism Hypoparathyroidism is a disorder in which the parathyroid glands do not produce enough parathyroid hormone (PTH). PTH helps control serum levels of calcium, phosphorus, and vitamin D.

A client is admitted with a diagnosis of myocardial infarction (MI) and reports having chest pain. The nurse provides care based on the knowledge that pain associated with an MI is related to which of the following findings? Insufficient oxygenation of the cardiac muscle Fluid volume excess Arrhythmia An electrolyte imbalance

Insufficient oxygenation of the cardiac muscle Due to ischemia of the heart muscle, the client will experience pain. This happens because destroyed myocardial tissue can block or interfere with the normal cardiac circulation.

The nurse is caring for a client who had a laparoscopy with excision of endometriosis. Where would the nurse expect to find the incision?

Laparoscopy is the most common procedure used to diagnose and treat endometriosis. A small incision is made near the belly button and the abdomen is filled with CO2 gas; the lighted laparoscope is then inserted into the abdomen. (Two other small incisions are also typically made in the abdomen.)

What is the drug classification for: bisacodyl

Laxatives are typically classified as either bulk-forming agents, osmotics, salines, stimulants (such as bisacodyl) or stool softeners. They are used to treat or prevent constipation or to prepare the bowel for radiologic or endoscopic procedures.

Drug classification for atorvastatin

Lipid-lowering Agents reduce LDL ("bad") cholesterol by inhibiting the enzyme in the liver (HMG-CoA reductase) responsible for making cholesterol. Along with diet and exercise, they are used to reduce blood lipids in an effort to reduce the morbidity and mortality of atherosclerotic cardiovascular disease and its sequelae.

Drug classification for furosemide

Loop Diuretics work in the ascending limb of the loop of Henle (where magnesium & calcium are reabsorbed). Disrupting the reabsorption of these 2 ions brings about increased urine production (which lowers blood volume) and results in lowered blood pressure. They also cause the veins to dilate, which lowers blood pressure mechanically. They are used to treat acute pulmonary edema and manage edema; they can also be used to reduce intracranial pressure and to treat hyperkalemia.

A client treated for depression tells the nurse at the mental health clinic, "I recently purchased a handgun because I am thinking about suicide." Which of these should be the FIRST action taken by the nurse? Respect the client's confidential disclosure Phone the family to warn them of the risk Suggest inpatient psychiatric care Notify the primary care provider immediately

Notify the primary care provider immediately This client has two critical points in suicide guidelines: a report of suicidal intent and a formulated plan with steps to implement it. The primary care provider, with other members of health care team, will arrange for treatment given the client's serious risk for self-destructive behavior. Hospitalization with family therapy is indicated. The nurse should never agree to help a client "keep secrets" from the health care team.

The nurse is assessing a child with suspected lead poisoning. Which assessment should a nurse expect to find? Auditory wheezes with expiration Numbness and tingling in feet Excessive perspiration A history of difficulty sleeping

Numbness and tingling in feet A child who has unusual neurologic complaints, such as neuropathy or footdrop that cannot be attributed to other causes, may be affected by lead poisoning. This may occur when a child ingests or inhales paint chips from lead-based paint or dust during remodeling in older buildings. Other findings of lead poisoning are appearance of bluish gum line, hyperactivity and developmental delays.

What is the drug classification for: mannitol

Osmotic Diuretics are low-molecular-weight substances that produce a rapid loss of sodium and water by inhibiting their reabsorption in the kidney tubules and the loop of Henle. They also increase the osmolality of plasma, which increases diffusion of water from the intraocular and cerebrospinal fluids. They are mainly used in the management of cerebral edema to decrease intracranial pressure.

The only FDA-approved type of medication to treat this disorder are SSRIs

PTSD Sertraline and paroxetine are FDA-approved to treat PTSD. Other medications may be used for off-label or as adjunct treatment. For example, prazosin may be used to decrease migraines.

tiny, silvery or grayish white nits can be seen at the base of the hair shaft

Pediculosis capitus Pediculosis capitus is caused by parasitic insects that lay eggs (called nits) on people's heads; also known as head lice.

Drug classification for spironolactone

Potassium-sparing Diuretics are used to conserve potassium in clients receiving thiazide or loop diuretics; they decrease sodium reabsorption in the collecting tubules of the kidneys. They are often used in clients with heart failure; they do not significantly lower blood pressure.

A nurse is teaching a client about the healthy use of ego defense mechanisms. An appropriate goal for this client should be which of these? Increase independence and communicate more often Avoid conflict and leave unpleasant situations Minimize anxiety and delay apprehension Reduce fear and protect self-esteem

Reduce fear and protect self-esteem Ego defense mechanisms are unconscious proactive barriers that are used to manage instinct and affect in the presence of stressful situations. Healthy reactions are those in which clients admit that they are feeling various emotions.

cough difficulty breathing fatigue fever greater than 100.4 headache myalgia

Severe Acute Respiratory Syndrome (SARS) SARS is a viral infection causing acute respiratory distress and sometimes death.

Chancres flu-like symptoms hair loss palmar rash

Syphilis Syphilis is a sexually transmitted disease (STD) with 4 distinct stages. In the primary stage, painless sores appear. Skin rashes and flu-like symptoms occur in the secondary stage. If untreated, syphilis is eventually fatal.

The nurse is teaching a client about an oral hypoglycemic medication. The nurse should place primary emphasis on which of the following points? Increasing the dosage based on blood glucose Distinguishing hypoglycemia from hyperglycemia Recognizing the findings of toxicity Taking the medication at specified times

Taking the medication at specified times A regular interval between doses should be maintained because oral hypoglycemics stimulate the islets of Langerhans to produce insulin. The other actions would be discussed after this point.

chest pain muffled heart and lung sounds mediastinal shift respiratory distress

Tension Pneumothorax A tension pneumothorax occurs when air gets trapped in the pleural cavity and as the pressure increases, it pushes the mediastinum to the other side of the chest, which compresses the other lung. This is a life-threatening condition.

A nurse is assigned to a client diagnosed with multiple trauma and head injury with a blood pressure on admission of 140/70 mm Hg. Four hours later the blood pressure is 179/68 mm Hg. What is the difference in the pulse widths or pulse pressures that would lead the nurse to suspect the client may be developing increased intracranial pressure? (Answer the question using whole numbers.)

The first pulse width is (140 - 70) = 70. The second pulse width is (179 - 68) = 111. The difference between them is 41, which is an increase that is called a widened pulse pressure.

Anemia, episodes of pain and frequent infections. Due to: Atrial fibrillation Myocardial infarction Cardiac tamponade Occlusive arterial disease hemophilia Raynaud's phenomenon Heart Failure Thrombophlebitis DIC Sickle cell disease

The most common finding of sickle cell disease is pain (sickle cell crisis). This is caused by the characteristic crescent-shaped red blood cells getting stuck and blocking blood vessels. Clients with sickle cell disease often also have anemia and are more prone to infections.

During teaching to use the metered-dose inhaler (MDI) to a client newly diagnosed with asthma, the client asks: "When will I know the canister is empty?" The nurse's response should include which information? Shake the canister to detect any fluid movement The number of doses that remain will be on the inhaler Drop the canister in water to observe floating Estimate how many doses are usually in the canister

The number of doses that remain will be on the inhaler Newer MDI products may have counters on them (for example, Ventolin and Proventil). If the inhaler does not have a counter, the client should write down the date a refill is needed (usually in about 30 days, if the client uses it on a regular basis). None of the manufacturers recommend the floating technique. Also, it is not always possible to determine if an inhaler is empty by shaking it because some propellant will always remain in the canister.

The nurse is having a discussion with the parents of a newborn who was diagnosed with hypospadias. The nurse should communicate which point? The surgery may be performed in stages over a period of time The postoperative appearance of the penis will be normal Circumcision can be performed at any time The initial repair is delayed until six to eight years of age

The surgery may be performed in stages over a period of time Hypospadias is a condition in which the urethral opening is located on the ventral surface or the underside of the penis. Mild defects may be repaired in one procedure, while severe defects may require two or more procedures. It is corrected in stages as soon as the child can tolerate surgery and before the child turns school age.

What is the drug classification for: streptokinase

Thrombolytics convert plasminogen to plasmin, which then degrades fibrin in clots. They are used for the acute management of coronary thrombosis (MI), massive pulmonary emboli, deep vein thrombosis, and arterial thromboembolism.

When the fetus is active, its heart rate should increase by about 15 beats per minute. True or False

True When the fetus is active, its heart rate will accelerate by about 15 beats per minute above the baseline. Average fetal heart rate is about 130 BPM when near term.

A nurse is caring for a 10 year-old child who will be started on heparin therapy. Which assessment is critical for the nurse to make before initiating this therapy? Skin turgor Vital signs Weight Lung sounds

Weight Check the client's weight because the dosage for anticoagulants in children is calculated on the basis of weight.

The nurse is caring for a client with a colostomy pouch. During a teaching session, when should the nurse teach that the pouch should be emptied? Prior to meals At the same time each day When it is one-third to one-half full After each fecal elimination

When it is one-third to one-half full If the pouch becomes more than half full it may separate from the flange. Emptying before meals would possibly decrease the appetite. After each fecal elimination is unrealistic to fit into activities of life. At the same time each day does not account for the pouch being empty at that time.

During the admission process, the client reports heavy alcohol use for at least one year. What effect does the nurse anticipate the hospitalized client will experience when alcohol consumption stops? Bradycardia Somnolence Withdrawal Craving

Withdrawal The findings of alcohol withdrawal develop within 24 to 48 hours after people either stop or significantly reduce their alcohol consumption. Findings of withdrawal can range from "mild" (shaking or sweating, or perhaps nausea, headache, anxiety, tachycardia or hypertension) to severe (delirium tremens or DTs), which are characterized by rapid heartbeat, fever, hallucinations or seizures.

The nurse is caring for a client following a thyroidectomy. The laboratory results indicate hypocalcemia, probably related to parathyroid gland damage when the thyroid gland was removed. Identify the part of the body the nurse should check to assess Chvostek's sign.

a twitch of the facial muscles following gentle tapping over the facial nerve in front of the ear that indicates hyperirritability of the facial nerve

At a senior citizen's group meeting the nurse talks with a client who has type 1 diabetes. Which statement by the client during the conversation is most predictive of a potential for impaired skin integrity? a. "I had a penny in my shoe all day last week, and I didn't even realize it until I took my shoes off!" b. "I give my insulin to myself in my thighs and belly and alternate sites." c. "Here are my glucose test readings that I wrote on my calendar." d. "If I bathe more than once a week my skin feels too dry."

a. "I had a penny in my shoe all day last week, and I didn't even realize it until I took my shoes off!" Peripheral neuropathy can lead to lack of sensation in the lower extremities. Clients who do not feel pressure and/or pain are at high risk for skin impairment.

A client is admitted with severe injuries resulting from an auto accident. The client's vital signs are BP 120/50, pulse rate 110, and respiratory rate of 28. What should be the initial nursing intervention? a. Administer oxygen as ordered b. Initiate continuous blood pressure monitoring c. Initiate the ordered intravenous therapy d. Institute continuous cardiac monitoring

a. Administer oxygen as ordered Early findings of shock are associated with hypoxia and manifested by a rapid heart rate and rapid respirations. Therefore, oxygen is the most critical initial intervention; the other interventions are secondary to oxygen therapy.

While caring for a client, the nurse notes a pulsating mass in the client's periumbilical area. Which of these assessments is appropriate for the nurse to perform on the mass? a. Auscultate b. Palpate c. Measure the length d. Percuss

a. Auscultate Auscultation of the abdomen and the finding of a bruit would confirm the presence of an abdominal aneurysm. This would form the basis of information to be given to the health care provider. The mass should not be palpated or percussed because of the risk of rupture.

The nurse is caring for a client who is in the advanced stage of multiple myeloma. Which action should be included in the plan of care? a. Careful repositioning b. Administer diuretics as ordered c. Place in protective isolation d. Monitor for hyperkalemia

a. Careful repositioning Multiple myeloma occurs when abnormal plasma cells (myeloma cells) collect in several bones. This disease may also harm other tissues and organs, especially the kidneys. This type of cancer causes hypercalcemia, renal failure, anemia,and bone damage. Because multiple myeloma can cause erosion of bone mass and fractures, extra care should be taken when moving or positioning a client due to the risk of pathological fractures.

The client is transported to the emergency department with minor injuries suffered during a home fire. The client experiences intense anxiety after learning his home was completely destroyed. What is the most important initial intervention for this client? a. Determine available community and personal support resources b.Explore the feelings of grief associated with the loss c. Provide a brochure on methods to promote relaxation d. Suggest that the client rent an apartment with a sprinkler system

a. Determine available community and personal support resources Although the sudden loss of a home can cause significant emotional distress, the most important initial intervention focuses on identifying (community) resources and obtaining assistance for housing and other immediate needs. Information on home safety, relaxation exercises, and grief counseling can wait until the client's basic need for shelter is met.

A client arrives in the emergency department after a radiologic accident at a local factory. After placing the client in a decontamination room, the nurse gives priority to which intervention? a. Ensure physiologic stability of the client b. Double bag the client's contaminated clothing c. Wrap the client in blankets to minimize staff contamination d. Begin decontamination procedures for the client

a. Ensure physiologic stability of the client The nurse must initially assist in the stabilization of a client prior to the performance of any other tasks related to radiologic contamination. A radiation survey meter reading above background radiation levels indicates the possibility of contamination. Radiologic contamination rarely results in loss of consciousness or immediate visible signs of injury. Thus, other causes of injury or illness should be ruled out.

A 12 yo child is admitted with a fracture of the arm and is told that surgery is required. A nurse finds the child crying and unwilling to talk. What is the most appropriate approach by the nurse? a. Give the child privacy b. Try to distract the child with hand-held electronic game c. Tell the child that the surgery will have no problems d. Make arrangements for the friends to visit as soon as possible

a. Give the child privacy A 12 year-old child needs the opportunity to express any emotions privately. A nurse should facilitate privacy in whatever manner possible.

The client is diagnosed with cystic fibrosis (CF). The nurse would expect the client to be treated with oral pancreatic enzymes and which type of diet? a. High fat, high-calorie b. Gluten-free, low fiber c. Dairy-free d. Sodium-restricted

a. High fat, high-calorie CF affects the cells that produce mucus, sweat and digestive juices. Someone with CF needs a high-energy diet that includes high-fat and high-calorie foods, extra fiber to prevent intestinal blockage and extra salt (especially during hot weather.) People with CF are at risk for osteoporosis and need calcium and dairy products. Someone with celiac disease or with a gluten intolerance, not CF, needs a gluten-free diet.

The nurse is caring for a client with a pressure ulcer on the heel that is covered with black hard tissue. Which would be an appropriate goal in planning the care of this client? a. Keep the tissue intact b. Heal the infection c. Debride the eschar d. Protection for the granulation tissue

a. Keep the tissue intact If the black tissue (eschar) is dry and intact, no treatment is necessary; the stable eschar serves as the body's natural cover. If the area changes with cellulitis or pain, then this is a sign of infection and requires debridement.

A nurse is caring for a client who is diagnosed with Hodgkin's disease and is scheduled for radiation therapy. The nurse recognizes that, as a result of the radiation therapy, the client is most likely to experience which difficulty? a. Nausea b. Night sweats c. Face and neck edema d. High fever

a. Nausea As a result of radiation therapy, which is at the lymph nodes throughout the body, nausea often results. Night sweats are a finding in this disease process. These clients are not likely to have a high fever because the lymphatic or immune system is not fully functional.

Today's prothrombin time for a client receiving warfarin 20 seconds. The normal range listed by the lab is 10 to 14 seconds. What is an appropriate nursing action? a. Recognize that this is a therapeutic level b. Assess for bleeding gums or IV sites c. Notify the health care provider immediately d. Observe the client for hematoma development

a. Recognize that this is a therapeutic level For the client on warfarin therapy, this prothrombin level is within the therapeutic range. Therapeutic levels for warfarin are usually 1 1/2 to 2 times the normal levels.

The nurse is caring for a client diagnosed with anemia and confusion. Which task could the nurse assign to the unlicensed assistive person (UAP)? a. Test stool for occult blood and urine for pH and report the results b. Suggest foods that are high in iron and prepare a list of the client's likes and dislikes c. Report mental status changes and level of mental clarity d. Assess and document skin turgor and skin color changes

a. Test stool for occult blood and urine for pH and report the results UAP can perform routine tasks that have known or expected outcomes because these tasks typically do not require nursing judgment or decision-making. Any nursing intervention that requires independent, specialize nursing knowledge, skill or judgment cannot be assigned to UAP.

The nurse is assessing a client with a history of hypertension. Which of these questions is a priority for the nurse to ask? a. "Describe your usual exercise and activity patterns." b. "What over-the-counter medications do you take?" c. "Describe your family's cardiovascular history." d. "Tell me about your usual diet for one day."

b. "What over-the-counter medications do you take?" Over-the-counter medications (OTC), especially those that contain cold preparations, can increase the blood pressure to the point of aggravation of the hypertension. The nurse would ask the other questions, but the answers to these questions don't have as great a risk for the client as the question about OTC medications.

A client diagnosed with testicular cancer is scheduled for a right orchiectomy. The nurse is able to answer the client's questions about this procedure with the understanding that a unilateral orchiectomy involves which of the following approaches? a. A dissection of related lymph nodes by the testes b. A surgical removal of one testicle c. A partial surgical removal of the perineal area d. A surgical removal of the entire scrotum

b. A surgical removal of one testicle The affected testicle is surgically removed along with its tunica and spermatic cord. The other genitals and the perineal area are not involved.

The nurse is teaching a group of women in a community clinic about prevention of osteoporosis. Which over-the-counter medication should the nurse recognize as having the most elemental calcium per tablet? a. Calcium citrate b. Calcium carbonate c. Calcium gluconate d. Calcium chloride

b. Calcium carbonate Calcium carbonate contains 400 mg of elemental calcium in 1 gram of calcium carbonate. Calcium carbonate is inexpensive and convenient; it is found in many over-the-counter antacid products, including Tums and Rolaids.

A nurse is assessing an 8 month-old infant with a malfunctioning ventriculoperitoneal shunt. Which of these findings should the nurse anticipate the infant might exhibit? a. Lethargy b. Irritability c. Sunken anterior fontanelle d. Negative Moro reflex

b. Irritability Irritability is an initial finding for cerebral hypoxia, which would occur from the retained fluid in the brain that results in increased intracranial pressure. Signs of increased intracranial pressure in infants include bulging fontanel, irritability, high-pitched cry, and continual crying when held. Changes in the pulse are variable, e.g., rapid to slow and bounding to feeble. Respirations are more often slow, deep and irregular.

The nurse is performing an assessment on an infant with severe airway obstruction. Which assessment finding would the nurse anticipate? a. Rapid, shallow respirations b. Nasal flaring c. Chest pain aggravated by respiratory movement d. Cyanosis and mottling of the skin

b. Nasal flaring When the trachea or bronchioles become partially blocked, air flow is restricted. Nasal flaring is an exaggerated opening and closing of the nostrils with breathing, and is considered a subtle but important sign of acute respiratory distress in an infant. This is an emergency and requires rapid medical intervention.

The parents of a 7 year-old tell the nurse that their child has started to "tattle" on siblings. In interpreting this new behavior, how should the nurse explain the child's actions to the parents? a. Insecurity and attention getting are common motives b. The ethical sense and feelings of justice are developing c. Attempts to control the family using new coping styles d. Complex thought processes help to resolve conflicts

b. The ethical sense and feelings of justice are developing The child is developing a sense of justice and a desire to do what is right. At 7, children are increasingly aware of family roles and responsibilities. They also do what is right because of parental direction or to avoid punishment. This age group, 6 to 12 years of age, is called the school-aged group.

The nurse is explaining an illness to a 10yo child. What should the nurse keep in mind about the cognitive development of children at this age? a. Children of this age are able to make simple association of ideas b. They are able to think logically in the organization of facts c. Interpretation of events originate from their own perspective d. Conclusions are based on previous experiences

b. They are able to think logically in the organization of facts Children in concrete operations stage, according to Piaget, are capable of mature thought when they are allowed to mentally or physically manipulate and organize objects.

The nurse is preparing to administer a feeding through a percutaneous endoscopic gastrostomy (PEG) tube. What nursing action is needed before starting the infusion? (Select all that apply.) a. Palpate the abdomen b. Verify the length and placement of the tube c. Milk or massage the tube d. Keep the feeding product refrigerated until ready to use e. Elevate the head of the bed 30-45 degrees f. Flush the tube with 30 mL of warm water

b. Verify the length and placement of the tube e. Elevate the head of the bed 30-45 degrees f. Flush the tube with 30 mL of warm water Prior to starting every feeding, the nurse should verify the length and placement of the tube, flush the tube with 30 mL of warm (not hot and not cold) water, and elevate the head of the client's bed at least 30 degrees. The nurse should also verify the presence of bowel sounds before starting the infusion. There's no need to milk the tube unless it's obstructed. Feeding products should be brought to room temperature before the infusion to prevent gastrointestinal discomfort.

After an explosion at a factory, one of the employees approaches the nurse and says, "I am a CNA at the local hospital." Which of these tasks would be appropriate for nurse to assign to this worker who is assisting in the care of the injured. a. take temp b. palpitate pulses c. measure BP d. check alertness

b. palpitate pulses The heart rate and regularity would indicate if the patient is in shock or has the potential for shock. If pulses cannot be easily palpitated or are irregular, those patients would be seen first and further assessment by the nurse could be done (including measuring BP). Taking the temperature is not a priority at this time

A female client is admitted for a breast biopsy. She says, tearfully to a nurse, "If this turns out to be cancer and I have to have my breast removed, my partner will never come near me." Which of these statements would be the best response by the nurse? a. "You sound concerned that your partner will reject you." b. "I hear you saying that you have a fear for the loss of love." c. "Are you worried that the surgery will lead to changes?" d. "Are you wondering about the effects on your sexuality?"

c. "Are you worried that the surgery will lead to changes?" By simply asking about changes, the nurse is encouraging further discussion without focusing on a specific issue. This technique of therapeutic communication will allow the client to decide what to talk about next. It is best to allow the client to identify the exact nature of the problem.

A 15 year-old client has been placed in a Milwaukee brace. Which statement made by the client is incorrect and indicates a need for additional teaching? a. "I should inspect my skin under the brace every day" b. "The brace has to be worn all day and night." c. "I will only have to wear this for six months." d. "I can take it off when I shower or take a bath."

c. "I will only have to wear this for six months." The brace must be worn long-term, during periods of growth, usually for one to two years. It is used to correct scoliosis, the lateral curvature of the spine.

The paramedics are transporting a poisoning victim to the local hospital. In which of these cases does the nurse anticipate that hyperbaric O2 therapy will be used? a. 6yo found sitting on bathroom floor beside an empty bottle of diazepam b. 21 yo with suspected ethanol intoxication c. 35 yo found unconscious with suspected CO poisoning 2 yo who ate an undetermined amount of crystal drain cleaner

c. 35 yo found unconscious with suspected CO poisoning CO poisoning is the leading cause of poisoning in the US. It causes severe hypoxia which is why treatment includes high-dose oxygen. In severe poisoning, hyperbaric O2 therapy may be used. Treatment for: -crystal drain cleaner and diazepam may include gastric lavage and/or activated charcoal -alcohol intoxication may include gastric lavage, IV fluids, and supportive care

Which individual is at greatest risk for the development of hypertension? a. 40 year-old Caucasian nurse b. 60 year-old Asian-American shop owner c. 45 year-old African-American attorney d. 55 year-old Hispanic teacher

c. 45 year-old African-American attorney The incidence of hypertension is greater among African-Americans than other groups in the United States. The incidence among the Hispanic population is rising.

A nurse manager suspects a staff nurse of substance use disorder (SUD). Which approach would be the best initial action by the nurse manager? a. Confront the nurse about the suspicions in a private meeting b. Counsel the employee to resign to avoid investigation and rumors c. Consult with human resources personnel about the issue and needed actions d. Schedule a staff conference, without the nurse present, to collect information

c. Consult with human resources personnel about the issue and needed actions The nurse manager needs to consult with human resources to determine the proper procedures for documenting and reporting the nurse's behavior. The nurse manager could also consult the EAP if one is available. If the staff nurse is also suspected of diversion, and a written policy exists, the nurse manager would follow these procedures. Attempts should be made to help the nurse with SUD by providing counseling and treatment for this disease.

A nurse is caring for a client with schizophrenia who has been treated with quetiapine for one month. Today the client is increasingly agitated and reports having muscle stiffness. Which of these additional findings should be reported to the health care provider? a. Decreased pulse and blood pressure b. Mental confusion and general weakness c. Elevated temperature and sweating d. Muscle spasms and seizures

c. Elevated temperature and sweating Neuroleptic malignant syndrome (NMS) is a rare disorder that can occur as a side effect of antipsychotic medications. It is characterized by muscular rigidity, tachycardia, hyperthermia, sweating, altered consciousness, autonomic dysfunction, and increased creatine phosphokinase (CPK). This is a life-threatening complication that can occur anytime during therapy with antipsychotic medications.

A nurse prepares for Denver Screening II of a 3yo child in the clinic when the mother asks the nurse to explain the purpose of the test. What is the nurse's best response about the purpose of the Denver Screening II? a. It measures a child's intelligence b. It helps to determine problems c. It assesses a child's development d. It evaluates psychological responses

c. It assesses a child's development The Denver Development Test II is a screening test to assess children from birth through 6 years of age in the personal/social, fine motor adaptive, language and gross motor development. During this test a child experiences the fun of play. This screening test determines the highest level of functioning in these areas at the time of the examination.

The 54 year old client is scheduled for a coronary angiography. The client's medical history includes angina, type 2 diabetes mellitus and mild renal insufficiency. Which of the following orders does the nurse anticipate? a. Metformin (Glucophage) 500 mg by mouth pre-procedure b. Restrict oral fluid intake post-procedure c. Monitor serum creatinine levels pre- and post-procedure d. Ibuprofen (Motrin) 800 mg by mouth PRN for pain post-procedure

c. Monitor serum creatinine levels pre- and post-procedure Coronary angiography requires the use of a contrast dye. Persons with diabetes and/or impaired kidney function are at high risk for developing contrast media-induced nephrotoxicity (CIN). Adequate hydration helps maintain renal blood flow and reduces the time the contrast media is in contact with the renal tubules and, therefore, will help prevent CIN. Serum creatinine levels are used to monitor for the development of CIN. Nephrotoxic drugs, such as ibuprofen, should not be used for procedures requiring contrast media. The oral hypoglycemic drug metformin increases the risk of lactic acidosis if CIN were to occur; it should be held the day of the procedure until kidney function returns to baseline (as determined by serum creatinine).

The nurse reviews the most recent lab results for a client on telemetry who is experiencing premature ventricular beats at 12 per minute. Which lab test would require immediate action by the nurse? a. Magnesium 2.4 mg/dL (1.2 mmol/L) b. Partial thromboplastin time (PTT) 70 seconds c. Potassium 2.5 mEq/L (2.5 mmol/L) d. Calcium 9 mg/dL (2.25 mmol/L)

c. Potassium 2.5 mEq/L (2.5 mmol/L) The client low potassium levels is at high risk for ventricular dysrhythmias (normal lab values are 3.5 to 5.0 mEq/L [3.5 - 5 mmol/L]). Premature ventricular contractions may also be caused by low magnesium levels, digoxin and aminophylline toxicity, and hypoxia. Normal values for magnesium are about 1.5 to 2.4 mg/dL (0.75-1.2 mmol/L); normal values for calcium are about 8.5-10.3 mg/dL (2.12 - 2.57 mmol/L)

A 55 year-old woman is taking prednisone and aspirin (ASA) as part of the treatment for rheumatoid arthritis. Which approach would be an appropriate intervention for the nurse? a. Assess the pulse rate every four hours b. Monitor the level of consciousness every shift c. Test stools for occult blood d. Discuss fiber in the diet to prevent constipation

c. Test stools for occult blood Both prednisone and ASA can lead to gastrointestinal bleeding. Therefore, monitoring for occult blood is indicated.

A patient referred for mammography questions the nurse about the cancer risks from radiation exposure. What is the appropriate response by the nurse? a. You have nothing to worry about, it is less than tanning in the nude b. A chest x-ray gives you more radiation exposure c. The radiation from a mammography is equivalent to one hour of sun exposure d. Exposure to mammography every 2 years is not dangerous

c. The radiation from a mammography is equivalent to one hour of sun exposure A patient would have to have several mammograms in a year's time to be at a risk for cancer. The radiation exposure from one mammogram session is thought to be equivalent to being out in natural sunlight for one hour. This answer is concise and gives the patient a point of reference. To say not to worry is judgmental and non therapeutic. In the other 2 options one is not accurate and can cause further concern about radiation exposure and one does not clearly address the patient's question.

Which statement is the correct stage of cognitive development for Piaget's: Infant - Sensorimotor stage a. Concepts are attached to concrete situations b. Analyzes situations and uses abstract logic and reasoning c. Uses sucking, grasping, listening, and looking to earn about the environment d. Uses magical thinking and imagination

c. Uses sucking, grasping, listening, and looking to earn about the environment

A nurse is observing a client during an excretory urogram. Which of these observations indicate there is a complication? a. Within two minutes of the dye injection the client states, "I have a feeling of getting warm." b. A client complains of a salty taste in the mouth when the dye is injected c. Within one minute after the dye is injected the client's entire body turns a bright red color d. Five minutes into the procedure the client gags and states, "I am getting sick."

c. Within one minute after the dye is injected the client's entire body turns a bright red color This observation suggests anaphylaxis from the dye injection, which can cause massive vasodilation and shock. Other findings of anaphylaxis are immediate wheezing and/or respiratory arrest. The salty taste in the mouth, the feeling of warmth and the complaint of nausea are expected side effects of the injection of the dye.

A nurse and a client are talking about the client's progress towards understanding the client's behaviors during stressful situations. This is typical of which phase in a therapeutic relationship? a. Orientation b. Termination c. Working d. Pre-interaction

c. Working During the working phase, alternative behaviors and techniques are explored mutually with a nurse and a client. A discussion of the meaning behind behaviors is one of many approaches during the working phase.

The nurse administer a new medication to the patient. Which of the following actions best demonstrates an awareness of safe and proficient nursing practice? a. verify order prior to administration. ask for patient name b. verify patient's allergies on chart and name on door, ask date of birth c. ask name and allergies, then check wristband and allergy band d. ask name then check wristband

c. ask name and allergies, then check wristband and allergy band A dual check is always done for the patient's name. This would involve verbal and visual checks. Because this is a new medication an allergy check is appropriate. The other option have parts that might be correct actions. However, to be the correct answer all the parts of an option need to be correct.

A nurse is performing well-child assessments at a day care center when a staff member interrupts the exam for assistance with another child. The nurse finds a 3 yo child on the floor with bleeding gums and 2 unlabeled open bottles nearby. What should the nurses first action be? a. call poison control and then 911 b. administer syrup of Ipecac to induce vomiting c. ask the staff member about the contents of the bottles d. give the child milk to coat the stomach

c. ask the staff member about the contents of the bottles The nurse needs to asses the situation and determine what the child ingested. Once the substance is identified, the poison control center and the emergency medical services should be called.

The nurse is caring for a patient who is not oriented to time, place, or person and has repeatedly attempted to pull out IV line and a feeding tube. The nurse receives an order from HCP to apply a vest and soft wrist restraints. Which of the following actions by nurse are appropriate? Select all that apply a. release the restraints and provide care Q4 b. call HCP for new order Q48 c. document which alternative interventions were used or attempted d. tie restraints using quick release knots e. explain the rationale for restraints to patient f. conduct a thorough assessment of the patient

c. document which alternative interventions were used or attempted d. tie restraints using quick release knots e. explain the rationale for restraints to patient f. conduct a thorough assessment of the patient Prior to applying restraints, the nurse must first conduct a thorough assessment of the patient and document the behavior and/or events leading up to the use of the restraints. The nurse should also document which alternatives to restraints were tried and the patient's response to those measures. Even though the patient is confused, nurse must still explain the reason for applying the restraints. A physician's order is renewed daily. Many policies state that the patient in restraints must be assessed hourly, care is given and documented at least every 2 hours.

A nurse is talking with a client. The client abruptly says to the nurse, "The moon is full. Astronauts walk on the moon. Walking is a good health habit." The client's remarks most likely indicate which finding? a. word salad b. neologisms c. loose associations d. flight of ideas

c. loose associations Though the client's statements are not typical of logical communication, the second and third remark contain elements of the preceding sentence (moon, walk). Neologisms refers to making up words that have personal meaning to the client. Flight of ideas defines nearly continuous flow of speech, jumping from one unconnected topic to another. Word salad refers to stringing together real words into nonsense "sentences" that have no meaning for the listener.

The nurse is caring for a 4 year-old child with a greenstick fracture. In explaining this type of fracture to the parents, the best comment by the nurse should include which point? a. "Compression of porous bones produces a buckle or torus type break." b. "Bone fragments often remain attached by a periosteal hinge." c. "A child's bone is more flexible and can be bent 45 degrees before breaking." d. "Bones of children are more porous than adults' and often have incomplete breaks."

d. "Bones of children are more porous than adults' and often have incomplete breaks." This allows the pliable bones of growing children to bend, buckle, and break in a "greenstick" manner. A greenstick fracture occurs when a bone is angulated beyond the limits of bending. The compressed side bends and the tension side develops an incomplete fracture.

The nurse, who is participating in a community health fair, assesses the health status of attendees. When would the nurse conduct a mental status examination? a. The individual reports memory lapses b. There are obvious signs of depression c. The individual displays restlessness d. As part of every health assessment

d. As part of every health assessment A mental status assessment is a critical part of baseline information and should be a part of every examination.

The nurse is evaluating a developmentally challenged 2 year-old child. During the evaluation, what goal should the nurse stress when talking to the child's mother? a. Help the family decide on long-term care b. Prepare for independent toileting c. Teach the child self-care skills d. Promote the child's optimal development

d. Promote the child's optimal development The primary goal of nursing care for a developmentally challenged child is to promote the child's optimal development.

Nursing students are reviewing the various types of oxygen delivery systems. Which oxygen delivery system is the most accurate? a. A nasal cannula b. A partial nonrebreather mask c. The simple face mask d. The Venturi mask

d. The Venturi mask The most accurate way to deliver oxygen to the client is through a Venturi system such as the Venti Mask. The Venti Mask is a high flow device that entrains room air into a reservoir device on the mask and mixes the room air with 100% oxygen. The size of the opening to the reservoir determines the concentration of oxygen. The client's respiratory rate and respiratory pattern do not affect the concentration of oxygen delivered. The maximum amount of oxygen that can be delivered by this system is 55%.

The nurse is preparing to administer total parenteral nutrition (TPN) through a central line. Indicate the correct order in which the following nursing actions should be performed by dragging and dropping the options below. a. Set the infusion pump at the prescribed rate b. Connect the tubing to the central line c. Thread the intravenous tubing through an infusion pump d. Use aseptic technique when handling the injection cap e. Check the solution for cloudiness or sediment f. Select and prime the correct tubing and filter

e. Check the solution for cloudiness or sediment f. Select and prime the correct tubing and filter c. Thread the intravenous tubing through an infusion pump d. Use aseptic technique when handling the injection cap b. Connect the tubing to the central line a. Set the infusion pump at the prescribed rate TPN solution should not be cloudy or have any kind of particles or sediment. The nurse should prepare the equipment by priming the tubing and threading it through the pump. To prevent infection, the nurse must use aseptic technique when inserting the connector into the injection cap and connecting the tubing to the central line. The nurse should then set the pump at the prescribed rate.

blisters dry to form honey-colored, adherent crusts

impetigo contagiosa Impetigo is a highly contagious bacterial infection that's usually found on the face; hands, feet and legs may also be affected.

The prescription is linezolid 600 mg IV in 300 mL of D5W to infuse over 2 hours. The IV tubing drip rate is 10 gtts/mL. What drip rate should the nurse use?

the drip rate is 25 gtts/minute

Drug classification for lorazepam

Antianxiety Agents act at many levels in the CNS to produce anxiolytic effect. They may produce CNS depression and the effects may be mediated by GABA (an inhibitory neurotransmitter). They are used in the treatment of Generalized Anxiety Disorder (GAD) and Panic Disorder; they are also used in the management of anxiety associated with depression.

A myelogram is a painless test that measures the electrical activity in muscles. True False

False A myelogram uses a special dye (oil-based, water-soluble and even air-contrast) and an x-ray (fluoroscopy) to make pictures of the bones and the subarachnoid space between the bones in the spine. Electrical activity in muscles is measured by an electromyogram (EMG); this may be a painful test.

A nursing intervention for prolapsed cord is to gently move the cord back into the vagina. True False

False A prolapsed cord is a medical emergency. The nurse may insert a sterile gloved hand in the vagina and gently push the presenting part away from the umbilical cord. The nurse should never attempt to push the cord back into the vagina.

The Schick test is used to test for allergies. True False

False A variety of different allergy tests can be used. One or more allergen-specific IgE antibody tests may be performed by either intradermal injection or by scratching the skin. Alternately, a radioallergosorbent test (RAST) can measure antibodies in the blood. A Schick test detects the presence of diphtheria toxin.

The nurse should inject approximately 30 mL of air through the nasogstric (NG) tube while auscultating the abdomen to confirm placement of a NG tube. True False

False Evidence-based practice recommends aspirating gastric contents to test the pH (which should be below 5) and/or obtaining an x-ray if there are concerns about the placement (and before instilling any feedings or medications).

The skin of a client with heat stroke is pale and moist. True False

False Findings of heat stroke include reddened skin and lack of perspiration. Body temperature may be greater than 106 F (41 C). This is a true medical emergency.

The nurse should write everything down for a patient with Wernicke's aphasia True or false

False Patients with Wernicke's aphasia may have no understanding of language in any modality, whether spoken or written. They can speak, by what they say makes no sense. Communication may be more effective using non-verbal techniques, such as actions, movements, props, and gestures.

A patient with gout is prescribed a pureed diet. True or false

False Patients with gout should eat a low-purine diet because purines are turned into uric acid, which aggravates the symptoms of gout. Almost any food contains purines: organ meats, anchovies and sardines in oil, mushrooms, spinach, dried beans and peans contain higher levels. The pureed diet is indicated when chewing or swallowing is difficult or causes discomfort.

Primitive defense mechanisms are very effective for long-term use. True or false

False People use defense mechanisms to protect themselves from things they don't want to think about or deal with. Primitive defense mechanisms, such as denial, regression, acting out and projection, are often used by children and can have short-term advantages, but become less effective when used long term.

An indication for total hip replacement is peripheral vascular disease associated with uncontrolled diabetes. True False

False Peripheral vascular disease lead to an amputation, often of the foot, but indications for total hip replacement include osteoarthritis, rheumatoid arthritis, trauma (such as fracture of the femoral head), failure of a prosthesis or avascular necrosis of the femur due to steroid use.

A woman cannot become pregnant when she is breastfeeding. True or false

False Pregnancy can occur with unprotected intercourse at or become the first menstrual cycle after birth. Nurses should caution women to avoid pregnancy for the first three months after delivery to allow the body time to heal.

A woman cannot become pregnant when she is breastfeeding. True or False

False Pregnancy can occur with unprotected intercourse at or before the first menstrual cycle after birth. Nurses should caution women to avoid pregnancy for the first three months after delivery to allow the body time to heal.

Most pregnancy tests measure the level of estrogen in the woman's blood. True or False

False Pregnancy tests measure the hormone human chorionic gonadotropin (hCG) in the urine or in the blood. Levels can be first detected about 12 to 14 days after conception and peak in the first 8 to 11 weeks of pregnancy.

Most pregnancy tests measure the level of estrogen in the woman's blood. True or false

False Pregnancy tests measure the hormone human chorionic gonadotropin (hcG) in the urine or in the blood. Levels can be first detected about 12 to 14 days after conception and peak in the first 8 to 11 weeks of pregnancy.

Women who are planning on becoming parents need about 200 ug daily of folic acid. True or false

False Prior to conception and during early pregnancy, women need to add 400 ug of folic acid (vitamin B9) each day. Research has demonstrated that this significantly decreases the risk of neural tube defects.

Pulse oximetry replaces the need to obtain arterial blood gases (ABGs). True False

False Pulse oximetry estimates oxygen (O2) saturation (SpO2) of capillary blood and these estimates typically correlate to arterial O2 saturation measurements (SaO2) when used correctly. But, pulse oximetry cannot detect hypercapnia or acidosis. An ABG is needed for accurate measurements of PaO2, PaCO2 and blood pH.

The nurse should use open-ended questions during admission of cognitively impaired clients. True False

False Questions requiring a simple yes or no response are used if thinking abilities are impaired.

The tonic component of the generalized seizure lasts longer than the clonic component. True False

False Rigid contracture of muscles (the tonic phase) is usually brief. The clonic component is the rhythmic shaking that occurs during the seizure; it lasts longer than the tonic component. A generalized tonic-clonic seizure is also known as a grand mal seizure.

Examples of Schedule I medications include morphine and secobarbital. True or False

False Schedule I drugs have no medical use and there is a high potential for abuse. Examples of Schedule I drugs include heroin and LSD. While morphine and short-acting barbiturates like secobarbital also have a high risk for abuse, they also have safe and accepted uses; they are examples of Schedule II drugs.

To screen for scoliosis, look at the child's silhouette and note asymmetries in the trunk and legs. True False

False Scoliosis is commonly detected during the preadolescent growth spurt. In a child who has scoliosis, there will be uneven hips or shoulders, an abnormal lateral curvature of the spine, ribs that are higher on one side or a waistline that may be flat on one side.

Neuropathic pain is also called musculoskeletal pain. True or false

False Somatic pain is also known as musculoskeletal pain because it originates in the tissues such as the skin, muscle, joints, bones, and ligaments. Neuropathic pain originates from the nervous system.

Primary enuresis develops several years after a person has learned to control his or her bladder. True False

False Someone with primary enuresis has never been able to control his or her bladder. Secondary enuresis is a condition that develops at least 6 months after learning to control the bladder. Common underlying problems associated with bedwetting include constipation and cystitis; many times the cause is idiopathic.

An infant with spina bifida may have a meningocele and neurological deficits and may experience seizures. True False

False Spina bifida is the most frequently occurring and permanently disabling birth defect in the U.S. Findings vary depending on the level of the lesion and type of defect, but spina bifida is not associated with seizures.

Stress activates the parasympathetic nervous system True or False

False Stress activates the sympathetic nervous system (norepinephrine and epinephrine) and the endocrine system (especially the pituitary gland). The sympathetic nervous system is responsible for stimulating the "fight-or-flight" response often associated with stress. The process under which the body confronts stress is the General Adaptation Syndrome.

The nurse should frequently suction the airway of clients with pneumonia and bronchitis. True False

False Suctioning should only be done when clinically necessary and when the client is physically unable to cough up secretions on his or her own. Clinical indicators for suctioning include coarse breath sounds, noisy breathing, increased or decreased pulse, increased or decreased respiration, and prolonged expiratory breath sounds.

The endocardium is a sac that surrounds and supports the heart. True False

False The "endo (inside) cardium (heart)" is the layer inside the myocardium.

The QRS complex of the ECG is when the atria depolarize. True False

False The QRS complex of the ECG represents the time it takes for depolarization of the ventricles. The duration of the QRS complex is normally 0.06 to 0.12 seconds. The P wave represents depolarization of the atria.

The nurse can use the deltoid muscle, vastus lateralis muscle, ventrogluteal muscle and dorsogluteal muscle to administer an intramuscular injection. True or False

False The accepted sites for intramuscular injections include the deltoid muscle, vastus lateralis muscle and ventrogluteal muscle. The dorsogluteal muscle should not be used for IM injections due to the risk of sciatic nerve damage.

The aorta is a vein. True False

False The aorta is the largest artery of the body. It exits the left ventricle to distribute oxygenated blood to the body.

Croup syndromes are treated with antibiotics and cool air/mist. True False

False The most common form of croup is acute laryngotracheobronchitis or viral croup, which is an infection of both the upper and lower respiratory tracts. The classic "barky" harsh cough, stridor and fever are treated with antipyretics and cool air/mist.

An APGAR score of 2 for appearance means the newborn's fingers and toes are bluish in color. True or False

False The normal color all over for the newborn is pink; a pink baby earns a score of 2. A baby who is pink with pale blue toes/feet and fingers/hands will receive a score of 1 on the APGAR test.

An APGAR score of 2 for appearance means the newborn's fingers and toes are blush in color. True or false

False The normal color all over the newborn is pink; a pink baby earns a score of 2. A baby who is pink with pale blue toes/feet and fingers/hands will receive a score 1 on the APGAR test.

The pain of cluster headaches comes on slowly and takes days to resolve. True False

False The pain of a cluster headache comes on suddenly and usually subsides quickly, before even over-the-counter pain relievers such as ibuprofen or acetaminophen can start working. Triptans can provide effective acute treatment for cluster headaches.

There is a valve between the portal vein and the left atrium. True False

False The portal vein has no direct connection to the left atrium, which receives blood from the pulmonary vein.

The safest time for the fetus is to give the mother analgesia when her cervix is dilated 8 to 10 centimeters. True or False

False The safest time to offer analgesia is when dilation is between 4 to 7 centimeters.

The safest time for the fetus is to give the mother analgesia when her cervix is dilated 8 to 10 centimeters. True or false

False The safest time to offer analgesia is when dilation is between 4 to 7 centimeters.

The 4th stage of labor is placental separation and expulsion True or False

False The third stage of labor is placental separation and expulsion and lasts about 5 to 30 minutes. The fourth stage of labor is maternal adaptation, occurring 1 to 2 hours after birth.

The fourth stage of labor is placental separation and expulsion. True or false

False The third stage of labor is placental separation and expulsion and lasts about 5 to 30 minutes. The fourth stage of labor is maternal adaptation, occurring 1 to 2 hours after birth.

The grieving process lasts for approximately one year. True or false

False The time span of the grieving process varies and there is not set time limit for how long an individual grieves. Also, the stages of grieving are not linear. They may pass and later return.

One of the first signs of pregnancy is Chadwick's sign, which is the softening of the cervix. True or False

False There are several findings of pregnancy during the first trimester. Increased vascularity in the vagina is called Chadwick's sign; the increased vascularization and softness of the uterine isthmus is Hegar's sign; and the softening of the cervix is Goodell's sign.

Juvenile idiopathic arthritis is best managed by diet, hormones and range-of-motion exercises. True False

False This autoimmune disorder is managed with exercise, as well as physical and occupational therapies, electrical stimulation, heat and/or whirlpool treatments. NSAIDs can help control symptoms; corticosteroids, disease-modifying antirheumatic drugs, immunosuppressives and cytotoxic agents are also used. Neither diet modification nor hormones are used to treat this disorder.

The treatment for frostbite includes massaging the affected skin. True False

False Tissue damage can occur if the skin with frostbite is massaged. The affected site should be rewarmed gently and gradually with warm water or wet heat.

Urinary incontinence is a normal part of aging. True or false

False Urinary incontinence is not normal, regardless of the patient's age. Kegal exercises, medications (anticholinergics, topical estrogen), medical devices (pessary), or surgery (sling procedures, bladder neck suppression) can help to counteract incontinence.

Clients with gestational diabetes are treated with oral hypoglycemic agents. True False

False Use of oral hypoglycemic agents is not recommended due to their potential teratogenicity. Instead, insulin therapy is used during pregnancy.

Hypothyroidism is a congenital disease that may manifest in children as lethargy, constipation, feeding problems and slow growth. True False

True An underactive thyroid is a congenital disease in children; symptoms may not appear until the child is 2 or 3 years old. Treatment includes a client taking levothyroxine for the rest of his/her life.

The nurse applies mild pressure to the inner canthus of the eye after instilling eye drop medication. True or False

True Applying pressure to the inner aspect of the eye for about a minute or so helps decrease systemic absorption of the medication.

Less than 4 to 8 wet diapers may be a sign of dehydration in a baby. True or false

True Babies should have a minimum of 4 to 8 wet diapers daily. Babies can become quickly dehydrated. Other signs of dehydration in infants include sunken fontanel, decrease or absence of tears, dry mouth and irritability.

Sensor pads may be used on beds of individuals who are a fall risk True or False

True Bed alarms and sensor pads can be used to alert caregivers when a patient is attempting to get up from a bed or chair, especially for a patient that is at risk for a fall. This is an effective alternative to the use of restraintts

Preeclampsia is clinically defined by hypertension and proteinuria. True False

True Mild preeclampsia includes a blood pressure of greater than 140/90 mmHg on 2 occasions and dipstick readings of excessive (1+ or 2+) protein on random urine samples after the 20th week of pregnancy. If preeclampsia-associated seizures develop, this condition is called eclampsia.

Colon cancer may develop from adenomatous polyps. True False

True Most colon polyps are benign growths. But some growths can turn into colon cancer. Polyps found during a colonoscopy can be removed and examined.

A common cause of increased intracranial pressure (ICP) in infants is hydrocephalus. True False

True Most hydrocephalus occurs in infancy and may be associated with a myelocele or myelomeningocele. Common findings of increased ICP include full or bulging fontanels, macrocephaly, poor feeding, vomiting and irritability.

One way to treat heat exhaustion is to get the client into a cool place. True False

True Moving the client to a cool place allows for lowering of the client's body temperature. If possible, also have the client lie down and rest and offer cool water or fruit juice. It is also helpful for the client to loosen or remove any unnecessary clothing.

Neonates with tracheoesophageal fistula (TEF) may develop copious amounts of fine white frothy bubbles of mucus in the mouth and nose. True False

True Neonates with TEF develop these secretions, which recur despite suctioning. They may also develop rattling respiration and episodes of coughing, choking and cyanosis.

About 5 days after delivery, loch is pink-brown in color. True or false

True Normal bleeding and discharge should be more watery and pink-brown colored (lochia serosa) about 3 to 5 days after delivery. It may take up to 2 to 4 weeks for discharge to taper off completely.

About 5 days after delivery, lochia is pink-brown in color. True or False

True Normal bleeding and discharge should be more watery and pink-brown colored (lochia serosa) about 3 to 5 days after delivery. It may take up to 2 to 4 weeks for discharge to taper off completely.

Some symptoms of osteomalacia include kyphosis, difficulty walking, deformation of weight-bearing bones and pain in the low back and hips. True False

True Osteomalacia is softening of bone. It may be caused by poor dietary intake or poor absorption of calcium and other minerals; it is a characteristic feature of vitamin D deficiency in adults. The more obvious effects may appear in major weight-bearing joints such as the back, hips and legs.

Paget's Disease (neuropathic joint disease) is characterized by overactive osteoclasts. True False

True Paget's Disease attacks the mechanism that replaces old cells with new ones. The overactive osteoclasts rapidly restore bone cells and, as a result, the bone that is formed is abnormal, i.e., enlarged, not as dense, brittle and prone to fractures.

If a draining wound tests positive for MRSA, the patient is placed on contact precautions True or False

True Patients with abscess or draining wounds who tests positive for MRSA are placed on contact precautions

The initial finding of placenta previa is sudden, painless bleeding from the vagina. True False

True Placenta previa is where the placenta lies low in the uterus and either partially or completely covers the cervix. The most common symptom is painless bleeding during the 3rd trimester.

The fetus receives more oxygenated blood when the laboring mother lies on her side. True or False

True Positioning the laboring mother on her (left) side usually results in a higher fetal oxygen saturation. Other measures to increase fetal oxygenation (and placental perfusion) include administering oxygen to the laboring woman.

The fetus receives more oxygenated blood when the laboring mother lies on her side. True or false

True Positioning the laboring mother on her (left) side usually results in a higher fetal oxygen saturation. Other measures to increase fetal oxygenation (and placental perfusion) include administering oxygen to the laboring woman.

Religious beliefs influence decisions about health. True or false

True Religious beliefs impact all aspects of a client's life, including health and illness. Research supports that worship and prayer contribute to positive emotions, including hope and spiritual contentment.

A person who has migraine headaches should avoid foods containing monosodium glutamate, tyramine and caffeine. True False

True Many things can trigger migraine headaches, including hormonal changes, stress, sensory stimuli and sleep (too much or too little). Common food triggers include alcohol, aged cheeses (which contain tyramine), chocolate, overuse of caffeine and MSG.

Narrow or obstructed valve related to pulmonary stenosis. Etiology Finding

Etiology Pulmonary stenosis is, by definition, a narrowing of the pulmonary valve.

What is the drug classification for: nitroglycerin

Antianginals are vasodilators - they dilate the blood vessels, improving blood flow and allowing more oxygen-rich blood to reach the heart muscle and they also relax the veins. They are used to treat and prevent attacks of (acute) angina.

Rheumatic fever related to endocarditis. Etiology Finding

Etiology Rheumatic fever is usually caused by an untreated streptococcal infection. The risk for developing endocarditis increases if a person had rheumatic fever or rheumatic heart disease (usually as a child).

The school nurse is teaching the faculty the most effective methods to prevent the spread of lice (Pediculus humanus capitis) in the school. The information that would be most important to include is reflected in which of these statements? "The treatment medication may require reapplication in 8 to 10 days." "Nit combs should be used to remove the eggs (nits) from your child's hair." "Children should not share hats, scarves and combs." "Bedding and clothing should be washed in hot water and dried in the dryer."

"Children should not share hats, scarves and combs." Head lice live only on human beings and can be spread easily by sharing hats, combs, scarves, coats and other items of clothing that touch the hair. All of the options are correct statements. However they do not best answer the question of how to prevent the spread of lice in a school setting.

A mother of a burned child asks the nurse to clarify what is meant by a third-degree burn. The best response by the nurse should include which point? "The top layer of the skin is destroyed." "All layers of the skin were destroyed in the burn." "The skin layers are swollen and reddened." "Muscle, tissue and bone have been injured."

"All layers of the skin were destroyed in the burn." A third-degree burn is a full thickness injury to dermis, epidermis and subcutaneous tissue.

A clinic nurse is discussing health promotion with a group of parents when a mother expresses concern about Reye's syndrome and asks about prevention. Which comment demonstrates appropriate teaching by the nurse? "Avoid use of aspirin for viral infections in children." "Immunize your child against this disease." "Report exposure to this illness as soon as possible." "Seek medical attention for serious injuries."

"Avoid use of aspirin for viral infections in children." The link between aspirin use and Reye's syndrome has not been confirmed. However, evidence suggests that the risk is sufficiently grave to include the warning on aspirin products. A study by Belay et al., (1999) concluded that Reye's syndrome, a severe neurologic disease that causes death or long-term neurologic sequelae in about one-third of patients, was shown to be epidemiologically associated with the ingestion of aspirin, a widely used analgesic and antipyretic agent.

The registered nurse is giving instructions to an unlicensed assistive person (UAP) regarding client care activities for the shift. Which directive provides the best information about the assigned tasks? "Stop by room 215 A and let me know how the new admission is doing and tell me if you need any help." "You will need to frequently take an oral temperature for the client in room 212 B today and report the results to me immediately if it is too high." "Beginning at 0800, empty the urinary catheter bag hourly for the client in room 210 A and write the amount with the time on the whiteboard." "Ambulate the client in room 214 A today and replace the sequential compression device (SCD) afterwards."

"Beginning at 0800, empty the urinary catheter bag hourly for the client in room 210 A and write the amount with the time on the whiteboard." When assigning tasks, directions must be clear, concise, correct and complete. Emptying the catheter bag and recording the amount hourly starting at 0800 meets these guidelines. The options related to ambulation and measuring the client's temperature are vague and incomplete. The option about the new admission is also vague and requires assessing the client; only nurses can assess clients.

A client was admitted to the psychiatric unit for severe depression. After several days, the client continues to withdraw from the other clients. Which of these statements by the nurse would be the most appropriate to promote interaction with other clients? "Come play a few games of Chinese Checkers with Gloria and me." "Your team here thinks it's good for you to spend time with others." "It is important for you to participate in group activities." "Come with me so you can paint a picture to help you feel better."

"Come play a few games of Chinese Checkers with Gloria and me." This gradually engages the client in interactions with others in small groups rather than large groups. In addition, a focus on an activity is less anxiety-provoking than unstructured discussion. The statement is an example of a positive behavioral expectation.

The nurse is providing instructions for a client diagnosed with bacterial pneumonia. What is the most important information to convey to the client? "Complete all of the antibiotics, even if you feel better." "Take your temperature every day." "Take at least two weeks off from work." "You will need another chest x-ray in six weeks."

"Complete all of the antibiotics, even if you feel better." To avoid a recurrence of infection, the client must complete all of the prescribed medications at the prescribed dosing intervals. It should be explained to the client that it may take two weeks or more for his energy level to return to normal, but he may not necessarily need to be off of work for two weeks. The health care provider may order a follow-up CXR, but this is not the priority. It is also not important to take his temperature daily unless he experiences worsening symptoms (such as chills, shortness of breath, chest pain, night sweats.)

An unlicensed assistive person (UAP), who usually works in pediatrics, is reassigned to work on an adult medical-surgical unit. Which of these questions should the charge nurse ask prior to assigning nursing activities and tasks to the UAP? "Do you have your competency checklist that we can review?" "How long have you been a UAP?" "What type of client care did you give in pediatrics?" "How comfortable are you caring for adult clients?"

"Do you have your competency checklist that we can review?" The UAP must be competent to accept assigned task. Using a checklist is the most comprehensive and appropriate approach to determine the UAP's skill set. In order to assign the correct types of tasks to the UAP, you must further assess his/her qualifications. The length of time in a position does not guarantee competency. Client care in pediatrics may not necessarily be relevant on an adult unit. Finally, although the charge nurse should be sensitive to the UAP's feelings, this is not a priority when assigning tasks or activities.

A client is prescribed trimethoprim/sulfamethoxazole for recurrent urinary tract infections. Which information would be important to reinforce during client teaching? "It is safe to take with oral contraceptives." "Drink at least eight glasses of water a day." "Be sure to take the medication with food." "Stop the medication after five days."

"Drink at least eight glasses of water a day." Trimethoprim/sulfamethoxazole (Bactrim) is a highly insoluble drug and clients should drink plenty of fluids while taking this medication to lower the risk of developing kidney stones. It is not necessary to take it with food, unless it causes stomach upset. When taking antibiotics, women who normally use oral contraceptives should be counseled to use additional forms of birth control. Clients should take the medication for the prescribed length of time.

A client is being discharged with a prescription for an iron supplement. What statement indicates a need for further teaching by the nurse? "I will have greenish-black stools from the medication." "I should not take antacids with my iron supplement." "I should take the iron supplement with a full glass of milk." "I should take vitamin C with the iron supplement."

"I should take the iron supplement with a full glass of milk." Iron should also be taken with vitamin C or orange juice because this increases the absorption of the medication; conversely, antacids, milk, caffeinated beverages, and calcium supplements can decrease the absorption of iron. Iron will cause the client's stool to turn greenish-black and tarry. Iron should be taken one hour before or two hours after meals to enhance absorption, although clients with GI intolerance may take the pills with food.

Which statement by an older adult with chronic obstructive lung disease (COPD) indicates an understanding of the major reason to use pursed-lip breathing for episodes of dyspnea? "This position of my lips helps to keep my lungs open." "I can breathe better when I pucker up my lips because I can control how fast I breathe in and out." "I can breathe better using pursed-lip breathing because less air will be trapped in my chest." "My mouth doesn't get as dry when I breathe with pursed lips."

"I can breathe better using pursed-lip breathing because less air will be trapped in my chest." Clients with chronic obstructive pulmonary disease (COPD) have difficulty exhaling fully as a result of air trapping in the alveoli due to the weakened alveolar walls from the disease process. Alveolar collapse can be avoided with the use of pursed-lip breathing, allowing the client to exhale more effectively. This technique facilitates appropriate gas exchange as carbon dioxide-rich air that has been trapped in the lungs is blown off, allowing oxygen-rich air to be inhaled. This is the major reason to use pursed-lip breathing. The other options are additional beneficial effects of this breathing technique.

Which statement made by a client to an admission nurse suggests that the client is experiencing a manic episode? "I think all of my contacts last week have attempted to poison me." "I think all children should have their heads shaved." "I have been restricted in thought and harmed." "I have powers to get you whatever you wish, no matter the cost."

"I have powers to get you whatever you wish, no matter the cost." This statement reflects grandiosity which is characteristic of a manic episode. Thinking that someone has been attempted to be poisoned is a paranoid thought.

A 6 year-old child diagnosed with acute glomerulonephritis (AGN) presents with anorexia, moderate edema and elevated blood urea nitrogen (BUN) levels. The child requests a peanut butter sandwich for lunch. How should the nurse respond to this request? "I'm sorry, that is not a good choice, but you could have pasta." "That's a good choice, and I know it is your favorite. You can have it today." "I know that is your favorite, but let me help you pick another lunch." "You cannot have the peanut butter until you are feeling better."

"I know that is your favorite, but let me help you pick another lunch." Children with AGN who have edema, hypertension, oliguria and azotemia may have dietary restrictions limiting sodium, fluids, protein and potassium. Because peanuts are made of protein, fats, and carbohydrates, and the sodium content of a tablespoon of peanut butter can be as high a 80 mg, a different choice for lunch might be best. Giving the child a short explanation and offering to talk about an alternative food is appropriate for this age.

A 54 year-old female explains to the health care provider that she experiences approximately 10 vasomotor symptoms of menopause ("hot flashes") throughout the day and night. Different treatment options are discussed. Which statement by the client indicates she needs further instruction from the nurse? "I will take gabapentin ER at bedtime." "I should avoid spicy foods, alcohol, and caffeine." "I can use a fan at home and in the workplace." "I may need to take estrogen and progesterone for many years."

"I may need to take estrogen and progesterone for many years." In addition to menopausal hormone therapy (MHT), medications for epilepsy (gabapentin), depression (SSRIs) and hypertension can be used to treat hot flashes. Extended release gabapentin is taken at bedtime to treat insomnia due to hot flashes. Although the risk of low-dose estrogen is small, there is still a risk of breast cancer, heart attack, and blood clots with menopausal hormone therapy (MHT), which is why it should only be a short-term treatment option. Non-medical interventions include avoiding spicy foods, alcohol, and caffeine. Clients should also dress in layers, use fans for cooling and try taking slow, deep breaths when a hot flash starts.

A nurse is teaching a client about digoxin toxicity. Which statement made by the client is incorrect and indicates more teaching is needed? "High levels of digoxin can cause vision changes." "I must immediately report a strong pulse of 62 bpm." "I should report nausea and vomiting lasting more than a few days." "I'll let you know if my pulse feels uneven and misses beats."

"I must immediately report a strong pulse of 62 bpm." Digoxin helps to make the heart beat stronger and with a more regular rhythm, which is why clients should understand that they should not take the drug if their pulse rate is less than 60 (or above 100). Clients should know to report the common symptoms of digoxin toxicity, such as irregular pulse, loss of appetite, nausea and vomiting and vision changes. The nurse needs to help the client understand that he or she does not need to contact the health care provider when his or her pulse is 62 bpm.

A client diagnosed of bipolar disorder has been referred to a local halfway house for placement consideration. How should the nurse respond when the social worker calls to obtain information about the client's mental status and adjustment during hospitalization? "I can never give any information out by telephone. How do I know who you are?" "Since this is a referral, I can give you a limited amount of information." "I need to get the client's written consent before I release any information to you." "I am sorry. Referral information can only be provided by the client's providers."

"I need to get the client's written consent before I release any information to you." In order to release information about a client there must be a signed consent form designating to whom information can be given, and what information can be shared. The other actions are incorrect approaches to this request.

The nurse is teaching the client about the patient controlled analgesia (PCA) planned for postoperative care. Which statement by the client is incorrect and indicates that further teaching is needed? "I will receive a continuous dose of medication." "I should call the nurse before I take additional doses." "The machine will prevent an overdose of the medication." "I will call for assistance if my pain is not relieved."

"I should call the nurse before I take additional doses." Patient controlled analgesia offers the client more control in the prevention and relief of severe pain. The client should be instructed to initiate additional doses as needed when the pain is increased. The client needs to know to call for assistance when insufficient control of the pain is present. The other statements illustrate correct knowledge.

While planning care for a toddler, the nurse teaches the parents about the expected developmental changes for this age. Which statement by the mother shows that she understands the child's developmental needs? "I understand our child's need to use those new skills." "I intend to keep control over our child's behavior." "I want to protect my child from any falls." "I will set limits on exploring the house."

"I understand our child's need to use those new skills." Erickson describes the stage of the toddler as being the time when there is normally an increase in autonomy. The child's developmental needs are to use motor skills for an exploration of the environment.

A staff nurse complains to the nurse manager that an unlicensed assistive personnel (UAP) consistently leaves the work area untidy and does not restock supplies. The initial response by the nurse manager should be which of these statements? "I would like for you to approach the UAP about the problem the next time it occurs." Correct! "I can assure you that I will look into the matter in due time." "I will add this concern to the agenda for the next unit meeting so we can discuss it." "I will arrange for a conference with you and the UAP within the next week"

"I would like for you to approach the UAP about the problem the next time it occurs." Part of the manager's role is to help the staff manage conflict among themselves. It is appropriate to urge the nurse to confront the other staff member to work out problems without a manager's intervention when possible. This is an approach at the first level of management. If the two staff members cannot resolve the issue then the manager would have a conference with the two staff to facilitate a negotiation for a win-win result.

A nurse is providing information to a young adult client recently diagnosed with testicular cancer and asks the client to state his understanding of the disease. Which of the statements made by the client indicates a need for additional correct information? "If I have cancer at stage III it means I have a better chance of recovery than if it were stage I or II." "After surgery, I can have a prosthesis placed inside my scrotum that will look and feel like the real thing." "The surgeon will remove one or both of my testicles and maybe the lymph nodes in my lower belly, too." "I may want to have my sperm frozen and preserved before starting chemotherapy."

"If I have cancer at stage III it means I have a better chance of recovery than if it were stage I or II." There are five stages of testicular cancer, ranging from Stage 0 to Stage IV; the higher the number, the more extensive the disease. Stage III testicular cancer means the cancer has spread beyond the lymph nodes (possibly to the liver, lungs or brain). The information in the other statements indicates appropriate understanding of this diagnosis and its treatment.

The nurse is caring for a client with inflammatory bowel disease who admits to using complementary therapies, including herbal remedies and peppermint tea. Which of the following statements made by the nurse is the most appropriate response? "It is important to inform your health care provider of the use of these therapies." "These therapies are probably not harmful but may be costing you unnecessary money." "These therapies are known to interfere with prescribed medications so it is important to stop using them." "I would suggest that you discontinue the use of these therapies as they may be dangerous."

"It is important to inform your health care provider of the use of these therapies." Herbal remedies and peppermint tea have been shown to provide some relief from symptoms but may interact with prescription medications. Therefore, the health care provider needs to be aware of the use of all complementary & integrative health therapies.

A pregnant client asks a nurse about the purpose of a blood test for alpha-fetoprotein (AFP). What would be the nurse's best response? "Possible neurological defects may be identified." "It tells us how far along your pregnancy is." "Placental well-being is being evaluated." "The results help determine if the baby is growing normally."

"Possible neurological defects may be identified." A fetus with neural tube defects loses alpha-fetoprotein (AFP) to the amniotic fluid and hence the maternal blood. High levels in the blood indicate the possibility of defects such as spina bifida and meningocele. Further evaluative tests are indicated if a test is positive.

During the check-up of a 2 month-old infant at a well-baby clinic, a mother expresses concern to a nurse because a flat pink birthmark on the baby's forehead and eyelid has not gone away. What is an appropriate response by the nurse? "Telangiectatic nevi are normal and will disappear as the baby grows." "Port wine stains are often associated with other malformations." "Mongolian spots are a normal finding in dark-skinned children." "The child is too young for consideration of surgical removal of these at this time."

"Telangiectatic nevi are normal and will disappear as the baby grows." Telangiectatic nevi, salmon patch or stork bite birthmarks, are a normal variation and the facial nevi will generally disappear by the time the child is 1 to 2 years old.

The nurse assesses a client who has been re-admitted to the psychiatric inpatient unit with a diagnosis of schizophrenia. The client's symptoms have been managed for several months with fluphenazine (Prolixin). Which should be the initial focus of the questioning during the admission assessment? "How long have you been outside in the hot weather this prior week?" "Tell me about your medication routine and when do you take it?" "What stressors do you have living in your home by yourself?" "How much alcohol in the form of beer, wine of hard liquor do you use each day?"

"Tell me about your medication routine and when do you take it?" Prolixin is an antipsychotic/neuroleptic medication useful in the management of the symptoms of schizophrenia. A side effect of this medication is increased sensitivity to sunlight, but asking about being outside in the hot weather would not be an initial focus. Medication compliance is a priority to investigate in the initial part of the admission assessment and it is a priority to convey this noncompliance to the health care provider because it may be associated with the need for admission to an inpatient unit.

The home health aide calls the nurse to report information about a client. Which of these findings should the nurse act on? "The client reports not sleeping well for the past week." "The partner says the client has gotten slower when doing things every other day." "The urine in the urinary catheter bag is of a deeper amber, almost brown color." "The family wants to discontinue the home meal service called Meals on Wheels."

"The urine in the urinary catheter bag is of a deeper amber, almost brown color." Home health aides often report diverse information to nurses through phone calls and documentation. The nurse who develops the plan of care for a specific client, and supervises the aide, must identify potential danger signs that require immediate action and follow-up. The most significant finding is the color of the urine for the one client; this requires follow-up and additional data collection prior to contacting the health care provider. The other options may need further assessment but are not the priority.

An emergency department nurse admits a child who experienced a seizure at school. When the parent comments that this is the first occurrence and denies any family history of epilepsy, what is the best response by the nurse? "Long-term treatment will prevent future seizures." "This seizure may or may not mean your child has epilepsy. Further evaluation is needed." "Since this was the first convulsion, it may not happen again." "Do not worry. Epilepsy can be treated with medications."

"This seizure may or may not mean your child has epilepsy. Further evaluation is needed." There are many possible causes for a childhood seizure. Some causes are transient, others require long-term treatment to prevent further seizures. Causes of seizure in childhood include include fever, central nervous system conditions, trauma, metabolic alterations and idiopathic (unknown) etiologies. EEG, electroencephalogram, is a test commonly used to evaluate seizure disorders.

A client diagnosed with bipolar disorder is reluctant to take lithium (Lithane) as prescribed. What is the most therapeutic response by a nurse to a refusal of the medication? "What is it about the medicine that you don't like or that you are afraid of?" "I can see that you are uncomfortable right now. I'll wait until tomorrow to discuss this with you." "You need to take your medicine. This is how you will get well." "If you refuse your medicine, we'll just have to give you a shot."

"What is it about the medicine that you don't like or that you are afraid of?" Nursing interventions for clients with psychotic disorders are aimed at the establishment of a trusting relationship, clear communications, presentation of reality and reinforcement of appropriate behaviors. This response validates the reluctance of the client with steps to work through the reluctance for an outcome of acceptance that medication is critical for treatment.

The nurse is assessing an 8 month-old infant diagnosed with atonic cerebral palsy. Which statement from the parent supports this diagnosis? "When I put her on her back to sleep, she's still in the same position a few hours later." "My baby doesn't seem to follow when I shake toys in front of her face." "When I put my finger in her left hand she doesn't respond with a grasp." "When it thundered loudly last night she didn't even jump."

"When I put her on her back to sleep, she's still in the same position a few hours later." Cerebral palsy is known as a condition whereby motor dysfunction occurs secondary to damage in the motor centers of the brain. Inability to roll over by eight months of age would illustrate one delay in the infant's attainment of developmental milestones. Cerebral palsy is most commonly associated with cerebral hypoxia during the birth process.

The nurse is reviewing the surgeon's discharge instructions with a client who experienced a myocardial infarction. The client asks the nurse why the waiting period is four to six weeks before having sexual intercourse. Which response best explains this instruction? "If you can maintain an active walking program, you will have less risk." "Have a glass of wine to relax you, then you can try to have sex." "You need to regain your strength before attempting such exertion." "When you can climb two flights of stairs without problems, it is generally safe."

"When you can climb two flights of stairs without problems, it is generally safe." Although it depends on the client's overall medical condition, most experts say that sexual activity is physically therapeutic and heart-healthy. If the client can comfortably climb up a flight of stairs without feeling tired, short of breath or having chest pain, then s/he is probably ready to resume having sex.

One hour before the first treatment is scheduled, a client becomes anxious and states, "I do not wish to go through with electroconvulsive therapy." Which response by the nurse is most appropriate? "You have the right to change your mind. You seem anxious. Can we talk about it?" "I'll go with you and will be there with you during the treatment." "You'll be asleep and won't remember anything." "I'll call the health care providers to notify them of your decision."

"You have the right to change your mind. You seem anxious. Can we talk about it?" This response indicates acknowledgment of the client's rights and the opportunity for the client to clarify and ventilate concerns. After this, if the client continues to refuse, the health care providers should be notified.

A client diagnosed with tuberculosis is started on rifampin and isoniazid. Which statement by the nurse would be most important to include in teaching the client about rifampin? "You should not skip doses or stop your medicine even if you feel better." "You should avoid drinking alcohol while taking this medication." "You may experience some nausea if you take the medication with food." "You may notice an orange-red color to your urine."

"You may notice an orange-red color to your urine." Rifampin can cause a harmless reddish-orange discoloration of saliva, sweat, tears, feces, urine and skin; clients should not wear soft contact lenses while taking this medication. Even though all responses are correct, this is the most important effect that clients should know about. Rifampin should be taken with a full glass of water on an empty stomach, one hour before or two hours after a meal. Clients should not drink alcohol while taking this medication due to its effect on the liver. As with any antibiotic, clients need to understand that they must continue taking the medication even if they start to feel better, especially since clients are taking this medication for many months.

A client with paranoid delusions stares at a nurse over a period of several days. The client suddenly walks up to the nurse and shouts, "You think you're so perfect and pure and good." Which statement would be the most appropriate response for the nurse to make? "Is that why you've been staring at me?" "You seem angry right now." "Perfect? I don't quite understand." "You seem to be in a really bad mood."

"You seem angry right now." The nurse recognizes the underlying emotion with a matter of fact attitude. The nurse should avoid telling the clients how the nurse feels. A general rule for interactions between clients with a psychiatric diagnosis and staff members is to focus on feelings first when giving responses to behaviors.

The order reads: start nitroglycerin intravenously for chest pain; titrate to keep client pain free and keep the systolic blood pressure greater than or equal to 90 mm Hg. The pharmacy sends a 250 mL bottle of D5W with 25 mg nitroglycerin added. The client weighs 215 pounds and the nurse has titrated the nitroglycerin drip to 47 micrograms per minute (mcg/min). What is the infusion rate for the nitroglycerin? (Use a whole number to answer the question and write only the number.)mL/hour.

(mg on hand/mL on hand) (convert mg to mcg) (25 mg/250 mL) = 0.1 mg/mL = 100 mcg/mL mL/hr = [(desired dose in mcg/min)/ (concentration in mcg/mL)]x(60) X= (47/100) (60) X= 28.2 or 28 mL/hr

The nurse should assist a client with an above the knee amputation to lie in the prone position several times a day. True False

True Lying on the stomach will help stretch the hip flexor muscles. The client should lie in the prone position for about 20 minutes, 3 to 4 times a day.

The charge nurse on a cardiac step-down unit makes assignments for the team that consists of a registered nurse (RN), a licensed practical nurse (LPN), and an unlicensed assistive person (UAP). Which client should be assigned to the LPN? A 58 year-old with a history of hypertension, diagnosed with possible angina A 65 year-old scheduled for discharge after angioplasty and stent placement A 35 year-old who is 12 hours post cardiac catheterization A 49 year-old diagnosed with a new onset atrial fibrillation with a rapid ventricular response

A 58 year-old with a history of hypertension, diagnosed with possible angina The client with a history of hypertension and suspected angina is the most stable. Moreover, there's minimal risk of instability for this client when compared to the clients in the other options. RNs should be assigned to care for any clients who require education and teaching, including clients who are scheduled for discharge or clients who require in-depth assessment, such as new admits or those returning from surgery.

To which nursing home resident could a nurse safely administer tricyclic antidepressants (TCAs) without questioning the health care provider's order? A 65 year-old Asian-American female with mild hypertension An 85 year-old Caucasian male with narrow-angle glaucoma An Hispanic female with coronary artery disease (CAD) An African-American male with benign prostatic hypertrophy (BPH)

A 65 year-old Asian-American female with mild hypertension Tricyclics can be safely administered to the hypertensive client. The expected anticholinergic effects of tricyclic antidepressants include difficulty in urination, which is why TCAs are contraindicated with BPH. TCAs are also contraindicated in narrow-angle glaucoma (they can cause elevated pressure in the eyes) and for certain heart abnormalities.

The nurse is providing care to a nullipara client who, at 12-weeks gestation, has just begun prenatal care. Her lab results indicate she is positive for human immunodeficiency virus (HIV). Which of the following statements is most important for the client to understand about caring for herself and her baby? A cesarean section will be scheduled before the woman's membranes rupture or she goes into labor Breast-feeding is recommended because the health benefits of breast-feeding outweigh the risks of HIV transmission Pregnancy is known to accelerate the course of HIV disease in the mother Medication for HIV infection should be started immediately after birth for both mother and baby

A cesarean section will be scheduled before the woman's membranes rupture or she goes into labor According to evidence-based research, administration of antiviral medications during pregnancy, a cesarean birth before membrane rupture and labor, and no breast-feeding have significantly reduced the incidence of perinatal transmission of HIV. Pregnancy is not known to accelerate HIV disease in the mother. The most important nursing action is to engage the mother in prenatal care, to educate her about the benefits of medication for HIV during pregnancy and to prepare her for a cesarean birth.

Sublingual medications avoid the first-pass effect. True or False

True Medications given sublingually and rectally bypass metabolism by gastrointestinal and hepatic enzymes. When a medication is given orally, the amount of available medication is reduced before it reaches the general circulation due to the first-pass effect.

Electroconvulsive therapy (ECT) is used to treat a severe form of this disorder

Severe depression ECT can be used to as a treatment for severe depression when medication does not ease the symptoms of clinical depression. ECT is not a cure for depression. ECT can also be sued to treat patients with symptoms of delusions, hallucinations or suicidal thoughts.

The geriatric social worker is working with the nurse to assess the client's ability to perform instrumental activities of daily living (IADL). Which of the following skills are considered instrumental activities of daily living? (Select all that apply.) Ability to take medications Ability to cook meals Ability to eat independently/feed self Ability to bathe self Ability to write checks

Ability to take medications Ability to cook meals Ability to write checks Activities of daily living (ADLs) are basic self-care tasks, such as feeding, toileting, grooming, bathing, putting on clothes. Instrumental activities of daily living (IADLs) are slightly more complex skills and include a series of life functions necessary for living independently, such as the ability to use a telephone, shopping, doing housework, preparing meals, handling finances, and being responsible to take medications. ADLs and IADLs are part of an older adult's functional assessment.

During a female patient's breast exam, you see a cluster of very tiny dimples near one nipple. Abnormal or expected findings

Abnormal There should be no dimples, in fact "orange peel" skin is a late sign of breast cancer

A patient can tell you her name, but does not know the day of the week week. Abnormal or expected findings

Abnormal Normal mental function includes orientation to person, place, and time.

Auscultation reveals bowel sounds in 2 of the 4 abdominal quadrants. Abnormal or expected findings

Abnormal findings Normally, you should hear bowel sounds in all 4 quadrants in a healthy patient.

The nurse is reviewing various group activities with the health care team. When planning a therapeutic milieu, what is the most important factor when selecting a group activity? Match them to the clients' preferences Provide consistency with clients' skills Build the skills of group participation Achieve clients' therapeutic goals

Achieve clients' therapeutic goals Activity groups are used to enhance the therapeutic milieu and to meet the clinical and social needs of clients, eg., to minimize withdrawal and regression and to develop self-care skills.

CD4 count less than 200 Kaposi's sarcoma pneumocystosis

Acquired Immune Deficiency Syndrome (AIDS) When the CD4 cell (a type of lymphocyte) count falls below 200 cells/microliter, clients are at risk for developing opportunistic infections; antiretroviral treatment should be started or changed.

The client is diagnosed with depression. Which therapeutic communication skill is most likely to encourage the client to express feelings? Projective identification Direct confrontation Active listening Reality orientation

Active listening Use of therapeutic communication skills such as silence and active listening encourages verbalization of feelings. Reality orientation is used with clients who may have cognitive impairment. Direct confrontation is usually not used except in cases where a risk of physical harm to the client or others is anticipated. Projective identification is used to project the bad object into (not onto) another person so it becomes a part of that person. The person then identifies with that other person, and hence has means for control.

Indicate the location of pain that supports the diagnosis of suspected pancreatitis.

Acute pancreatitis produces sharp pain in the epigastric area because of the anatomical position of the pancreas behind the stomach. It is not unusual for a client to report "my stomach hurts" yet be holding the pelvic or periumbilical region. The client may even report upper back pain. Nurses need to clearly assess correct locations regardless of how the client identifies it verbally.

dehydration fatigue and muscle weakness hyper pigmentation of skin unintentional weight loss

Addison's disease Addison's disease is a disorder that occurs when the adrenal glands do not produce enough hormones (glucocorticoid hormones, mineralcorticoid hormones and sex hormones).

A client with chronic kidney disease (CKD) is scheduled for hemodialysis at 9 am. It is now 6:30 am and the client is eating breakfast. How should the nurse help the client to prepare for hemodialysis? (Select all that apply.) Hold all oral medications Administer prescribed vitamin D Administer prescribed phosphate binder Weigh the client Assess patency of the access site Ensure the client eats a high fiber, high protein breakfast

Administer prescribed vitamin D Administer prescribed phosphate binder Weigh the client Assess patency of the access site The nurse should administer a phosphate binder, such as sevelamer, with breakfast. Vitamin D may be prescribed with the phosphate binder to help control both serum calcium and phosphate levels. Some medications will be withheld; dialyzable meds and meds that lower blood pressure are held until after the procedure. The client should eat an easily digestible meal at least 2 hours before treatment begins, avoiding foods high in fiber or protein. The nurse should assess the patency of the access site (for presence of bruit, palpable thrill, distal pulses, and circulation), weigh the client, and measure vital signs.

The parents of a 2 year-old child report that the child has been holding the breath during temper tantrums. What is an appropriate approach for the response by a nurse? Recommend that the parents give in when the child holds the breath to prevent anoxia Advise the parents to ignore breath holding because breathing will begin as a reflex Instruct the parents on how to reason with the child about possible harmful effects Teach the parents how to perform cardiopulmonary resuscitation

Advise the parents to ignore breath holding because breathing will begin as a reflex If temper tantrums are accompanied by breath holding, the parents need to know that this behavior will not result in harm to the child. Ignoring the breath holding is the best response to this benign behavior.

The client with lymphoma is admitted for treatment of tumor lysis syndrome. Which medication will protect the kidneys by minimizing uric acid deposits? Mannitol (Osmitrol) Hydralazine Allopurinol (Aloprim, Zyloprim) Furosemide (Lasix)

Allopurinol (Aloprim, Zyloprim) Tumor lysis syndrome (TLS) is a complication of cancer treatment and is a medical emergency. When large number of neoplastic cells are killed rapidly, they release their intracellular contents, including uric acid, into the blood stream. TLS results in metabolic disturbances, including hyperurecemia. Of those listed below, only allopurinol (Aloprim) reduces uric acid production; this medication is commonly used to treat gout.

What is the drug classification for: gentamicin

Aminoglycosides are bactericidal; they primarily act by inhibiting protein synthesis in bacteria and compromising the structure of the bacterial cell wall. They are used in the treatment and prophylaxis of severe infections, such as septicemia, severe urinary tract infections, and hospital-acquired respiratory infections, caused by aerobic, gram-negative bacteria, e.g., as Escherichia coli and Klebsiella species.

The nurse works in the pediatric emergency department. In which situation would a child be treated by using enemas followed by an antitoxin? A school-aged child who has swallowed a handful of iron-fortified vitamins A toddler who has eaten an undetermined number of ibuprofen tablets A preschooler who bit into a laundry detergent pod An infant who is diagnosed with botulism

An infant who is diagnosed with botulism Food-borne botulism can be treated by removing whatever contaminated food is in the stomach by using enemas (or by inducing vomiting) and administering a Botulinum antitoxin. Children with iron poisoning and who are breathing normally can be given a strong laxative fluid; severe poisonings require IV chelation therapy. For NSAID poisoning, sometimes activated charcoal is given (usually within 1 hour of ingestion); massive overdoses may require orogastric lavage because there is no specific antidote for ibuprofen. Since laundry detergent is an alkaline substance, the most commonly used therapy is dilution/irrigation/wash, especially for burns to the skin and eyes. Tracheal intubation with ventilation may be required if the child swallowed the laundry detergent.

What is the drug classification for: losartan

Angiotensin Receptor Blockers block the action of angiotensin II by preventing angiotensin II from binding to angiotensin II receptors on blood vessels. As a result, blood vessels enlarge (dilate) and blood pressure is reduced. They are primarily used to control high blood pressure and treat heart failure. In addition, they slow the progression of kidney disease due to high blood pressure or diabetes.

What is the drug classification for: montelukast

Antiasthmatics either relax the smooth muscles that line the airway (bronchodilators), block the inflammation that narrows the airways (corticosteroids), counteract substances that cause the air passages to constrict and secrete mucus (leukotriene modifiers), or prevent allergic reactions or asthma symptoms. Montelukastis aleukotriene antagonist; it can also be categorized as a Bronchodilator. They are used in the management of acute and chronic episodes of reversible bronchoconstriction associated with asthma. The goal of therapy is to treat acute attacks (short-term control) and to decrease incidence and intensity of future attacks (long-term control).

What is the drug classification for: enoxaparin

Anticoagulants work by inhibiting clotting factor synthesis, inhibiting thrombin, or by interfering with blood platelet formation. Enoxaparin is classified as a low-molecular-weight heparin (LMWH). They are used to prevent or treat blood clots associated with stroke, heart attack, heart valve disease, coronary artery disease, heart failure, arrhythmia, atrial fibrillation, deep vein thrombosis, and pulmonary embolism.

What is the drug classification for: loperamide

Antidiarrheals work in a variety of ways. Some slow the passage of stools through the intestines (like loperamide). Others decrease the secretion of fluid into the intestine and inhibit the activity of bacteria (bismuth subsalicylate). They are used to control and to provide symptomatic relief of acute and chronic nonspecific diarrhea.

What is the drug classification for: clotrimazole

Antifungal agents are also called antimycotic agents; they kill or inactivate fungi. They are used to treat systemic, localized, or topical fungal infections (including yeast infections).

What is the drug classification for: fexofenadine

Antihistamines compete with histamine for histamine receptor sites and when they occupy the histamine receptor sites, they prevent histamine from causing allergic symptoms. They are used for relief of symptoms associated with allergies (including rhinitis, urticaria and angioedema) and as adjunctive therapy in anaphylactic reactions. Some are used to treat insomnia (diphenhydramine), motion sickness (dimenhydrinate and meclizine), Parkinson-like reactions (diphenhydramine), and other nonallergic conditions.

What is the drug classification for: methotrexate

Antineoplastics inhibit or prevent the development, maturation or spread of neoplastic cells by various different mechanisms of action. Many damage the DNA of cancer cells; others interfere with the cancer cell's metabolism or affect cell division (methotrexate is classified as an antimetabolite); still others create an unfavorable environment for cancer cell growth (hormones). Methotrexate is also listed in the drug category: Antirheumatic. They are used in the treatment of various solid tumors, lymphomas, and leukemias. They may also be used in some autoimmune disorders (such as rheumatoid arthritis).

What is the drug classification for: clopidogrel

Antiplatelet Agents block the formation of blood clots by preventing the clumping of platelets. They are used to treat and prevent thromboembolic events, e.g., stroke, myocardial infarction, peripheral vascular disease. They are used after stent, artificial heart values, and other devices that are placed inside the heart or blood vessels.

What is the drug classification for: risperidone

Antipsychotics work by blocking a specific subtype of the dopamine receptor (the D2 receptor). The 2nd generation of these medications not only block D2 receptors but also a specific subtype of serotonin receptor (5HR2A receptor). Risperidone (Risperdal) is a 2nd generation antipsychotic. They are used in the treatment of acute and chronic psychosis, especially when accompanied by increased psychomotor activity. Off-label uses include Tourette's syndrome, substance abuse, stuttering, obsessive-compulsive disorder, post-traumatic stress disorder, depression, bipolar disorder and personality disorders.

What is the drug classification for: isoniazid (INH)

Antituberculars have various actions that affect mycobacteria, with most having bactericidal (for example, rifampin) and/or bacteriostatic (for example, isoniazid) actions. They used in the treatment and prevention of tuberculosis (TB).

What is the drug classification for: acyclovir

Antivirals are designed to work in one of two ways - they either inhibit the ability to multiply or they mimic the virus attachment protein, disrupting the replication process. They are commonly used in the management, prevention, and/or treatment of viral infections, such as HIV, herpes simplex and cytomegalovirus, pneumonia, measles and mumps, and influenza strains (including swine flu).

The client is being treated for tuberculosis (TB). Which assessment would indicate that the client is having a possible adverse response to isoniazid? Appearance of jaundice Decreased hearing Severe headache Tachycardia

Appearance of jaundice Clients who are being treated with isoniazid are at risk for developing drug-induced hepatitis. The appearance of jaundice may indicate an elevation of the client's serum bilirubin levels; liver enzymes (AST and ALT) will also be elevated. A small number of adults taking isoniazid develop severe hepatitis that may progress to liver failure and even death unless the drug is stopped immediately.

The charge nurse is planning assignments on a surgical unit. A client with which need could be assigned to an unlicensed assistive person (UAP)? Review dietary needs with client prior to transfer to long-term care facility Change post-op hip dressing after removal of a drainage tube Assist with meals and monitor ability to swallow following a mild stroke Apply compression stockings and ambulate in hall three times a day

Apply compression stockings and ambulate in hall three times a day UAP can be assigned routine tasks that have predictable outcomes. Many of the tasks a UAP can do involve activities of daily living (ADLs), such as personal hygiene (shaving, bathing, oral hygiene, hair care and toileting), assisting with dietary needs, and measuring vital signs. Sometimes a UAP may be allowed to change a dry, nonsterile dressing. Although UAP routinely assist clients with delivering and setting up food trays, UAP cannot assess a client's ability to swallow. Client teaching prior to discharge is a nursing responsibility.

A nurse is suctioning a tracheostomy tube in a client. The nurse should take what action in order to prevent unnecessary hypoxia during this procedure? Maintain sterile technique throughout the procedure Withdraw catheter in a circular motion with intermittent suction Apply suction for no more than 10 seconds Lubricate three to four inches of the catheter tip

Apply suction for no more than 10 seconds Although all responses are correct and important steps of the suctioning process, hypoxia can result from applying suction for more than 10 sections. The nurse should be sure to apply oxygen immediately before and after suctioning and allow the client to rest a bit if more suctioning is indicated.

The nurse is caring for an 81 year-old client with colorectal cancer. Until now, the client's pain has been managed using acetaminophen with codeine. Because of increased pain, intravenous morphine has now been ordered. What should the nurse recognize about the validity of this order? Inappropriate because of potential respiratory depression Inappropriate and demonstrates poor knowledge of pain control Appropriate despite the expected effect of mental confusion Appropriate pain management around-the-clock

Appropriate pain management around-the-clock Older adults with cancer pain are frequently undermedicated. Pain management with IV morphine is appropriate and should be offered throughout the day and night.

A nurse is caring for a trauma victim who has experienced a significant blood loss. Immediately following multiple transfusions, what is the most accurate indicator of oxygenation? Pulse oximetry Hemoglobin and hematocrit (H and H) Complete blood count (CBC) Arterial blood gases (ABGs)

Arterial blood gases (ABGs) Arterial blood gases (ABGs) are the most accurate measure of oxygenation at this time. An ABG measures PaO2, PaCO2, pH, HCO3 and oxygen saturation. Pulse oximetry would not be as accurate during and after blood replacement therapy because it is a peripheral test; when a person is in shock, the peripheral extremities are typically vasoconstricted. A CBC examines all components of blood, including hemoglobin and hematocrit.

A 35-year-old client of Puerto Rican-American descent is diagnosed with ovarian cancer. The client states, "I refuse both radiation and chemotherapy because they are 'hot.'" Which action should the nurse take next? Ask the client to talk about concerns regarding "hot" treatments Report the situation to the health care provider Document the situation and client response in the notes Talk with the client's family about the situation

Ask the client to talk about concerns regarding "hot" treatments In Hispanic folk medicine, it is believed that disease is caused by an imbalance between hot and cold principles. Health is maintained by avoiding exposure to extreme temperatures and by consuming appropriate foods and beverages. Examples of "hot" diseases or states include pregnancy, hypertension, diabetes and indigestion. "Cold" diseases include pneumonia. These designations are symbolic and do not necessarily indicate temperature or spiciness. Care and treatment regimens can often be negotiated with clients within this framework. Also note that the correct response is the best answer because it is client-centered.

The nurse is planning care for a 3 month-old infant in the immediately postoperative period after the placement of a ventriculoperitoneal shunt for hydrocephalus. The nurse should take which action with anticipation of complications of the procedure? Pump the shunt at intervals to assess for proper function Maintain the infant in an upright position in a car seat Begin formula feedings when infant is alert Assess for abdominal distention or taunt abdominal wall

Assess for abdominal distention or taunt abdominal wall The nurse should observe for abdominal distention or a taunt abdominal wall because cerebrospinal fluid could cause peritonitis or a postoperative ileus as a complication of distal catheter placement. The child may be in a car seat afterwards. However, it does not answer the question being asked about potential complications. The infant would be started on clear liquids initially, not formula. The shunt will not be pumped.

A 5 month-old is hospitalized with a diagnosis of bronchiolitis related to respiratory syncytial virus (RSV). The parent reports the baby has been sneezing and wheezing, has had a runny nose for two days, and has not eaten for more than nine hours. Vital signs are: temperature 100.2 F (38 C), pulse 102, respiratory rate 32. Place the nurse's actions in order of priority by dragging and dropping the options below. a. Administer prescribed medications b. Provide family teaching c. Assess for respiratory distress d. Promote adequate tissue oxygenation e. Promote desired fluid intake f. Institute droplet isolation precautions

Assess for respiratory distress Promote adequate tissue oxygenation Institute droplet isolation precautions Administer prescribed medications Promote desired fluid intake Provide family teaching When caring for a client with bronchiolitis related to RSV, the primary nursing interventions are aimed at promoting oxygenation. Remember the ABCs. Assessing respiratory distress and promoting adequate oxygenation address basic physiologic needs. Instituting droplet precautions is required to prevent transmission of the disease to others. Administering prescribed medications will alleviate symptoms such as wheezing and will promote healing. It is important to maintain fluid and electrolyte balance by promoting fluid intake. Even though the infant has not eaten for a long time, this is less important than the respiratory status. Finally, the nurse will need to provide support to the parents, who may have a lot of anxiety and concern about their child. They will need to be taught about their infant's diagnosis and treatments so that they can care for the child at home. Involving them in the care while the child is hospitalized will facilitate this learning process while alleviating anxiety.

The nurse is preparing to administer medications to a client diagnosed with heart failure and atrial fibrillation. Which intervention is most important for the nurse to implement before administering digoxin? Monitor oxygen saturation on room air Monitor intake and output ratios and daily weight Assess for peripheral edema and auscultate the lungs for crackles Assess the apical pulse, counting for a full minute

Assess the apical pulse, counting for a full minute The nurse should monitor the client's pulse before the administration of digoxin; bradycardia is a sign of digoxin toxicity. The correct technique for taking an apical pulse is to use the stethoscope and listen for a full 60 seconds. Digoxin is held for a pulse below 60 beats per minute. The other assessments are correct assessments in the client with heart failure, but are not as great a priority and are not related to the use of digoxin.

Postoperative orders for a client who had a mitral valve replacement include monitoring pulmonary artery pressure together with pulmonary capillary wedge pressure with a pulmonary artery catheter. What is the purpose of these actions by a nurse? Determine the changes in an acid-base balance Establish coronary artery stability Assess the left ventricular end-diastolic pressure Compare the right ventricular pressures

Assess the left ventricular end-diastolic pressure The pulmonary capillary wedge pressure is reflective of left ventricular end-diastolic pressure. Pulmonary artery pressures are an assessment tool used to determine the ability of the heart to receive and pump blood effectively.

A nurse is caring for a client in the coronary care unit. The display on the cardiac monitor indicates ventricular fibrillation. What should the nurse do first? Initiate CPR Assess the pulse Perform defibrillation Assess the level of consciousness

Assess the level of consciousness Artifact (interference) can mimic ventricular fibrillation on a cardiac monitor. Always treat the patient, not the monitor. If the client is truly in ventricular fibrillation the client will be unresponsive and no pulse will be present. The standard of care is to verify the monitor display with an assessment of the client's level of consciousness, shaking and shouting to arouse followed by a carotid pulse check. If the client is unresponsive without a pulse in ventricular fibrillation, the most effective treatment will be electricity or defibrillation. This should be the priority, supplementing circulation using chest compressions until the defibrillator is set up and ready to deliver the shock.

A 28 year-old is transferred to the emergency department (ED) via ambulance with a traumatic head injury. The client is awake and reports having a headache and some amnesia. What are the priority nursing interventions for this client? (Select all that apply.) Assess the wound for presence of drainage or bruising on the head Assess vital signs and neurological function Assess the airway Prepare for CT imaging of the head Position this client in high Fowler's position

Assess the wound for presence of drainage or bruising on the head Assess vital signs and neurological function Assess the airway Prepare for CT imaging of the head Remember primary emergency trauma assessment using "A, B, C, D and E". The ED nurse will assess airway, breathing, circulation, and disability/neurological function on a person who has experienced a traumatic head injury. The nurse will also examine the client for the presence of any bruising or drainage, particularly of the ears and nose. A supine position is best; the head of the bed may be elevated slightly if not contraindicated. A CT scan is required if the client presents with an abnormal mental status, clinical signs of skull fracture, history of vomiting, or headache.

An elderly client is admitted to a home care agency following hospitalization for exacerbation of heart failure. The client lives alone, has difficulty completing activities of daily living (ADLs), and is unable to drive. List the order of the steps in the case management process by dragging and dropping the options below. a. Reassessment of health status and ADL ability b. Assessment of biophysical and sociocultural considerations c. Evaluation of progress towards client's goals d. Referral to personal care attendant and transportation services e. Identification of nursing diagnoses

Assessment of biophysical and sociocultural considerations Identification of nursing diagnoses Referral to personal care attendant and transportation services Reassessment of health status and ADL ability Evaluation of progress towards client's goals Case management is a collaborative process that assesses, plans, implements, coordinates, monitors, and evaluates options and services to meet an individual's health needs.

The charge nurse is making assignments on the day shift for a registered nurse (RN), a licensed practical nurse (LPN) and a certified nursing assistant (CNA). Which assignments are the most appropriate for a client who fell during the night and now has a skin tear on his arm and a hematoma on his hip, and is scheduled for an x-ray of his hip? (Select all that apply.) Assign medication administration to the LPN Assign wound care to the RN Assign complete care to the LPN Assign the CNA to assist with personal hygiene tasks Assign the LPN to report confusion or headache

Assign medication administration to the LPN Assign wound care to the RN Assign the CNA to assist with personal hygiene tasks Assign the LPN to report confusion or headache The RN can assign clients to LPNs as long as the care of the client is not too complex and there is a low likelihood of an emergency. Since this client fell during the night, the RN should not assign complete care to the LPN. But the LPN could administer medications to this client and should report observations and assessment data to the RN. The CNA can assist the client with personal care activities.

A 60 year-old male client underwent inguinal hernia repair in a day-surgery clinic. He is awake and alert, but has not been able to void since he returned from surgery six hours ago. He has received 1000 mL of IV fluid. Which action would most likely help him to void? Perform Credé's method on the bladder from the bottom to the top Wait two hours and have the client attempt to void again Assist the client to stand by the side of the bed to void Have him drink several glasses of water

Assist the client to stand by the side of the bed to void When a male client is not able to use a urinal in the bed, the client should stand by the side of the bed to attempt voiding. This is the most natural physiological position of normal voiding for male clients. Also, given the client's age, the client most likely has some degree of prostate enlargement which may interfere with ease of voiding.

The charge nurse on the night shift at an urgent care center has to admit clients of a higher acuity than usual because of a large fire in the nearby area. Which style of leadership would be best under these circumstances? Assume an autocratic, decision-making role Apply an integrative leadership approach Engage in collaborative practice Adopt a transformational, nondirective approach

Assume an autocratic, decision-making role Leadership style may need to change depending on the situation. In an emergency or crisis situation, there's not much time for input from staff, which is why an autocratic or authoritarian leadership style is best for decision making in these situations. Integrative leadership is an approach that fosters collaboration with the major sectors of society (business, government, nonprofits); transformational leadership is where a leader works with subordinates to motivate them to make agreed-upon changes.

The nurse enters the client's room and finds the client, who was previously alert, lethargic and slow to respond. Prioritize the nursing actions by dragging and dropping the options below. a. Complete a quick neurological assessment: orientation, pupil response, ability to follow commands b. Call the rapid response team and report the client's situation; request immediate assistance c. Attempt to elicit a response by physically shaking the client and loudly stating "open your eyes and talk to me." d. Remain with the client; send another staff member to get the list of medications and the chart

Attempt to elicit a response by physically shaking the client and loudly stating "open your eyes and talk to me." Complete a quick neurological assessment: orientation, pupil response, ability to follow commands Call the rapid response team and report the client's situation; request immediate assistance Remain with the client; send another staff member to get the list of medications and the chart When a client exhibits an acute change in condition, such as a reduced level of consciousness, it is an emergency situation and requires immediate intervention. First, the client's status must be determined through quick focused assessment. Most facilities have a rapid response team to assist the staff nurse so this asset should be mobilized next. Because the client should not be left alone, the nurse should call for help and ask a colleague to bring the list of medications along with the client's chart before the rapid response team arrives.

Which finding should the nurse identify as the most characteristic of an acute episode of reactive airway disease? Audible expiratory wheezing Frequent dry coughing Auditory gurgling Inspiratory laryngeal stridor

Audible expiratory wheezing In an acute episode of reactive airway disease, breathing is likely to be characterized by wheezing on expiration. This sound is made as air is forced through the narrowed passages and often can be heard by the naked ear without a stethoscope. Remember, reactive airway disease is a general term that does not indicate a specific diagnosis - it simply means there is wheezing due to airway reactivity to a trigger. The trigger can be an allergy, virus, cold air, humidity, exercise or any number of other respiratory triggers.

The child with this disorder has difficulties with social interaction and verbal and nonverbal communication and also exhibits repetitive behaviors.

Autism spectrum disorder A child with autism spectrum disorder (ASD) has difficulty with social interactions and verbal and nonverbal communication; the child also exhibits repetitive behaviors. ASD is considered a neurodevelopmental disorder.

The nurse is caring for a newly admitted client with a diagnosis of hyperosmolar hyperglycemic nonketotic state (HHNS). Which interventions would the nurse expect the health care provider to order? (Select all that apply.) a. BUN and creatinine levels b. Check blood glucose levels every four hours c. Rapid infusion of intravenous fluids d. NPH insulin as IV bolus and then titrated by weight

BUN and creatinine levels Immediate labs would include arterial blood gases, complete blood count with differential, urinalysis, plasma glucose, BUN, electrolytes, chemistry profile and creatinine levels. The client will be very dehydrated in this hyperosmolar hyperglycemic state; therefore, rapid infusion of intravenous fluid will be nded to correct the hypovolemia. IV infusion of regular insulin may be used to rapidly correct the hyperglycemia, and glucose checks should be performed hourly until blood sugar levels have reached a more normal level. At that point, a long-acting insulin, such as glargine (Lantus), can be started to provide consistent baseline insulin coverage, supplemented with rapid-acting insulin dosed according to a sliding scale based on blood glucose readings.

Hypotension and tachycardia, with muffled heart sounds and jugular vein distention. Due to: Atrial fibrillation Myocardial infarction Cardiac tamponade Occlusive arterial disease hemophilia Raynaud's phenomenon Heart Failure Thrombophlebitis DIC Sickle cell disease

Beck's triad (hypotension; jugular vein distention; and distant/muffled heart sounds) are the classic symptoms of cardiac tamponade. Cardiac tamponade is where blood or fluid accumulates in the pericardial space and acts to compress and constrict the heart.

A nurse is working with a client in an extended care facility. Which bed position is preferred for a client, who is at risk for falls, as part of a prevention protocol? Bed in lowest position, wheels locked, bed placed against the wall Knees bent, head slightly elevated, bed in lowest position Four side rails up, wheels locked, bed closest to door Lower side rails up, upper wheels locked, bed facing doorway

Bed in lowest position, wheels locked, bed placed against the wall Restraints are anything that will limit the movement of a client and side rails should not be used as a substitute for protective restraints. If a client is at risk for getting out of bed unsupervised and/or is a fall risk, then other precautions must be made, such as using a bed that is low to the ground (sometimes pads are also placed on the floor) or placing the bed against the wall (but the side rail cannot be up on the side that's not against the wall.) Bed alert systems and/or arranging for a sitter are other alternatives.

difficulty starting a urine stream dribbling after urination urinary retention

Benign Prostatic Hypertrophy An enlarged prostate does not necessarily raise the risk of cancer, but men may develop urinary tract infections, hematuria, bladder stones, and bladder or kidney damage.

What is the drug classification for: midazolam

Benzodiazepinesde press the CNS, probably by potentiating GABA, which is an inhibitory neurotransmitter. Midazolamcan also be categorized as a Sedative/Hypnotic. These are all Schedule IV drugs. They are primarily used to produce sedation, induce sleep, relieve anxiety and muscle spasms, and to prevent seizures. Midazolam is used as an agent for sedation/anxiolysis/amnesia

What is the drug classification for: metoprolol

Beta Blockers block norepinephrine and epinephrine from binding to beta receptors on nerves. By blocking the effects of these neurotransmitters, they reduce heart rate and reduce blood pressure by dilating blood vessels. They are used to treat hypertension; heart failure; arrhythmias; angina (but not for immediate relief); glaucoma (ophthalmic); sometimes used in heart attack patients to prevent future heart attacks; also used prophylactically for migraine headaches.

What is the drug classification for: atenolol

Beta Blockers block norepinephrine and epinephrine from binding to beta receptors on nerves. By blocking the effects of these neurotransmitters, they reduce heart rate and reduce blood pressure by dilating blood vessels. The spelling of many beta blockers often end with "lol." They are used to treat hypertension; heart failure; arrhythmias; angina; glaucoma (ophthalmic); sometimes used in heart attack patients to prevent future heart attacks; also used prophylactically for migraine headaches.

The nurse is instructing a client with moderate persistent asthma on the proper method for using metered-dose inhalers (MDIs) and various types of medications. Which medication should the nurse advise the client to administer first? Beta agonist Steroid Anticholinergic Mast cell stabilizer

Beta agonist The beta-agonist drugs are bronchodilators that relieve bronchospasm by relaxing the smooth muscle of the airway. These drugs should be taken first so that other medications can more deeply and effectively penetrate the lungs.

A client with this disorder may experience drastic changes in mood accompanied by extreme changes in energy, activity, sleep and behavior.

Bipolar disorder Patients with bipolar disorder may experience mood swings ranging from mania to depression, with periods of normal mood and activity in between. Sometimes the mood swings can be unusually intense or extreme; at other times, they are less extreme.

The pregnant woman asks how a health care provider (HCP) can tell she is pregnant "just by looking inside." What is the best explanation for this? Bluish coloration of the cervix and vaginal walls Pronounced softening of the cervix Slight rotation of the uterus to the right Plug of very thick mucus

Bluish coloration of the cervix and vaginal walls Chadwick's sign is a bluish-purple coloration of the cervix and vaginal walls. It develops after the 6 to 8 weeks and is caused by increased blood supply to the area. Other early signs of pregnancy include Hegar's sign (a softening of the cervical isthmus) and Goodwell's sign (a softening of the cervix), but the HCP would need to compress the tissue to assess these findings. The HCP would not see the mucus plug; the mucus plug dislodges, breaks up and passes out of the body just prior to labor.

What is the drug classification for: risedronate

Bone Resorption Inhibitors bind to hydroxyapatite in bone and inhibit bone resorption by decreasing the number and activity of osteoclasts. They are primarily used in the prevention and treatment osteoporosis in postmenopausal women; they are also used to treat osteoporosis due to other causes, e.g., Paget's disease of the bone and corticosteroid therapy.

What is the drug classification for: albuterol

Bronchodilators relax bronchial smooth muscle. Relaxing these muscles makes the airways larger and allows air to pass through the lungs. Some also increase mucociliary clearance (beta agonists). Albuterol (Proventil) is a short-acting (or rescue) medication and can also be categorized as an Antiasthmatic. Short-acting medications are used as needed as asthma "rescue" medications. Long-acting medications are used every day to control asthma in conjunction with an inhaled steroid.

The client is seen in the emergency one day after falling in his bathroom at home. The client reports having "a few drinks" prior to the fall. Which finding requires the nurse's immediate attention? Bruise behind one ear Blurred vision Nausea and vomiting Headache

Bruise behind one ear Bruising behind one ear (over the mastoid process) requires the nurse's immediate attention. Known as "Battle's sign", this injury is seen a day or so following a basilar skull fracture. A CT scan of the brain will confirm a skull fracture. The client may report loss of hearing, smell or vision and he may have blood leaking from the ear. The vomiting and headache could be due to his alcohol intake, as well as the skull fracture.

The nurse is assessing a client who has been treated long-term with glucocorticoid therapy. Which finding might the nurse expect? Jaundice Peripheral edema Buffalo hump Increased muscle mass

Buffalo hump The most common side effects of glucocorticoid therapy include increased appetite (and weight gain), increased blood sugar, change in body shape (increase in fatty tissue on the trunk with thinner legs and arms), acne, thinning of the skin and easy bruising. The client may also have a hump behind the shoulders; the hump is an accumulation of fat on the back of neck.

Russell's sign can be observed with this disorder

Bulimia nervosa A person who repeatedly self-induces vomiting will have scraped or raw areas on the knuckles. Bulimia nervosa is a type of eating disorder that involves binging (eating large amounts of food) and purging (vomiting).

The patient with a sigmoid colectomy will have semi-liquid stool collect in a colostomy bag. True or false

False A colectomy is the primary treatment for colon cancer. The cancerous part of the large intestine is removed and remaining bowel is joined together (anastomosis). The patient will not need a colostomy. Bowel managements may be more frequent after a colectomy.

The health care provider orders the antidepressant trazodone ER 150 mg at bedtime. Which common side effect of this drug should the client understand? Reduces arthritic pain Relieves nasal stuffiness Causes drowsiness Decreases acne breakouts

Causes drowsiness This medication is chemically unrelated to the SSRIs, TCAs or MAO inhibitors, even though it inhibits the uptake of serotonin by nerves in the brain. The sedative effects of this antidepressant is why this medication is also successfully used to treat insomnia. People with insomnia may sleep better immediately, but it may take a week or two before maximum antidepressant effects are noticed. Other common side effects of trazodone include dry mouth, stuffy nose, constipation or change in sexual interest/ability.

A client with a history of asthma and kidney stones is admitted with a diagnosis of recurrent renal calculi. The client experiences shortness of breath following a lithotripsy. The nurse auscultates the client's lungs and finds decreased air movement but no wheezing. The arterial blood gas (ABG) results are pH 7.31, PaO2 53 mm Hg, PaCO2 50 mm Hg, and O2 sat 82%. Which of the following actions are appropriate for the nurse to take? (Select all that apply.) Call respiratory therapy Administer a short-acting bronchodilator via nebulizer Start high flow oxygen via face mask Start oxygen via nasal cannula Increase IV fluids Contact the health care provider Prepare for possible intubation

Call respiratory therapy Administer a short-acting bronchodilator via nebulizer Start high flow oxygen via face mask Contact the health care provider Prepare for possible intubation This client needs emergency treatment to open the airways and improve gas exchange. The absence of lung sounds without wheezing indicates a severe narrowing of the airways in asthma with minimal air movement. Emergent intervention to open the closed airway including possible intubation are indicated. The high PaCO and low pH indicate respiratory acidosis due to inadequate gas exchange. The low oxygen saturation and PaO2 indicate severe hypoxemia requiring high flow oxygen via mask.

The nurse is having difficulty reading the health care provider's written order that was written just before the shift change. What action should the nurse take? Leave the order for the oncoming staff to follow up on Ask the pharmacy for assistance in the interpretation Call the provider for clarification of the order Contact the charge nurse for an interpretationswertytrewiuytrewesdfrdeseed

Call the provider for clarification of the order Relying on another's interpretation is very risky. When in doubt, check with the health care provider who wrote the difficult-to-read order. Order entry systems help to minimize these types of problems.

A 36 year-old female client has a hemoglobin level of 14 g/dL and a hematocrit of 42%, 24 hours after a dilation and curettage (D&C). Which of the following findings should the nurse expect when assessing the client? Complaints of fatigue with ambulation Capillary refill of less than three seconds Respirations 36 breaths per minute Pale mucous membranes

Capillary refill of less than three seconds Because the hemoglobin and hematocrit are within the normal limits for an adult female, any additional assessments should also be normal. This capillary refill time is normal. The other options could be findings of anemia.

What is the drug classification for: cephalexin

Cephalosporins belong to a group of broad spectrum, semi-synthetic beta-lactam antibiotics derived from the mold Cephalosporium. The mechanism of action is the same as penicillins (they interfere with bacterial cell wall synthesis). An example of a 1st generation Cephalosporin is cephalexin. They are mainly used in the treatment and prophylaxis of a wide variety of bacterial infections, such as respiratory tract infections, skin and soft-tissue infections, and urinary tract infections.

Upon entering an adult client's room, the client is found to be unresponsive. After calling for help, what is the next action that should be taken by the nurse? Give two rescue breaths Deliver five abdominal thrusts Maintain an open airway Check for a carotid pulse

Check for a carotid pulse According to the American Heart Association's basic life support, the first step after determining a victim is unresponsive is to call for help. The next step is to check for a pulse (for no more than 10 seconds). If there is no pulse, the rescuer should begin CPR (30 chest compressions followed by 2 ventilations).

The registered nurse (RN) needs to make assignment for the unlicensed assistive persons (UAP). Which activity should the RN ask the UAP to perform? Adjust the rate of a gastric tube feeding Check the blood pressure of a two-hour postoperative client Ask a client receiving chemotherapy about pain Record a history on a newly admitted client

Check the blood pressure of a two-hour postoperative client UAPs must be assigned tasks that are routine, have expected outcomes and require no nursing judgment or decision-making situations. Vital signs on stable clients are commonly assigned to unlicensed staff.

Malabsorption syndrome and Wernicke-Korsakoff syndrome are associated with this disorder.

Chronic Alcoholism Nutritional deficiencies are common among clients who suffer from chronic alcohol abuse and are related to malabsorption of fat, nitrogen, sodium, water, thiamine, folic acid and vitamin B12. Wernicke-Korsakoff syndrome (also called Wernicke encephalopathy) is caused by a lack of thiamine (vitamin B1).

Pain localizes to the right upper quadrant, but may radiate to the right shoulder or scapula. Cirrhosis GERD Cholecystitis Crohns Diverticulitis Hepatitis Ulcerative colitis Pancreatitis Intestinal obstruction Peptic Ulcer

Cholecystitis is inflammation of the gallbladder caused by an obstruction of the cystic duct (usually by stones). Clients may experience nausea, vomiting and fever. Risk factors include increasing age, obesity or rapid weight loss, medication and pregnancy.

A nurse is reviewing with a client how to collect a clean catch urine specimen. What is the appropriate sequence to teach the client? Clean the meatus, then urinate into the container Clean the meatus, begin voiding, then catch urine stream Void a little, clean the meatus, then collect specimen Void continuously and catch some of the urine midstream

Clean the meatus, begin voiding, then catch urine stream A clean catch urine is difficult to obtain and requires clear directions. Instructing the client to carefully clean the meatus, then void naturally with a steady stream prevents surface bacteria from contaminating the urine specimen. As starting and stopping flow can be difficult, once the client begins voiding it's best to just slip the container into the stream. Other responses do not reflect correct techniques.

The nurse is caring for a client who had a closed reduction of a fractured right wrist followed by the application of a fiberglass cast 12 hours ago. Which finding requires the nurse's immediate attention? Client reports burning and tingling in the right hand and arm Slight swelling of fingers of right hand Skin warm to touch and normally colored Capillary refill of fingers on right hand is about three seconds

Client reports burning and tingling in the right hand and arm Burning and tingling as well as intense pain out of proportion to the injury may be an indication of compartment syndrome, requiring immediate action by the nurse to prevent permanent muscle damage. The other findings are normal for a client in this situation.

Due to a recent outbreak in the community, the nurse is speaking to a group of parents and elementary school teachers about rheumatic fever. Which information is most important for the nurse to emphasize? Home schooling is preferred to classroom instruction Most play activities will be restricted indefinitely Clumsiness and behavior changes should be reported Children may remain strep carriers for years

Clumsiness and behavior changes should be reported Sydenham chorea is a major sign of acute rheumatic fever; it may be the only sign of rheumatic fever in some clients. Symptoms include jerky, uncontrollable, and purposeless movements that look like twitches (these disappear during sleep); loss of fine motor control (causing changes in handwriting); and loss of emotional control (as evidenced by inappropriate crying or laughing). Sydenham chorea usually clears up in a few months and no complications are expected.

The nurse is reviewing the medical records for a newborn and sees that the first APGAR score was an 8 and the second score was a 9. Which category of the APGAR test is most likely the reason for the improved score? Muscle tone Color Cry Heart rate

Color There are five categories of the APGAR test and each category is scored with a 0, 1, or 2, depending on the observed condition. If the body is pink and the extremities are blue (acrocyanosis), the infant scores 1 in the "skin color" category. This is the most common APGAR score deduction.

The school nurse suspects that a third-grade child might have attention deficit hyperactivity disorder (ADHD). Prior to referring the child for further evaluation, what should the nurse do? Consult with the teacher about how to assist with impulse control Compile a history of behavior patterns and developmental accomplishments Observe the child's behavior on at least two occasions Compare the child's behavior with classic signs and symptoms of ADHD

Compile a history of behavior patterns and developmental accomplishments A complete behavioral and developmental history will play an important part in helping to diagnose this disorder.

Decentralized scheduling is used on a nursing unit. What is the advantage of this management strategy? Conserves time spent on planning Considers client and staff needs Frees the nurse manager to handle other priorities Allows requests for special privileges

Considers client and staff needs Decentralized staffing takes into consideration specific client needs and staff abilities and interests. This means the staffing is decided on the lowest level which is at the unit level.

During the admission process, the staff nurse realizes that the information on the identification (ID) bracelet does not match the information on the client's admission face sheet. What action should the nurse take? Use a permanent marker to change the incorrect information on the ID bracelet Contact the admission department to create a new ID bracelet Write the corrected information on the whiteboard in the client's room Communicate with staff that the two-identifiers requirement must be verified using the admission face sheet

Contact the admission department to create a new ID bracelet The admissions office has the responsibility to verify the client's identity, apply the correct bracelet and keep all the records in the system consistent. While the Joint Commission does not require the use of arm bands, correct identifying information must be attached to the client at all times. The other options are unsafe practices.

A mother wants to switch her 9 month-old infant from an iron-fortified formula to whole milk. Upon further assessment, a nurse finds that the baby eats table foods well, but drinks less milk than before. What information is the priority for the nurse discuss with the mother? q Offer fruit juice in a sippy cup occasionally Continue introducing new food Change the baby to whole milk

Continue with the present infant formula Even though the baby is eating table food and may be drinking less formula that she was before, she still needs to get most of her nutrition from formula (or breast milk.) The recommended age for switching from formula to whole milk is 12 months. That's because babies can't digest cow's milk as completely or easily until they are 12 months and cow's milk doesn't have the right amounts of nutrients (drinking cow's milk may even cause iron-deficiency anemia.)

The nurse reviews an order to administer Rh (D) immune globulin to an Rh negative woman after the birth of her Rh positive newborn. Which assessment is a priority before the nurse gives the injection? Gravida and parity Coombs test results Previous RhoGAM history Newborn's blood type

Coombs test results Rh (D) immune globulin (RhoGAM®) is given only if antibody formation has not occurred; a negative indirect Coombs test confirms that antibodies have not been formed in the new mother. An Rh negative woman should receive RhoGAM® within 72 hours after birth. Rh negative women should also receive RhoGAM® at about 28 weeks of pregnancy, after a miscarriage, abortion, ectopic pregnancy or an amniocentesis. RhoGAM® prevents sensitization and Rh incompatibility. The effects of Rh immune globulin last about 12 weeks, which is why repeat administration is needed in future pregnancies.

What is the drug classification for: fluticasone

Corticosteroids mimic the effect of hormones produced naturally by the adrenal glands. When the dose exceeds the body's usual hormone levels, they will suppress inflammation, as well as the immune system; they are also used for their antineoplastic activity. Oral forms are used to treat inflammation and pain associated with arthritis and autoimmune diseases (such as lupus, Crohn's). Inhaled medications are used to treat asthma and allergies. Topical applications help heal skin conditions. The injected forms are used treat the pain and inflammation of arthritis, gout and other inflammatory diseases.

A low protein and low calorie diet is indicated for children with cystic fibrosis (CF). True False

Cystic fibrosis is caused by a defective gene. This gene causes the body to produce thick, sticky mucus, which builds up in the lungs and the pancreas. Children with CF need a diet high in protein, fat and calories. Many children with CF also take pancreatic enzymes (to help absorb fats and protein), and supplements for vitamins A, D, E and K.

The thirst center is located in the parathyroid center. True or false

False A dry mouth and dehydration will activate the thirst center, which is located in the hypothalamus. As a result, there will be a conscious desire to drink. There will also be a series of subconscious steps to correct the dehydration, including vasopressin secretion by the pituitary gland.

The nurse is applying silver sulfadiazine topical to severe burns on the arms and legs of an adult. Which side effect should the nurse monitor for? Decreased neutrophils Hardened eschar Skin discoloration Increased neutrophils

Decreased neutrophils Silver sulfadiazine (Silvadene) is a broad spectrum antimicrobial and is especially effective against pseudomonas. When applied to extensive areas, however, it may cause a transient neutropenia, a decrease in neutrophils, as well as renal function changes with sulfa crystal production and kernicterus.

The 17 year-old female is diagnosed with bulimia nervosa. Which lab result does the nurse anticipate? Increased serum glucose Increased sodium retention Decreased serum potassium Decreased blood urea nitrogen (BUN)

Decreased serum potassium Bulimia nervosa is an eating disorder characterized by a cycle of eating a very large amount of food (bingeing) and compensatory behaviors such as self-induced vomiting or laxative abuse. The recurring binge-and-purge cycles damage the digestive system; purging behaviors such as vomiting can lead to tooth decay. Common findings of bulimia include electrolyte imbalances, such as hypokalemia, hypocalcemia, hypochloremia, and hyponatremia; BUN is increased. Metabolic acidosis is expected with laxative abuse; metabolic alkalosis is due to vomiting.

The nurse is caring for a 15 year-old client with a lengthy confining illness. This client is most at risk for altered psychoemotional growth and development due to what issue? Lack of trust Insecurity Loss of control Dependence

Dependence The illness-client role fosters dependency. Adolescents may react to dependency with rejection, uncooperativeness or withdrawal.

A nurse is assigned to provide care in the pediatric unit. What must be the priority consideration for nurses when communicating with children? Developmental level Present environment Nonverbal cues Physical condition

Developmental level While each of the factors affect communication, nurses should recognize that developmental differences have implications for processing and understanding information. Consequently, a child's developmental level must be considered to select communication approaches.

The nurse is performing a physical assessment on a client with type 1 diabetes. Which finding calls for immediate nursing action? Painful hematoma on thigh Intense thirst and hunger Reduced lower leg sensation Diaphoresis and shakiness

Diaphoresis and shakiness Diaphoresis is a sign of hypoglycemia, which warrants immediate intervention to raise the blood glucose and prevent diabetic coma. Diabetic neuropathy develops over time; some people have no symptoms while others experience pain, tingling or numbness. Early symptoms of diabetes include extreme hunger, thirst and urination, but these are not as critical as the findings associated with hypoglycemia.

A client is admitted to the emergency department during an acute asthma attack. Which assessment finding would support this diagnosis? Loose, productive cough Fever and chills Diffuse auditory expiratory wheezing Forced expiratory volume (FEV1) 60% of the predicted value

Diffuse auditory expiratory wheezing In asthma, two situations are of concern. First, the airways are narrowed making it difficult to get air into the lungs, resulting in wheezing. An auditory wheeze is one that is heard with normal hearing of the ear without a stethoscope. This is an emergency situation. The second concern is thick, tenacious secretions. A forced expiratory volume (FEV1) is very concerning if it is 50% of predicted. Fever and chills are not consistent with asthma attacks.

The order is for ibuprofen oral drops 10 mg/kg of body weight. The client weighs 62 lbs. Motrin oral drops are supplied in bottles containing 40 mg/mL. How many milliliters will the nurse administer? (Report to the nearest whole number.) mL.

Dimensional analysis: X mL = 1 mL/40 mg X 10 mg/kg X 1 kg/2.2 lbs X 62 lbs = 620/88 = 7.05 or 7 mL Ratio : 62 lbs/x = 1 kg/2.2 lbs = 28.19 kg 10 mg/x = 1 mL/40 mg = 10/40 = 0.25 0.25 X 28.19 = 7.05 or 7 mL

During an initial home visit, a nurse is discussing with family members the care of their 86 year-old mother who is newly diagnosed with Alzheimer's disease. Which of these interventions would be most helpful at this time? List actions to improve the client's daily nutritional intake Leave a book about relaxation techniques Write out a daily exercise routine for them to assist the client to do Discuss communication strategies to try using with the client

Discuss communication strategies to try using with the client Alzheimer's disease is a progressive chronic illness. Communication between caregivers and clients can be some of the most challenging issues. At this initial visit a nurse can help the family to know when to use communication strategies to enhance their ability to relate to the client. By use of select verbal and nonverbal communication strategies, the family can best support the client's strengths and cope with any aberrant behavior. The other actions are more specific and should be addressed when the need arises.

A client with heart failure is newly referred to a home health care team. The nurse discovers that the client has not been following the prescribed diet. What should be the appropriate nursing action? Discuss diet with the client to learn the reasons for not following the diet Discharge the client from home health care because of noncompliance Notify the provider of the client's failure to follow prescribed diet Make a referral to Meals-on-Wheels for a weekly delivery of a proper meal

Discuss diet with the client to learn the reasons for not following the diet When new problems are identified during client care, it is important for the nurse to collect accurate assessment data. This means that the nurse should go to the client first. Before reporting information to a health care provider, the nurse should have a complete collection of information such as an understanding of the client's behavior and feelings as a basis for future teaching and intervention.

The nurse is assessing the newborn of a mother with diabetes. The nurse should understand that hypoglycemia is related to what pathophysiological process? Maternal insulin dependency Pancreatic insufficiency Disruption of fetal glucose supply Reduced glycogen reserves

Disruption of fetal glucose supply After delivery, high glucose levels, which crossed the placenta to the fetus, are suddenly stopped. The newborn continues to secrete insulin in anticipation of the glucose. When oral feedings begin, the newborn will adjust insulin production within a day or two.

The order is for 900 mg of nafcillin and the nurse has a powder in a vial labeled "Nafcillin 1 gram, dilute with 3.4 mL of sterile water to produce 1 gram in 4 mL." How many milliliters will the nurse administer? Report the answer to the nearest tenth mL

Don't get confused by the quantity of the diluent (3.4 mL). When you are calculating reconstituted medication, only the final concentration directs your answers (in this case, 1 g in 4 mL). Using dimensional analysis: 4 mL/1 gram x 1 gram/1000 mg x 900 mg/1 = 3600/1000 = 3.6 mL Using ratio proportion: First convert the 1 g to 1000 mg 1000 mg/4 mL = 900 mg/x mL x = 3.6 mL

A nurse is providing discharge teaching to a client who has a new diagnosis of renal calculi. Which point should be included as a dietary recommendation to prevent recurrence of this condition? Consume foods high in vitamin E Increase dietary vitamin C Eat calcium-rich foods several times per day Increase foods rich in animal proteins

Eat calcium-rich foods several times per day Diet is one of several key factors that can promote or inhibit kidney stone formation. Calcium oxalate stones are the most common type of stone. People who form calcium oxalate stones should include 1000-1200 mg of calcium in their diet every day, either through diet or supplements; calcium in the digestive tract binds to oxalate and keeps it from forming stones. Other dietary recommendations include restricting animal proteins, which contain purines and can increase the risk of uric acid and calcium stones. There is no evidence that increasing vitamins E or C reduces the formation of kidney stones.

A client continually repeats phrases that others have just said. The nurse should document this behavior as which term? Autistic Echopraxis Echolalia Catatonia

Echolalia Echolalia is repeating words or phrases heard before. Catatonic behavior is defined as that of extreme inactivity or activity that's disconnected from the environment or encounters with other people. Catatonic behaviors are associated with autism and schizophrenia. Autism is one of a group of serious developmental problems called autism spectrum disorders (ASD) that appear in early childhood — usually before age three. Although symptoms and severity vary, all autism disorders affect a child's ability to communicate and interact with others. Echopraxis is the the involuntary imitation of the actions of others.

recurrent weeping and oozing skin papule and vesicles that later become crusted and scaly

Eczema Eczema is a superficial dermatitis of unknown cause resulting in recurrent inflammation of the skin.

barrel chest chronic cough shortness of breath wheezing weight loss

Emphysema Emphysema is a chronic and progressive disease of the lungs that causes shortness of breath due to over-inflation of the alveoli

The nurse is preparing a client diagnosed with deep vein thrombosis (DVT) for a venous doppler evaluation. Which of these actions should be necessary to prepare the client for this test? Determine if the client has any allergies to the contrast material Ask client not to eat or drink anything after midnight Administer a sedating medication prior to the test Ensure the client is wearing a hospital gown prior to the test

Ensure the client is wearing a hospital gown prior to the test A venous doppler examination uses ultrasound to create a 2-dimensional picture of the veins in the legs. The purpose is to detect blood clots. This is a noninvasive test and does not require sedation; a venography would require injecting contrast material into a vein. The client may eat or drink prior to the test.

A nurse is caring for a 13 year-old after a spinal fusion to treat scoliosis. Which nursing intervention is appropriate in the immediate postoperative period? (Select all that apply.) Maintain bedrest with the head of the bed elevated at least 30 degrees Assist the client to stand and walk to the bathroom as needed Encourage passive leg and ankle exercises Position the client flat in bed and logroll every 2 to 4 hours Encourage use of patient-controlled analgesia Perform neurovascular checks every 8 hours

Encourage passive leg and ankle exercises Position the client flat in bed and logroll every 2 to 4 hours Encourage use of patient-controlled analgesia The client should remain flat in bed for at least 6 hours and turned from side to side every 2 to 4 hours. The day after surgery, the client can sit up in bed a few times; the client will get out of bed to sit in a chair on the second or third day after surgery. Clients should be encouraged to perform isometric exercises right after surgery. Neuro checks will be performed every 2 hours for the first 24 hours.

A client diagnosed with schizophrenia, talks animatedly and the nursing staff are unable to understand what the client is attempting to communicate. The client is observed mumbling to self and speaking to the radio. A desirable outcome for this client's care should be which of these behaviors? Engages in meaningful and understandable verbal communication Expresses feelings appropriately through verbal interactions Interprets accurately the events and behaviors of others Demonstrates improved social relationships within the unit

Engages in meaningful and understandable verbal communication An outcome should be related to the medical diagnosis and supporting data. The client's exhibited behaviors support a nursing diagnosis of impaired verbal communication deficit. No information is presented related to feelings or to thinking processes. Disorganized verbalizations are typically taken to indicate disorganized thinking.

Which is the appropriate injection site for an adult receiving enoxaparin?

Enoxaparin (Lovenox) is a low-density anticoagulant that can cause significant bruising if not administered correctly. The nurse needs to avoid the area 2 inches (5 cm) from the umbilicus. The manufacturer recommends gently pinching the skin on one of the anterolateral surfaces of the abdomen (think "love handles") and holding the skin fold throughout the injection. The injection site should not be rubbed after completion of the injection.

A client initially experiences a large local reaction with swelling of the entire leg after being stung by a bee. A concerned family member drives the client to the emergency department. The client is now having difficulty breathing and has swelling of the tongue. Which of the following medications should be administered first? Methylprednisolone (Solu-Medrol) IV Albuterol (Proventil) inhaler Diphenhydramine (Benadryl) subQ Epinephrine (Adrenaline) IV

Epinephrine (Adrenaline) IV Difficulty breathing and swelling of the face, eyes or tongue are severe and life-threatening allergic reactions to the allergen. Epinephrine, 0.3-0.5 mL of a 1:1000 solution may be administered IM but airway obstruction due to angioedema, respiratory compromise due to bronchospasm, or circulatory collapse (or combination of these 3 conditions) requires IV administration. The other medications are more appropriate for mild-to-moderate distress: antihistamines, such as diphenhydramine or hydroxyzine, or oral steroids can help reduce the severity of the itching and albuterol may be used for treatment of bronchospasm without obstruction.

A 14 month-old child ingested half a bottle of aspirin tablets. Which finding should the nurse expect to see in this child? Dyspnea Hypothermia Edema Epistaxis

Epistaxis A large dose of aspirin inhibits prothrombin formation and lowers platelet levels. With an overdose, clotting time is prolonged. Spontaneous bleeding often occurs from the nose or mucous membranes in the mouth.

Mitral valve prolapse related to mitral valve regurgitation. Etiology Finding

Etiology Any disease or problem that weakens or damages the valve or heart tissue around the valve can cause mitral valve regurgitation (also called mitral valve insufficiency).

Aging heart related to aortic stenosis. Etiology Finding

Etiology Aortic stenosis can be a congenital disorder, but it is more commonly caused by the buildup of calcium deposits that narrow the valve, which is seen in older adults. Another cause of aortic stenosis is rheumatic fever.

Marfan syndrome related to aortic valve insufficiency. Etiology Finding

Etiology Causes of aortic insufficiency include ankylosing spondylitis, endocarditis, hypertension, Marfan syndrome, syphilis and systemic lupus erythematosus. Marfan syndrome is an inherited disease of connective tissues. In the past, the main cause of aortic insufficiency was rheumatic fever.

Mononucleosis related to myocarditis. Etiology Finding

Etiology Myocarditis is an inflammation of the myocardium (the middle layer of the heart wall) and it is usually caused by a viral infection (it can also be caused by bacteria, parasites and fungi). Other viruses associated with myocarditis include the common cold (adenovirus), rubella, parvovirus B19 (which causes fifth disease) and HIV.

Upper respiratory infection related to pericarditis. Etiology Finding

Etiology Pericarditis is inflammation of the pericardium (the thin, sac-like covering of the heart). A common cause of pericarditis includes viral infections, including pneumonia and influenza; other causes include bacterial or fungal infections.

Congenital disorder related to pulmonary stenosis. Etiology Finding

Etiology Pulmonary stenosis is a congenital defect affecting the pulmonary valve. It is the second more common congenital heart defect.

The nurse is assessing a client diagnosed with chronic obstructive pulmonary disease (COPD). The client is on oxygen for low PaO2 levels. Which assessment is a nursing priority to evaluate the outcome of the therapy? Assess lung sounds Observe for skin color changes Assess coughing frequency and sputum characteristics Evaluate oxygen saturation (SaO2) levels frequently

Evaluate oxygen saturation (SaO2) levels frequently The best method to evaluate a client's oxygenation is to evaluate the SaO2. The oxygen saturation should be around 88% to 91% for someone with COPD. This method is equally as effective as an arterial blood gas reading to evaluate oxygenation status, and is less traumatic and expensive. Assessment of lung sounds, coughing and sputum and color should also be components of the respiratory assessment for a client with COPD, but are less precise indicators of the response to oxygen therapy than the oxygen saturation level.

During examination of the mouth of a 3 year-old child, the nurse discovers that the teeth have chalky white-to-yellowish staining with pitting of the enamel. Which of these conditions would most likely explain these findings? Excessive fluoride intake Oral iron therapy Ingestion of tetracycline Poor dental hygiene

Excessive fluoride intake The described findings are indicative of fluorosis, a condition characterized by an increase in the extent and degree of the enamel's porosity. This problem can be associated with repeated swallowing of toothpaste with fluoride or the drinking water with high levels of fluoride.

An 88 yo has decreased muscle strength in his bilateral upper extremities. Abnormal or expected findings

Expected A common age-associated change with musculoskeletal system is the decline in muscle mass and strength.

A 60 yo male has a left scrotal sac that is lightly lower than the right. Abnormal or expected findings

Expected Asymmetry in the scrotum is normal, with the left usually larger or having lower than the right.

The adolescent's spine is straight and posterior ribs are symmetrical when the patient bends forward. Abnormal or expected findings

Expected The adolescent patient should be assessed for scoliosis by asking the patient to bend forward and touch his or her toes. The patient's spine should be straight and without curvature or asymmetry.

The nurse has performed the initial assessments of four clients admitted with an acute episode of asthma. Which assessment finding would cause the nurse to call the health care provider immediately? Expiratory wheezes that are suddenly absent in one lobe Expectoration of large amounts of purulent mucous Appearance of the use of abdominal muscles for breathing Prolonged inspiration with each breath

Expiratory wheezes that are suddenly absent in one lobe Acute asthma is characterized by expiratory wheezes caused by obstruction of the airways. Wheezes are high-pitched musical sounds produced by air moving through narrowed airways. Clients often associate wheezes with the feeling of tightness in the chest. However, sudden cessation of wheezing is an ominous or bad sign that indicates an emergency - the small airways are now collapsed. Inspiratory wheezes are an emergency, whereas expiratory wheezes may not be as high a priority because some clients experience this with emphysema.

A nurse is preparing a 5 year-old for a scheduled tonsillectomy and adenoidectomy when the parents express anxiety and concern about the child's reaction to impending surgery. Which nursing intervention should best prepare the child? Encourage the child to bring a favorite toy to the hospital Explain the surgery one week prior to the procedure Introduce the child to all staff the day before surgery Arrange a tour of the operating and recovery rooms

Explain the surgery one week prior to the procedure A 5 year-old can understand the surgery, and should be prepared well before the procedure. Most of these procedures are "same day" surgeries and do not require an overnight stay.

A gravida 3, para 3 woman should be rushed to the delivery room once engagement has occurred. True or false

False Engagement means that the baby's head no longer floats freely, but has dropped down into the pelvis. In the multipara, engagement normally occurs about 2 weeks before birth.

The nurse is teaching a 10 year-old child prior to heart surgery. Which form of explanation meets the developmental needs of this age child? Explain the surgery using a model of the heart Introduce the child to another child who had heart surgery three days ago Provide the child with a booklet to read about the surgery Provide a verbal explanation just prior to the surgery

Explain the surgery using a model of the heart According to Piaget, the school-age child is in the concrete operations stage of cognitive development. The use of something concrete, like a model will help the child understand the explanation of the heart surgery.

During a nursing staff meeting, there is discussion of perceived inequities in weekend staff assignments. As a follow-up, the nurse manager should initially take which action? Help staff see the complexity of issues Facilitate creative thinking about staffing Allow the staff to change assignments Clarify reasons for current assignments

Facilitate creative thinking about staffing The manager, as a change agent, can facilitate the staff's solving the problem. Referred to as the "moving phase" of Lewin's change theory, the problem is first viewed from a different perspective and a variety of solutions are examined and decided upon; a new approach for weekend assignments can then be tried out.

A nurse is performing a developmental assessment on an 8 month-old infant. Which finding should be reported to the health care provider? Lifts head from the prone position Responds to parents' voices Falls forward when sitting Rolls from abdomen to back

Falls forward when sitting Sitting without support is expected in infants at 6 months-old to older.

15 to 30 pounds (6.8 to 13.6 kg) of traction is recommended for Buck's skin traction. True False

False 5 to 7 pounds (2.3 to 3.2 kg) of traction is recommended for Buck's skin traction - any more weight and the boot will be pulled off the leg! Buck's traction is used to immobilize, position and align the lower extremity. It is one of the most commonly used types of traction.

Only young patients suffer from abuse True or false

False Abuse can affect patients across the lifespan. Children to older adults. Abuse can be physical, emotional, or sexual. Depending on the jurisdiction, nurses may be mandated to report elder abuse. All US states have enacted laws and policies related to child abuse and neglect. Domestic abuse is not mandatory to report unless there is a threat to a child or unborn fetus.

The nurse should use alcohol or iodine-based products to clean around the pins used in skeletal traction. True False

False Alcohol and iodine-based products can accelerate corrosion of the metal and can cause skin staining. Skeletal traction pins can be cleaned with normal saline, sterile water or even plain soap and water.

Hands can be cleaned with alcohol-based hand rub after caring for a patient with C. diff True or False

False Alcohol does not kill C diff spores and soap and water should be used for hand hygiene as recommended by CDC

The pulmonary vein takes blood away from the heart to the lungs. True False

False All veins lead to the heart; it is arteries that lead away from it. The pulmonary vein returns oxygenated blood from the lungs to the left atrium.

The nurse-patient relationship is mutually defined, social relationship. True or false

False Although it is mutually defined, the nurse-patient relationship is time-limited, goal-directed and bounded by standards of care and of professional practice. It is not a local relationship. In fact, one of the blocks to therapeutic communication is the social response.

An intravenous cholangiogram (IVC) is an iodine-based contrast study designed to visually study the function of the kidneys. True False

False An IVC shows the bile ducts. It is the intravenous pyelogram (IVP) that visualizes the kidneys and urinary system.

A barium study and a computerized tomography (CT) scan can be completed within 24 to 48 hours of each other. True False

False Barium takes up to four days to be completely excreted so that its radio occlusive properties do not interfere with the CT scan.

Blood flows from the ventricles to the atria. True False

False Blood flows from the atria (sometimes incorrectly called "auricles") to the ventricles.

Treatment for bronchiolitis almost always involves surgery. True False

False Bronchiolitis is usually caused by the respiratory syncytial virus (RSV). RSV can also cause croup, ear infections and pneumonia in young children. Mild cases usually respond to rest, fluids and humidified air. Hospitalization may be required for severe cases of RSV.

Central diabetes insipidus may be caused by damage to the pancreas. True False

False Central diabetes insipidus may be caused by damage to the hypothalamus (or pituitary gland).

The client with Charcot's joint will benefit from regular aerobic exercise. True False

False Charcot's joint is a degenerative condition affecting one or more joints and results in joint instability and hypermobility, along with numbness and tingling or loss of sensation in the affected joints (usually in the feet). Treatment includes casting (for up to 12 weeks) and no weight-bearing on the foot followed by wearing a brace.

The prognosis for childhood cancers is generally poor. True False

False Childhood cancers, if diagnosed and treated early, are highly curable.

Children's bones are more brittle than those of adults, because the bones have not yet fully calcified. True False

False Children's bones are more flexible and porous than those of adults; in fact, fractures are very rare before age 1.

Chloasma is the first milk the new mother produces. True or False

False Chloasma is a skin discoloration of pregnancy. The first breast milk is called colostrum. Colostrum is low in fat, high in carbohydrates, protein and antibodies and is easy for the newborn to digest.

Chloasma is the first milk the new mother produces. True or false

False Chloasma is a skin discoloration of pregnancy. The first breast milk is called colostrum. Colostrum is low in fat, high in carbohydrates, protein and antibodies and is easy for the newborn to digest.

An overweight client who has been newly diagnosed with gout should be advised to lose weight as quickly as possible. True False

False Clients diagnosed with gout and who are overweight should lose weight slowly. Quick weight loss may cause uric acid kidney stones to form.

Clients should take levothyroxine at bedtime. True False

False Clients should take this medication in the morning with a full glass of water. Levothyroxine should be taken on an empty stomach, at least one hour before any other medications or vitamins.

Complementary and integrated health therapies replace the need for pharmacologic interventions. True or false

False Complementary and integrated health therapies are widely used among patients for various disorders and are often used in conjunction with medial treatment. Pain management may consist of pain medication, as well as relaxation or acupuncture. Complementary and integrated health therapies may also be effective at reducing anxiety, improving mood and increasing a patient's sense of control over the environment.

You should quickly remove contaminated clothing by pulling it over your head True or False

False Contaminated clothing should never be removed quickly, but it should be cut off instead of pulled over your head. place contaminated clothing inside a plastic bag, seal the bag, and then place inside another plastic bag

Delirium is a chronic condition affecting brain function. True False

False Delirium, or acute confusional state, is not a disease but a transient and potentially reversible disorder of cognition. It is often mistaken for a neurocognitive disorder (formerly referred to as dementia) or even an acute schizophrenic reaction.

One monoclonal antibody drug can be used to attack a variety of different types of cancer. True False

False Each monoclonal antibody recognizes only one particular protein, so different antibodies have to be used to target different types of cancer. For example, trastuzumab (Herceptin®) is used to treat certain breast cancers.

Rheumatoid nodules are the same thing as Heberden's nodes. True False

False Firm, non-tender, subcutaneous nodules develop in some chronic active cases of rheumatoid arthritis. They are serious extra-articular manifestations found in the lungs, eyes and blood vessels. Heberden's and Bouchard's nodes are bony enlargements of the joints involving the hand; these nodes are strongly familial (inherited) and are characteristic of osteoarthritis.

Guillain-Barré results from an acute infection and inflammation of the peripheral nerves. True False

False Guillain-Barré is a progressive, inflammatory autoimmune response occurring in the peripheral nervous system. The autoimmune response results in damage to myelin sheath and slows or alters nerve conduction. It is not caused by an acute infection.

Restraints can be ordered prn by health care providers True or False

False HCP are required to specify duration and circumstances for which restraints are required and for how they should be used. Nurses and HCPs must frequently monitor patients to reassess for the continued need for restraints.

A classic finding of Hirschsprung's disease is diarrhea. True False

False Hirschsprung's disease is a congenital defect with innervation of the rectum and/or colon. Areas without nerves cannot push waste material through the bowel, which causes a blockage. Findings in newborns include failure to take liquids, constipation and bile-stained vomitus.

To treat a conscious adult with a serum glucose below 50 mg/dL, the nurse should prepare to administer 1 mg glucagon. True False

False If the client is conscious and alert, the nurse can offer 15-20 g of carbohydrates (4 ounces of juice or regular soda, or 1 tablespoon honey) to the client. Glucagon is used if the client is unconscious.

Acromegaly is the result of excess growth hormone secretion in children. True False

False In children, too much growth hormone causes gigantism, resulting in an abnormal increase in height and bone growth. Acromegaly occurs in adulthood.

Normally, the blood pressure is lower in systole than in diastole. True False

False In systole the heart muscle squeezes or contracts; in diastole, it relaxes.

Hypotension is a finding of the initial stage of shock. True False

False In the initial stage of shock, only subtle changes in clinical signs may be seen. Hypotension does not typically occur until the progressive stage of shock. Pallor, cool and clammy skin, altered level of consciousness and irregular heart rhythms are the other classic findings of the progressive stage.

Fetal movement count during the third trimester should be at least 5 movements per day. True or false

False In the third trimester, an awake healthy fetus should move at least 3 times per hour. If the baby does not move, the mother should drink a glass of juice and then start a new count.

Fetal movement count during the third trimester should be at least 5 movements per day. True or False

False In the third trimester, an away, healthy fetus should move at least 3 times per day. If the baby does not move, the mother should drink a glass of juice and then start a new count.

Altered level of consciousness (LOC) is often a late sign in a client with increased intracranial pressure (ICP). True False

False Intracranial pressure is the pressure inside the skull and brain tissue. Altered LOC is often one of the earliest signs that a client has increased ICP. LOC is also the most important component of the neurological assessment in a high acuity and emergent client situation. Increased ICP can be caused by trauma, hemorrhage, tumors, edema or inflammation.

The nurse should expect to hear bowel sounds when assessing the client who is one day post-op following colostomy surgery. True False

False It may take three or four days for the bowel to return to normal function after a colostomy.

Mental health disorders and substance use disorder rarely occurs together. True or false

False Mental health problems can often lead to alcohol or drug use and abuse. Many clients who suffer from substance use disorder are also diagnosed with mental health disorders (and vice versa). Mental and substance use disorders share some underlying causes, including changes in brain composition, genetics and early exposure to stress and trauma.

Estrogen, calcitonin, bisphosphonates and bone-forming agents can reverse the damage of osteoporosis. True False

False Nothing can reverse the damage already done by osteoporosis. These medications can sometimes slow or halt the progress of the disease.

Osteoarthritis is an autoimmune disease that causes progressive loss of cartilage in the joints. True False

False Osteoarthritis is typically the result of normal aging and wear and tear on the joints; it is not an autoimmune disease. With osteoarthritis, there is loss of cartilage in the joints. Eventually, the cartilage wears away and bones rub against each other, causing pain, swelling and stiffness.

Pacemakers use high-energy electrical pulses to treat life-threatening arrhythmias. True False

False Pacemakers use low-energy electrical pulses to speed up a slow heart rhythm, help control abnormal or fast rhythms, and coordinate electrical signaling between the chambers of the heart. Implantable cardioverter defibrillators (ICD) use both low-energy and high-energy electrical pulses (these high-energy pulses treat the life-threatening arrhythmias).

Parkinson's disease (PD) affects intellectual ability. True False

False Parkinson's disease does not initially affect intellectual ability; however, some clients with PD may eventually experience changes in memory, thinking or reasoning. Also, many clients may develop depression later in the disease process, which is characterized by withdrawal, sadness, loss of appetite and sleep disturbances.

A stress fracture is an example of a pathological fracture. True False

False Pathological means that a bone or joint was weakened by a disease process. Stress fractures are caused by the repetitive application of force (such as overuse) or by a condition that weakens the bone (osteoporosis); stress fractures are most common in the weight-bearing bones of the lower leg and foot.

Infantile osteochondritis of the hip occurs when the infant's femur is insecurely seated in the acetabulum. True False

False When the head of the femur is improperly seated in the acetabulum, it is called hip dysplasia. During the physical assessment, the nurse will hear a click or feel a popping sensation (Ortolani's sign) when rotating or abducting the affected hip. This condition is treated using bracing, casting and/or surgery.

When the nurse suspects compartment syndrome, the casted limb should be elevated about the level of the heart. True False

False When the nurse suspects compartment syndrome, the cast should be split and constrictive bandages released. The limb should not be elevated above the level of the heart because this compromises arterial perfusion, which compounds the ischemic problem.

Type 2 diabetes results when pancreatic beta cells stop producing insulin. True False

False With type 2 DM, the pancreas produces insulin, but it's not enough and/or the body cannot use it properly (insulin resistance). Type 1 DM is when beta cells stop producing insulin.

Respiratory distress syndrome (RDS) in infants is caused by weakness or underdevelopment of chest muscles. True False

False v

The nurse can crush the oral medication disopyramide CR and mix it with applesauce. True or False

False CR means 'controlled release' and this medication must not be crushed. Do not crush any oral medication that ends in the following abbreviations: CR (controlled release), CD (controlled delivery), LA (long acting), SR (sustained release), XL (extended release), XR (extended release) or XT (extended release).

Cardioversion is used in the emergency treatment of ventricular fibrillation. True False

False Cardioversion is an elective procedure that is used to treat dysrhythmias, like atrial fibrillation. It involves synchronized shocks specific to the arrhythmia. Defibrillation is used for the immediate treatment of life-threatening arrhythmias, like ventricular fibrillation. It involves non-synchronized shocks during the cardiac cycle.

A gravida 3, para 3 woman should be rushed to the delivery room once engagement has occurred. True or False

False Engagement means that the baby's head no longer floats freely, but has dropped down into the pelvis. In a multipara, engagement normally occurs about two weeks before birth.

Hypovolemia is a risk for the client receiving whole blood products. True or False

False Hypervolemia (circulatory or fluid overload) develops when too large a volume of blood is given too quickly. To avoid hypervolemia, blood products should be infused at a rate no faster than 2 to 4 mL/kg/hour (but not to exceed a 4 hour hang time).

The nurse will hold the tube feeding when the gastric residual is greater than 100-150mL True or false

False If the residual volume exceeds the amount of formula given in the previous 2 hours, it may be necessary to consider reducing the rate of feeding. Current protocols state not to stop feedings unless there are other signs of intolerance, such as bloating abdominal pain, emesis, or nausea.

Obesity is a risk factor for the development of rheumatoid arthritis. True False

False Obesity, because it stresses joints, is a risk factor in the development of traumatic osteoarthritis. Rheumatoid arthritis, although not fully understood, seems to be an autoimmune disorder.

One if the first signs of pregnancy is Chadwick's sign, which is the softening of the cervix. True or false

False There are several findings of pregnancy during the first trimester, increased vascularity in the vagina is called the Chadwick's sign; the increased vascularization and softness of the uterine isthmus is Hegar's sign; and the softening of the cervix is Goodall's sign.

A child is sent to the school nurse by a teacher who has a written note that fifth disease is suspected. Which of the following should the nurse expect to find if the child has this condition? a. Macule that rapidly progresses to papule and then vesicles b. Discrete rose pink macules will appear first on the trunk and fade when pressure is applied c. Bright red cheeks, with a "slapped face" appearance d. Koplik spots appear first followed by a rash that appears first on the face and spreads downward

Fifth disease is also referred to as parvovirus infection or erythema infectiosum. Some people may call it slapped-cheek disease because of the face rash that develops resembling slap marks. It is also commonly called fifth disease because it was fifth of a group of once-common childhood diseases that all have similar rashes. The other four diseases are measles, rubella, scarlet fever, and Dukes' disease. People will not know that a child has parvovirus infection until the rash appears, and by that time the child is no longer contagious.

A parent has numerous questions regarding normal growth and development of a 10 month-old infant. Which growth or development parameter should be of the most concern to a nurse? Cries when the parents leave the room Able to stand up briefly in play pen Fifty percent increase in birth weight Head circumference is about the same as chest size

Fifty percent increase in birth weight Although a lot of factors affect weight gain, birth weight usually doubles by 5 months and triples by about 12 months. Between 6 months and 2 years, head circumference and chest size are about the same. A 10 month-old may be able to stand alone (very briefly) or even walk unassisted. Stranger anxiety also starts around this time.

Cardiomyopathy related to myocarditis. Etiology Finding

Finding After the initial infection subsides, the body's immune system continues to damage the heart muscle, weakening the heart. Myocarditis is a common cause of dilated cardiomyopathy.

Dyspnea, chest pain, and syncope related to pulmonary stenosis. Etiology Finding

Finding Dyspnea, chest pain, and syncope are the three classic findings of pulmonary stenosis.

Dyspnea related to mitral valve regurgitation. Etiology Finding

Finding Findings of mitral valve regurgitation depend on the severity and how quickly the condition develops. Common findings include heart murmurs, shortness of breath, fatigue and paroxysmal nocturnal dyspnea.

Dyspnea, syncope and angina related to aortic stenosis. Etiology Finding

Finding In aortic stenosis, the aortic valves do not open fully, causing decreased blood flow from the heart. Dyspnea, syncope and angina are the three classic findings of aortic stenosis.

Sharp, stabbing chest pain related to pericarditis. Etiology Finding

Finding The main finding of pericarditis is a sharp, stabbing pain in the center or left side of the chest. By definition, chest pain is a finding.

Peripheral edema related to pulmonary stenosis. Etiology Finding

Finding The most common symptoms of pulmonary stenosis are rapid or heavy breathing, dyspnea, tachycardia and peripheral edema (swelling in the feet, ankles, face, eyelids and/or abdomen).

A client, who had his entire stomach surgically removed six months ago, is now readmitted. Which of the following assessment findings would indicate that the client is experiencing complications associated with this surgery? Decreased night vision Findings consistent with fatigue Poor wound healing Tendency to bruise easily

Findings consistent with fatigue When clients have the stomach surgically removed, they no longer have the stomach's production of intrinsic factor, leading to poor Vitamin B12 absorption. This results in anemia with symptoms of fatigue, due to the decreased number of red blood cells to carry oxygen to the body. The client with gastrectomy or gastric bypass surgery is also at risk of experiencing dumping syndrome with abdominal cramping pain, diarrhea, lightheadedness, tachycardia and hypoglycemia. Dumping syndrome is usually associated with eating too much or too rapidly, and can be avoided by following the proper diet (five to six small meals per day, high protein, low carbohydrate and fat, eaten slowly) and by avoiding fluids with meals that move food rapidly into the small intestine.

A client says, "It's raining outside and it's raining in my heart. Did you know that St. Patrick drove the snakes out of Ireland? I've never been to Ireland." What should the nurse document this behavior as? Perseveration Flight of ideas Neologisms Circumstantiality

Flight of ideas Flight of ideas is characterized by over productivity of talk and verbally skipping from one idea to another. It is classic with clients diagnosed with bipolar disorder and occurs in the manic state of this disease. Flight of ideas can also occur with the diagnosis of schizophrenia and in clients who are intoxicated with psychoactive substances.

A nurse documents "effective use of guided imagery to change pain from a 4 to a 1 (on a 10-point scale)." Which definition best describes this technique? Focus on pleasant, relaxed mental pictures Closure of the eyes to focus on the back of the eyelids or blank screen Inhalation to a count of four and exhalation to a count of four The repetition of a word to self with the eyes closed

Focus on pleasant, relaxed mental pictures Guided imagery is a technique that uses pleasant mental visuals that can be recalled by the client to reduce stress, anxiety or pain. Focusing on the back of the eyelids or a repetition of a word or phrase describes meditation.

A nurse is teaching a child and family members about the medication phenytoin prescribed for seizure control. Which side effect is most likely to occur? Vertigo Drowsiness Gingival hyperplasia Vomiting

Gingival hyperplasia Overgrowth of the gums (gingival hyperplasia) is the most common side effect of long-term use of phenytoin (Dilantin); excessive hair on the face or body, acne, and coarseness of facila features are also common with long-term use. Good oral hygiene and regular visits to the dentist should be emphasized.

A nurse is teaching a parent how to administer oral iron supplements to a 2 year-old child. Which intervention should be included in the teaching? Stop the medication if the stools become tarry green Add the medicine to a bottle of formula Give the medicine with orange juice and through a straw Administer the iron with your child's meals

Give the medicine with orange juice and through a straw Absorption of iron is facilitated in an environment rich in vitamin C. Because liquid iron preparation will stain teeth, a straw should be used. Parents should be informed that dark, tarry stools are expected outcomes of taking iron supplements. Iron is best absorbed on an empty stomach (but it may be given after meals if the child experiences an upset stomach).

A mother, who has been exclusively breastfeeding her 6 month-old, requests more information about meeting the nutritional needs of her infant. What information will the nurse provide? Begin a regular schedule of meals and snacks, offering a variety of foods Offer finger foods to encourage self-feeding during family meals Cut back on the number of times a day the infant receives breastmilk Gradually begin adding pureed iron-rich meat and/or cereal as the first foods

Gradually begin adding pureed iron-rich meat and/or cereal as the first foods The nurse should recommend increasing the number of times a day that complementary foods are offered while continuing to breastfeed. Pureed iron-rich meat, meat alternatives, and/or iron-fortified cereal should be the first complementary foods. After pureed foods, the next transition should be to add strained or mashed foods and then finger foods. From about one year of age, young children begin to have a regular schedule of meals and snacks.

The nurse is preparing a client and her healthy newborn for discharge. The nurse provides information about hormonal effects in newborns and tells the client to expect which of the following conditions in her baby? Edema of the scrotum Gynecomastia Mongolian spots Lanugo

Gynecomastia Exposure to maternal hormones in utero may cause temporary conditions in the newborn. About three days after birth, both newborn boys and girls may experience swelling of the breasts as a result of the withdrawal of maternal estrogen. This should go away by the second week after birth.

The nurse is caring for a client in a home setting. Which approach is the most effective way to prevent infections when providing care to clients in the home? Using a barrier between the client's furniture and the nurse's bag Handwashing before and after examination of clients Wearing non-powdered latex-free gloves to examine the client Wearing a mask with a shield during any eye/mouth/nose examination

Handwashing before and after examination of clients Handwashing remains the most effective way to avoid spreading infection. However, too often nurses do not practice good handwashing techniques and do not teach families to do so. Nurses need to wash their hands before and after touching the client and before entering the nursing bag. All of the options are correct, and the sequence of priorities would be to wash hands in stated manner, use of a barrier for the nurse's bag, wear gloves, and wear a mask during head examinations.

A newly admitted adult client has a diagnosis of hepatitis A. The charge nurse should reinforce to the staff members that the priority routine in infection control strategy, in addition to handwashing, is which of these approaches? Wear a gown to change linens soiled from incontinence Have gloves on while handling bedpans with feces Use a mask with a shield if there is a risk of fluid splash Place appropriate precaution signs outside and inside the room

Have gloves on while handling bedpans with feces The specific measure to prevent the spread of hepatitis A is careful handling and protection while working with fecal material. All of the other actions are correct but not the priority routine in infection control strategy that is used with hepatitis A.

The nurse is caring for a client who received tenecteplase to open an occluded coronary artery following an acute myocardial infarction. Which finding should cause the greatest concern for the nurse? Red-colored tint of the urine Hematemesis Pink frothy sputum Serosanguineous drainage from gums

Hematemesis Hemorrhage, or bleeding, is the most common risk associated with any thrombolytic. Tenecteplase (TNKase) is currently indicated for the management of acute myocardial infarction (AMI). Minor bleeding from the gums or nose can occur in about 25% of people who receive these drugs. Remember that the spelling of many of the generic thrombolytic agents end with "ase."

The nurse is caring for a 4 year-old who will have surgery for tetralogy of Fallot tomorrow. Which laboratory report must receive priority attention by the nurse? White blood cell count Erythrocyte sedimentation rate Hemoglobin and hematocrit Arterial blood gases

Hemoglobin and hematocrit Tetralogy of Fallot is a cyanotic heart defect, which can manifest in polycythemia due to release of erythropoietin. Hemoglobin values of up to 20 and hematocrit's of 60 or higher may be seen. These must be noted and reported to the health care provider.

The nurse is planning care for a client who is taking cyclosporine modified. What is an appropriate nursing diagnosis for this client? Alteration in body image High risk for infection Impaired physical mobility Altered growth and development

High risk for infection Cyclosporine modified (Gengraf, Neoral) inhibits normal immune responses and is used with other medications to prevent transplant rejection. Clients who receive cyclosporine modified are at higher risk for infection with a fatal outcome.

The nurse is working to improve relationships with clients. To establish the feeling of trust in a nurse-client relationship, the nurse should exhibit which of the following qualities? Flexibility and kindness Confidence and optimism Sympathy and understanding Honesty and consistency

Honesty and consistency The client needs to understand that the nurse-client relationship will be safe, confidential, reliable, and consistent, with appropriate and clear boundaries. The relationship must always be focused on the client's problem and needs. Developing a positive relationship is one of the best predictors of outcomes in therapy.

A nurse is interviewing the parents of a child diagnosed with asthma. During data collection, the nurse should have as a priority to assess the child's environment for what factor? q New furniture Plants such as cactus Lead-based paint

Household pets Animal dander is a very common allergen affecting children and adults with asthma. Other triggers for an asthma or asthma attacks may include pollens, carpeting, secondhand cigarette smoke and household dust.

abdominal pain amenorrhea decreased libido osteoporosis sensitivity to cold

Hypopituitarism These findings are associated with a deficiency of luteinizing hormone (LH) & follicle stimulating hormone (FSH), which are pituitary hormones. Findings of hypopituitarism are directly related to the missing hormone (thyroid-stimulating hormone (TSH), LH, growth hormone (GH), adrenocorticotropic hormone (ACTH), and/or prolactin).

Which of these client's behaviors would indicate that the nurse-client relationship has passed from the orientation phase to the working phase? Identifies feelings about situations and expresses them appropriately Revitalizes a relationship with the family to help cope with the death of a child Expresses a desire to be cared for and pampered Recognizes regressive behaviors as a defense mechanism

Identifies feelings about situations and expresses them appropriately The working phase of the nurse-client relationship is also called exploration or the identification stage. That's because the client identifies his/her problems and works with the nurse to solve problems and develop coping skills, a positive self concept and, eventually, independence. These skills will help the client to adapt and behave more appropriately.

What is the drug classification for: azathioprine

Immunosuppressants inhibit cell-mediated immune responses. Azathioprine can also be categorized as an Antirheumatic. Most of these drugs are used in the prevention of transplantation rejection reactions; others are used in the management of selected autoimmune diseases (for example, nephritic syndrome of childhood and severe rheumatoid arthritis).

Bleeding, blood clots, bruising and drop in blood pressure. Due to: Atrial fibrillation Myocardial infarction Cardiac tamponade Occlusive arterial disease hemophilia Raynaud's phenomenon Heart Failure Thrombophlebitis DIC Sickle cell disease

In disseminated intravascular coagulation (DIC), the proteins that control blood clotting become overactive, increasing a client's risk for serious bleeding. This can be due to inflammation, infection or cancer.

A client is in the post anesthesia care unit (PACU). The vital signs are now much lower than when the client arrived in the PACU: T = 98 F (36.6 C), apical pulse = 115, respirations = 14, blood pressure = 82/46 mm Hg. The client's skin is cold and clammy. Rank the interventions the nurse should perform from first to last. Increase the intravenous (IV) rate Reassess vital signs Assess the area dependent to the surgical incision Assess the surgical dressing Elevate the lower extremities

Increase the intravenous (IV) rate Elevate the lower extremities Assess the surgical dressing Assess the area dependent to the surgical incision Reassess vital signs The client is exhibiting signs of hypovolemia. The first intervention should be to increase the IV rate to maintain circulatory volume. The nurse should then elevate the lower extremities to bring fluid from the lower body to the core, assess the surgical dressing for bleeding (as well as any sites dependent to the incision), and then reassess the vital signs to evaluate the interventions.

A nurse is caring for a client admitted to the hospital with a diagnosis of right lower lobe (RLL) pneumonia. On assessment, the nurse notes rhonchi and a loose but weak cough. The client has significant pleuritic pain and is unable to take a deep breath to cough effectively. Which nursing diagnosis would be most appropriate for this client based on this assessment data? Ineffective airway clearance related to sputum production and ineffective cough Impaired gas exchange related to acute infection and sputum production Anxiety related to hospitalization and role conflict Ineffective breathing pattern related to acute infection

Ineffective airway clearance related to sputum production and ineffective cough Ineffective airway clearance is defined as ineffective mobilization and clearance of airway secretions, usually due to a weak cough. Movement of secretions in the airway produces rhonchi on auscultation. Respiratory secretions obstruct the airway and need to be cleared to promote appropriate gas exchange and resolution of the infection. While the other diagnoses may be appropriate for this client, this is the only one supported directly by the assessment data provided.

A nurse is taking a health history from parents of a child admitted with possible Reye's syndrome. Which recent illness would the nurse recognize as increasing the risk of developing Reye's syndrome? Rubeola Meningitis Influenza Hepatitis

Influenza Varicella (chickenpox) and influenza are viral illnesses that have been identified as increasing the risk for Reye's syndrome. Use of aspirin with viral infections is contraindicated in children (from birth to 19 years of age) as it increases the risk of developing Reye's syndrome.

The nurse is performing pulmonary assessment on a client. Indicate the correct sequence of pulmonary assessment by dragging and dropping the steps below into the correct order. Percussion Inspection Auscultation Palpation

Inspection is first, observing for pattern of breathing, symmetry, anteroposterior and transverse chest diameters, as well as skin color, sounds and odors. The nurse will then palpate the posterior and anterior chest, noting any tenderness, crepitus, or tactile fremitus. The next step is percussion, which is performed in a systematic manner, usually from side to side from apex (top) of lungs to base (bottom), listening to changes in tone from one area to another. Lastly, the lungs are auscultated, listening over the anterior, posterior, and lateral chest for expected or adventitious sounds.

A client is admitted with a tentative diagnosis of congestive heart failure (CHF). Which assessment finding, consistent with this diagnosis, would the nurse expect? Inspiratory crackles Cyanosis Chest pain Heart murmur

Inspiratory crackles CHF is when the pumping action of the heart is reduced or weakened, which leads to a buildup of fluid in the lungs and surrounding body tissues. Early symptoms may include shortness of breath or chronic coughing. The person may also feel fatigued or lightheaded, and have difficulty breathing (dyspnea) and anorexia. When auscultating the lungs of a client with CHF, the nurse can expect to hear fine crackles (usually over the posterior lung bases) that do not clear after a cough. Crackles are the result of pulmonary edema or fluid in the alveoli due to heart failure.

Clients may develop this condition after some types of surgery and when using certain drugs, especially narcotics. Cirrhosis GERD Cholecystitis Crohns Diverticulitis Hepatitis Ulcerative colitis Pancreatitis Intestinal obstruction Peptic Ulcer

Intestinal obstruction is a partial or complete blockage of the bowel. It is usually due to either a mechanical reason or an ileus (a condition where there is no structural problem causing the bowel not to contract correctly - such as following surgery).

The nurse provides regular mouth care to the hospice client who is actively dying at home. The family wants to know why the doctor doesn't order an IV since the client's mouth seems so dry. What information can the nurse provide to the family that best answers their question. The client will need to be hospitalized if an IV is started The client will need to have a indwelling catheter inserted if an IV is started Intravenous hydration will increase episodes of delirium Intravenous hydration can delay death

Intravenous hydration can delay death Clients who are dehydrated may experience delirium and may benefit from IV therapy. However, intravenous hydration does not improve dry mouth and can even delay death. The nurse should explain that the client's comfort can be enhanced by providing frequent mouth care and that decreased oral intake is a natural and nonpainful part of the dying process.

Iron is one of the macro minerals fond in a healthy human body. True or false

Iron is considered a trace mineral. The seven major minerals in the body are calcium, magnesium, sodium, potassium, phosphorus, sulfur, and chlorine.

A nurse is assessing a hospitalized newborn who was delivered in the home several days ago. The infant's mother has a known history of heroin addiction. Which finding should the nurse expect to find in the infant? Central nervous system depression Lethargy with excessive sleepiness Large for gestational age Irritability

Irritability Findings of withdrawal in the newborn may take several days and the severity of the findings depends on how much and how often the mother was using heroin during the pregnancy. The more common findings are irritability and continual crying, decreased sleep, fever, diarrhea, and seizures. Initial treatment may be primarily supportive; the infant is swaddled to decrease sensory stimulation, offered small frequent feedings of hypercaloric formula to supply the additional caloric requirements (these babies are often hypoglycemic and are small for gestational age), and the nurse will monitor sleep habits, temperature, neuro status and weight.

The client with coronary artery disease has a prescription for nitroglycerin transdermal patches. What is the best reason the client should not wear a patch for more than 12 to 14 hours each day? It can cause severe headaches It may no longer work as well It will cause profound hypotensive effects it will irritate the skin

It may no longer work as well Nitroglycerin patches may not work as intended when they are used continuously. To prevent tolerance to the medication, clients should apply a patch once a day and remove it after 12 to 14 hours. Some of the more common side effects of wearing a nitroglycerin patch may include headache, dizziness, lightheadedness, nausea, redness or irritation of the skin that was covered by the patch.

A client who is being treated for paranoid schizophrenia becomes loud and boisterous. The nurse immediately places the client in seclusion, and the client willingly complies. How might the nurse's action be interpreted by the case manager? It was appropriate in view of the client's behaviors of violence It may result in charges of unlawful seclusion and restraint It leaves the nurse vulnerable for charges of assault and battery It was necessary to maintain the therapeutic milieu of the unit

It may result in charges of unlawful seclusion and restraint Seclusion should only be used when there is an immediate threat of violence or threatening behavior toward the staff, the other clients or the client himself. In this situation, the client exhibited loud behaviors without any suggestion of threat to the staff or self.

A nurse is caring for a client with sickle cell disease who is scheduled to receive a unit of packed red blood cells. Which is an appropriate action for the nurse during the administration of the infusion? Slow the rate of infusion if the client develops a fever or chills Assess vital signs every 15 minutes throughout the entire infusion Store the packed red cells in the refrigerator while starting IV line Limit the infusion time to a maximum of four hours

Limit the infusion time to a maximum of four hours Infuse the specified amount of blood within four hours; blood is not to be infused for longer than four hours because of the risk of bacterial growth in the bag. Similarly, blood should not be stored in an unapproved refrigerator because of the concern of bacterial growth. If the client develops fever or chills, the nurse does not slow the rate of infusion; this would be considered a reaction and the blood should be stopped and the blood bank and health care provider notified. Vital signs are per agency protocol but are usually not this frequent for the duration of the infusion.

A nurse is planning care for a client with increased intracranial pressure. What is the best position for this client? Side-lying with head flat Supine Trendelenburg Low Fowler's

Low Fowler's Positioning the client in low Fowler's position, with the head of the bed elevated 15 to 20 degrees, reduces cerebral venous congestion. This causes a decrease in intracranial pressure (ICP) compared to the supine (flat) position. Trendelenburg, with the head lower than the feet, is contraindicated in clients with increased ICP, as it will further increase the ICP level.

ST elevation on ECG. Due to: Atrial fibrillation Myocardial infarction Cardiac tamponade Occlusive arterial disease hemophilia Raynaud's phenomenon Heart Failure Thrombophlebitis DIC Sickle cell disease

MI STEMI is a type of heart attack when the coronary artery is completely blocked off by an occlusion. The client will experience crushing, non-remitting retrosternal pain, diaphoresis, nausea/vomiting and dyspnea.

A nurse admits a premature infant who has been diagnosed with respiratory distress syndrome (RDS). In planning care for the infant, the nurse understands that the pathophysiology of this disorder affects the infant's ability to do what? Stabilize thermoregulation Regulate intrapulmonary airway pressures Maintain alveolar surface tension Adequately clear thick, sticky mucus from the lungs

Maintain alveolar surface tension RDS is primarily a disease related to a developmental delay in lung maturation. Although many factors may lead to the development of the disorder, the central factor is the lack of a normally functioning surfactant system in the alveolar sac from immaturity in lung development because the infant is premature. A lack of surfactant production results in the collapse of the alveolar sacs.

A client reports to the nurse that she is experiencing a sudden, deep and throbbing pain in one leg. What is the most appropriate first action to be taken by the nurse? Maintain the client on bed rest Apply ice to the extremity Ambulate for several minutes Suggest isometric exercises

Maintain the client on bed rest The finding suggests deep vein thrombosis (DVT). The client must be maintained on bed rest and the health care provider should be notified urgently. Deep vein thrombosis can lead to pulmonary embolism, which is a medical emergency that can cause severe problems with gas exchange and cardiac output and can even cause cardiac arrest. Anticoagulants are used to treat DVT, initially being administered by IV (heparin drip) or subcutaneous injection (low-molecular weight heparin). This is then followed by long-term oral anticoagulation with warfarin.

The nurse is providing care for a client diagnosed with sickle cell crisis. Which medication ordered for pain control should be questioned by the nurse? Morphine sulfate Fentanyl (Sublimaze) Codeine Meperidine (Demerol)

Meperidine (Demerol) Meperidine (Demerol) is not recommended in clients with sickle cell disease. Normeperidine, a metabolite of meperidine, is a central nervous system stimulant that produces anxiety, tremors, myoclonus, and generalized seizures when it accumulates with repetitive dosing. Clients with sickle cell disease are particularly at risk for normeperidine-induced seizures.

The client is diagnosed with heart failure and oral digoxin is prescribed. What is the priority nursing assessment for this medication? Monitor serum electrolytes and creatinine Measure apical pulse prior to administration Maintain accurate intake and output ratios Monitor blood pressure every 4 hours

Measure apical pulse prior to administration Digoxin is an antiarrhythmic and an inotropic drug. It works to increase cardiac output and slow the heart rate, which is why the nurse should measure the apical pulse for one minute prior to administering the drug. The nurse will withhold the dose and notify the health care provider if the apical heart rate is less than 60 bpm (adult). Intake and output ratios and daily weights should be monitored for clients in heart failure, but this is not the priority. Impaired renal function may contribute to drug toxicity, which is why the nurse should monitor serum electrolytes, creatinine and BUN; the nurse should also monitor serum digoxin levels.

A 78 year-old client has just returned from having an intravenous pyelography. Which information is a priority for the nurse to reinforce? Rest for the next 24 hours because the preparation and the test are tiring Eat a light diet for the rest of the day Measure the urine output for the next day and immediately notify the health care provider if it is less than usual Drink at least one 8-ounce glass of fluid every hour while awake for the next two days

Measure the urine output for the next day and immediately notify the health care provider if it is less than usual It is important for the client to monitor urine output because this information would alert the client to the complication of acute renal failure. Renal failure may occur as a complication of the dye used during the procedure. Renal failure occurs most often in older adult clients who are often dehydrated before the dye injection. Unless contraindicated, they should force fluids for 24 hours after the test.

A client in acute respiratory distress is admitted with arterial blood gas results of: PH 7.30; PO2 58, PCO2 34; and HCO3 19. The nurse should make which conclusion about these results? Metabolic acidosis Respiratory alkalosis Respiratory acidosis Metabolic alkalosis

Metabolic acidosis These lab values indicate metabolic acidosis: the PH is low, PCO2 is normal, and bicarbonate level is low. The oxygen level is not used during the determination of ABG interpretation.

What is the drug classification for: potassium bicarbonate & potassium citrate

Minerals/electrolytes/pH modifiers are taken to correct imbalances of substances in the blood (minerals and electrolytes) or to make the urine more alkaline (pH modifiers). They are used in the prevention and treatment of any deficiencies or excesses of electrolytes. Acidifiers and alkalinizers are also used to prevent crystals from forming in the urine and inhibit the formation of kidney stones. Magnesium sulfate is used for pre-eclampsia and eclampsia. Some of these meds are used to neutralize gastric acid.

creamy white patches on tongue, palate, and inner aspects of cheek

Moniliasis candidiasis Moniliasis candidiasis is a fungal infection that commonly occurs in babies and also people with impaired immune systems, taking antibiotics or using inhaled steroids; also known as thrush.

What is the drug classification for: phenelzine

Monoamine oxidase inhibitors (MAOIs) prevent the enzyme monamine oxidase from breaking down the neurotransmitters norepinephrine and serotonin (also known as monoamines) in the brain. They are typically used to treat depression.

Ptosis; difficulty chewing & swallowing; weakness in arms & legs; shortness of breath

Myasthenia Gravis Myasthenia gravis is a chronic autoimmune neuromuscular disease characterized by varying degrees of weakness of the skeletal muscles. It is caused by a defect in the transmission of nerve impulses to muscles. Muscles that control eye and eyelid movement, facial expression, chewing, talking, swallowing, and breathing are often affected.

facial puffiness macroglossia ptosis coarse, sparse hair confusion hypothermia bradycardia

Myxedema coma Myxedema coma/crisis usually affects older women who have long-standing, undiagnosed or undertreated hypothyroidism. The crisis is triggered by a significant stress, such as infection, a systemic disease, certain medications or exposure to a cold environment.

An infant has just had a pyloromyotomy. Initial postoperative nursing care would include which of these approaches? Intravenous fluids for three to four days Bland diet appropriate for age NPO then glucose and electrolyte solutions Formula or breast milk as tolerated

NPO then glucose and electrolyte solutions Pyloric stenosis is caused when a muscle between the stomach and duodenum grows too large and thick, blocking food from being pushed from the stomach into the duodenum. During a pyloromyotomy, the surgeon cuts through the thickened muscle. Postoperatively, the initial feedings for infants are small quantities of clear liquids, such as glucose water or water with electrolytes in it. If the infant tolerates clear liquids, caregivers will give watered-down breast milk or formula; feedings are then advanced to regular breast milk or formula.

A client is receiving erythromycin 500 mg PO every six hours for the treatment of pneumonia. What is the most common side effect of this anti-infective? Nausea/vomiting Severe headache Blurred vision Muscle aches

Nausea/vomiting Nausea is a common side effect of erythromycin, for both oral and intravenous forms.

A 71 year-old client is admitted for mitral valve replacement surgery. The client has a history of mitral valve regurgitation and mitral stenosis since he was a teenager. During the admission assessment, the nurse should ask if the client experienced which health issue as a child? Hay fever Measles Rheumatic fever Encephalitis

Rheumatic fever Clients that present with mitral stenosis often have a history of rheumatic fever or bacterial endocarditis, with valvular damage due to the infection.

A client with this disorder recognizes their behavior is excessive and unreasonable but cannot stop the behavior.

OCD Clients with obsessive compulsive disorder (OCD) cannot control their obsessions and/or compulsions, even though they recognize that they are unreasonable or excessive.

A nurse is talking to parents about nutrition for their school-aged children. What is the most common nutritional disorder found in this age group? Anorexia Bulimia Malnutrition Obesity

Obesity Many factors contribute to the high rate of obesity in school-aged children. These include heredity, sedentary lifestyle, social and cultural factors and poor knowledge of balanced nutrition.

A nurse is completing the initial assessment for a child immediately after a surgical correction of a ventricular septal defect. Which nursing assessment should be a priority during the process? Auscultate for pulmonary congestion Observe for postoperative arrhythmias Blanch nail beds for color and refill Monitor for the equality of peripheral pulses

Observe for postoperative arrhythmias The atrioventricular bundle (bundle of His), a part of the electrical conduction system of the heart, extends from the atrioventricular node along each side of the interventricular septum and then divides into right and left bundle branches. Surgical repair of a ventricular septal defect consists of a purse-string approach or a patch sewn over the opening that involves this electrical circuit.

Pain, pallor, paresthesia, pulselessness, paralysis or poikilothermia. Due to: Atrial fibrillation Myocardial infarction Cardiac tamponade Occlusive arterial disease hemophilia Raynaud's phenomenon Heart Failure Thrombophlebitis DIC Sickle cell disease

Occlusive arterial disease (also known as peripheral artery disease) is caused by arteriosclerosis. The classic symptoms appear during walking or exercise and are relieved with rest.

The client is using nonsteroidal anti-inflammatory drugs (NSAIDs), such as ibuprofen, to manage arthritis pain. The nurse should caution the client about which common side effect? Urinary incontinence Nystagmus Constipation Occult bleeding

Occult bleeding Nonsteroidal anti-inflammatory drugs (NSAIDs) taken for long periods of time may cause serious side effects, including bleeding in the gastrointestinal tract. Clients should be instructed to take the medication with meals if stomach upset occurs. To avoid esophageal irritation, the client should take the drug with a full glass of water and to avoid lying down for 30 to 60 minutes after taking a dose.

A client with schizophrenia receives haloperidol 5 mg three times a day. The client's family is alarmed and calls the clinic nurse when "his eyes rolled upward." The nurse should recognize this finding as what type of side effect? Dysphagia Nystagmus Tardive dyskinesia Oculogyric crisis

Oculogyric crisis This refers to involuntary muscles spasm of the eye. There are medications to treat such side effects, for example trihexyphenidyl or benztropine.

A nurse is caring for a client with extracellular fluid volume deficit. Which assessment would the nurse anticipate finding? Rapid respirations Bounding pulse Oliguria Distended neck veins

Oliguria Kidneys maintain fluid volume through adjustments in urine volume. When clients are dehydrated the kidney will conserve fluid and the urine output will be decreased.

The adult client is alert and cooperative. The client has a short leg cast and can only partially bear weight on the casted leg. Which technique can be safely used to transfer the client from the bed into a chair? One caregiver applies a transfer belt and uses the stand-and-pivot technique One caregiver applies a gait belt and transfers the client toward the weak side Two caregivers lift the client from the bed and move the client into the chair Two caregivers use a friction-reducing device and wide base of support when transferring the client

One caregiver applies a transfer belt and uses the stand-and-pivot technique The algorithm for safe client handling and transferring an alert and cooperative client to a chair states: one caregiver applies a gait/transfer belt, uses the stand-and-pivot technique and transfers the client toward the strong side. A friction-reducing device is placed under the client to assist in turning or moving the person in bed, not transferring to a chair. A two person lift is unsafe.

A newborn presents with a pronounced cephalohematoma after a birth in the posterior position. Which nursing diagnosis should guide the plan of care? Injury related to intracranial hemorrhage Impaired mobility related to bleeding Parental anxiety related to knowledge deficit Pain related to periosteal injury

Parental anxiety related to knowledge deficit This hematoma is caused by pressure and/or trauma during labor; it is often caused by forceps used in the delivery. This painless condition is usually benign and resolves on its own in four to six weeks. The swelling does not cross the suture lines. Parental anxiety must be addressed by listening to their fears and explaining the nature of this common alteration.

The nurse is caring for a client who has a wound on the leg from a motorcycle accident. During a home visit, the nurse should use which assessment parameter as an indication that this client is experiencing normal wound healing? Pebbled red tissue in the wound base Eschar over the surface White patches on the outside edges Green drainage from the center

Pebbled red tissue in the wound base As the wound granulates, pebbled red tissue in the wound base indicates healing. Any of the other findings would indicate that the wound was not healing properly.

What is the drug classification for: ampicillin

Penicillins belong to a group of antibiotics called beta-lactams, which exert bactericidal action by inhibiting bacterial cell wall production. Currently the group includes more than 20 antibiotics. Used in the treatment and prophylaxis of wide range of bacterial infections including streptococcal infections, syphilis and Lyme disease.

The nurse is assessing a client who sustained multiple fractures, contusions, and lacerations in a motor vehicle accident three days ago. The client suddenly becomes confused. Which of the following findings would support the nurse's suspicion that the client has developed a fat embolism? (Select all that apply.) Hypertension Petechiae on the upper anterior chest Elevated temperature Dyspnea Low oxygen saturation

Petechiae on the upper anterior chest Elevated temperature Dyspnea Low oxygen saturation Manifestations of acute confusion, hypoxia, fever and hypotension may indicate fat embolism in a client who has sustained multiple fractures, particularly fractures of the long bones. The occlusion of dermal capillaries by fat with increased friability of the capillaries can result in skin petechiae. This is most common on the chest, neck, upper arm, axilla, shoulder, oral mucous membranes and conjunctiva.

A client is admitted to the psychiatric unit with a diagnosis of bipolar disorder. This client constantly "bothers" other clients, tries to help the housekeeping staff, demonstrates pressured speech and demands constant attention from the staff. Which activity should the nurse attempt to get the client to do? Ping-pong Checkers Reading Cards

Ping-pong Ping-pong provides an outlet for physical energy and requires limited attention. The other options would over-tax the client's level of self-control because the client has a need to be active.

The client underwent a total hip arthroplasty 48 hours ago. The client has been up in a chair and is prescribed physical therapy twice daily. What type of nursing care is needed for this client? (Select all that apply.) Remind the client not to bend the knee of the affected leg when sitting Empty the wound suction drainage device every 4 hours Place a soft foam triangular pillow between the client's legs when in bed Assist the client with a clear liquid diet Provide a seat riser for the toilet or commode Encourage client to perform leg exercises when in bed

Place a soft foam triangular pillow between the client's legs when in bed Provide a seat riser for the toilet or commode Encourage client to perform leg exercises when in bed On the first post-operative day following a total hip arthroplasty, the client will be up in a chair. The client should bend the affected leg at the knee when sitting in a chair - not keep it straight. Two days after surgery, the client will be walking in the hallway. When in bed, the client should continue to perform leg exercises and use a pillow or foam wedge between his or her legs (to keep the legs abducted.) The drain is usually removed the second day after surgery; there should be little-to-no drainage on the second post-op day. The client can eat a regular diet after surgery.

A client with this disorder experiences hallucinations and delusional thoughts.

Schizophrenia A client with schizophrenia experiences hallucinations and delusional thoughts. There are different types of schizophrenia, but often the client is unable to think rationally, communicate properly, make decisions or remember information.

The nurse enters a client's room just as the client begins to experience a generalized tonic clonic seizure. What action should the nurse take? Place the client on one side Elevate the head of the bed Hold the client's arms at the side Insert a padded tongue blade in client's mouth

Place the client on one side Clients should be positioned on their side. This position keeps the airway patent and allows saliva to drain from the mouth, which prevents aspiration. The nurse should also protect the client from injury by clearing furniture (if the client is on the floor). The client should not be restrained nor should anything be forced in the client's mouth.

The client who is diagnosed with autism begins to eat the food on his plate using his hands. Because the client regularly uses utensils to eat his food, which response by the nurse would be best? Place the spoon in the client's hand and state, "Use the spoon to eat your food." Make the comment: "I believe you know better than to eat with your hands." Remove the food and state: "You can't have anymore food until you use the spoon." Jokingly state: "Well I guess fingers sometimes work better than spoons."

Place the spoon in the client's hand and state, "Use the spoon to eat your food." This action identifies instruction and verbal expectation with adaptive behavior. The other options are incorrect and non-therapeutic approaches.

The mother of a child diagnosed with poison ivy tells the nurse that she does not know how her child contracted the rash because the child had not been playing in wooded areas. As the nurse asks questions about possible contact, which of these situations should the nurse recognize as the highest risk for exposure to poison ivy? Throwing a ball to a neighborhood child who has poison ivy Playing with cars on the pavement near burning leaves Eating small amounts of grass while playing "farm" Playing with toys in a backyard flower garden

Playing with cars on the pavement near burning leaves Smoke from burning leaves or stems of the poison ivy plant can produce a reaction. Direct contact with the toxic oil, urushiol, is the most common cause for this dermatitis. Throwing a ball to a child with the rash is not a highest risk because direct contact has the greatest risk.

fever chills productive cough dyspnea pleuritic pain use of accessory muscles

Pneumonia Pneumonia is an inflammatory process that results in edema of the lung tissues. Fluid moves into the alveoli and this can cause hypoxia. Pneumonia is caused by bacteria, viruses, fungi or chemicals.

The nurse is working with an adolescent diagnosed with morbid obesity. The nurse should recognize that obesity in adolescence is most often associated with what other finding? Sexual promiscuity Poor body image Drug experimentation Dropping out of school

Poor body image As the adolescent gains weight, there is a lessening sense of self-esteem and poor body image.

A client is treated in the emergency department for diabetic ketoacidosis (DKA) and a glucose level of 650 mg/dL (36 mmol/L). The nurse would expect which serum lab value to be altered as a result of therapy associated with the client's condition? Magnesium Creatinine Calcium Potassium

Potassium Potassium is lost in diabetic ketoacidosis during rehydration and insulin administration. Initial laboratory studies for DKA would include blood test for glucose and serum electrolytes every 1-2 hours until the client is stable; initial blood urea nitrogen (BUN) and initial arterial blood gas (ABG) measurements are also ordered. Repeat potassium and glucose (and sometimes phosphorus) are critical during treatment. An ECG may be used to assess the cardiac effects of extremes in potassium levels.

A nurse arranges for an interpreter to facilitate communication between a health care team and a non-English speaking client. To promote clearer communication, which of these would be an appropriate action for the nurse to use when working with an interpreter? Plan that the encounter will take more time than if the client spoke English Promote verbal and nonverbal communication with both the client and the family Speak only a few sentences at a time and then pause for a few moments Ask the client to speak slowly and to look at the person spoken to

Promote verbal and nonverbal communication with both the client and the family The nurse should communicate with the client and the family and not with the interpreter. Culturally appropriate eye contact, gestures and body language toward the client and family are important factors to enhance rapport and understanding. Maintain eye contact, if culturally correct, with both the client and interpreter to elicit feedback and to read nonverbal cues.

The nurse manager interviews several nurses for a staff position and the best qualified nurse is one with a sensory impairment. To better understand the issue of accommodations needed for this nurse, the nurse manager meets with the director of human resources. Which approach is suggested for the nurse manager? Make every necessary accommodation for the nurse with the disability Maintain an environment that's free from associated hazards Provide reasonable accommodations for the nurse with the disability Don't offer the position to the nurse with the disability due to financial burden

Provide reasonable accommodations for the nurse with the disability In the U.S., for example, the Americans with Disabilities Act (ADA) is designed to permit individuals with motor, cognitive, psychiatric or sensory impairment access to job opportunities. Employers must evaluate an applicant's ability to perform the job on a case-by-case basis; however, employers cannot discriminate on the basis of a disability. Employers are required to make "reasonable accommodations."

After a successful alcohol detoxification, a client remarked to a friend, "I've tried to stop drinking but I just can't. I can't even work without having a drink." The client's belief that he needs alcohol indicates the dependence is primarily of which type? Psychological Socialcultural Biological Physical

Psychological With psychological dependence, clients think thoughts and have attitudes toward alcohol or the desired substance. The results is that these thoughts produce cravings and behavior that reinforce compulsive use of the substance.

A client was admitted to the psychiatric unit diagnosed with major depressive disorder (MDD) after a suicide attempt. Which of the following findings of MDD would the nurse expect the client to exhibit? Anxiety from unconscious anger with hostility Psychomotor retardation or agitation Guilt coupled with indecisiveness from a poor self-concept Meticulous attention to grooming and hygiene

Psychomotor retardation or agitation Somatic or physiologic findings of depression include: fatigue, psychomotor retardation or agitation, chronic generalized or local pain, sleep disturbances, disturbances in appetite, gastrointestinal complaints and impaired libido. The other factors in the options are not a consideration for major depression.

A nurse is administering lidocaine to a client with a myocardial infarction. Which assessment finding requires the nurse's immediate action? Respiratory rate of 22 Pulse rate of 48 beats per minute Central venous pressure reading of 11 Blood pressure of 144/92

Pulse rate of 48 beats per minute Some of the side effects of lidocaine is bradycardia, heart block, cardiovascular collapse and cardiac arrest. This medication should not be administered without continuous cardiac monitoring.

A nurse is caring for a client with a new order for bupropion hydrochloride for treatment of depression. The order reads "Wellbutrin 175 mg twice a day for four days." What is the appropriate action? Observe the client for mood swings Question this medication dose Monitor neurologic signs frequently Give the medication as ordered

Question this medication dose Bupropion should be started at 100 mg twice a day for three days then increased to 150 mg twice a day. When used for depression, it may take up to four weeks for effective results. Common side effects are dry mouth, headache and agitation. Doses should be administered in equally spaced time increments throughout the day to minimize the risk of seizures.

flaky, white-to-yellowish scales form on oily areas of skin

Seborrheic dermatitis Seborrheic dermatitis is an inflammatory skin disorder affecting the scalp, face or ear; may occur with or without reddened skin. Also known as 'cradle cap' in babies.

The nurse is performing the initial assessment of a client with asthma at the beginning of the shift. The client has oxygen running at 2 liters per minute per nasal cannula. Which assessment finding would the nurse be most concerned about? Crackles at the base of the lungs on auscultation Pulse oximetry reading of 89% Excessive thirst with a dry cracked tongue Rapid shallow respirations with intermittent wheezes

Rapid shallow respirations with intermittent wheezes The most common symptoms of asthma include tightness in the chest, labored breathing, coughing and wheezing. Rapid and shallow respirations associated with labored breathing indicate the client is losing the strength required to breathe. The intermittent wheezes indicate increased narrowing of the small airways and a worsening condition. This client requires prompt and aggressive respiratory intervention to avoid respiratory failure, including bronchodilators (such as nebulized albuterol), increased oxygen supplementation to maintain a SpO2 of at least 92%, and anti-inflammatory medications (such as IV corticosteroids). The increased mucus in the airways stimulates coughing and can cause coarse crackles; the anti-inflammatory medication and bronchodilator will make breathing and mucus removal easier.

What would a nurse expect to see in a client who reports symptoms associated with tardive dyskinesia? Rapid tongue movements Repetitive slapping movements Behavioral changes Uncontrolled hand tremors during meals

Rapid tongue movements Tardive dyskinesia is a syndrome of involuntary movements of the face, mouth, tongue, trunk, and limbs that may occur after years of treatment with neuroleptic agents. Predisposing factors include older age, many years of cigarette smoking, long-term phenothiazine treatment and a diagnosis of diabetes mellitus.

What is the drug classification for: lispro

Rapid-acting insulin, such as insulin lispro, covers insulin needs for meals eaten at the same time as the injection. Short-acting insulin covers insulin needs for meals eaten within 30 to 60 minutes. Intermediate-acting insulin covers insulin needs for about half of the day or overnight (and is often combined with rapid- or short-acting insulin). Long-acting insulin covers insulin needs for about one full day. Insulin is used in the treatment of type 1 diabetes mellitus and may be used to treat type 2 diabetes mellitus.

Sequence of color changes in skin in response to cold or stress. Due to: Atrial fibrillation Myocardial infarction Cardiac tamponade Occlusive arterial disease hemophilia Raynaud's phenomenon Heart Failure Thrombophlebitis DIC Sickle cell disease

Raynaud's phenomenon is a condition where cold temperatures or strong emotions cause blood vessel spasms, which block blood flow to the fingers, toes, ears and nose.

A client is scheduled for a CT scan with contrast. What interventions should be taken by the nurse prior to sending the client to the imaging department? (Select all that apply.) Reassess the client's allergies Administer prescribed medication to sedate the client Confirm that a signed consent is in the chart Ask the client to remove all metal jewelry Ensure the client is well-hydrated

Reassess the client's allergies Confirm that a signed consent is in the chart Ask the client to remove all metal jewelry Usually the client is NPO prior to a CT scan, particularly when contrast material is being used. Allergies and past reactions to contrast media should be reviewed with the client. Any metal, including body piercings, jewelry, hearing aids and removable dental work should be removed and safely stored prior to the test. Sedation is necessary only in cases of extreme anxiety.

The nurse recognizes that cultural practices affect health outcomes. Which statement best reflects what nurses can do to improve health outcomes in clients from different cultures? Incorporate high personal standards and values for all interactions Use conventional wisdom to gain a deeper understanding about the client's health practices Reinforce the correct Western medical perspective Recommend a plan that meets client goals as well as professional nursing standards

Recommend a plan that meets client goals as well as professional nursing standards Nurses must provide culturally and linguistically appropriate care in order to help ensure successful outcomes. Nurses should advocate for clients from diverse ethnic and cultural groups by asking them about their health practices. As long as these practices are not harmful, the nurse can recommend a plan that both meets the goals of the client and professional nursing standards. Nurses can emphasize the science behind the plan of care without disparaging the client's culture.

A nurse is examining an infant in a clinic. Which nursing assessment for the infant is most valuable in the identification of serious visual defects? Cover test Visual acuity Red reflex test Pupil response to light

Red reflex test A brilliant, uniform red reflex is an important sign because it virtually rules out almost all serious defects of the cornea, aqueous chamber, lens, and vitreous chamber.

A client who is diagnosed with cirrhosis of the liver is started on lactulose. What should the nurse understand about the main action of the drug? Control portal hypertension Add dietary fiber Stimulate peristalsis Reduce ammonia levels

Reduce ammonia levels Lactulose (Kristalose) is a synthetic disaccharide that can be given orally or rectally. It blocks the absorption and production of ammonia from the gastrointestinal tract and secondarily stimulates bowel elimination (with the goal of two to four diarrhea stools per day). Lactulose is used to prevent portal-systemic encephalopathy.

A client has undergone electroconvulsive therapy (ECT). What is an appropriate postprocedure nursing intervention? Offer frequent sips of clear liquids Permit the client to sleep for four to six hours Expect long-term memory loss for a few hours Remain with client until oriented to time, place and person

Remain with client until oriented to time, place and person These clients usually awaken postprocedure 20 to 30 minutes after ECT and will appear groggy and confused. The nurse should remain with the client until the client is oriented and able to engage in self-care. The timeframe will vary and will be less than several hours.

Hematuria severe pain in low back/flank pain

Renal Calculi Although kidney stones (nephrolithiasis) may be small, passing one can be very painful. Dehydration is a major risk factor for kidney stone formation.

A client is being discharged with a prescription for warfarin. Which information is critical to be included in the nurse's discharge teaching? Take Tylenol for minor pains Use a soft toothbrush Don't increase your intake of green leafy vegetables Report any nose or gum bleeds

Report any nose or gum bleeds The client should notify the health care provider if blood is noted in stools or urine, or if any other signs of bleeding occur. This is the most important information to include in the teaching. The other options are correct but are not the priority. The client should not increase intake of dark green leafy vegetables because these are high in vitamin K. Recommending the use of a soft-bristled toothbrush and taking Tylenol for pain are also indicated for someone taking an anticoagulant.

A nurse has asked the second staff nurse to sign for a wasted narcotic, which was not witnessed by another person. What should be an appropriate initial action? Counsel the colleague about the risky behaviors Report this immediately to the nurse manager Confront the nurse about the suspected drug use Sign the narcotic sheet and document the event in an incident report

Report this immediately to the nurse manager The incident must be reported to the appropriate manager, for both ethical and legal reasons. This is not an incident that a coworker can resolve without referral to a manager.

A client with an IV antibiotic infusing is scheduled to have blood drawn at 1:00 pm for a peak antibiotic level measurement. The nurse notes that the IV infusion is running behind schedule and won't be infused until 1:30 pm. What action should the nurse take? Reschedule the laboratory test for 2:00 pm Stop the infusion at 1:00 pm and get the blood drawn Increase the infusion rate to finish it by 1:00 pm Notify the client's health care provider

Reschedule the laboratory test for 2:00 pm If the antibiotic infusion will not be completed at the time the peak blood level is scheduled to be drawn, a nurse should ask that the blood sampling time be adjusted. Typically the peak level should be drawn about 30 to 60 minutes after completion of the infusion. The infusion should not be increased because in this situation the volume of fluid to be infused is unknown; rates for IV infusions should not be increased or decreased by more than 10% of the ordered rate. Trough and/or peak levels are commonly drawn for aminoglycosides (such as vancomycin, gentamicin, and tobramycin.)

The nurse is caring for a client on peritoneal dialysis. While performing a dialysate exchange which finding(s) would alert the nurse that the client has developed an acute complication? Client sleeps throughout fluid exchange Pulse 86 and blood pressure 112/74 Catheter dressing saturated with clear fluid Respiration rate of 30 with crackles throughout the lung fields

Respiration rate of 30 with crackles throughout the lung fields The development of an increased respiratory rate with crackles bilaterally indicates fluid overload, which is an acute complication of peritoneal dialysis. The vital signs are normal. Sleeping throughout the fluid exchange is normal and indicates the client is comfortable. Clear fluid on the dressing around the catheter indicates leakage of the dialysate fluid and can be controlled by instilling less fluid with each exchange.

A client with chronic obstructive pulmonary disease (COPD) and a history of coronary artery disease is receiving aminophylline 25 mg/hour. Which finding would be associated with side effects of this medication? Flushing and headache Restlessness and palpitations Increased heart rate and blood pressure Decreased blood pressure and respirations

Restlessness and palpitations Aminophylline is a bronchodilator often used to treat symptoms of asthma, bronchitis, and emphysema. Side effects include restlessness and palpitations (it is related chemically to caffeine).

The nurse cares for a newborn with tracheoesophageal fistula (TEF) and esophageal atresia (EA.) Which nursing diagnosis is the highest priority? Risk for infection Activity intolerance Imbalanced nutrition: less than body requirements Risk for aspiration

Risk for aspiration With TEF, there's an abnormal opening between the trachea and esophagus; fluids are easily aspirated into the trachea and lungs. With EA, the esophagus ends in a blind pouch and doesn't attach to the stomach, so food can't get from the esophagus into the stomach. The 3 C's of TEF are choking, coughing and cyanosis; symptoms of EA include vomiting and drooling. The priority is to prevent aspiration and maintain an open airway. TEA with EA is a clinical and surgical emergency.

A client has expressed to the nurse feelings of hopelessness. Which of the following nursing diagnoses becomes a priority when a client expresses a sense of hopelessness? Powerlessness Risk for self-directed violence Fatigue Ineffective individual coping

Risk for self-directed violence Although the other diagnoses may apply to a client expressing a sense of hopelessness, the priority is that of self-directed violence.

A slowly progressing disease in which healthy tissue is replaced with scar tissue, which may result in the need for a transplant. Cirrhosis GERD Cholecystitis Crohns Diverticulitis Hepatitis Ulcerative colitis Pancreatitis Intestinal obstruction Peptic Ulcer

Scar tissue from cirrhosis in loss of liver function and can cause: portal hypertension, hepatic encephalopathy, gastrointestinal bleeding, infection, ascites and hepatorenal syndrome.

Drug classification for temazepam

Sedatives/Hypnotics are substances that moderate activity and excitement while inducing a calming effect (and may be anxiolytic) or substances that may induce drowsiness and sleep. Most all are Schedule IV drugs. Temazepamis a intermediate-acting benzodiazepine. They are used to provide sedation, usually prior to procedures. Selected agents are useful as anticonvulsants, skeletal muscle relaxants, adjuncts in general surgery and adjuncts for the treatment of alcohol withdrawal syndrome.

What is the drug classification for: sertraline

Selective Serotonin Reuptake Inhibitors (SSRIs) block the reabsorption (reuptake) of serotonin. They are used primarily to treat moderate-to-severe depression and chronic fatigue syndrome; they may also be used to treat premenstrual dysphoric disorder, obsessive-compulsive disorder, panic disorder, post-traumatic stress disorder, and generalized anxiety disorder.

The home health nurse makes a scheduled visit to provide wound care and finds the client lethargic and confused. The client's partner states the client fell down the stairs two hours ago. What action should the nurse take next? Place a call to the client's health care provider for instructions Reassure the client's partner that the symptoms are transient Instruct the client's partner to call the health care provider if symptoms return Send the client via ambulance to the emergency department for evaluation

Send the client via ambulance to the emergency department for evaluation This client requires immediate evaluation. Possible causes of the findings include intracranial bleeding, concussion or other head injury. A delay in treatment could result in further deterioration of the client's condition and possibly permanent injury. Home care nurses must prioritize interventions based on assessment findings that are in the client's best interest. After emergency care is implemented, the nurse should also inform the client's health care provider about the event.

The nurse is planning care for a 2 year-old hospitalized child. Which situation would the nurse expect to most likely affect the child's behavior? Unfamiliar toys and games Separation from parents Presence of other toddlers Strange bed and surroundings

Separation from parents Separation anxiety is most evident and at a peak from 6 months to 30 months of age. It is the greatest stress imposed on a toddler by hospitalization. If separation is avoided, young children have a tremendous capacity to withstand other stress.

What is the drug classification for: duloxetine

Serotonin and Norepinephrine Reuptake Inhibitors (SNRIs) block or delay the reuptake of serotonin and norepinephrine by the presynaptic nerves. The increased levels of these neurotransmitters elevates mood. They are primarily used to treat depression, but are also used to treat anxiety disorder, panic disorder and other mood disorders.

A hospitalized 8 month-old infant is receiving gentamicin. While monitoring the infant for drug toxicity, the nurse should focus on which laboratory result? Platelet counts Serum creatinine Thyroxin levels Growth hormone levels

Serum creatinine Toxicity to the aminoglycoside antibiotic gentamicin is seen in increased BUN and serum creatinine levels. Kidney damage may be reversible if the drug is stopped at the first sign of toxicity. In addition to nephrotoxicity, this medication has a Black Box warning for neurotoxicity and ototoxicity.

A client with a panic disorder has a new prescription for alprazaolam. In teaching the client about the drug's actions and side effects, which point should the nurse emphasize? Short-term relief can be expected Dosage will be increased as tolerated The medication acts as a stimulant Initial side effects often continue unchanged

Short-term relief can be expected Alprazaolam (Niravam, Xanax) is a short-acting benzodiazepine useful to quickly control panic symptoms.

What is the drug classification for: cyclobenzaprine

Skeletal Muscle Relaxants act centrally on the spinal cord or brain stem and inhibit neuronal transmission; dantrolene is the only one that acts directly on skeletal muscle. These medications are typically classified by their pharmacologic properties as either antispasticity (baclofen & tizanidine) or antispasmodic (cyclobenzaprine & carisoprodol) agents. These drugs are used to treat spasticity associated with spinal cord diseases (such as cerebral palsy, multiple sclerosis) or lesions; they may also be used as adjunctive therapy in the symptomatic relief of acute painful musculoskeletal conditions

A nurse is assessing a young child at a clinic visit for a mild respiratory infection. Koplik spots are noted on the oral mucous membranes. What should the nurse assess next? Lungs Skin Urine Sputum

Skin A characteristic sign of rubeola is Koplik spots (tiny white spots). These are found on the buccal mucosa in the mouth about a few days before the onset of the measles rash (which appears as small red, irregularly shaped spots with a bluish white center). Although the nurse should assess the child's lungs with any reports of a respiratory infection, these spots would indicate that the skin should be checked for the presence of a rash. Sometimes a complication of measles is pneumonia, but it may be a bit premature to do a sputum culture.

At the day treatment center, a client diagnosed with schizophrenia-paranoid type sits alone alertly watching the activities of clients and staff. The client is hostile when approached and asserts that the health care provider prescribes medication to control the client's mind. The client's behavior most likely indicates what associated nursing diagnosis? Impaired verbal communication related to impaired judgment Feelings of increased anxiety related to paranoia Sensory perceptual alteration related to withdrawal from environment Social isolation related to altered thought processes

Social isolation related to altered thought processes Hostile alertness and absence of involvement with people are findings supporting a nursing diagnosis of social isolation. The psychiatric diagnosis and the client's idea about the purpose of medication suggest altered thinking processes.

The nurse is preparing a handout on infant feeding to be distributed to families visiting the clinic. Which notation should be included in the teaching materials? Solid foods are to be introduced one at a time beginning with cereal Egg white is added early to increase protein intake Solid foods should be mixed with formula in a bottle A variety of ground meat should be started early to provide iron

Solid foods are to be introduced one at a time beginning with cereal Solid foods should be added, one at a time, between 4 to 6 months. If the infant is able to tolerate the food, another is then added each week. Iron-fortified cereal is the recommended first food; rice cereal is recommended due to the low risk of food allergies. Teach parents to mix the cereal flakes with either breast milk or formula. After the baby is eating cereal, pureed meat, vegetables and fruits can be introduced. Egg whites and wheat products should not be given before the baby is at least a year old because these foods are more commonly associated with allergies.

What is the drug classification for: flecanide

Some Antiarrhythmics slow down the heart (the calcium channel blockers, digoxin, and beta-blockers); other slow the heart's electrical impulses by blocking the heart's potassium channels (amiodarone, sotalol, dofetilide). They are generally classified by their effects on cardiac conduction tissue (Class IA, IB, IC, II, III, IV). Flecainide is in Class IC. They are used in the suppression of (potentially lethal) cardiac arrhythmias.

What is the drug classification for: clonazepam

Some Anticonvulsants are thought to generally depress central nervous system function. Others (such as GABA inhibitors) are thought to target specific neurochemical processes, suppress excess neuron function, and regulate electrochemical signals in the brain. Clonazepam is also categorized as a Benzodiazepine. They are primarily used to help control epileptic seizures; they are also used to treat neuropathic pain (associated with diabetes, shingles, and fibromyalgia), migraine headaches, and bipolar disorder.

What is the drug classification for: promethazine

Some Antiemetics may inhibit the chemoreceptor trigger zone in the medulla by blocking dopamine receptors; others act by decreasing the sensitivity of the vestibular apparatus. Phenergan has different effects on the brain - both antihistamine and anticholinergic activity. They are used to manage the various causes of nausea and vomiting, including surgery, anesthesia, antineoplastic and radiation therapy, and motion sickness.

What is the drug classification for: infliximab

Some Antirheumatics relieve pain (analgesics), some reduce inflammation (NSAIDs & steroids), while others control the underlying disease (disease modifying rheumatoid arthritis drugs or DMARDs & biologic drugs, like infliximab). DMARDs are used as long-term solutions to control symptoms of rheumatoid arthritis by slowing down joint destruction and preserving joint function. Biologic agents (IM or IV only) target specific components of the immune system; biologic agents may be used alone, but are often given with other DMARDs to increase the benefits and limit potential side effects.

What is the drug classification for: glimepiride

Some Oral Antidiabetic Agents (sulfonylureas and meglitinides) work by stimulating insulin release from the beta cells of the pancreas - glipizide is a sulfonylurea. Other (biguanides) improve insulin's ability to move glucose into cells, especially muscle cells. Some (thiazolidinediones) enhance insulin effectiveness in both muscle and adipose tissue. Others (alpha-glucosidase inhibitors) block enzymes that help digest starches, slowing the rise in blood sugar. These medications are used to treat type 2 diabetes mellitus.

What is the drug classification for: sumatriptan

Some Vascular Headache Suppressants(ergot derivatives) directly stimulate alpha-adrenergic and serotonergic receptors, producing vascular smooth muscle vasoconstriction. Others (5-HT1 or selective serotonin receptor agonists, such as sumatriptan) work by narrowing dilated blood vessels and blocking nerves from transmitting signals of pain to the brain. They are used for the treatment of vascular headaches (migraines and cluster headaches).

Drug classification for pantoprazole

Some of the Antiulcer Agents (PPIs) block the secretion of gastric acid by the gastric parietal cells (one example is pantoprazole). Others (H-2 receptor blockers) stop the action of histamine on the gastric parietal cells, which inhibits the secretion of gastric acid. Remember, the spelling of PPIs often end with "prazole." These drugs are used in the treatment and prophylaxis of peptic ulcer and gastric hypersecretory conditions, e.g., Zollinger-Ellison syndrome and also to manage the symptoms of gastroesophageal reflux disease (GERD).

Drug classification for donepezil

Some of these Anti-Alzheimer's Agents (cholinesterase inhibitors, like donepezil) are thought to prevent the breakdown of acetylcholine by blocking the activity of acetylcholinesterase. Others (NMDA receptor antagonists) helps regulate the activity of glutamate, a chemical involved in the processing, storage and retrieval of information. They are used to treat Alzheimer's disease.

Drug classification ropinorole

Some of these Antiparkinson Agents replenish dopamine, while others mimic the role of dopamine or block the effects of other chemicals that cause problems in the brain when dopamine levels drop. Ropinirole (Requip) is a dopamine agonist. They are used to help relieve the symptoms of parkinsonism (such as tremor or trembling in the hands, arms, legs, jaw, and face; stiffness or rigidity of the arms, legs, and trunk; bradykinesia; poor balance and coordination).

What is the drug classification for: rosiglitazone

Some of these Oral Antidiabetics (sulfonylureas and meglitinides) work by stimulating insulin release from the beta cells of the pancreas. Other (biguanides) improve insulin's ability to move glucose into cells, especially muscle cells. Some (thiazolidinediones - like rosiglitazone) enhance insulin effectiveness in both muscle and adipose tissue. Others (alpha-glucosidase inhibitors) block enzymes that help digest starches, slowing the rise in blood sugar. These medications are used to treat type 2 diabetes mellitus.

A 14 month-old had cleft palate surgical repair several days ago. The parents ask the nurse about feedings after discharge. Which lunch is the best example of an appropriate meal at this time? Peanut butter and jelly sandwich, chips, pudding, milk Hot dog, carrot sticks, gelatin, milk Soup, blenderized soft foods, ice cream, milk Baked chicken, applesauce, cookie, milk

Soup, blenderized soft foods, ice cream, milk In a child with cleft palate repair several days ago, parents should prepare soft foods. Any foods with particles that might traumatize the surgical site should be avoided. The other choices include items such as carrots, chips and cookie and are either stiff or rough types of food.

The nurse is reviewing the list of medications for a client who is scheduled for electroconvulsive therapy (ECT). Which medication does the nurse recognize as the one that will promote skeletal muscle relaxation? Methohexital (Brevital) Propofol (Diprivan) Atropine Succinylcholine (Anectine)

Succinylcholine (Anectine) ECT is performed under full general anesthesia and muscle relaxation. The sequence of administration is to give the anesthetic induction agent first, followed by the muscle relaxant. Two of the most commonly used anesthetics for this procedure are propofol and methohexital. These drugs are all well-suited for short procedures, such as ECT (which typically takes less than 10 minutes). Succinylcholine is the drug of choice for skeletal muscle relaxation in ECT, due to its brief duration of action. Atropine is an anticholinergic drug and may be used for ECT to help reduce the risk of arrhythmias and to minimize oral or other secretions and to prevent bronchial constriction.

The nurse is using the Glasgow Coma Scale to assess a client diagnosed with a traumatic brain injury. When the client does not obey verbal commands to move, which technique should the nurse use to evaluate motor function? Squeeze the trapezius muscle firmly Lift the client's arm and observe for pronation and drift Apply finger tip pressure for 10 seconds Rub the sternum with the knuckles

Squeeze the trapezius muscle firmly If the client's eyes do not open spontaneously or to sound, the nurse will assess eye opening using physical peripheral pressure, such as finger tip pressure. But a stronger, central pressure is needed to assess motor function when the client does not obey commands to move, such as the trapezius pinch. If there is no response to the trapezius pinch, the nurse can then apply pressure to the supraorbital notch to elicit a motor response. Using the Glasgow Coma Scale, the client's response on the motor scale is scored from 1 (no movement) to 6 (obeys 2-part verbal request). Rubbing the sternum with the knuckles is no longer used since it can easily bruise the soft tissue. Observing for pronation and drift is used in neurologic assessments to detect subtle arm movement in clients who can obey commands.

A child is brought to the emergency department with suspected ingestion of a toxic substance. Place the following actions in priority order by dragging and dropping the options. a. Start an IV infusion b. Reverse or eliminate the toxic substance c. Stabilize the child d. Obtain a history of the ingestion

Stabilize the child Start an IV infusion Obtain a history of the ingestion Reverse or eliminate the toxic substance The first priority is to assess the ABCs. Provide supplemental oxygen (and ventilator support, if needed). Next, an IV infusion is started using a large bore needle; this will allow for blood to be drawn for a toxicology screen as well as IV therapy. Then, a history of the ingestion is needed to guide the provider in planning care. Once the substance is identified, or there is a high index of suspicion, then treatment to reverse or eliminate the toxic substance is begun.

A client reports bilateral knee pain from osteoarthritis and is taking the prescribed nonsteroidal anti-inflammatory drug (NSAID). The nurse should instruct the client to make which lifestyle change to manage this condition? Avoid foods high in citric acid Start a regular exercise program Rest the knees as much as possible Keep the legs elevated when sitting

Start a regular exercise program A regular exercise program is beneficial in the treatment of osteoarthritis. It can restore self-esteem and improve physical functioning.

There's an order to check the pH of aspirate every four hours for a client who has a continuous tube feeding. The nurse checks the aspirate at the designated time and the pH is 8. What action should the nurse take? Continue the tube feeding as scheduled Hold the tube feeding and notify the health care provider Irrigate the tube with water and reassess pH Stop the tube feeding for about an hour and then reassess aspirate

Stop the tube feeding for about an hour and then reassess aspirate A pH of less than 4 indicates that the tube is in correct placement in the stomach, which is a highly acidic environment. A higher pH (alkaline pH) indicates either small intestine placement or even in the lungs. However, to promote accuracy, continuous tube feedings should be stopped one hour prior to obtaining aspirate, so the best response would be to stop the feeding and then recheck the aspirate. If the pH of the aspirate remains high, the health care provider should be contacted. Although not indicated in this question, the nurse should understand that several things can alter the pH of aspirate. For example, clients taking H2 receptor blockers or those diagnosed with HIV, pernicious anemia, and GERD will have higher pH concentrations.

A client who is a former actress enters the day room wearing a sheer nightgown, high heels, numerous bracelets, bright red lipstick and heavily rouged cheeks. Which nursing action is the most therapeutic in response to the client's attire? Tactfully explain appropriate clothing for the unit meetings Quietly point out to her how the other clients are dressed on the unit Straightforwardly assist the client to her room for appropriate apparel Gently remind her that she is no longer on stage

Straightforwardly assist the client to her room for appropriate apparel This action assists the client to maintain self-esteem while modifying her behavior. The other options would result in embarrassment or possibly conflict.

What is the drug classification for: trimethoprim-sulfamethoxazole

Sulfonamides are bacteriostatic and have a broad spectrum of activity against both gram-positive and gram-negative bacteria. They are typically used in the treatment of urinary tract infections and also some types of bacterial pneumonia (Pneumocystis Carinii), shigellosis, as well as some protozoal infections.

The nurse is caring for a child diagnosed with nephrotic syndrome. What finding should the nurse expect when assessing the child? Increased appetite Increased activity levels Weight loss Swelling around the eyes

Swelling around the eyes Nephrotic syndrome in children causes excess excretion of protein and retention of fluid, causing edema (around the eyes, feet, ankles) and weight gain. In this type of kidney disease, large amounts of protein are lost in the urine (proteinuria). Children may be more tired and irritable than usual.

Dyspnea, fatigue and weakness, and edema in legs, ankles and feet. Due to: Atrial fibrillation Myocardial infarction Cardiac tamponade Occlusive arterial disease hemophilia Raynaud's phenomenon Heart Failure Thrombophlebitis DIC Sickle cell disease

Systolic heart failure is when the heart muscle cannot pump/eject the blood out of the heart. Diastolic heart failure is when the heart muscles are stiff and do not fill up with blood easily. As the heart's pumping action becomes less effective, blood backs up in other areas of the body (congestive heart failure).

The nurse is teaching a client about the difference between tardive dyskinesia (TD) and neuroleptic malignant syndrome (NMS). Which statement is true about tardive dyskinesia? TD more commonly develops in children and young adults diagnosed with Tourette syndrome TD occurs within minutes of the first dose of any antipsychotic drug but it is reversible TD can occur in clients taking antipsychotic drugs longer than two years TD can easily be treated with anticholinergic drugs

TD can easily be treated with anticholinergic drugs Tardive dyskinesia (TD) is an extrapyramidal side effect that appears after prolonged treatment with antipsychotic medication. Early symptoms of TD are fasciculations of the tongue or constant smacking of the lips. Neuroleptic malignant syndrome is a more serious side effect of antipsychotic medications in which the client presents with hyperthermia, rigidity, and autonomic dysregulation (hypertension, tachycardia, tachypnea, agitation, diaphoresis). TD can be treated with the anticholinergic medication benztropine; therapy is started with a low dose and gradually increased to find the smallest amount necessary for relief. Tourette syndrome is a movement disorder, but it is unrelated to TD.

A nurse is teaching about nonsteroidal anti-inflammatory agents (NSAIDs) to a group of clients diagnosed with arthritis. The nurse should emphasize which of these actions to minimize a side effect of these drugs? Continue to take aspirin for short-term pain relief Use alcohol in moderation when driving or operating heavy machinery Take the medication after meals or with food Report joint stiffness in the morning

Take the medication after meals or with food Taking NSAIDs after meals or with food should help to minimize gastric irritation. The client should also take the medication with a full glass of water and remain in an upright position for 15 to 30 minutes after administration. Clients should be cautioned to avoid concurrent use of aspirin or alcohol with these medications to minimize possible gastric irritation; three or more glasses of alcohol per day may increase the risk of GI bleeding.

What is the drug classification for: doxycycline

Tetracyclines exert their bacteriostatic effect by inhibiting protein synthesis in bacteria. They are broad spectrum anti-infectives. They are typically used in the treatment of respiratory tract infections, acne and skin infections, genital infections (syphilis, Chlamydia), urinary tract infections, Lyme disease, mycoplasmal infections and rickettsial infections and the infection that causes stomach ulcers (helicobacter pylori).

Use the cursor to indicate the correct hand placement used to relieve an obstructed airway on a conscious person who is obese.

The Heimlich is usually performed on a conscious person who is in an upright position. In the event that the rescuer cannot encircle the victim's belly to reach the space between the navel and the xiphoid notch, the hands should be placed around the chest so that the fists lie at the mid-nipple line, and a chest thrust should be used to relieve the obstruction. The chest area has a smaller circumference than the abdomen in many people who are obese so it is often more accessible on a larger person; the same technique would be used on a pregnant female to avoid harm to the fetus. Should the person become unconscious, the rescuer would begin chest compressions.

Using a vibrating fork, the nurse will perform the Rinne test to assess the patient's hearing. Where will the nurse place the tuning fork to assess for bone conduction of sound?

The Rinne test helps distinguish between conductive and sensorineural hearing loss. To assess for bone conduction of sound, the nurse holds the tip of a vibrating tuning fork against the mastoid bone. Normally, air conduction is audible longer than bone conduction, but the reverse is true for someone with conductive hearing loss.

The charge nurse assigns the unlicensed assistive person (UAP) to measure vital signs. Despite written and verbal instructions not to take the blood pressure on the left arm of a client who is 48 hours postmastectomy, the charge nurse later observes a blood pressure cuff on that client's left arm. Which of these statements is accurate about this situation? The charge nurse has no accountability for this situation The UAP is covered by the charge nurse's license The charge nurse did not appropriately make assignments The UAP is responsible for following instructions given by the charge nurse

The UAP is responsible for following instructions given by the charge nurse The UAP is responsible for carrying out the activity correctly once directions have been clearly communicated both verbally and in writing. The licensed nurse retains accountability for the delegation and is responsible for tasks assigned to the UAP. However, the UAP is not covered under the nurse's license.

What is the drug classification for: clarithromycin

The action of Macrolides is mainly bacteriostatic. They are used in the treatment of various systemic and local bacterial infections of the respiratory tract, gastrointestinal tract, and soft tissues; they are also effective in treating severe acne and sexually transmitted infections. They are used prophylactically in the prevention of whopping cough and the prevention of endocarditis in dentistry.

A child has been tentatively diagnosed with Graves' disease (hyperthyroidism). Which of these findings noted on the initial nursing assessment requires quick intervention by the nurse? A report of the sudden onset of irritability in the past two weeks The appearance of eyeballs that appear to "pop" out of the client's eye sockets A comment by the client: "I just can't sit still." A report of 10 pounds weight loss in the last month

The appearance of eyeballs that appear to "pop" out of the client's eye sockets Exophthalmos or protruding eyeballs is a distinctive characteristic of Graves' disease. It can result in corneal abrasions with severe eye pain or damage when the eyelid is unable to blink down over the protruding eyeball. Eye drops or ointment may be needed.

There is an order to insert a urinary catheter. The client is an adult female. The nurse slips the catheter approximately 4-5 inches (10-12 cm) into an opening, but no urine is obtained. What is the most probable reason for this outcome? No urine is present in the bladder The catheter is not inserted far enough into the canal The catheter is located in the vaginal canal The bladder is overdistended without the ability to empty

The catheter is located in the vaginal canal For an adult female, a urinary catheter is inserted about 2-3 inches (5-7 cm) in the urinary meatus until the urine flow begins. If urine does not flow, the catheter can be rotated gently and carefully inserted a bit further. When a catheter is inserted 4-5 inches (10-12 cm) with no urine return, the catheter is probably in the vaginal canal.

The nurse is using the image of descending colostomy to explain and clarify information in teaching the client about a new colostomy. Based on this image, which of the following statements about the consistency of the drainage is correct? The feces are mushy (liquid to semiformed) The feces are liquid to semiliquid and the discharge is often irritating to the skin around the stoma The feces are semiformed to formed The feces have a normal, formed consistency

The feces are semiformed to formed This is an image of a descending colostomy. The feces will be semiformed to formed because much of the water has already been absorbed.

A 14 month-old infant is brought to the emergency department with irritability, lethargy over two days, dry skin, and increased pulse. What additional question would be most important for the nurse to ask in assisting the provider with determination of the diagnosis? Use of daycare Reverse of sleep-wake cycles Change in eating habits The number of wet diapers in the past two days

The number of wet diapers in the past two days Based on these clinical findings, the nurse might suspect that the infant is dehydrated. Asking about the number of wet diapers would assess for decreased urine output, a key finding in dehydration. Asking about increased concentration of the urine would also be appropriate. The other questions, while appropriate, would not provide the most helpful diagnostic information.

The nurse is assessing the uterine fundus of a client who delivered a healthy neonate 10 hours ago. Identify the area where the nurse would expect to feel the fundus.

The uterus should be felt at the level of the umbilicus from about 1 to 24 hours after birth.

A client is about to undergo a plaster cast application. Prior to the cast application, the nurse should be sure to include what teaching point in the discussion? The wet cast should be handled with the palms of hands until fully dry The cast material will be dipped several times into the tepid water The casted extremity will be placed on a cloth-covered surface The cast should be covered with cotton material until it fully dries

The wet cast should be handled with the palms of hands until fully dry The cast will be handled with the palms of the hands and need to be lifted at two points of the extremity while it is drying. This will prevent stress of the injury and pressure indentation areas on the cast. The cast should be uncovered and be placed on a firm surface.

Caused by a bacterial infection, toxins and viruses, this condition can cause inflammation, cirrhosis or cancer of the liver. Cirrhosis GERD Cholecystitis Crohns Diverticulitis Hepatitis Ulcerative colitis Pancreatitis Intestinal obstruction Peptic Ulcer

There are several causes of inflammation of the liver, but hepatitis is usually caused by a virus. Hepatitis can heal on its own with no significant consequences or it can progress to scarring of the liver (cirrhosis).

Excessive bruising, swollen and painful joints and lengthy bleeding. Due to: Atrial fibrillation Myocardial infarction Cardiac tamponade Occlusive arterial disease hemophilia Raynaud's phenomenon Heart Failure Thrombophlebitis DIC Sickle cell disease

There are several types of hemophilia. All types can cause abnormal or exaggerated bleeding and poor blood clotting. Common sites for bleeding are the joints, muscles and gastrointestinal tract.

The nurse is talking with the family of an 18 month-old toddler who is newly diagnosed with retinoblastoma. Which point is a priority when discussing this diagnosis with the parents? Suggest that total blindness may follow surgery Prepare them for their child's permanent disfigurement There is a need for genetic counseling Inform them that even aggressive treatment is usually ineffective

There is a need for genetic counseling Aggressive treatment of retinoblastoma can be effective. If the tumor does not respond to chemotherapy and/or radiation therapy, the eye may need to be removed; however, that does not necessarily mean the child will be permanently disfigured. Regardless, the oncologist is the person who will discuss treatment options and anticipated outcomes with the parents. The parents should be prepared for the effects of the cancer on their child, but they should also understand that retinoblastoma is a rare cancer that runs in families and there is a high risk for future offspring to be affected.

Drug classification for chlorothiazide

Thiazide Diuretics are derived from a chemical called benzothiadi(A)zene. They work in the distal convoluted tubule by decreasing the kidney's reabsorption of sodium and chloride (which results in increased urine production) and they also help dilate blood vessels. They are used alone or in combination with loop diuretics in the treatment of hypertension or edema due to heart failure or other causes.

Skin redness, swelling, warmth and tenderness over a vein. Due to: Atrial fibrillation Myocardial infarction Cardiac tamponade Occlusive arterial disease hemophilia Raynaud's phenomenon Heart Failure Thrombophlebitis DIC Sickle cell disease

Thrombophlebitis is a swollen or inflamed vein due to a blood clot.

red rash becomes ring-shaped with a red-colored, raised border and clearer center

Tinea corporis Tinea corporis is a contagious, superficial fungal infection that lives on, not in, the skin. It is also known as ringworm

A progressive exercise routine is the best therapy for the client with chondromalacia patellae. True False

True Although this degenerative disorder cannot be cured, it can be traced to a trauma or repeated stress. Selective strengthening of the inner portion of the quadriceps muscle will help normalize the tracking of the patella. Cardiovascular conditioning can be maintained by stationary bicycling, pool running or swimming.

Clients should remove all metal objects, including any and all piercings, prior to a magnetic resonance imaging (MRI) scan. True False

True An MRI uses powerful, magnetic fields and radiofrequency energy to create clear pictures of internal body structures. Because of these magnetic fields, clients must remove all metal objects. Clients with shrapnel, a pacemaker or any surgically implanted joints may not be able to have this test.

difficulty swallowing ear pain fever and chills headache sore throat

Tonsillitis Tonsillitis is an inflammation of the tonsils, due to either viral or bacterial infections or immunologic factors. Findings are similar to pharyngitis (sore throat).

This condition may be caused by gallstones, chronic alcohol use, infections, medications and trauma. Cirrhosis GERD Cholecystitis Crohns Diverticulitis Hepatitis Ulcerative colitis Pancreatitis Intestinal obstruction Peptic Ulcer

Treatment for pancreatitis includes IV fluids, antibiotics and pain relievers; many clients will require an endoscopic retrograde cholangiopancreatography (ERCP) to examine the bile and pancreatic ducts.

A client has chronic renal failure and is being treated at home. During weekly home visits, which factor is the most accurate indicator of fluid balance? Trends in daily weights Skin turgor over at least two areas of the body Changes in mucous membrane moistness Difference between intake and output

Trends in daily weights The most accurate indicator of changes in fluid balance is the daily weight. A 1-kilogram (or 2.2 pounds) of weight gain is equal to approximately 1000 mL of retained fluid. Other options are considered as part of data collection for fluid balance, but they are not the most accurate indicators of fluid balance.

What is the drug classification for: amitriptyline

Tricyclic Antidepressants inhibit the nerve cell's ability to reuptake serotonin and norepinephrine, resulting in increased levels of these neurotransmitters in the brain. They also block the action of acetylcholine and histamine (which causes many of the side effects of these meds). They are used to relieve depression and may also be used to treat obsessive compulsive disorder and bedwetting. Off-label uses include panic disorder, bulimia, and chronic pain (migraine, diabetic neuropathy & post herpetic neuralgia).

The patient using a cane should hold it on his strong side and move the can at the same time as the weaker leg. True or false

True A cane can provide stability when walking. The standard cane is fine if it's only needed for balance but if the patient needs the cane to bear weight, an offset can with four tips might be best.

Foam dressings are ideal for draining full-thickness wounds. True False

True A foam dressing with a fluid-proof backing is a good choice for treating full thickness wounds. This dressing will absorb moderate amounts of drainage and cushion the wound site. A secondary dressing, such as Kerlix, may be needed to secure the primary dressing in place.

A priority for septic shock is to treat the cause of infection. True False

True Along with fluid replacement and medications to increase cardiac output, this type of shock must be treated with the appropriate anti-infective agent(s).

Glucose is the only fuel used by brain cells. True or false

True Although most energy needs could be met by fats and proteins, the brain requires carbohydrates, specifically glucose. Neurons need a constant supply of glucose since they cannot store it.

Liquid medications are best for patients who are on suicide precautions True or false

True Although the nurse can inspect the client's mouth after giving oral medications in tablet form, medications given in oral liquid form can prevent the client from hiding and hoarding medications.

The infant born with bladder exstrophy will most likely experience additional congenital defects. True False

True Bladder exstrophy is a rare congenital birth defect where the bladder is turned "inside out" and exposed outside of the body. The infant will most likely have many other problems, including clubfoot or major deformity of a lower extremity, hip dislocation, abdominal wall defects (such as inguinal hernias) and epispadias.

Acute myocardial infarction (MI) is the most common cause of cardiogenic shock. True False

True Cardiogenic shock typically develops following an acute MI, especially a ST-segment elevation MI (STEMI). However, cardiogenic shock can result from any cardiac dysfunction that causes acute myocardial ischemia.

An abnormal immune reaction to gluten damages the small intestine in people with celiac disease. True False

True Celiac disease is an inherited autoimmune disease in which the lining of the small intestine is damaged from eating gluten and other proteins found in wheat, barley, rye and possibly oats. Management includes adopting a gluten-free diet.

Clients diagnosed with systemic lupus erythematosus (SLE) should avoid exposure to sunlight and ultraviolet light. True False

True Clients with SLE often experience photosensitive rashes. Exposure to sunlight can also cause migraine headaches, nausea and joint pain.

The first sign of mild hypothermia is shivering. True False

True Cold-induced shivering is stimulated when body temperatures drop below the set point (which is governed by the hypothalamus). Shivering is activated to raise the body temperature.

Disaster triage differs from route emergency department triage True or False

True Disaster triage categories range from most urgent (first priority), urgent, nonurgent (the walking wounded), and dead/catastrophic/coma.

An esophageal manometry may be ordered to confirm dysphagia or gastroesophageal reflux (GERD). True False

True During an esophageal manometry a thin, pressure-sensitive tube is passed into the esophagus. As the client swallows, the tube measures the pressure of the muscle contractions. This test is used to determine the cause of dysphagia, to evaluate for signs of GERD or to evaluate chest pain that may be coming from the esophagus.

The mitral valve should open during diastole. True False

True During diastole (ventricular filling), the mitral valve is open to allow blood to flow from the atria to the ventricles.

A client's hemodynamic status (blood pressure) is continuously monitored during hemodialysis. True False

True During hemodialysis, blood is diverted from the body; if it is not replaced at the proper rate and osmolality, complications such as shock could develop.

The ELISA test is used to detect antibodies in the blood. True False

True ELISA stands for enzyme-linked immunosorbent assay (it is also known as EIA, or enzyme immunoassay). This laboratory test is used to detect antibodies in the blood. It is used for clinical diagnosis, screening blood products and testing individuals who believe they may have been exposed to other infectious substances or viruses, such as HIV.

Management of mild hyperparathyroidism includes increasing oral fluid intake to prevent the development of kidney stones. True False

True Extra parathyroid hormone (PTH) results in hypercalcemia. There is also increased calcium in the urine, which may cause kidney stones. If the client's serum calcium levels are only slightly elevated, s/he should drink plenty of fluids to minimize the risk of developing kidney stones.

Paradoxical chest wall movement is a key assessment finding in the client with a flail chest. True False

True Flail chest results when two or more rib fractures occur in two or more places, causing the flail segment to separate from the rib cage. It often occurs from blunt trauma associated with accidents. Paradoxical respirations are the inward movement of a part of the thorax during inspiration and the outward movement during expiration. Clients also have severe chest pain, dyspnea and possible tachycardia and hypotension with flail chest.

Findings of a partial thickness burn include moist, red skin that's painful to the touch. True False

True For a superficial partial thickness burn, the skin is red, moist and has blisters; the skin is very painful to the touch. There may be no blisters with a deep partial thickness burn.

Anemia, thrombocytopenia and acute renal failure are the classic findings of acute hemolytic-uremic syndrome (HUS). True False

True HUS typically develops after a gastrointestinal infection involving Escherichia coli bacteria (E coli 0157:H7). The bacterium produces toxic substances that destroy red blood cells, resulting in anemia. HUS often begins with vomiting and bloody diarrhea. It is one of the most common causes of acute kidney failure in children.

The most curable form of cancer is Hodgkin disease. True False

True Hodgkin disease is a malignancy of the of lymph tissue (found in the lymph nodes, spleen, liver and bone cancer).

Protein is the body's only source of nitrogen True or false

True In a healthy did, a nitrogen balance is achieved when dietary intake is balanced by excretion of urea wastes. A negative nitrogen balance occurs if excretion is greater than the nitrogen content of the diet, as seen in burns, infections, injuries, fever or starvation.

Cyanotic heart defects are more dangerous than acyanotic defects. True False

True In acyanotic (or 'pink') heart defects, blood is shunted from left to right within the heart, so oxygenated blood is recirculated. In contrast, cyanotic (or 'blue') heart defects involve a shunt that recirculates some venous blood, thus starving the body tissues of needed oxygen.

During systole, the pulmonic valve is open but the tricuspid valve is closed. True False

True In systole, the ventricles contract. So the right ventricle should be pushing blood through the pulmonic valve, not backward through the tricuspid.

Patients with insomnia either have difficulty falling asleep or staying asleep. True or false

True Insomnia is a sleep disorder. There are 2 types of insomnia: primary and secondary.

Ischemic strokes are the most common type of stroke. True False

True Ischemic strokes account for about 87% of all stroke cases and are caused by an obstruction within a blood vessel supplying blood to the brain (either a thrombosis or embolism).

The nurse will give Rh immune globulin (RhoGAM) to a Rh negative women after a miscarriage (spontaneous abortion). True or false

True RhoGam is administered to Rh negative women after any possible exposure to fetal blood such as after each ectopic pregnancy, miscarriage, abortion or amniocentesis, RhoGAM will be given to help prevent problems associated with incompatible blood types in future pregnancies.

Sarcomas are cancers that begin in the cells of the immune system. True False

True Sarcomas are cancers that begin in connective or supportive tissue, e.g., bone, cartilage, fat or muscle. Lymphomas and myelomas are cancers that begin in the cells of the immune system.

Apnea is a symptom of any number of different etiologies. True False

True Some of the more common causes of apnea include apnea of prematurity, obstructive sleep apnea, and apnea secondary to head trauma, infections or toxins. Apnea is an unexplained episode of cessation of breathing for 20 seconds or longer, or a shorter respiratory pause associated with bradycardia, marked hypotonia, and cyanosis and/or pallor.

Standard precautions also includes respiratory/cough etiquette True or False

True Standard precautions are used to reduce the risk of transmission of bloodborne and other pathogens from both recognized and unrecognized sources. Respiratory hygiene/cough etiquette is now considered part of standard precautions

A baby tapped briskly on the bridge of the nose will close both eyes. True or false

True Tapping on the glabella (flat bone between the eyebrows) causes a neurologically healthy baby to close both eyes. This is referred to as the glabellar reflex.

The aorta carries oxygenated blood. True False

True The aorta leaves the left ventricle, which contains oxygen-rich blood from the lungs.

The heart muscle (myocardium) gets oxygenated blood via the aorta. True False

True The aorta's first two branches are the right and left coronary arteries, which bring blood to the myocardium.

The arteries stay open when cut. True False

True The arteries contain a fibrous outer layer (tunica adventitia) that is stiff enough to hold them open when cut. The veins collapse when cut.

The sequence of actions in the initial assessment for trauma care is: airway, cervical spine stabilization, breathing and then circulation. True False

True The cervical spine must be simultaneously stabilized when assessing the airway, and before breathing and circulation are assessed.

Clients should fast 8 to 12 hours before having blood drawn for lipid blood tests. True False

True The client should fast for at least 8 to 12 hours before a lipid panel blood draw; the client can drink clear liquids. Lipids include cholesterol, triglycerides, high-density lipoprotein (HDL) and low-density lipoprotein (LDL).

Medical management of hypovolemic shock includes rapid fluid replacement. True False

True The essential treatment for clients with hypovolemic shock is to restore fluid volume and blood pressure. The client may also need medications to help increase cardiac output and mean arterial pressure, such as dobutamine (Dobutrex) and norepinephrine (Levophed).

The 3 elements of radiation protection are time, duration, and shielding True or False

True The farther away people are from a radiation source, the less their exposure; as a rule, if you double the distance, you reduce the exposure by a factor of four. The amount of radiation exposure typically increases with the time people spend near the source of radiation

The mitral (bicuspid) valve is an atrioventricular valve. True False

True The mitral valve controls flow between the left atrium and left ventricle.

The lower legs and upper back are the most common sites for melanoma in fair-skinned women. True False

True The most common sites for melanoma in fair-skinned women (including fair-skinned Hispanics) are the lower legs and upper back. For fair-skinned men, the most common site for melanoma is the upper back. Melanomas in dark-skinned people often appear in the mouth, palms of the hands, soles of the feet and under the nails.

Common issues on the first postpartum day include afterpains and episiotomy discomfort and swelling. True or False

True The nurse should provide information about interventions that will help the new mother cope with the common physical and emotional changes she is experiencing. For example, the client can apply ice or a cold pack to the perineum and use a gentle squeeze of warm water for cleansing after voiding.

Common issues on the first postpartum day include afterpains and episiotomy discomfort and swelling. True or false

True The nurse should provide information about prevention that will help the new mother cope with common physical and emotional changes she is experiencing. For example, the patient can apply ice or a cold pack to the perineum and use a gentle squeeze of warm water for cleaning after voiding.

The septum is a valve between the two ventricles. True False

True The septum is a wall; normally there should be no opening between the ventricles.

A clean catch urine specimen can be used to detect the presence of blood cells, protein and bacteria. True False

True The urine obtained from a clean catch urine specimen can be used for a variety of tests, including urinalysis, cytology and urine culture. The nurse will need to instruct (or reinforce teaching to) the client about the correct procedure to obtain a clean catch specimen.

There are two veins called venae cavae. True False

True There is an anterior (superior) vena cava and a posterior (inferior) vena cava; both bring blood from the body back to the heart.

Complications of orthopedic surgery include deep vein thrombosis, fat embolism, pulmonary embolism, thrombophlebitis, hemorrhage and wound infection. True False

True These are all potential complications of orthopedic surgery.

Sudden onset of fever, headache, photosensitivity and stiff neck are common findings of meningitis. True False

True These are some of the classic findings of meningitis, which can occur quickly or over several days after exposure. However, infants may present with high fever, constant crying, excessive sleepiness or irritability and poor feeding.

The basic metabolic panel (BMP) is a group of 8 specific tests used to determine the status of the kidneys, blood sugar, electrolyte and acid/base balance. True False

True This commonly-ordered test includes: glucose, calcium, sodium, potassium, carbon dioxide (CO2), chloride, blood urea nitrogen (BUN) and creatinine. A related test is the comprehensive metabolic panel (CMP), which consists of 14 specific tests.

In the TNM classification (staging) system, the 'M' stands for metastasis. True False

True This staging system signifies the extent or severity of a client's cancer. The T stands for the extent of the primary tumor and N is for lymph node involvement.

The human papillomavirus 9-valent vaccine, recombinant (Gardasil®9) can prevent cervical cancer. True False

True This vaccine, which is given to adolescent boys and girls, can prevent diseases cause by the human papillomavirus (HPV). This includes several different types of cancer and genital warts.

An elderly client is more sensitive to the active substance in a transdermal patch than a younger adult. True or False

True Transdermal medication application requires adequate tissue perfusion to absorb and distribute the medication. Skin permeability varies based on hydration, temperature, age (the skin of babies and the elderly is more permeable than that of other age groups) and ethnicity (the skin of Caucasians is more permeable than that of African Americans). Therefore an older client would be more sensitive to medication administered this way than a younger adult.

Anticonvulsants and skeletal muscle relaxants are used in the management and treatment of trigeminal neuralgia. True False

True Trigeminal neuralgia is one of more common causes of chronic and excruciating facial pain. Anticonvulsants help to decrease pain impulses and produce pain relief. The muscle relaxant baclofen may be used as an adjunct to anticonvulsants.

The birthing client should be placed on her left side if late decelerations are noted. True False

True Turning the client on her left side will increase placental perfusion. Other interventions include administering oxygen to increasing fetal oxygenation and increase IV fluids to improve intravascular volume.

A woman is more likely to develop type 1 osteoporosis if she is postmenopausal, smokes, drinks alcohol and is not taking hormone replacement therapy. True False

True Type 1 osteoporosis is related to decreased estrogen levels in postmenopausal women. Risk factors include a family history of osteoporosis, low body weight, smoking and drinking a large amount of alcohol.

Many clients are able to manage type 2 diabetes through diet and exercise. True False

True Type 2 diabetes can often be managed with proper diet and exercise. Some clients also require additional treatment with oral hypoglycemic agents, like metformin.

Peripheral veins contain valves that keep the blood flowing back to the heart. True False

True Unlike arteries, veins do not contain an elastic membrane lining; they instead rely on valves to keep blood flowing in a single direction, back to the heart.

Ventilator support in infants can lead to bronchopulmonary dysplasia (BPD). True False

True Ventilators use pressure to blow air into the airways and lungs. Although ventilator support can help premature infants survive, the machine's pressure may irritate the infant's lungs. Other causes of BPD include high levels of oxygen therapy, infections and heredity.

Newborns are fitted with tamperproof security sensors during their stay at the hospital True or False

True Wearing a tamper proof safety device reduces the risk of abduction. The sensor shows the location of the infant and the security system can activate other devices (such as cameras, door locks, public address systems, sirens, and other alarms) in the event of an attempted abduction

X-rays should be taken both before and after a closed reduction of a fracture. True False

True X-ray images are necessary to first show where the bone should be moved and afterward to show whether it is positioned for ideal healing.

External beam radiation is used to damage cancer cell DNA. True False

True Using high-energy radiation, the DNA of the cancer cells is damaged and the cell will die. X-rays, gamma rays and charged particles are types of radiation used for cancer treatment. Internal radiation therapy (brachytherapy) is another treatment for cancer.

Many clients experience some soreness and shoulder pain following a diagnostic laparoscopy. True False

True A laparoscopy involves injecting carbon dioxide or nitrous oxide gas into the abdominal cavity to expand the area for better viewing. Many people experience shoulder pain for a few days after the procedure because the gas irritates the diaphragm, which shares some of the same nerves as the shoulder.

A 7yo child can be taught to self-catherize him or herself. True or false

True Children with neurogenic bladder complications or spina bifida may successfully learn self-catherization as young as 6 or 7 yo. Training initially starts with performing the procedure using a doll.

Anticholinesterase inhibitor medications should be given 30 to 60 minutes before a meal for clients diagnosed with myasthenia gravis. True False

True Clients diagnosed with myasthenia gravis experience progressive muscle weakness. To minimize the risk of aspiration and to facilitate chewing and swallowing, anticholinesterase inhibitors, such as pyridostigmine (Mestinon), should be taken before meals.

Assistive devices are used when a caregiver is required to lift more than 35 lbs/15.9 kg true or false

True During any patient transferring task, if any caregiver is required to lift a patient who weighs more than 35 lbs/15.9 kg, then the patient should be considered fully dependent, and assistive devices should be used for transfer

A nurse should ask a client with emphysema to bear down during the insertion of a non-tunneled central venous catheter (CVC). True or False

True Intravenous pressure must exceed atmospheric pressure during CVC insertion to prevent air from entering the catheter and traveling to the heart and lungs. Any client, regardless of his or her diagnosis, should be asked to bear down during CVC insertion.

Nurses must be aware of their own cultural values and beliefs to avoid biases when providing care to clients. True or false

True Nurses must be aware of and sensitive to the cultural needs and beliefs of their clients and their families, as well as themselves. Nurses must engage in self-awareness and critical reflection of their own beliefs to provide culturally sensitive care to all clients. This is especially true when caring for clients with mental health disorders because biases can hinder the therapeutic relationship.

The nurse will give Rh immune globulin (RhoGAM®) to a Rh negative women after a miscarriage (spontaneous abortion). True or False

True RhoGAM® is administered to Rh negative women after any possible exposure to fetal blood, such as after each ectopic pregnancy, miscarriage, abortion or amniocentesis. RhoGAM® will be given to help prevent problems associated with incompatible blood types in future pregnancies.

A baby tapped briskly on the bridge of the nose will close both eyes. True or False

True Tapping on the glabella (flat bone between the eyebrows) causes a neurologically healthy baby to close both eyes. This is referred to as the glabellar reflex.

One of the initial signs of a stroke is weakness on one side of the body. True False

True The American Stroke Association lists this as one initial warning sign of a stroke, along with sudden confusion, sudden trouble speaking or understanding, sudden trouble seeing or sudden headache.

The nonrebreather mask with reservoir bag can deliver oxygen concentrations near 100%. True False

True The nonrebreather mask delivers the highest percentage of oxygen of any of the high flow systems. It should be used only in medical emergencies, for a relatively short time (6 to 8 hours); any longer and the client risks pulmonary oxygen toxicity.

Serum bilirubin and urobilinogen measure how well the liver and gallbladder are functioning. True False

True The prefix "bil-" refers to bile, a product of the liver that is stored in the gallbladder and excreted into the small intestine. Testing for bilirubin in the blood helps identify liver disease and any obstruction of the gallbladder or bile ducts. Urobilinogen is a breakdown product of bilirubin and can be detected with a urinalysis.

Clients increase their risk of adverse effects if they use herbal supplements along with prescription medications. True or False

True There is an inherent risk of adverse effects when a client combines herbal agents with prescription drugs. Because herbal remedies have drug actions of their own, the client taking prescription drugs should not take herbal supplements or over-the-counter drugs until they have discussed these with their health care providers.

The water deprivation test is used to diagnose diabetes insipidus. True False

True This test is used to determine the cause of polydipsia and polyuria - central diabetes insipidus (DI), nephrogenic DI or psychogenic polydipsia.

When the fetus is active, its heart rate should increase by about 15 beats per minute. True or false

True When the fetus is active, its heart rate will accelerate by about 15 beats per minute above the baseline. Average fetal heart rate is about 130 ppm when near term.

The client diagnosed with syndrome of inappropriate antidiuretic hormone (SIADH) should be placed on seizure precautions. True False

True With SIADH, hypersecretion of antidiuretic hormone (ADH) causes hyponatremia. As serum sodium drops, extra water enters cells and causes them to swell. Convulsions, shock, coma and death may occur with cerebral edema and increased brain cell volume.

After a myocardial infarction, a client is placed on a sodium-restricted diet. When the nurse is teaching the client about the diet, which meal plan would be the most appropriate to suggest? Turkey 3 oz., a fresh sweet potato, 1/2 cup fresh green beans, milk, and an orange Broiled fish 3 oz., a baked potato, ½ cup canned beets, an orange, and milk Canned salmon 3 oz., fresh broccoli, a biscuit, tea, and an apple A bologna sandwich, fresh eggplant, 2 ounces of fresh fruit, tea, and apple juice

Turkey 3 oz., a fresh sweet potato, 1/2 cup fresh green beans, milk, and an orange Canned fish and vegetables and cured meats are high in sodium. The correct meal does not contain any canned fish and/or vegetables or cured meats. A low-sodium diet is 2 grams of sodium as compared to a normal sodium diet of 4 grams of sodium.

A client is scheduled for an intravenous pyelogram (IVP). Which information from the client's history indicates the greatest potential hazard for this test? Type 2 diabetic taking metformin (Glucophage) Urge incontinence Constipation Hypertension

Type 2 diabetic taking metformin (Glucophage) The elderly and those with diabetes and/or heart disease are at greater risk of developing kidney failure following administration of the dye. To avoid this complication, kidney function should be tested (creatinine). If kidney damage occurs, metformin can cause additional problems, which is why clients must stop taking metformin prior to and for 48 hours after an IVP. Feces or gas in the colon can interfere with the IVP, but these are not necessarily a hazard.

Inflammation in the lining of the colon leads to abdominal discomfort and blood or pus in diarrhea. Cirrhosis GERD Cholecystitis Crohns Diverticulitis Hepatitis Ulcerative colitis Pancreatitis Intestinal obstruction Peptic Ulcer

Ulcerative colitis is a disease that causes inflammation and sores (ulcers) in the lining of the rectum and colon (large intestine).

The nurse attends an interdisciplinary meeting on the topic of fall prevention. What specific tactics can be used to reduce falls in health care settings? (Select all that apply.) Use "low beds" for at-risk clients Raise side rails Install and use bed alarms Identify vulnerable clients Use a "two to transfer" policy Regularly reorient clients

Use "low beds" for at-risk clients Install and use bed alarms Identify vulnerable clients Use a "two to transfer" policy Regularly reorient clients Fall prevention involves managing a client's underlying fall risk factors and then implementing strategies to reduce falls. Using restraints, including side rails, can actually increase the risk of fall-related injuries and deaths. Clients with neurocognitive disorders cannot process the information we provide when we attempt to reorient them to our reality. The other techniques listed are used (in combination) to help prevent falls in health care facilities.

Which nursing practice best reduces the chance of communication errors that might otherwise lead to negative client outcomes? Use standardized forms for client handoffs Speak using a professional tone on the telephone Maintain respectful working relationships with all staff Document nursing care at the end of the shift

Use standardized forms for client handoffs Standardized forms improve information for communication between caregivers. Most problems/poor outcomes involve some element of poor communication. The options of keeping good working relationships and using a professional tone of voice on the phone is good practice but not as useful for minimizing the chance of errors. Documenting at the end of the shift is incorrect practice and may lead to poor communication, as critical findings may be forgotten and not recorded.

The order is for vancomycin 2 grams every 12 hours IV. The pharmacy sends the medication mixed in a 200 milliliter (mL) bag with directions to infuse it over 1.5 hours. The nurse will use an infusion pump to deliver the medication. What is the infusion rate for the vancomycin (Vancocin)? (Round to the nearest whole number and write only the number.)mL/hour.

Vancomycin must be run with an infusion pump. 200 mL/1.5 hours = X mL/hour X = 133 mL/hour

Drug classification for epinephrine

Vasopressors are potent vasoconstrictors, producing a rise in blood pressure (increase in mean arterial pressure). They are used to control blood pressure in hypotensive states, such as (cardiogenic, septic) shock, drug reactions, spinal anesthesia. They can also be used to prolong anesthesia and to treat certain heart rhythm problems, including cardiac arrest.

The client is admitted to the emergency department with hypertensive crisis. Which finding requires immediate action by a nurse? Weakness in left arm Jugular vein distension Crackles at the lung bases Lower extremity pitting edema

Weakness in left arm In a client who has uncontrolled hypertension, weakness in the extremities is a sign of cerebral involvement with the risk for cerebral infarction or stroke. Cerebral infarctions account for about 80% of the strokes in clients with hypertension. The remaining three choices indicate mild fluid overload, which may be associated with heart failure due to hypertensive crisis, but they are not medical emergencies. Crackles throughout the lung fields with acute onset of dyspnea and orthopnea would indicate acute pulmonary edema, which would also be considered a medical emergency requiring immediate action.

During the change-of-shift report, the assigned nurse notes a client of the Catholic religion is scheduled to be admitted for the delivery of a ninth child. Which comment made by a nurse indicates an attitude of prejudice? a. "All those people indulge in large families!" b. "I guess she doesn't understand how to use birth control." c. "I wonder who is paying for this trip to the hospital?" d. "I think she needs to go to the city hospital."

a. "All those people indulge in large families!" Prejudice is a hostile attitude toward individuals simply because they belong to a particular group presumed to have objectionable qualities. Prejudice refers to preconceived ideas, beliefs, or opinions about an individual, group or culture that limit a full and accurate understanding of the individual, culture, gender, race, event or situation.

A client with a diagnosis of depression has recently been acting suicidal and is now more social and energetic than usual. Smiling, the client tells a nurse, "I've made some decisions about my life." What should be the nurse's initial response? a. "Are you thinking about killing yourself?" b. "You need to discuss your decisions with your therapist." c. "You've made some decisions. You seem happier now. " d. "I'm so glad to hear that you've made some decisions."

a. "Are you thinking about killing yourself?" Sudden mood elevation and energy may signal a higher risk of suicide. The nurse must validate suicidal ideation as a beginning step in evaluating degree of the risk. A more direct approach is used because a threat to harm exists in the client.

A parent calls the hospital hot line and is connected to the triage nurse. The caller states: "I found my child with odd stuff coming from the mouth and an unmarked bottle nearby." Which of these comments would provide the best information to help the nurse to determine if the child has swallowed a corrosive substance? a. "Ask the child if the mouth is burning or throat pain is present." b. "Has the child had vomiting, diarrhea or stomach cramps?" c. "Take the child's pulse at the wrist and see if the child has trouble breathing lying flat." d. "What color are the child's lips and nails and has the child voided today?"

a. "Ask the child if the mouth is burning or throat pain is present." Local irritation of tissues indicates a corrosive poisoning. The other comments may be helpful in determining the child's overall condition. However, the question concerns evaluation for ingestion of a caustic substance.

A client newly diagnosed with type 1 diabetes mellitus asks: "What is the purpose of the test that measures that funny glucose value over time?" Which response best answers the client's question? a. "Called glycolsolated hemoglobin, it is the average blood glucose for the past two to three months." b. "The oxygen carrying capacity of your red cells is reflected in this type of test." c. "It indicates the presence of glucose attaching itself to red cells over the past two weeks." d. "It compares the risk for cardiac complications with the risk for cerebral infarctions associated with sugar levels."

a. "Called glycolsolated hemoglobin, it is the average blood glucose for the past two to three months." By testing the portion of the hemoglobin that absorbs the glucose, it is possible to determine the average blood glucose over the life span of the red cell, which is 120 days. The expected glucose control target is less than 6% for anyone with glucose balance challenges. The test is also known as the hemoglobin A1c test of HbA1c.

A client is started on long-term corticosteroid therapy. Which comment by the client indicates a need for more teaching? a. "For one week every month I will stop taking the medication." b. "I will keep a weekly weight record." c. "The medication needs to be taken with food." d. "I will be sure to eat foods high in potassium."

a. "For one week every month I will stop taking the medication." To suddenly stop taking a steroid may result in a sudden drop in the blood pressure from a loss in fluid volume associated with adrenal crisis. Clients should be warned not to abruptly stop taking the medication. Corticosteroids can lower the amount of potassium in the body so the client should eat more potassium-rich foods. Weight gain is an expected effect of corticosteroid therapy; clients should regularly keep track of their weight. Normally corticosteroid medications are taken with breakfast.

A client who lives in an assisted living facility tells the nurse, "I am so depressed. Life isn't worth living anymore." What is the best response by the nurse to this statement? a. "Have you thought about hurting yourself?" b. "Did you tell any of this to your family?" c. "Come on, it is not that bad." d. "Think of the many positive things in life today."

a. "Have you thought about hurting yourself?" It's important to determine if someone, who has voiced thoughts about death, is considering a suicidal act. This response is most therapeutic under the circumstances. To respond by saying things are not so bad, denies the validity of the client's statement. To ask if the family or anyone knows of these feelings lacks focus on the client and would also be in violation of the rights of the client. Many times, when there doesn't seem to be an urgent physiologic need, look for a response that focuses on the safety of the client.

After working with a client, an unlicensed assistive personnel (UAP) tells the nurse, "I have had it with that demanding client. I just can't do anything that pleases him. I'm not going in there again." The nurse should respond with which statement? a. "He may be scared and taking it out on you. Let's talk to figure out what to do next." b. "I will talk with him and try to figure out what to do or what the problem is." c. "Ignore him and get the rest of your work done. Someone else can care for him the rest of the day." d. "He has a lot of problems. You need to have patience with him."

a. "He may be scared and taking it out on you. Let's talk to figure out what to do next." This response explains the client's behavior without belittling the UAP's feelings. The UAP is encouraged to contribute to the plan of care to help solve the problem. The other responses are incorrect because they either belittle the UAP or ignore the problem and do not include the UAP in planning of how to deal with the issue.

The nurse has given discharge instructions to parents of a child who will be taking liquid phenytoin. Which statement made by the parents suggests that the teaching was effective? a. "Our child should brush after every meal and floss daily." b. "We will shake the liquid before pouring it into a tablespoon." c. "We will call the health care provider if the child develops acne." d. "When our child is seizure-free for six months, we can stop the medication."

a. "Our child should brush after every meal and floss daily." Phenytoin causes thickening or enlargement of gingiva (gingival hyperplasia). Although this may be dose-related, it can be minimized by good dental care, such as frequent brushing and daily flossing. Any liquid medication must be measured using a measuring spoon or medicine cup, not a tablespoon. Phenytoin is an anticonvulsant drug that is used to treat seizures.

The nurse is offering safety instructions to a parent with a 4-month old infant and a 4 yo child. Which statement by the parent indicates a correct understanding of the appropriate precautions to take with the children. a. "I have the 4 yo hold and help feed the 4 month old a bottle with me." b. "I place my infant in the middle of the living room floor on a blanket to play with my 4yo while I make supper in the kitchen." c. My sleeping baby lies so cute in the crib with the little buttocks stuck up in the air while the 4 yo naps on the sofa." d. "I strap the infant car seat on the front seat to face backwards."

a. "I have the 4 yo hold and help feed the 4 month old a bottle with me." The infant seat should be placed on the rear seat. Small children and infants should not be left unsupervised. Infants are to be placed on their backs when they are sleeping or lying in a crib. A 4 yo could assist with the care of an infant, such as feeding with proper direct supervision.

A client expresses anger when a call light is not answered within five minutes. The client demanded a blanket. How should the nurse respond? a. "I see this is frustrating for you. I have a few minutes so let's talk." b. "I am surprised that you are upset. The request could have waited a few more minutes." c. "Let's talk. Why are you upset about this?" d. "I apologize for the delay. I was involved in an emergency."

a. "I see this is frustrating for you. I have a few minutes so let's talk." This is the best response because it gives credence to the client's feelings and then concerns. To say "let's talk" and ask a why question is not a therapeutic approach because it does not acknowledge or validate the client's feelings. To apologize and not notice the client's feelings is inappropriate. To say it could have waited a few minutes is rude and non-accepting of the client's verbalized needs.

The health care provider writes a new order for a fentanyl (Sublimaze) patch to manage the chronic pain experienced by the client in hospice care. The nurse is teaching a client and family members about the fentanyl patch and knows that teaching was effective when the client makes which of the following statements? (Select all that apply.) a. "If my pain is too great while I am on the patch, I can take a supplemental pain medication." b. "I should cut up the patch before I throw it away so no one else can use it." c. "It may take up to a half day or longer for the patch to start working, the first time I use it." d. "I will take the old patch off before I apply the new patch on." e. "I can soak in a hot tub to help decrease my pain."

a. "If my pain is too great while I am on the patch, I can take a supplemental pain medication." c. "It may take up to a half day or longer for the patch to start working, the first time I use it." d. "I will take the old patch off before I apply the new patch on." Fentanyl patches are slowly absorbed via the subcutaneous tissue at a predetermined rate for up to 72 hours. Due to the slow absorption rate, the first patch may take 12 to 24 hours before effective analgesia is felt; a short-acting opioid may be given for breakthrough pain. The client can shower or bathe with the patch, but it should not be exposed to heat (hot tubs, heating pads) because it speeds up the absorption of the medication. Old patches are removed and the new patch is applied to a different skin area. Old patches are disposed by folding the old patch in half, not by cutting them up and throwing them in the trash (which may be dangerous for people and pets).

At a routine health assessment, a client tells the nurse that she is planning a pregnancy in the near future. The client asks about preconception diet changes. Which of these statements made by the nurse is the best approach in this situation? a. "Increase your intake of green leafy vegetables." b. "Eat at least one serving of fish weekly." c. "Drink a glass of milk with each meal." d. "Include fiber in your daily diet."

a. "Increase your intake of green leafy vegetables." Folic acid sources should be included in the diet and are critical in the preconceptual and early gestational periods to foster neural tube development and prevent birth defects such as spina bifida.

The nurse manager is interviewing a prospective employee who just completed the agency application. Which approach should the nurse manager use to assess skills competencies of this potential employee? a. "Let's review your skills checklist for type and level of skill for tasks." b. "Let's talk about your comfort zone for working independently." c. "What degree of supervision for basic care do you think you need?" d. "What types of complex client-care tasks or assignments do you prefer?"

a. "Let's review your skills checklist for type and level of skill for tasks." The nurse needs to know that the potential employee has competence in certain tasks that are common on the unit. One way to do this is to do mutual review of the agency list of skills. The other questions might be asked during the skills checklist review.

The nurse is examining a 2 year-old child with a tentative diagnosis of Wilm's tumor. The nurse would be most concerned about which statement by the mother? a. "Urinary output seems to be less over the past two days." b. "The child prefers some salty foods more than others." c. "My child has lost three pounds in the last month." d. "All the pants have become tight around the waist."

a. "Urinary output seems to be less over the past two days." Wilm's tumor is a malignant tumor of the kidney that can lead to kidney dysfunction; therefore, a recent decrease in urinary output should be investigated further as it may be a sign of renal dysfunction. Increasing abdominal girth is a common finding in Wilm's tumor, but does not require immediate intervention by the nurse.

A client has had a positive reaction to purified protein derivative (PPD). When the client asks, "What does this mean?" the nurse should respond with which statement? a. "You have been exposed to the organism Mycobacterium tuberculosis." b. "This means you have never had or been around someone with tuberculosis." c. "You are mostly likely have a natural immunity to the bacteria." d. "You most likely have a resistant form of active tuberculosis."

a. "You have been exposed to the organism Mycobacterium tuberculosis." The PPD skin test is used to determine the presence of tuberculosis antibodies. In an otherwise healthy person, an induration greater than or equal to 15 mm is considered a positive skin test. This indicates that the client has been exposed to the organism Mycobacterium tuberculosis. Additional tests such as a chest x-ray and sputum culture will be needed to determine if active tuberculosis is present. The sputum cytology test is the only definitive test to confirm a diagnosis of active TB.

A nurse is conducting a community-wide seminar on childhood safety issues. Which of these children is at the highest risk for poisoning a. 20 month old who has just learned to climb the stairs b. 10 yo who occassionally stays at home unattended c. 15 yo who likes to repair bicycles d. 9 month old who stays with a sitter 5 days a week

a. 20 month old who has just learned to climb the stairs Toddlers, aged 1-3 years, are at highest risk for poisoning because they are increasingly mobile, need to explore and engage in autonomous behavior

The health care provider has ordered a vanillylmandelic acid test and catecholamine test for a middle-aged client. Which of the following points should the nurse discuss with the client prior to these tests? (Select all that apply.) a. A 24-hour urine collection procedure is required b. Continue taking all prescribed medications c. Avoid caffeinated beverages, bananas, chocolate, cocoa, licorice and citrus fruit d. Avoid excessive physical exercise several days prior to the test e. Identify and minimize factors contributing to stress and anxiety

a. A 24-hour urine collection procedure is required c. Avoid caffeinated beverages, bananas, chocolate, cocoa, licorice and citrus fruit d. Avoid excessive physical exercise several days prior to the test e. Identify and minimize factors contributing to stress and anxiety The nurse should confirm what the client has discussed with the health care provider about medications. Many times the client is instructed to stop taking antihypertensive medications prior to the test. Clients should be instructed to avoid anything that would increase urinary catecholamine levels and alter the test results before starting this 24-hour urine test, which is used to help diagnose a tumor in the adrenal glands. Catecholamines (dopamine, epinephrine, norepinephrine) increase heart rate, blood pressure, breathing rate, muscle strength and mental alertness.

The nurse receives report on the following client assignments. Which client should the nurse assess first? a. A client diagnosed with peptic ulcer disease (PUD) who reports feeling dizzy b. A client who underwent a partial gastrectomy and reports feeling lightheaded c. A client diagnosed with gastroesophageal reflux disease (GERD) reporting difficulty swallowing d. A client reporting severe gastric distress after taking ibuprofen

a. A client diagnosed with peptic ulcer disease (PUD) who reports feeling dizzy Dizziness with PUD may indicate hemorrhaging. The findings in the other options are either expected and are not life-threatening: clients may feel lightheaded when they are not drinking enough fluids after a gastrectomy; difficulty swallowing can be a symptom of GERD; gastrointestinal symptoms are the most common side effects of NSAIDs.

The hospital sounded the call for disaster drill on the evening shift. Which of these patients would the nurse put first on the list for discharge in order to make room for new admissions? a. A middle-aged patient with 7-year history of being ventilator dependent and who was admitted with bacterial pneumonia 5 days ago. b. An older adult with a history of hypertension, hypercholesterolemia, and lupus, and who was admitted with Steven Johnson syndrome that morning c. A young adult with DM2 for more than 10 years and was admitted with antibiotic induced diarrhea d. An adolescent with a positive HIV test and was admitted with cellulitis of the lower leg 48 hours ago.

a. A middle-aged patient with 7-year history of being ventilator dependent and who was admitted with bacterial pneumonia 5 days ago. The best candidate for discharge is one who has a chronic condition and has an established plan of care. The patient who has been on a ventilator for years is most likely stable and could continue medication therapy at home. The other patients have a risk for instability or are unstable.

The client is admitted to the hospital with a diagnosis of exacerbation of right ventricular heart failure. Which of the following findings would the nurse expect with right-sided heart failure? (Select all that apply.) a. Abdominal discomfort b. Anorexia and nausea c. Cough d. Peripheral edema e. Orthopnea

a. Abdominal discomfort b. Anorexia and nausea d. Peripheral edema The classic findings of right-sided heart failure arise from blood backing up into the systemic circulation resulting in abdominal organ engorgement and dependent edema. The nurse would expect to find orthopnea and cough with left-sided heart failure.

A nurse is providing home care for a client with chronic bilateral heart failure. Which nursing diagnosis should have the priority when planning care for this client? a. Activity intolerance related to an imbalance in oxygen supply and demand b. Risk for infection related to ineffective mobilization of secretions c.Impaired skin integrity related to dependent edema d. Constipation related to immobility from challenges with breathing

a. Activity intolerance related to an imbalance in oxygen supply and demand Findings of bilateral heart failure may include dyspnea, fatigue, chronic cough, lack of appetite, mental confusion or impaired thinking, peripheral edema and weight gain. Due to an inadequate supply of oxygen and the stress of the extra heart muscle mass, the client will experience exertional dyspnea. Therefore, activity intolerance is the priority nursing diagnosis for this client. While there may be a concern about skin integrity, risk for infection and even constipation, oxygen needs are more important.

The client needs to be moved up in bed. The client is able to partially assist and weighs 135 pounds. Which action by the nursing staff best supports an awareness of ergonomics and safe client handling? (Select all that apply.) a. Adjust the height of the bed for caregivers b. Move the bed into the flat position c. Pull the client up from the head of the bed d. Use a friction-reducing device e. Coordinate lifting the client by counting to 3

a. Adjust the height of the bed for caregivers b. Move the bed into the flat position d. Use a friction-reducing device The algorithm for safe client handling and repositioning a client from side-to-side or up in bed states: use 2 to 3 caregivers for a client who can partially assist and who weighs less than 200 pounds, use a friction-reducing device, move the bed so that it's flat and at a comfortable height for the caregivers. The client should not be pulled from the head of the bed. There really is no safe method to manually lift another adult.

The clinic nurse is counseling a postpartum client who has a substance-abuse problem and is at risk for continued cocaine use. In order to provide continuity of care, which nursing diagnosis should be a priority? a. Altered parenting b. Social isolation c. Ineffective coping d. Sexual dysfunction

a. Altered parenting The mother who abuses cocaine puts her newborn and any other children at risk for neglect and abuse. The continued use of drugs has the potential to impact parenting behaviors. Social service referrals are indicated for evaluation and follow-up.

The client, diagnosed with an acute anterior MI, has a triple lumen infusing with nitroglycerin, alteplase and heparin. The client reports experiencing angina. Which intervention is the priority? a. Administer intravenous morphine sulfate as ordered b. Administer antidysrhythmic drugs as indicated c. Review and compare serial ECG strips d. Auscultate heart and lung sounds

a. Administer intravenous morphine sulfate as ordered Nitrates are useful for pain control due to their coronary vasodilating effects. The nurse will titrate the intravenous nitroglycerin infusion for chest pain according to standing orders but if chest pain is unrelieved by the nitroglycerin infusion, the nurse can administer morphine intravenously (IM injections are avoided because they can alter the CPK.) Morphine not only relieves pain and reduces anxiety, but also dilates blood vessels. After giving the pain reliever, the nurse can do a more in-depth assessment of the client (auscultate heart and lung sounds, review ECGs, vital signs and labs.) There is no need to administer an antidysrhythmic drug if the client is asymptomatic.

A 67 year-old client is admitted with substernal chest pressure that radiates to the jaw. The admitting diagnosis is acute myocardial infarction (MI). What should be the priority nursing diagnosis for this client during the first 24 hours? a. Altered tissue perfusion b. Activity intolerance c. Anxiety d. Risk for fluid volume excess

a. Altered tissue perfusion In the immediate post MI period, altered tissue perfusion is priority, as an area of myocardial tissue has been damaged by a lack of blood flow and oxygenation. Interventions should be directed toward promoting tissue perfusion and oxygenation. The other problems are also relevant, but tissue perfusion is the priority.

At the beginning of the shift, the nurse is reviewing the status of each of the assigned clients in the labor and delivery unit. Which of these clients should the nurse check first? a. An adolescent who is 18-weeks pregnant with a report of no fetal heart tones and is coughing up frothy sputum b. A middle-aged woman with a history of two prior vaginal term births and who is 2 cm dilated c. A young woman who is a grand multipara, cervical dilation to 4 cm and is 50% effaced d. A young woman, first-time para, cervical dilation to 1 cm and contractions 15 minutes apart

a. An adolescent who is 18-weeks pregnant with a report of no fetal heart tones and is coughing up frothy sputum The 18 year-old client has an actual complication of left-sided heart failure and a possible stillborn birth. The other clients present with findings of potential, but not actual, complications.

The nurse is assessing a client with a stage II skin ulcer. Which of these approaches should be most effective to promote healing? a. Apply a hydrocolloid or foam dressing b. Use hydrogen peroxide soaks c. Cover the wound with a dry dressing d. Leave the area open to dry

a. Apply a hydrocolloid or foam dressing In prior years an accepted treatment was the use of a transparent cover. However, evidence-based nursing outcomes now recommends the use of the foam (DuoDerm) dressings to keep the wound somewhat moist. One could eliminate the two options that have the word "dry" in them. This is called the elimination of "similar - dissimilar" options. Hydrogen peroxide should not be used full strength on any type of wound because it is destructive to the cells.

A nurse is caring for a client with end-stage heart failure. The family members are distressed about the client's impending death. Which intervention should the nurse take first? a. Assess the family's patterns for dealing with death b. Ask about their present religious affiliations c. Explain the stages of death and dying to the family d. Recommend an easy-to-read book on grief

a. Assess the family's patterns for dealing with death When a new problem is identified, it is important for the nurse to first collect accurate information. This is crucial to ensure that the client and the family's needs are adequately identified in order to plan and implement nursing care. Once the situation has been assessed and a plan has been established, the nurse can focus on teaching or referral to other resources.

A client is transported by a family member to the emergency department following a boating accident. The client is conscious, shivering, and confused. The client is still wearing wet clothes. Which interventions does the nurse implement? (Select all that apply.) a. Apply warm blankets b. Monitor vital signs c. Infuse warm IV solutions as ordered d. Remove wet clothes e. Massage cold extremities f. Give sips of warm fluids

a. Apply warm blankets b. Monitor vital signs c. Infuse warm IV solutions as ordered d. Remove wet clothes This client is at risk for hypothermia. In a conscious client, wet clothing should be removed carefully. External rewarming, using blankets or heat packs placed under the arms and on the neck, chest, and groin, is appropriate. In-hospital treatment also includes monitoring core temperature and cardiac rhythm, ventilating with warm humidified air/oxygen to help stabilize core temperature and administering warm IV fluids. Sips of warm fluids may be given to the conscious and alert client only after his condition is stabilized. Extremities should never be massaged.

The nurse is assessing the mental status of a patient admitted with possible dementia. Which of these options would best assess the functioning of the patient's short-term memory? a. Ask the patient to recall 3 words the nurse had previously asked the patient to remember b. Ask the patient to copy an image of 2 simple, intersecting geometric shapes c. Ask the patient to calculate simple arithmetic operations d. Ask the patient to name the last four presidents

a. Ask the patient to recall 3 words the nurse had previously asked the patient to remember Short-term memory refers to the temporary storage of information in memory and the management of the information so that it can be used for more complex cognitive tasks, such as learning and reasoning. Tests of cognitive function are used to evaluate cognitive impairment. The Mini-Mental Status Exam, for example, measures orientation to time and place, calculation, language, short-term verbal memory, and immediate recall. To help determine short-term memory functioning, the health care practitioner would ask the client to recall three words that the client had previously been asked to remember.

During an interview of a new admission, the nurse notices that the client is shifting positions, wringing the hands, and avoiding eye contact. It is important for the nurse to take which of these approaches? a. Ask the patient what the patient is feeling at this moment b. Refocus the discussion on a less anxiety-provoking topic c. Assess the patient for auditory hallucinations d. Recognize the behavior as a side effect of medication

a. Ask the patient what the patient is feeling at this moment An initial step in anxiety intervention is using the senses to observe, identify, and assess anxiety behaviors. The nurse then should seek client validation of the accuracy of observations and interpretations. The nurse should avoid drawing conclusions based on limited data. In the situation above, the client may simply need to use the restroom but be reluctant to communicate this elimination need!

A nurse is to collect a sputum specimen for acid-fast bacillus (AFB) from a client. Which action should the nurse take first? a. Assist with oral hygiene without the use of mouthwash b. Ask client to cough sputum into container c.Have the client take several deep breaths d. Provide an appropriate specimen container

a. Assist with oral hygiene without the use of mouthwash The nurse should obtain a specimen after mouth care, early in the morning, so that particles of food will be removed. Mouthwash should not be used because it may have alcohol in it and this could alter the sputum as it travels through the mouth. The other actions would follow this first action: The client should take several deep breaths then cough into the appropriate sterile container to obtain the AFB specimen of the sputum.

The client, who is diagnosed with dementia, wanders throughout the long-term care facility. How can the nurse best ensure the safety of a client who wanders? a. Attach a monitoring band to the client's wrist b. Explain the risk of walking with no purpose c. Frequently reorient the client to time, person, place d. Apply a restraint to keep keep the client in a chair when awake

a. Attach a monitoring band to the client's wrist A wander management system is used to give people with dementia and other "at risk" clients the ability to move freely where they live. The sensor in the bracelet trips an alarm that's attached to exterior doors if the client attempts to leave the facility. It is inappropriate to use restraints or other restrictive devices to keep clients in chairs or beds (unless they are potentially harmful to themselves or others.) Reality orientation is inappropriate for someone with dementia.

The nurse is caring for a child who is diagnosed with coarctation of the aorta. Which finding would the nurse expect when assessing the child? a. Bounding pulses in the arms b. Diminished carotid pulses c. Strong pedal pulses d. Normal femoral pulses

a. Bounding pulses in the arms Coarctation of the aorta, which is a narrowing or constriction of the descending aorta, causes increased blood flow to the upper extremities, resulting in a bounding pulse in the arms. Cardinal signs include resting systolic hypertension, absent or diminished femoral and pedal pulses, and a widened pulse pressure.

The client is admitted in stable condition from the emergency department. Based on an EKG strip of Atrial flutter, which of the following types of medications will be ordered? (Select all that apply.) a. Calcium channel blocker b. Beta blocker c. Diuretic d. Vasodilator e. Cardiac glycoside

a. Calcium channel blocker b. Beta blocker e. Cardiac glycoside ECG depicts atrial flutter, when the atria beat excessively fast (250-300 BPM). Medications used to slow the heart include calcium channel blockers (such as diltiazem), beta-adrenergic blockers (such as propranolol), and a cardiac glycoside (digoxin). An anticoagulant (such as warfarin) may also be ordered. Vasodilators and diuretics are used to lower blood pressure; vasodilators will increase heart rate.

A 62 year-old client is admitted to the emergency department. The client has a history of anemia and peptic ulcer disease and is now experiencing chest pain, nausea and dizziness. The nurse anticipates which laboratory tests to be ordered right away? (Select all that apply.) a. Cardiac enzymes b. Complete blood count (CBC) c. Helicobactor pylori (H. pylori) d. Lipid panel e. Toxicology screen

a. Cardiac enzymes b. Complete blood count (CBC) Chest pain with nausea and dizziness may be findings associated with angina or myocardial infarction. Cardiac enzymes, including creatinine kinase, myoglobin, and troponin, are indicated to determine if muscle damage has occurred (to help rule out a heart attack). Low hemoglobin (Hgb) can precipitate an angina attack if there is not enough Hgb to deliver oxygen to the myocardium; given the client's history, a CBC would be indicated. There are no findings of either overdose or poisoning, so a toxicology screen is not needed. A lipid panel is used to show one's risk for coronary heart disease and the presence of Helicobacter Pylori is associated with increased cardiovascular disease risk and lower HDL, but these tests are not used diagnostically for chest pain.

The nurse discusses nutrition with a pregnant woman who is iron-deficient and follows a vegetarian diet. The selection of which foods indicates the woman has learned food sources of iron? a. Cereal and dried fruits b. Fish and dairy products c. Carrots and other yellow-orange vegetables d. Whole grains and yellow vegetables

a. Cereal and dried fruits Cereals are often fortified with vitamins and iron. Fruits rich in iron include: apricots currants raisins dates figs prunes Other sources for iron: dark, leafy green vegetables beans lentils chick peas soybeans

Following a surgical procedure, a pneumatic compression device is applied to the adult client. The client reports that the device is hot and the client is sweating and itching. Which of the following steps should the nurse take? (Select all that apply.) a. Check for appropriate fit b. Confirm pressure setting of 45 mm Hg c. Explain that the health care provider ordered the device and it cannot be removed d. Collaborate with health care provider for anti-embolism stockings to be worn under the sleeves of the device e. Inform the client that removing the device will likely result in the formation of deep vein thrombosis

a. Check for appropriate fit b. Confirm pressure setting of 45 mm Hg d. Collaborate with health care provider for anti-embolism stockings to be worn under the sleeves of the device In any situation in which a client has discomfort associated with a medical device, the nurse should ensure it is applied correctly and functioning safely. The usual safe and effective pressure range is 35 to 55 mm Hg. Explanations to the clients should support their informed decision-making capabilities and should not be phrased to intimidate or remove client autonomy. Applying anti-embolism stockings under the disposable sleeves of the device may help with the sweating and itching.

There is an order to administer a vesicant chemotherapy medication intravenously (IV). Which nursing action is the priority before starting the flow of the medication? a. Check for blood return in the intravenous line b. Use meticulous sterile technique with the intravenous line c. Apply warm compresses to the skin surrounding the insertion site d. Ensure that two nurses check the accuracy of the medication and dose

a. Check for blood return in the intravenous line Extravasation occurs when the IV line infiltrates and begins depositing the drug into the soft tissue. Many of the chemotherapeutic agents are vesicants and will cause serious soft tissue damage, and even necrosis, if they infiltrate. To protect against this, the nurse must always aspirate for positive blood return prior to use.

The nurse needs to accurately assess gastric placement of a nasogastric tube prior to the administration of an enteral feeding. What is the priority action the nurse should take before starting the infusion? a. Check the pH of the aspirate b. Measure the length of tubing from nose to epigastrium c. Auscultate the abdomen while instilling 10 mL of air into the tube d. Place the end of the tube in water to check for air bubbles

a. Check the pH of the aspirate Once the initial placement of the tube has been confirmed by x-ray, the nurse will check the pH of the aspirate before administering medications or enteral feeding solutions. Current practice recommendations include assessing the feeding tube placement by testing the pH of aspirates, measuring the external portion of the tube, and observing for changes in the volume and appearance of feeding tube aspirates. If tube placement is in doubt, an x-ray should be obtained. The other methods are older approaches that are no longer recommended.

The nurse is caring for a patient in labor. Which non-pharmacologic measures can the nurse implement to provide the laboring patient with a sense of control and comfort? Select all that apply a. Childbirth education b. Amnioinfusion c. Intrauterine pressure catheter d. Aromatherapy e. Coujnterpressure f. Lamaze breathing techniques

a. Childbirth education d. Aromatherapy e. Coujnterpressure f. Lamaze breathing techniques Nonpharmacologic labor pain management techniques incorporate special attention to all the senses. Aromatherapy - smell Relaxing music - auditory Coujnterpressure, massage, effleurage - tactile Childbirth education helps to prepare the patient and support person to understand what to anticipate prior to, during, and after L&D, giving the patient a sense of control, initiation of breathing techniques to close the gate to nerve stimulation caused by pain is also used. Intrauterine pressure catheter, which provides an exact measurement of contractions and amnioinfusion, which involves the infusion of fluid into the uterus during labor, are unrelated to pain management.

The nurse is caring for a young adult client with an acute attack of inflammatory bowel disease. Which of the following findings indicates a potential complication? (Select all that apply) a. Chills and fever b. Visible blood and mucus in the stool c. Abdominal distention d. Frequent diarrhea e. Abdominal pain and tenderness

a. Chills and fever c. Abdominal distention Expected findings in the client with inflammatory bowel disease include persistent diarrhea, abdominal pain and tenderness, and even visible blood and mucus in the stools. Abdominal distention, along with chills and fever, can indicate possible intestinal obstruction (toxic megacolon) or fistula formation.

The charge nurse sends a certified nursing assistant (CNA) to help a registered nurse (RN) with the admission of a client with multiple health problems. Which of the following tasks would be appropriate for the the CNA to perform with the nurse during the admission process? (Select all that apply.) 15of123 Ref # 4435 The charge nurse sends a certified nursing assistant (CNA) to help a registered nurse (RN) with the admission of a client with multiple health problems. Which of the following tasks would be appropriate for the the CNA to perform with the nurse during the admission process? (Select all that apply.) a. Collect a urine specimen b. Obtain routine vital signs (temperature, pulse, respirations, blood pressure) c. Assist the client to change into a gown d. Orient the client to the room e. Observe and document the client's ability to walk to the bathroom

a. Collect a urine specimen b. Obtain routine vital signs (temperature, pulse, respirations, blood pressure) c. Assist the client to change into a gown d. Orient the client to the room CNAs can obtain routine vital signs, measure height and weight, and obtain urine specimens. CNAs also routinely help clients with activities of daily living (ADLs). Although nursing assistants can measure vital signs, it's up to the RN to determine how to use this data when developing the plan of care. The CNA cannot assess clients or perform any of the other steps of the nursing process. Any nursing intervention that requires independent, specialized nursing knowledge, skill or judgment must be performed by a nurse.

Which statement is the correct stage of cognitive development for Piaget's: School-age concrete operations stage a. Concepts are attached to concrete situations b. Analyzes situations and uses abstract logic and reasoning c. Uses sucking, grasping, listening, and looking to earn about the environment d. Uses magical thinking and imagination

a. Concepts are attached to concrete situations

A client with a diagnosis of methicillin-resistant Staphylococcus aureus (MRSA) has died. Which type of precautions is appropriate to use when performing postmortem care? a. Contact precautions b. Droplet precautions c. Compromised host precautions d. Airborne precautions

a. Contact precautions The resistant bacteria remain alive for up to three days after the client dies. Therefore, contact precautions must still be used. The body should also be labeled as MRSA-contaminated so that the funeral home staff can protect themselves as well. Gown and gloves are required.

A client has a chest tube inserted immediately after surgery for a left lower lobectomy. During the repositioning of the client during the first postop check, the nurse notices 75 mL of a dark, red fluid flowing into the collection chamber of the chest drain system. What is the appropriate nursing action? a. Continue to monitor the rate of drainage b. Call the surgeon immediately c. Check to see if the client has a type and cross match d. Turn the client back to the original position

a. Continue to monitor the rate of drainage It is not unusual for blood to collect in the chest and be released into the chest drain when the client changes position this soon after surgery. The dark color of the blood indicates it is not active bleeding inside of the chest. Sanguinous drainage should be expected within the initial 24 hours postop, progressing to serosanguinous and then to a serous type. If the drainage exceeds 100 mL/hr, the nurse should call the surgeon.

The postpartum nurse is caring for a couplet four hours after a vaginal delivery with a partial abruption of the placenta prior to delivery. The nurse would immediately notify the health care provider (HCP) based on which of the following data? a. D-dimer test result is increased b. Hemoglobin is 10.4 g/dL (6.45 mmol/L) c. Maternal urine output is 280 mL/8 hours d. Infant is Rh positive

a. D-dimer test result is increased An increased D-dimer test following a partial abruption raises the concern of disseminated intravascular coagulation; the HCP should be notified right away. The urine output is a bit low; the nurse should continue to monitor this. Infant Rh+ would need to be addressed if the mother is Rh (-) but there's nothing to indicate the mother is Rh negative. Even so, this is not an immediate concern since this drug can be given within 72 hours following birth. The hemoglobin is adequate for a postpartum client, especially one who experienced increased bleeding due to an abruption. A hemoglobin less than 8 g/dL (4.96 mmol/L) might require a transfusion, but you don't know if this lab value is stable or decreasing.

A patient is diagnosed with gastroesophageal reflux disease (GERD). The nurse's instruction to the patient about the approaches to dietary changes should include which topic? a. Decrease intake of fatty foods b. Focus on 3 average meals daily c. Increase intake of milk products d. Avoid all raw fruits and vegetables

a. Decrease intake of fatty foods GERD may be aggravated by fatty diet. A diet low in fat would decrease the symptoms of GERD. Other agents that should also be decreased or avoided are: cigarette smoking - nicotine caffeine alcohol chocolate narcotic analgesic - meperidine (Demerol)

The clinic nurse is assisting with medical billing. The nurse uses the DRG (Diagnosis Related Group) manual for which purpose? a. Determine reimbursement for a medical diagnosis b. Identify findings related to a medical diagnosis c. Classify nursing diagnoses from the client's health history d. Implement nursing care based on case management protocol

a. Determine reimbursement for a medical diagnosis DRGs are the basis of prospective payment plans for reimbursement for Medicare clients. Other insurance companies often use it as a standard for determining payment.

The nurse is caring for a pregnant woman who is diagnosed with pregnancy induced hypertension (PIH) and is receiving magnesium sulfate intravenously. During assessment of the client, the nurse notes that respirations are 12, pulse and blood pressure have dropped significantly, and the eight-hour urine output is 200 mL. What should the nurse do first? a. Discontinue the magnesium sulfate b. Call the health care provider immediately c. Perform additional assessments d. Administer calcium gluconate

a. Discontinue the magnesium sulfate The assessments strongly suggest magnesium sulfate toxicity. The nurse must discontinue the medication immediately and take measures to ensure the safety of the client. Calcium gluconate would be given as an antidote.

The client with newly diagnosed irritable bowel syndrome (IBS) states: "All this fiber I have to eat now is making me full of gas! It makes me want to stop taking it." What instruction by the nurse will help the client manage this side effect and increase compliance with the diet? (Select all that apply.) a. Discuss a work-up for lactose intolerance with the health care provider b. Reduce intake of gas-forming foods c. Eat three regularly scheduled meals every day d. Cut back on fiber and then add it again slowly to the diet e. Eat a balanced and nutritious variety of foods

a. Discuss a work-up for lactose intolerance with the health care provider b. Reduce intake of gas-forming foods d. Cut back on fiber and then add it again slowly to the diet Adequate fiber intake is critical to controlling IBS but can result in bloating and gas if added to the diet too quickly. Increasing fiber intake by 2 to 3 grams per day will help reduce the risk of gas and bloating. Some foods, as well as dairy products, also contribute to gas formation. Eating a balanced nutritious diet is good self-care practice but does not decrease gas production. Large meals can cause cramping (and diarrhea) so eating four to five small meals a day is recommended instead of less-frequent big meals.

A client frequently admitted to the locked psychiatric unit repeatedly compliments and then invites one of the nurses to go out on a date. The nurse should take which of these approaches? a. Discuss the boundaries of a therapeutic relationship with the client b. Inform the client that the hospital policy prohibits staff to date clients c. Ask to not be assigned to this client or request to work on another unit d. Tell the client that such behavior is inappropriate and unethical

a. Discuss the boundaries of a therapeutic relationship with the client The nurse-client relationship is one with professional not social boundaries. Consistent adherence to the limits of the professional relationship builds trust. The client may need to be educated about the interactions in a therapeutic relationship.

The nurse is providing discharge teaching to a client who has had a total hip prosthesis implanted. During teaching, the nurse should include which content in the instructions for home care? a. Do not cross your legs at the ankles or knees b. Ambulate using crutches only c, Sleep only on your back and not on your side d. Avoid climbing stairs for three months

a. Do not cross your legs at the ankles or knees These clients should avoid the bringing of the knees together. Clients are to use a pillow between their legs when lying down and can lie on the back or side. Crossing the legs or bringing the knees together results in a strain on the hip joint. This increases the risk of a malfunction of the prosthesis where the ball may pop out. A walker or crutches may be used as assistive devices. These and other precautions are minimally followed for six weeks postoperative and sometimes longer as indicated.

A nurse is caring for a client with chronic renal failure who is treated with hemodialysis three times a week. The client becomes confused and irritable six hours before the next treatment. Which of these physiologic changes might explain the reason for the client's behavior? a. Elevated blood urea nitrogen (BUN) b. Metabolic alkalosis c. Calcium depletion d. Potassium loss

a. Elevated blood urea nitrogen (BUN) Confusion and irritability are signs of renal encephalopathy secondary to elevated levels of BUN and creatinine in the blood. The other options do not explain the client's behavior. Potassium levels are generally high in renal failure. Side effects of calcium depletion manifest as abdominal and muscle cramping and hyperactive reflexes. Metabolic acidosis, not alkalosis, is seen in renal failure.

The client states to the nurse: "I am ready to stop all of these treatments. I just want to go home and enjoy my family for the little bit of time I have left." Which action is most appropriate? a. Encourage the client to discuss this decision with the health care provider and family b. No action is needed at this time unless the client repeats the statement to another caregiver c. Tell the family members that the client's preference is to go home to die d. Call in a referral to a social worker and explain that the request will need to be discussed in more detail at a later time

a. Encourage the client to discuss this decision with the health care provider and family The client has the right to stop treatment and should be supported in clearly communicating this decision with the health care provider and family. The nurse needs to act as an advocate for the client. It is factually incorrect to wait until the request is repeated; clients should not need to express their wishes repeatedly before caregivers listen to them. The nurse should not be the one to share sensitive information with the family; the client controls that information. Social services may get involved but time is of the essence for those who are terminally ill.

A client diagnosed with a terminal condition is admitted to the nursing unit. What should be the initial action taken by the nurse? a. Ensure that the patient is free from pain, nausea, or dyspnea b. Refer the patient's family to the chaplain c. Discuss the options for Advanced directives with the patient and family d. collaborate with the multidisciplinary team members

a. Ensure that the patient is free from pain, nausea, or dyspnea Keep in mind the nursing process where the client is the center of attention. Also, recall that physiological needs are priority to psychosocial needs. Thus, the client should be kept as comfortable as possible. After the physiological needs are met, any of the other choices would be appropriate with the discussion of the advanced directives being first. This would not be the initial action because a "discussion" may take some time and the client needs would be ignored during any discussion.

A client taking isoniazid for tuberculosis (TB) asks the nurse about the side effects of this medication. The client should be instructed to report which of these findings? a. Extremity tingling and numbness b. Confusion and light-headedness c. Double vision and visual halos d. Photosensitivity and photophobia

a. Extremity tingling and numbness Peripheral neuropathy is a common side effect of isoniazid and other antitubercular medications and should be reported to the health care provider. Daily doses of pyridoxine (vitamin B6) may lessen or even reverse peripheral neuropathy due to isoniazid use.

The client is diagnosed with post-traumatic stress disorder (PTSD). What are the some of the more common treatment options for PTSD? (Select all that apply.) a. Eye movement desensitization and reprocessing (EMDR) b. Cognitive behavioral therapies c. Selective serotonin reuptake inhibitors (SSRIs) d. Opioid analgesics

a. Eye movement desensitization and reprocessing (EMDR) b. Cognitive behavioral therapies c. Selective serotonin reuptake inhibitors (SSRIs) The only two FDA approved medications for the treatment of PTSD are the SSRIs sertraline (Zoloft) and paroxetine (Paxil). There are other medications that are helpful for specific PTSD symptoms, but narcotics should not be used since they don't relieve psychogenic pain and there's a risk of dependence. Most people who experience PTSD undergo some type of psychotherapy, most commonly cognitive-behavioral therapy and/or group psychotherapy, EMDR and hypnotherapy.

A nurse is caring for a 74 year-old client with benign prostatic hypertrophy (BPH). Which finding would the nurse anticipate when assessing this client? a. Frequent urination b. Large volume of urinary output with each voiding c. Involuntary voiding with coughing and sneezing d. Urine is dark and concentrated

a. Frequent urination Clients with BPH have frequent urination in small amounts day and night, with difficulty starting the urinary stream.

A client is about to have an intravenous pyelogram (IVP). After the contrast material is injected, which client reaction should be acted upon immediately by a nurse? a. Hives with severe itching over the body b. Excessive salty taste in the mouth c. Face turning a deep ruddy red color d. Feeling of excessive warmth

a. Hives with severe itching over the body Hives over the body with severe itching is a sign of anaphylaxis and should be acted upon with the administration of epinephrine (adrenaline) immediately. The other reactions are considered normal after the dye injection. Prior to any dye injection procedure clients should be informed that these symptoms may occur.

A client diagnosed with diabetes mellitus has a blood glucose of 175 this morning. After the nurse reports this lab result along with the client's findings of being hungry and thirsty, what type of insulin should the nurse expect the health care provider to order? a. Humulin-R insulin b. Insulin glargine (Lantus) c. NPH insulin (Humulin-N) d. Mixture of insulin aspart (NovoLog) and insulin glargine (Lantus)

a. Humulin-R insulin Of these choices, the best answer is to administer the short-acting Humulin-R, or regular insulin; this insulin will begin to work in about 30 minutes. The client should wait to eat breakfast so that the insulin can begin working. Never mix insulin glargine (Lantus), which is a long-acting insulin, with any other kind of insulin. NPH is an intermediate-acting insulin and would not provide the needed quick response.

A nurse is providing foot care instructions to a patient with arterial insufficiency. The nurse would identify the need for additional teaching if the patient makes which statement? a. I will trim corns and calluses regularly b. I cannot go barefoot around my house c. I can only wear cotton socks d. I should ask a family member to inspect my fee daily

a. I will trim corns and calluses regularly Older adults should not cut their nails, corns, and calluses. They should have them trimmed by their HCP, nurses, or another provider who specializes in foot care. Older adult patients who have diagnosis of diabetes or vascular disease often have decreased circulation and sensation in one or both feet. Their vision may also be impaired. Therefore, they need to be taught to examine their feet daily or have someone else do so. They should wear cotton socks that have not been mended, and they should always wear shoes when out of bed.

The patient needs assistance to insert bilateral in-the-ear hearing aids. What action should the nurse take before inserting hearing aids? a. Identify the hearing aid programmed for each ear. b. Adjust the volume control to its highest setting c. Clean the hearing aid with plain soap and water d. Grasp the open battery door to use as a handle

a. Identify the hearing aid programmed for each ear. Since hearing aids are customized for each ear, the nurse should make sure that the correct hearing aid is inserted in the correct ear (a red dot indicates the right ear). The volume should be turned down when inserting the devices and adjusted after they are in the ear. Hearing aids should only be cleaned with soft cloth; water or alcohol can damage the device. The battery door should never be used as a handle.

Parents of a 4 year-old boy have just been informed that their son has a congenital neurologic demyelinating disorder that is terminal. The nurse anticipates their reaction to be in which phase of the crisis process? a. Impact phase b. Crisis phase c. Pre-crisis phase d. Resolution phase

a. Impact phase There is no data to determine their response phase except the time frame of recent bad news. The impact of crisis is indicative of high levels of stress, sense of helplessness, confusion, disorganization, and the inability to apply problem-solving behavior.

The nurse is assessing a client who had a stroke and underwent a carotid endarterectomy. The client is now experiencing motor deficits and communication problems. Which of the following findings requires immediate follow-up? a. Increased pulse and decreased blood pressure b. Slurred speech and inability to write clearly c. Drowsiness and difficulty in dressing d. Weak cough and difficulty walking

a. Increased pulse and decreased blood pressure Increased pulse and decreased blood pressure may indicate hemorrhage, which is a complication of the surgery. The other options are related to findings of the stroke.

A registered nurse (RN) works for a visiting nurse agency (VNA) and makes a home visit to admit a client newly diagnosed with type 1 diabetes. The client has a small foot ulcer that was debrided and needs daily wound care. Which of the following options is the most important intervention to ensure a successful outcome? a. Involve the client in making health care decisions b. Schedule daily RN visits to the client c. Refer to a local diabetes support group d. Arrange for a friend or relative to visit daily

a. Involve the client in making health care decisions Although all interventions may benefit the client, the involvement of the client in making health care decisions improves outcomes. The client is more motivated to adhere to recommendations when involved in the process, setting priorities and making decisions.

The nurse listens to report about a newly admitted client who has a skin ulcer that's tested positive for MRSA (methicillin-resistant Staphylococcus aureus). What precautions must be taken for this hospitalized client? (Select all that apply.) a. Keep all equipment in the client's room for his/her sole use Correct! b. Place the client in a single room Correct! c. Wear mask when providing routine care to the client d. Perform hand hygiene after direct contact with the client and before leaving the room Correct! e. Keep the door to the room closed, with a notice for visitors

a. Keep all equipment in the client's room for his/her sole use b. Place the client in a single room d. Perform hand hygiene after direct contact with the client and before leaving the room e. Keep the door to the room closed, with a notice for visitors Contact precautions are recommended in acute care settings for MRSA when there's a risk for transmission or wounds that cannot be contained by dressings. The client should be in a single room, with the door closed; the sign on the door instructs visitors to report to the nurse before entering the room. All equipment, such as stethoscopes and blood pressure devices, should be for the client's sole use and kept in the room. Health care workers must perform hand hygiene (wash hands with soap and water) after direct contact with the client and his/her environment and before leaving the isolation room. Contact precautions require health care workers to wear gloves and a gown; a face mask is not necessary for routine care.

The partner of a patient with Alzheimer's disease expresses concern about the burden of caregiving. Which of these actions by the nurse should be a priority? a. Link the caregiver with a support group b. Ask the friends to visit regularly c. Schedule a home visit each week d. Request anti-anxiety prescription

a. Link the caregiver with a support group Assisting caregivers to locate and join support groups will be most helpful and effective. Families share feelings and learn about services such as respite care. Health education is also available through local and national Alzheimer's Association chapters.

A nurse is caring for a patient in skeletal traction. Which nursing intervention is appropriate for this patient? a. Maintain correct body alignment b. Use alcohol or iodine-based products to clean around the pins c. Remove the weights when turning the patient d. Maintain a supine position at all times

a. Maintain correct body alignment The best response is to maintain correct body alignment. Skeletal traction is used to maintain proper alignment of the bones while healing. A pin or wire is inserted through the bone and weights are applied, using a system of ropes and pulleys attached to the bed frame to provide a constant pulling pressure - weights should not be removed or lived. Iodine-based solutions or alcohol should not be used for pin care because they can corrode the pins and/or stain the skin. Skeletal traction allows the patient to change position without interfering with the pull of traction, by the head of the bed must be completely lowered several times a day to prevent hip flexion contractures.

The nurse is providing care to a client who has just received an epidural for anesthesia during labor. The nurse recognizes which of the following as the most important nursing intervention following this procedure? a. Monitor maternal blood pressure for possible hypotension b. Monitor the fetus for possible tachycardia c. Monitor maternal pulse for possible bradycardia d. Reduce the intravenous fluid infusion to a keep open rate

a. Monitor maternal blood pressure for possible hypotension The most important nursing intervention for a woman who has received an epidural block is to frequently monitor the maternal blood pressure for signs of hypotension. After an epidural in the laboring client, IV fluids would be increased to prevent hypotension. The nurse would observe for signs of fetal bradycardia (not tachycardia) following an epidural and monitor for signs of maternal tachycardia, secondary to maternal hypotension.

A family arrives at the emergency department. A parent believes the child ingested an undetermined number of acetaminophen tablets approximately 1 hour ago. The serum acetaminophen level confirms acute poisoning. Which of these orders should be implemented first? a. N-acetylcysteine (NAC) (Mucomyst) b. Ondansetron (Zofran) 0.1 mg/kg for nausea c. Consultation with a medical toxicologist d. Oral activated charcoal therapy

a. N-acetylcysteine (NAC) (Mucomyst) The child should be started on the antidote for acetaminophen, N-acetylcysteine (NAC) because, without rapid intervention, acetaminophen toxicity can lead to liver failure and death. The amount ordered depends on the serum acetaminophen level. Although nausea and vomiting can be a side effect of acetaminophen poisoning (and even treatment with oral NAC), an antiemetic is not the priority. Activated charcoal is not recommended when more than 1 hour has elapsed after ingestion. Eventually, either a medical toxicologist or hepatologist can be consulted if the child experiences liver dysfunction.

A client has returned from a cardiac catheterization that was two hours ago. Which finding would indicate that the client has a potential complication from the procedure? a. No pulse in the affected extremity b. Increased blood pressure c. Increased heart rate d. Decreased urine output

a. No pulse in the affected extremity Loss of the pulse in the extremity would indicate a potential severe spasm of the artery or clot formation to the extent of an occlusion below the site of insertion. It is not uncommon that initially the pulse may be intermittently weaker from the baseline. However, a total loss of the pulse is a nursing emergency. The health care provider needs immediate notification.

The nurse is assessing a 4 year-old child who is in skeletal traction 24 hours after surgical repair of a fractured femur. The child is crying and reports having severe pain. The right foot is pale and there is no palpable pulse. What action should the nurse take first? a. Notify the health care provider b, Administer the ordered PRN medication c. Reassess the extremity in 15 minutes d. Readjust the traction for comfort

a. Notify the health care provider Pain and absence of a pulse within 48-72 hours after a severe injury to an extremity suggests acute compartment syndrome. This condition occurs when there's a build up of pressure within the muscles; this pressure decreases blood flow and can cause muscle and nerve damage. Acute compartment syndrome is a medical emergency. Surgery is needed immediately; delaying surgery can lead to permanent damage to the extremity.

A nurse is performing physical assessments on adolescents. What finding should the nurse anticipate concerning female growth spurts? a. Occur about two years earlier than for males b. Start just prior to the onset of puberty c. Characterized by an increase in height of 4 inches each year d. Begin about the same time for males

a. Occur about two years earlier than for males Normally, females in their teenage years experience a growth spurt about two years earlier than their male peers.

A nurse is performing nutritional assessment on a 2yo child. Which of these principles should the nurse apply? a. Serving size at this age is about 2 tablespoons b. Increased serum albumin or pre albumin levels indicate malnutrition c. An accurate measurement of intake is not reliable d. Total intake varies greatly every day

a. Serving size at this age is about 2 tablespoons In children, a greater guide to serving sizes is one tablespoon of solid food per year of age. Understanding this, the nurse can access adequacy of intake for any child.

The nurse is reviewing the laboratory results for several clients. Which of the laboratory result indicates a client with partly compensated metabolic acidosis? a. PaCO2 30 mm Hg b. Hemoglobin 15 g/dL (150 g//L) c. Sodium 130 mEq/L (130 mmol/L) d. Chloride 100 mEq/L (100 mmol/L)

a. PaCO2 30 mm Hg Metabolic acidosis can be caused by many conditions, including renal failure, shock, severe diarrhea, dehydration, diabetic acidosis, and salicylate poisoning. With metabolic acidosis, you should expect a low pH (less than 7.35) and a low HCO3 (less than 22 mEq/L.) Compensation means the body is trying to get the pH back in balance; therefore, a pure metabolic acidosis should elicit a compensatory decrease in PaCO2 (normal is 35-45 mm Hg.) The hemoglobin is within normal limits (WNL) for both males and females. The chloride and sodium results are also WNL.

The registered nurse (RN) is planning the care of an 80-year-old client with skin abrasions from a fall in the home. What aspect of this client's care is the primary responsibility of the nurse? a. Perform a head-to-toe assessment b. Apply lotion to areas of the skin not affected by the fall c. Report findings of any break in the skin's integrity d. Identify changes in skin color

a. Perform a head-to-toe assessment The RN is responsible to conduct a thorough assessment and evaluation of all body systems for this client. The nurse would document information collected during the focused assessment, such as changes in skin color and breaks in the skin's integrity. Applying lotion would not be a primary responsibility.

A client was just taken off the ventilator after surgery and has a nasogastric tube draining bile-colored liquids. Which nursing intervention would provide the most comfort to the client? a. Perform frequent oral care using a tooth sponge b. Allow the client to melt ice chips in the mouth c. Swab the mouth with glycerin swabs d. Provide mints to freshen the breath

a. Perform frequent oral care using a tooth sponge Frequent cleansing and stimulation of the mucous membrane is important for a client with a nasogastric tube to prevent development of lesions and to promote comfort. Ice chips or mints could be contraindicated with a nasogastric tube. Glycerin swabs have no mechanical or cleansing value and should not be used.

A 3 year-old child is brought to the health clinic. The grandmother reports that the child is always "scratching his bottom" and is "extremely irritable." Based on this information, which health issue would the nurse assess for initially? a. Pinworm b. Scabies c. Allergies d. Ringworm

a. Pinworm Findings of pinworm infection include intense perianal itching. The itching is usually worse at night, which is why the child will also exhibit poor sleep patterns, general irritability, restlessness, bedwetting, distractibility and a short attention span. The eggs will stick to a piece of clear cellophane tape placed against the rectum and the eggs can be seen under a microscope. The nurse can also take some samples from under the child's fingernails to look for eggs. Recall tip: the "P in worms" are found where the "pooh" comes out - the anal/rectal area. Scabies is an itchy skin condition caused by a tiny mite that burrows under the skin, causing small, itchy bumps or blisters; the most commonly affected areas of the body are the hands and feet. Ringworm is a fungus with characteristic round, itchy irritations on the skin.

The nurse is caring for a client in the coronary care unit who has developed acute renal failure as a consequence of cardiogenic shock. Which of the following findings are consistent with the diagnosis? (Select all that apply.) a. Pitting sacral edema b. Jugular vein distention c. Weight loss d. Oliguria e. Crackles on auscultation in bilateral bases

a. Pitting sacral edema b. Jugular vein distention d. Oliguria e. Crackles on auscultation in bilateral bases Findings related to fluid retention and heart failure are expected, because the kidneys are unable to function properly due to a decrease in glomerular filtration rate and tubular necrosis. In the bed-bound client, pitting sacral edema would be seen, since fluid follows gravity. Weight gain, jugular vein distention, oliguria and crackles in the lungs would also be expected with fluid overload in this client. Treatment consists of diuresis, with a possible small fluid challenge, if the client can tolerate it, to correct pre-renal azotemia. If these options are not effective or inappropriate for the client, dialysis or ultrafiltration may be used to remove excess fluid. In many cases, this type of treatment is temporary, and can be stopped as the kidney function improves with improved urine output and decreasing creatinine levels.

The nurse is caring for a neonate immediately following a vaginal delivery. Which of the following interventions will promote temperature regulation in the neonate? Select all that apply. a. Place the neonate under a radiant warmer b. Bathe the neonate to remove contaminants after delivery c. Wrap the neonate in blankets d. Dry the neonate in warm towels e. Encourage skin to skin contact with the mother

a. Place the neonate under a radiant warmer c. Wrap the neonate in blankets d. Dry the neonate in warm towels e. Encourage skin to skin contact with the mother After drying off the wet amniotic fluid, placing the neonate under the radiant warmer or placing the neonate skin to skin against the mother will provide a source of heat for the neonate. Wrapping the neonate in blankets will help to reduce heat loss. The neonate should not be bathed until the temperature is stabilized.

A patient is admitted to the rehab unit after having had a CVA with residual mild dysphagia. The appropriate intervention for this patient is which action? a. Place the patient in an upright position while eating b. Tilt the head back while eating to facilitate the swallowing reflex c. Offer finger foods such as crackers or pretzels d. Initially place patient on clear liquids

a. Place the patient in an upright position while eating An upright position facilitates proper chewing and swallowing. Liquid diet is contraindicated with patients with swallowing difficulties.

A Native-American chief visits his newborn son and performs a traditional ceremony that involves feathers and chanting. The charge nurse tells a colleague, "I wonder if he has any idea how ridiculous he looks - he's a grown man!" This statement is an example of which non-therapeutic approach? a. Prejudice b. Ethnocentrism c. Stereotyping d. Discrimination

a. Prejudice In this question, the nurse is reacting to the chief's behavior, which is an example of prejudice. Prejudice reflects the overall attitude and emotional response (both positive and negative, conscious and non-conscious) to a group. Discrimination refers to differences in actions towards different groups on the basis of prejudice. Stereotypes are cognitions or beliefs used to categorize others and systemize information in order to better predict behavior and react. Stereotypes can be used to develop prejudices toward others and to discriminate. Ethnocentrism is the practice of making judgments about other cultures based on the values and beliefs of one's own culture (especially related to language, customs and religion.)

The health care provider orders an osmotic diuretic for a client diagnosed with a traumatic brain injury (TBI). Why is this medication ordered? a. Reduce intracranial pressure b. Reduce pulmonary edema c. Prevent seizures d. Prevent electrolyte imbalance

a. Reduce intracranial pressure Osmotic diuretics, such as mannitol, are used to reduce intracranial or intraocular pressure. Osmotic diuretics reduce the amount of water normally reabsorbed by the renal tubules and loop of Henle, so urinary output is increased. Osmotic diuretics can cause excessive loss of water and electrolytes, which can lead to serious electrolyte imbalances. In addition to water intoxication and dehydration, adverse reactions of osmotic diuretics include pulmonary edema and circulatory overload. Anticonvulsants prevent seizures.

A client with goiter is treated preoperatively with potassium iodide. What should the nurse recognize as the purpose of this medication? a. Reduce vascularity of the thyroid gland b. Balance serum enzymes and electrolytes c. Correct chronic hyperthyroidism d. Destroy the thyroid gland function

a. Reduce vascularity of the thyroid gland Potassium iodide solution (Iosat, SSKI, ThyroSafe, ThyroShield) may be used preoperatively to reduce the size and vascularity of the thyroid gland.

There is an order for a 25-year-old client, who is unresponsive after suffering a traumatic brain injury, to be transferred from the hospital to a long-term care facility (LTC) today. To which staff member should the charge nurse assign this client? a. Registered nurse (RN) b. Licensed practical nurse (LPN) c. Nursing student in final semester before graduation d. Unlicensed assistive person (UAP)

a. Registered nurse (RN) The RN is responsible for facilitating continuity of care for clients and their families during the transfer from one health care setting to another. The transfer to a LTC facility often requires referrals and coordinating information from many different providers about treatments, therapies and medications.

The nurse performs a heel stick for a blood glucose check on a 1 hour old, full-term newborn who weighed 9 lbs at birth. The serum glucose reading is 45 mg/dL. What action is needed by the nurse? a. Repeat the test in 2 hours b. Give oral glucose water c. Notify the pediatrician d. Check the pulse oximetry reading

a. Repeat the test in 2 hours A serum glucose of 45 mg/dL is considered normal (normal range for the neonate is about 40-90). Neonatal hypoglycemia is defined as a blood glucose level of less than 30 in the first 24 hours of life and less than 45 in thereafter. Risk for hypoglycemia includes newborns who weigh more than 4 kg or less than 2 kg at birth, are large for gestational age; also gestational age less than 37 weeks and newborns suspected of hypoglycemia in the first hour of life. Due to the weight of the newborn, repeat blood glucose testing is indicated.

The 75 year-old female client is newly diagnosed with urge incontinence. She confides that she is often incontinent of large amounts of urine and expresses a fear of falling when rushing to the bathroom. What are the most appropriate nursing interventions to review with the client? (Select all that apply.) a. Restrict foods that may irritate the bladder b. Review preoperative instructions c. Perform pelvic floor muscle exercises d. Assist with pessary insertion e. Schedule urination

a. Restrict foods that may irritate the bladder c. Perform pelvic floor muscle exercises e. Schedule urination Urge incontinence involves periodic, but frequent, leakage of urine. Treatment involves treating the underlying cause, but in most cases, no cause can be found. Medication and behavioral interventions are tried before surgery is considered. Behavioral changes include bladder retraining with urge suppression, restricting foods that may irritate the bladder, drinking less than 8 ounces of fluid at one time and pelvic floor muscle exercises. A pessary is commonly used in the management of pelvic support defects such as cystocele and rectocele; it can also be used in the treatment of urinary stress incontinence, but not urge incontinence.

A nurse is reviewing laboratory results on a client diagnosed with acute renal failure. Which lab result should be reported immediately? a. Serum potassium 6 mEq/L (6 mmol/L) b. Hemoglobin of 9.3 mg/dL (93 g/L) c. Venous blood pH 7.30 d. Blood urea nitrogen 50 mg/dL (17.9 mmol/L)

a. Serum potassium 6 mEq/L (6 mmol/L) Although all of these findings are abnormal, the elevated potassium level is a life-threatening finding and must be reported immediately. Serious consequences of hyperkalemia include heart block, asystole and life-threatening ventricular dysrhythmias. Anemia (hemoglobin less than 13 g/dL [130 g/L] in men or less than 12 mg/dL [120 g/L] in women) is common with kidney disease. Blood urea nitrogen (BUN) will be increased in acute renal failure (7 to 30 mg/dL [2.5 to 10.7 mmol/L] is a considered normal).

A patient is admitted to an impatient crisis unit with the diagnosis of acute mania and has been placed in seclusion. The nurse is assigned to observe the patient at all times. It is now time for the patient's dinner. What action should the nurse take next? a. Serve dinner in the seclusion room maintaining close observation. b. Obtain a contract for safe behavior before accompanying the patient to the dining room c. Accompany the patient to the dining room and maintain observation d. Hold the meal until after the seclusion order is discontinued.

a. Serve dinner in the seclusion room maintaining close observation. Seclusion is ordered by the physician and requires continuous observation, unless the order is discontinued or amended. It is incorrect to amend the seclusion or mealtime. Meals can be eaten in the seclusion room with the nurse continuing 1:1 observation. Meals should be offered on time and should not be withheld. Contracts for safe behavior are meaningless in the presence of psychotic behavior - mania

The nurse is reviewing and reinforcing information about the use of a peak flow meter with a client newly diagnosed with asthma. Which of these statements should the nurse include when explaining how to use the peak flow meter? (Select all that apply.) a. Set the marker to the bottom of the numeric scale on the device b. Record the number achieved on the indicator; this is your peak flow rate c. Blow slowly into the mouthpiece d. Take a deep breath before placing your lips tightly around the mouthpiece e. Repeat the steps for a total of two times and record the highest peak flow reading of the two

a. Set the marker to the bottom of the numeric scale on the device b. Record the number achieved on the indicator; this is your peak flow rate d. Take a deep breath before placing your lips tightly around the mouthpiece The steps for correctly using the peak flow meter include: move the indicator to the bottom of the numbered scale, stand up, take a deep breath, place the mouthpiece in mouth and close lips around it, blow out hard and fast with a single blow, and record the number achieved on the indicator. The steps should be repeated two more times, for a total of three attempts and the highest peak flow reading of the three is recorded.

The nurse recognizes that obtaining accurate post anesthesia vital signs is extremely important. Which of the following client conditions are not appropriate for electronic blood pressure measurement? (Select all that apply.) a. Shivering b. Irregular heart rate c. Peripheral vascular obstruction d. Blood pressure greater than 140 mm Hg systolic

a. Shivering b. Irregular heart rate c. Peripheral vascular obstruction Clients with irregular heart rates, peripheral vascular disease, seizures, tremors, and shivering are not candidates for using an electronic blood pressure machine.

A nurse is teaching the parent of a 9 month-old infant about diaper dermatitis. Which of these actions would be appropriate for the nurse to include during the teaching? a. Stop any new food that was added to the infant's diet prior to the rash b. Use commercial baby wipes with each diaper change c. Do not use occlusive ointments on the rash d. Use only cloth diapers that are rinsed in bleach

a. Stop any new food that was added to the infant's diet prior to the rash The addition of new foods to the infant's diet commonly can cause diaper dermatitis. The other actions are incorrect to deal with this problem.

During a home visit, the nurse observes the mother of a school-aged child in a long leg synthetic cast using a cloth covered wooden spoon handle to relieve itching inside the cast. Which response by the nurse is most appropriate? a. Suggest placing an ice pack (protected by plastic) over the area that is itching b. Instruct them to blow hot air from a hand-held hair dryer into the cast c. Remind the mother and child that itching is normal d. No response is needed because the mother's behavior is appropriate

a. Suggest placing an ice pack (protected by plastic) over the area that is itching Because itching is a common and frustrating problem for a person with a cast, it would not be therapeutic to simply remind the mother and child that itching is normal. But using anything to scratch the skin inside the cast is not recommended because it can injury the skin and increase risk for infection. Patents can use a hair dryer on cool setting. Cool temperatures can constrict blood vessels, minimizing itching (just like heat vasodilates to intensify itching).

The client is prescribed dexamethasone by mouth every other day and asks the nurse for more information about the medication. What information would the nurse want to share with the client? (Select all that apply) a. Take the medication with food b. Do not get any immunizations or skin tests c. Take the medication at bedtime d. Avoid dairy products e. Mark your calendar to keep track of doses

a. Take the medication with food b. Do not get any immunizations or skin tests e. Mark your calendar to keep track of doses Dexamethasone is a glucocorticosteroid used for its anti-inflammatory properties. It is best to take the medication in the morning, before 9:00 am, with food or milk to avoid stomach upset. A low-sodium diet is usually prescribed because the drug can cause an elevation in blood pressure, salt and water retention, and increased potassium loss. Dexamethasone also causes calcium loss; the client should increase calcium in the diet and take a calcium supplement. Because the medication affects the immune system, it could make vaccinations ineffective and/or lead to serious infections. It's always a good idea for clients to keep track of medication administration, particularly when they are not taking the medication every day.

The registered nurse (RN) and the unlicensed assistive person (UAP) are caring for clients on a surgical unit. Which action(s) by the UAP warrant immediate intervention? (Select all that apply.) a. The UAP assists a client, who had a total knee replacement two days ago, to shave using a straight-edge razor b. The UAP assists a client, who received an IV narcotic analgesic 30 minutes ago, to ambulate in the hall c. The UAP applies moisture barrier cream to the client's excoriated perianal area d. The UAP empties the indwelling catheter bag for the client who had a transurethral resection of the prostate (TURP) yesterday e. The UAP applies a fingertip pulse oximeter on a client's finger with dark blue nail polish

a. The UAP assists a client, who had a total knee replacement two days ago, to shave using a straight-edge razor b. The UAP assists a client, who received an IV narcotic analgesic 30 minutes ago, to ambulate in the hall e. The UAP applies a fingertip pulse oximeter on a client's finger with dark blue nail polish The UAP can perform a number of nursing tasks, such as emptying an indwelling urinary catheter bag and applying moisture barrier cream after peri care. However, it is unsafe for the UAP to ambulate a client who recently received an IV push narcotic. Although UAP can shave clients, it is unsafe to shave someone using a straight-edge razor because a client who had knee replacement surgery is probably taking an anticoagulant; only an electric razor should be used. Pulse oximeter readings must be done on a finger that is warm and free from dark fingernail polish.

A nurse uses the New Ballard Scale to assess gestational age of a newborn. The assessment score total is very high. What is a reasonable interpretation of this result? a. The baby is post-term b. The baby is premature c. The baby experienced distress during labor d. The baby is large for gestational age

a. The baby is post-term Birth weight and gestational age are important indicators of the newborn's health and are used to identify any (potential) problems. The New Ballard Scale can help differentiate, for example, between a small for gestational age baby and one that is premature. The New Ballard Scale scoring system adds up the individual scores for 6 external physical assessments and 6 neuromuscular assessments; the total score may range from -10 to 50. Premature babies have lower scores; higher scores correlate with post-maturity. Fetal distress during labor can result in lower scores.

During a situation of pain management, which statement is a priority to consider for the ethical guidance of a nurse? a. The client's self-report is the most important consideration b. Cultural sensitivity is fundamental to pain management c. Clients have the right to have their pain relieved d. Nurses should not prejudge a client's pain using their own values

a. The client's self-report is the most important consideration Pain is a complex phenomenon that is perceived differently by each individual. Pain is whatever the client says it is. The other statements are correct but not the most important consideration.

The nurse is providing discharge teaching to the parents of a 15 month-old child diagnosed with Kawasaki disease (mucocutaneous lymph node syndrome or infantile polyarteritis). The child has received immunoglobulin therapy. Which instruction point would be appropriate to include during the discussion? a. The measles, mumps and rubella vaccine should be delayed b. Complete recovery is expected within several days c. High doses of aspirin will be continued for some time d. Active range of motion exercises should be done frequently

a. The measles, mumps and rubella vaccine should be delayed Discharge instructions for a child with Kawasaki disease should include the information that immunoglobulin therapy may interfere with the body's ability to form appropriate amounts of antibodies. Therefore, live or attenuated (weakened) immunizations should be delayed. The MMR vaccine contains three live attenuated viruses and should be delayed until the child's immune system recovers from this treatment.

A client has been taking alprazolam for three days. The nurse should expect to find which intended effect of this drug? a. Tranquilization and calming effects b. Increased coordination and ability to concentrate c. Relief of insomnia and phobias d. Sedation and long-term analgesia

a. Tranquilization and calming effects Alprazolam (Xanax) is a benzodiazepine used in the treatment of anxiety, panic disorder, and anxiety associated with depression; it is also beneficial to those suffering from sleep disorders. This medication is a central nervous system depressant, producing a drowsy or calming effect; it may cause a lack of coordination. Alprazolam has a very short half-life and produces immediate symptom relief. It does not cause analgesia nor is it used to treat phobias.

Which situation requires hand washing? Select all that apply a. after cleaning a wound b. after contact with inanimate objects in the immediate vicinity of a patient c. prior to eating d. before having direct contact with a patient e. after making a chart entry

a. after cleaning a wound b. after contact with inanimate objects in the immediate vicinity of a patient c. prior to eating d. before having direct contact with a patient Handwashing is still the simplest and most effective strategy to prevent the spread of infection. It is necessary to wash one's hands to protect oneself prior to eating, after removing gloves following any patient procedure, and even after having contact with intact ski or objects in the patient's room. However, it is not necessary to wash hands after handling every chart (although using an alcohol-based hand rub would be advisable).

The medication benztropine mesylate is ordered, but the nurse incorrectly administers carvedilol. What are the most important actions the nurse should take after making this medication error? Select all that apply a. document administration of carvedilol b. monitor and document the patient's BP c. notify HCP d. notify patient e. notify nurse manager

a. document administration of carvedilol b. monitor and document the patient's BP c. notify HCP e. notify nurse manager When the nurse makes a medication error, the patient's safety and well-being are the top priority. The nurse will document giving the beta blocker carvedilol and as well as any effects the medication has on the patient. The HCP must be notified; the nurse will document that the provider was called and that orders were implemented. The nurse manager must be notified. Once the patient is stable, the nurse will complete an incident/variance/quality assurance report within 24 hours of the incident. The initial disclosure of the medication error with the patient should occur as soon as reasonably possible after the event, usually 1-2 days after the event.

A patient diagnosed with gastroenteritis, caused by a salmonella infection. Which of these actions is the primary nursing intervention designed to limit the transmission of salmonella? a. hand hygiene before and after patient contact b. decontaminate with alcohol-based skin disinfectant c. wear 2 pairs of gloves when changing contaminated linens d. isolate the patient in a single room without a roommate

a. hand hygiene before and after patient contact Salmonella is a bacteria and of the causes for gastroenteritis. Gastroenteritis is characterized by acute onset of nausea, vomiting, abdominal cramps, and/or diarrhea. The CDC recommends using standard precautions for this illness, which is the why the primary nursing intervention is thorough handwashing before and after contact with patient using soap and water. Skin disinfectants can reduce the number of bacteria on the hands but cannot replace the importance of washing with soap and water. Contact isolation is not needed. Symptomatic patients can be cohorted. Double-gloving can be effective in surgery, but it is probably not needed when changing contaminated linens.

A neonate is having difficulty maintaining a temperature above 98F and is placed in an infant warming system. Which of the following actions will ensure the safety of the neonate? a. monitor temperature continuously b. avoid touching neonate with cold hands c. warm all medications and liquids before administration d. wrap the neonate snugly in a cotton blanket

a. monitor temperature continuously When using the warming device, the neonate's temperature should be continuously monitored using a probe that securely attached to the skin. Monitoring the neonate's temperature is the priority safety concern because the skin burns, permanent brain damage or even death can result due to improper use or monitoring of equipment. No clothing or swaddling is needed in the IWS; usually babies are dressed only in a diaper (although bubble wrap blankets or plastic wrap blankets can be used to minimize heat loss in high risk newborns). For healthy term newborns, nurses should warm their hands and stethoscopes prior to contact with the baby.

The client reports seeing spiders crawling on the walls, over the bed, and on the food tray, but denies feeling spiders crawling on the skin. The nurse determines that there are no spiders in the room. Which of the following assessments should the nurse use to document these findings? (Select all that apply.) a. spiders are not found in room b. incoherent speech c. visual hallucinations d. spiders reported to be crawling on patient e. delusional thinking f. tactile hallucinations g. spiders reported to be crawling on surfaces

a. spiders are not found in room c. visual hallucinations g. spiders reported to be crawling on surfaces Charting should be factual and not judgmental. It is important to evaluate the client's statements. The nurse looks to see if there are indeed spiders in the room surfaces. When the client sees something that is not present, this is called a visual hallucination. Because this client did not feel crawling spiders, tactile hallucinations is not an acceptable answer.

The preoperative nurse completes a health history on a client scheduled for a general anesthetic. Which of the following statements made by the client may indicate a risk for serious complications? (Select all that apply.) a."My lips have been itching ever since I blew up balloons for my daughter this morning." b. I am kind of sore all over; I get muscle cramps a lot." c. "I am scared of waking up during anesthesia." d. "My uncle got a fever and died unexpectedly during surgery."

a."My lips have been itching ever since I blew up balloons for my daughter this morning." b. I am kind of sore all over; I get muscle cramps a lot." d. "My uncle got a fever and died unexpectedly during surgery." Although anxiety is a pertinent consideration, it is not indicative of a serious complication. History of muscle cramps and/or unexplained death of a relative who experienced a febrile condition during surgery are both indicators of the inherited muscle disorder that is induced by many general anesthetic agents and leads to malignant hyperthermia. A history of itching or swelling of the lips is a possible indicator of a latex allergy.

The mother of a 4 month-old infant asks the nurse about the dangers of sunburn while they are on vacation at the beach. Which of these statements is the best advice about sun protection for this child? a. "Liberally apply a sunscreen with a minimum sun protective factor of 15 all over the body." b. "Dress the infant in lightweight long pants, long-sleeved shirts and brimmed hats." c. "Sunscreen should not be used on children." d. "You should keep the baby inside unless it's cloudy outside."

b. "Dress the infant in lightweight long pants, long-sleeved shirts and brimmed hats." Infants under 6 months of age should be kept out of the sun or shielded from it. Even on a cloudy day, the infant can be sunburned while near water. A hat and light protective clothing should be worn. Sunscreen is not generally recommended for infants under the age of 6 months; however, the American Academy of Pediatrics states that it can be applied to small areas of the baby's skin that are exposed to the sun (such as the baby's face or the back of the hands).

A client has just received an extracorporeal shock-wave lithotripsy (ESWL) procedure. What is the priority information the nurse should teach ? a. "Restrict milk and dairy products for one to two months." b. "Drink 3,000 to 4,000 mL of fluid each day for one month." c. "Increase intake of citrus fruits to three servings per day for two months." d. "Limit fluid intake to 1,000 mL each day for two months."

b. "Drink 3,000 to 4,000 mL of fluid each day for one month." Drinking three to four quarts (3,000 to 4,000 mL) of fluid each day will aid passage of fragments of the broken up renal calculi and help prevent formation of new calculi.

The 70 year-old male is recently diagnosed with osteoporosis. The nurse is teaching the client about this disease. Which of the following client responses requires further education by the nurse? (Select all that apply.) a. "I need to stop smoking." b. "Exercising in an aquatics class will make my bones much stronger." c. "I should ask for help to clean the gutters instead of climbing a ladder." d. "I don't believe the doctor because I heard that only women can get osteoporosis." e. "It sounds like I'll need to drink more milk and eat more cheese and yogurt."

b. "Exercising in an aquatics class will make my bones much stronger." d. "I don't believe the doctor because I heard that only women can get osteoporosis." Osteoporosis is commonly thought of as a "woman's problem", but after age 65 men and women are losing bone mass at about the same rate and calcium absorption decreases. Treatment for osteoporosis includes regular weight-bearing exercises, such as walking, in which bones and muscles work against gravity; aqua aerobics will not make bones stronger. The client needs adequate intake of calcium and vitamin D; while supplements may be needed, calcium is best absorbed from natural food sources. There is a direct link between tobacco use and decreased bone density.

The nurse is providing information to a client about a prescribed medication. Which one of these statements, if made by a client, indicates that teaching about propranolol (Inderal) has been effective? a. "I will expect to feel nervousness the first few weeks." b. "I can have a heart attack if I stop this medication suddenly." c. "I could have an increase in my heart rate for a few weeks." d. "I may experience seizures if I stop the medication abruptly."

b. "I can have a heart attack if I stop this medication suddenly." Propranolol is commonly used to treat hypertension, abnormal heart rhythms, heart disease and certain types of tremor. It is in a class of medications called beta blockers. Suddenly discontinuing a beta blocker can cause angina, hypertension, arrhythmias, or even a heart attack.

The nurse is performing the initial assessment of a client in the emergency department. Which statement by the client most strongly suggests domestic violence? a. "No one else in the family is as accident prone as I am." b. "I have tried leaving home, but have always gone back." c. "I have only been married for two months." d. "I am determined to leave my house in a week."

b. "I have tried leaving home, but have always gone back." Persons being abused or neglected often develop a high tolerance for abuse. They commonly blame themselves for being abused or neglected. All members in the family are affected by the behaviors of abuse, even if they are not the actual object of the abuse. For these reasons, persons who have been abused or neglected often have an extensive history of being abused. They struggle for a long time before actions are taken to leave permanently.

The nurse provides information to a client who is scheduled for a radiofrequency catheter ablation to control atrial fibrillation (AF). Which statement indicates the client correctly understands information about the procedure? a. "A cardioversion of my heart will be performed during the procedure." b. "I may need another ablation if this one doesn't work." c. "I'm a little concerned about having open heart surgery." d. "I am so glad I won't have to take that blood thinning medication any longer."

b. "I may need another ablation if this one doesn't work." Clients with AF who are unresponsive to antiarrhythmics or electrical cardioversion can be treated by radiofrequency catheter ablation. The procedure, which is usually performed under conscious sedation, destroys any abnormal pacemaker cells so that erratic electrical signals are normalized. Sometimes more than one ablation is needed. After an ablation, the client will need to continue taking an anticoagulant and even antiarrhythmic medication.

A nurse who cares for clients undergoing treatment for cancer might expect clients diagnosed with cancer to make the following statements. Based on an understanding of the stages of the grieving process, place the statements in the correct order. a. "If I eat a more balanced diet, I can live longer." b. "I think the tests got mixed up." c. "I am so mad at everyone for always reminding me that I have it." d. "I don't know where to go or what to do." e. "I will just go on with my life."

b. "I think the tests got mixed up." c. "I am so mad at everyone for always reminding me that I have it." a. "If I eat a more balanced diet, I can live longer." d. "I don't know where to go or what to do." e. "I will just go on with my life." The phases of loss or the grief process according to Dr. Kubler-Ross are: denial, anger, negotiation, depression and acceptance.

The nurse is working in a community health clinic answering telephone calls. Which client would the nurse recommend to be seen immediately by a health care provider? a. "I am an diabetic and today I have been going to the bathroom every hour." b. "I went to the bathroom and my urine looked very red but it didn't hurt when I went." c. "I started my period and now my urine has turned bright red." d. "I was started on some medicine yesterday for a urine infection and now my lower belly hurts when I go to the bathroom."

b. "I went to the bathroom and my urine looked very red but it didn't hurt when I went." The client with painless gross hematuria needs to be seen right away because this finding is closely associated with bladder cancer or kidney problems. The other complaints can be handled over the phone.

A client asks the nurse for information about a living will. Which statement made by the client demonstrates an understanding of a living will? (Select all that apply.) a. "It lists all my assets and how they should be divided among my family after I die." b. "I will need to identify someone to be my health care proxy." c. "My wishes for end of life treatment are stated in writing." d. "I should sit down and discuss my wishes for end of life care with my loved ones." e. "A living will is a legal document that becomes a permanent part of my health care record." f. "A living will must be renewed by a designated family member each time I am hospitalized."

b. "I will need to identify someone to be my health care proxy." c. "My wishes for end of life treatment are stated in writing." d. "I should sit down and discuss my wishes for end of life care with my loved ones." e. "A living will is a legal document that becomes a permanent part of my health care record." An advance health care directive is also known as a living will. It is a legal document in which a person specifies his or her wishes concerning medical treatments at the end of life, when s/he is unable to make those decisions. Advance care planning involves sharing personal values and wishes with loved ones and selecting someone, (called a medical power of attorney or health care proxy) who will eventually make medical decisions on the client's behalf. A living will does not expire; it remains in effect unless it is changed. A living will does not include information regarding assets or a person's estate.

The nurse is providing information to a 28 year-old female, who is a type 1 diabetic and planning a pregnancy. The nurse is assessing the client's understanding of insulin therapy during pregnancy. Which statement, made by the client, indicates a need for more teaching? a. "I will probably need to give myself more insulin during the second and third trimesters of my pregnancy." b. "I will need to increase my insulin dosage during the first three months of pregnancy." c. "I may be more likely to experience hypoglycemia during the first three months of pregnancy." d. "If I bottle-feed my baby, my insulin needs should return to normal within 7 to 10 days after birth."

b. "I will need to increase my insulin dosage during the first three months of pregnancy." Due to an increase in sensitivity to insulin, rapid fetal growth, and a reduction in eating associated with "morning sickness", women with pre-existing diabetes have a higher risk of hypoglycemia and will need decreased amounts of insulin. In the second trimester, the placenta is fully developed and hormone levels being to rise steadily, which increases maternal insulin needs. In the third trimester, insulin is absorbed more slowly and is less effective at lowering glucose, so the client may need larger doses of insulin. Insulin needs should return to normal soon after birth, if the client bottle feeds the baby.

A nurse is discussing Kawasaki disease with a group of student nurses. What statement made by a student about Kawasaki disease is incorrect and needs to be clarified? a. "In the second phase of the disease, findings include peeling of the skin on the hands and feet with joint and abdominal pain." b. "Kawasaki disease occurs most often in boys, who are younger than 5 years-old and of Hispanic descent." c. "Initial findings include a sudden high fever, often up to 104 F (40 C), which lasts one to two weeks." d. "It also called mucocutaneous lymph node syndrome because it affects the mucous membranes (inside the mouth, throat and nose), skin and lymph nodes."

b. "Kawasaki disease occurs most often in boys, who are younger than 5 years-old and of Hispanic descent." Kawasaki disease occurs most often in boys who are younger than 5 years-old and of Asian descent (especially Japanese). Findings in the initial phase include extremely red eyes (conjunctivitis); a rash on the main part of the body (trunk) and in the genital area; red, dry, cracked lips; a red, swollen tongue resembling a strawberry; swollen, red skin on the palms of the hands and the soles of the feet; swollen lymph nodes in the neck. Fever reduction signals the second phase, when the findings slowly go away. In the third phase, except for abnormal lab values, findings are gone (unless cardiac complications develop). The disease lasts from 2 to 12 weeks without treatment. With treatment, the child usually improves within 24 hours.

The parents of a 4 year-old hospitalized child tell the nurse, "We are leaving now and will be back at 6:00 pm." A few hours later the child asks a nurse when the parents will come again. What is the best response by the nurse? a. "When the clock hands are on the numbers 6 and 12." b. "They will be back right after you eat supper." c. "In about two hours, you will see them." d. "After you play awhile, they will be here."

b. "They will be back right after you eat supper." Time is not completely understood by preschoolers. Preschoolers interpret time with their own frame of reference of activities that they have experienced. Thus, it is best to explain time in relationship to a known and common event.

The nurse is assisting a client with substance use disorder (SUD) to deal with issues of guilt. Which response by the nurse would be best for this client? a. "You've caused a great deal of pain to your family and close friends. It will take time to undo all the things you've done." b. "What have you done that you feel most guilty about? What steps can you begin to take to help you lessen this guilt?" c. "Don't focus on the guilty feelings. These feelings will only lead to drinking and taking drugs." d. "Addiction usually causes people to feel guilty. Don't worry, it is a typical response due to your drinking behavior."

b. "What have you done that you feel most guilty about? What steps can you begin to take to help you lessen this guilt?" The correct response encourages the client to get in touch with his/her feelings and to utilize problem-solving steps to reduce guilt feelings; this is the only option that focuses on the client's actions. The other options are statements about general thoughts, with less focus on the client. Also note that three of the options are statements; if you have no idea about the correct answer, select the odd option (the question)

The client is newly diagnosed with gastroesophageal reflux disease (GERD). Which statement(s) made by the client indicates a need for further education about this disease? (Select all that apply.) a. "I am going to enroll in a smoking cessation class." b. "When I have a headache I'll be sure to take aspirin instead of acetaminophen." c. "I will take my omeprazole (Prilosec) as needed when I have heartburn." d. "I will drink less coffee and cola." e. "I'll wait a while after eating before I go exercise at the gym." f. "A bedtime snack may help me sleep better."

b. "When I have a headache I'll be sure to take aspirin instead of acetaminophen." c. "I will take my omeprazole (Prilosec) as needed when I have heartburn." f. "A bedtime snack may help me sleep better." GERD occurs as a result of gastric secretions from the stomach moving up the esophagus, usually because the lower esophageal sphincter is too relaxed. The client should eat meals several hours before lying down and give up those late night snacks which may trigger bedtime symptoms. Food and beverages that may trigger symptoms, such as caffeine and carbonated beverages, should be avoided. Proton pump inhibitors such as omeprazole (Prilosec) may take one to four days for their full effect, so they should be taken routinely, and a fast-acting antacid should be used for acute relief of heartburn symptoms. Aspirin and NSAIDs can aggravate GERD, so they should be avoided, and alternatives such as acetaminophen should be used instead. Avoiding tobacco and losing weight may also help improve heartburn.

At the client's request, the nurse performs a fingerstick to test the client's blood glucose and the results are 322 mg/dL (17.89 mmol/L). Following standing orders, the nurse administers 3 units of insulin lispro at 11 am. When does the nurse anticipate the insulin lispro will begin to act? a. 3:00 pm b. 11:15 am c. 1:00 pm d. 12 noon

b. 11:15 am The onset of action and peak for insulin lispro (Humalog), which is a rapid-acting insulin, is 10 to 15 minutes after administration. This type of insulin will peak in about 1.5 to 2.5 hours. It is designed to cover meals and lower high blood sugar readings.

The interdisciplinary team is reviewing charts for potential candidates for hospice care. Which of the following clients meet the criteria for hospice care? (Select all that apply.) a. 53 year-old client with chronic, unrelieved pain, who is addicted to narcotics following a back injury b. 8 year-old client with acute myelogenous leukemia, for whom all treatment options have failed c. 91 year-old with Alzheimer's disease, who is no longer able to eat or drink oral fluids d. 46 year-old with end stage liver disease, on a wait list for a donor organ e. 72 year-old with prostate cancer metastasized to the bone, who is receiving palliative radiation therapy

b. 8 year-old client with acute myelogenous leukemia, for whom all treatment options have failed c. 91 year-old with Alzheimer's disease, who is no longer able to eat or drink oral fluids e. 72 year-old with prostate cancer metastasized to the bone, who is receiving palliative radiation therapy Hospice care provides services for clients who are at the end of their life, usually with less than 6 months to live. There are no age requirements. Palliative care is provided by a multi-disciplinary team in a variety of settings, including the home, hospital or extended-care facilities. Clients actively seeking a cure or treatment for their disease do not meet the criteria for hospice care.

A nurse is to present information about Chinese folk medicine to a group of student nurses. Based on this cultural belief system, the nurse would explain that illness is attributed to which focus? a. The use of improper hot foods, herbs and plants b. A failure to keep the physiological processes of life in balance with nature and others c. Yin, the negative force that represents darkness, cold and emptiness d. Yang, the positive force that represents light, warmth and fullness

b. A failure to keep the physiological processes of life in balance with nature and others Chinese folk medicine proposes that health is regulated by the opposing forces of yin and yang. Under normal conditions, there is a dynamic equilibrium of these two physiological processes. Yin is the negative force characterized by darkness, cold and emptiness. Excessive yin predisposes one to nervousness. Yang is the positive force that represents light, warmth and fullness. Illness occurs when the balance between yin and yang is broken.

The charge nurse on the evening shift is asked to determine which patient is a candidate for discharge following an internal disaster in the hospital at 2100. which of these patients would the nurse select as a potential candidate for discharge? a. An older adult female who is actively dying and has a DNR b. A middle-aged adult with a history of DM1 and 1 day post DKA c. A young adult admitted at the beginning of the shift, with an asthma exacerbation d. An adolescent admitted on the day shift to rule out acute pancreatitis, who reports a history of alcohol abuse

b. A middle-aged adult with a history of DM1 and 1 day post DKA The patient selected to be discharged should be one whose condition is more stable than the others and where there is less of a risk for complications or instability after discharge. Although the patient with asthma has a chronic condition, she was just admitted and is experiencing acute exacerbation of the condition. The adolescent is experiencing an acute condition, probably brought on by alcohol abuse. Neither of these patients are stable enough for discharge. It is a humane choice to allow the patient who is in the process of dying to stay in the hospital.

A client who is unconscious is brought to the emergency department by an ambulance. What document should be given priority to guide the approach for the care of this client? a. Orders written by the health care provider in an emergency department b. A notarized original of the Advanced Directive brought in by the partner c. National statement of patient rights and the patient self-determination act d. Clinical pathway protocol of the agency and the emergency department

b. A notarized original of the advance directive brought in by the partner This document specifies the client's wishes of what actions are to be taken when the client becomes unable to make health care decisions. The advance directive often includes a living will and the power of attorney to whom will make the decisions for the client. The next document that would take precedent are the orders written by the heath care provider. The clinical pathways are used to evaluate the client's progress during therapy.

The nurse receives a client from the post anesthesia care unit following a left femoral-popliteal bypass graft procedure. Which of the following assessments requires immediate notification of the health care provider? a. Left foot is cool to the touch b. Absent left pedal pulse using Doppler analysis c. Inability to palpate the left pedal pulse d. Acute pain in the left lower leg

b. Absent left pedal pulse using Doppler analysis Although the inability to palpate the left pedal pulse, a cool extremity, and increased pain in the left lower leg are important findings, they all require additional nursing assessment prior to contacting the health care provider. In clients without palpable pedal pulses, the next step in the assessment is to perform a Doppler analysis. The inability to locate the left pedal pulse using the Doppler analysis requires immediately notifying the health care provider.

The nurse is making a home visit to a patient diagnosed with COPD. The patient tells the nurse that he used to be able to walk from the house to the mailbox without difficulty. Now, he has to pause to catch his breath halfway through the trip. Which nursing diagnosis is appropriate for the patient based on this assessment? a. Impaired mobility related to COPD as evidenced by a change b. Activity intolerance related to chronic tissue hypoxia as evidenced by fatigue c. Self-care deficit related to dyspnea as evidenced by fatigue d. Ineffective airway clearance related to increased bronchial secretions as evidenced by complaints

b. Activity intolerance related to chronic tissue hypoxia as evidenced by fatigue Activity intolerance describes a condition in which the patient's physiological capacity for activities is compromised. The other nursing diagnoses are not supported with data from the situation.

The charge nurse is making assignments for the shift. Which of these clients would be appropriate to assign to a licensed practical nurse (LPN)? a. A confused client whose family complains about the nursing care two days after the client's surgery b. An older adult client diagnosed with cystitis and has an indwelling urethral catheter c. A client admitted with the diagnosis of possible transient ischemic attack with unstable neurological signs d. A trauma victim with multiple lacerations that require complex dressing changes

b. An older adult client diagnosed with cystitis and has an indwelling urethral catheter The most stable client is the one diagnosed with cystitis. Care for this client has predictable outcomes and there is only a minimal risk for complications. The other clients require more complex care and independent, specialized nursing knowledge, skill or judgment that only an RN can provide.

Which statement is the correct stage of cognitive development for Piaget's: Adolescence-formal operations stage a. Concepts are attached to concrete situations b. Analyzes situations and uses abstract logic and reasoning c. Uses sucking, grasping, listening, and looking to earn about the environment d. Uses magical thinking and imagination

b. Analyzes situations and uses abstract logic and reasoning

The nurse is preparing to administer albuterol inhaled to a 11 year-old with asthma. Which assessment by the nurse indicates there is a need for the health care provider to adjust the medication? a. Temperature of 101 F (38.3 C) b. Apical pulse of 112 c. Lethargy d. Lower extremity edema

b. Apical pulse of 112 One of the more common adverse effects of beta adrenergic medications such as albuterol (AccuNeb, ProAir HFA, Proventil HFA, Ventolin HFA), is an increase in heart rate. Normal resting heart rate for children 10 years and older is the same as adults: 60-100 beats per minute.

A 3 year-old has just returned from surgery for application of a hip spica cast. What nursing action will be the priority? a. Drying the cast using a hair dryer set to "warm" b. Apply waterproof plastic tape to the cast around the genital area c. Use the crossbar to help turn the child from side to side d. Position the child flat in bed, repositioning from back to stomach every two to four hours

b. Apply waterproof plastic tape to the cast around the genital area The most important aspects of caring for the cast is to keep it clean and dry. Shortly after returning from surgery, waterproof plastic tape will be applied around the genital area to prevent soiling. The child should be turned every two hours to help facilitate drying, from side to side and front to back, with the head elevated at all times. If a crossbar is used to stabilize the legs, it should not be used to turn the child (it may break off). After the cast has completely dried and it becomes damp, it can be either exposed to air or a hair dryer (set to cool) may be used to help dry the cast.

There is an order to remove the client's nasogastric tube (NGT). What action is the safest when the tube is being removed? a. Offer sips of water in between every three-inch removal b. Ask the client to hold a breath until the tube is completely out c. Empty the tube of all drainage before pulling it out d. Bring the crash cart to the bedside

b. Ask the client to hold a breath until the tube is completely out Asking the client to hold his breath will close the epiglottis and help prevent aspiration. Occasionally, during insertion, the passing of the NGT is easier if the client swallows when it gets to the back of the throat. Emptying the tube of drainage is probably not possible. There's no need for the crash cart when removing a NGT.

Parents bring their special needs child to a community health center one day after an explosion occurred at the child's high school. It is determined that the child may be in a crisis state. Which of these interventions is appropriate to implement at this time? a. Discuss of variety of alternative approaches with the child b. Ask the parent to identify the major issues c. Make the child identify a specific problem d. Examine a variety of options with the patient

b. Ask the parent to identify the major issues If a client is unable to participate in problem solving because of developmental delays or altered mental status, then crisis intervention should not be attempted with the client. However, the family can be approached with the use of crisis intervention methods. The crisis intervention method includes five steps: identify the problem and then the alternatives, selection of an alternative, implementation, and evaluation of the outcome.

A client completes a fecal occult blood screening and the results come back positive. Which factor could have influenced this outcome? (Select all that apply.) a. Eating green, leafy vegetables b. Aspirin (ASA) therapy c. Drinking fruit juices that contain vitamin C d. Eating a steak dinner e. Teeth cleaning during regular dental visit f. Recent use of corticosteroids

b. Aspirin (ASA) therapy d. Eating a steak dinner e. Teeth cleaning during regular dental visit f. Recent use of corticosteroids Eating red meat, NSAIDs and steroid use can cause a false positive result; even bleeding gums can cause a false positive result. Using vitamin C supplements and fruit juices can cause a false negative result (because it interferes with the chemical reaction that indicates blood is present.) The test should be repeated and the client should be given specific instructions about special dental, dietary and drug restrictions.

The nurse is performing the following actions immediately following a delivery of a health, normal newborn. Indicate the correct sequence of actions by dragging and dropping the options below into the correct order. a. Administer Vitamin K b. Assess the infant's airway and breathing c. Perform bulb suctioning if excessive mucus is present d. Assess the infant's heart rate e. Place ID bands on infant and mother

b. Assess the infant's airway and breathing c. Perform bulb suctioning if excessive mucus is present d. Assess the infant's heart rate e. Place ID bands on infant and mother a. Administer Vitamin K Assessing the airway and respirations is the first action. Next if indicated, the baby should be suctioned. Then the heart rate is assessed. After these initial assessments, the identification bands are placed on both mother and baby. IM administration of vitamin K is recommended for the newborn but this can be done after the initial assessments and proper identification.

The client is diagnosed with gastroesophageal reflux disease (GERD). Which recommendation made by the nurse would be most helpful? a. Sit upright for at least half an hour after eating b. Avoid eating two hours before going to sleep c. Avoid liquids unless a thickening agent is used d. Maintain a diet of soft foods and cooked vegetables

b. Avoid eating two hours before going to sleep Eating before sleeping enhances regurgitation of stomach contents, which have increased acidity, into the esophagus. An upright posture should be maintained for about two hours after eating to allow for stomach emptying. Thickening agents and sitting upright for an hour after eating are interventions for clients with swallowing difficulties. Common dietary modifications in GERD include decreasing total fat intake; avoiding large meals, chocolate, caffeine or known irritants; a soft diet is not necessary.

The nurse is teaching a client with coronary artery disease about nutrition. What information should the nurse be sure to emphasize? a. Eat three balanced meals a day b. Avoid large and heavy meals c. Add complex carbohydrates to each meal d. Limit sodium to 7 grams per day

b. Avoid large and heavy meals Eating large, heavy meals can pull blood away from the heart for the digestion process. This may result in angina for clients with coronary artery disease. Sodium for clients with cardiac disease is limited to two grams per day. Three meals a day is a correct approach. However, it does not mention the size of the meal, which is more important.

A client with a brain tumor is scheduled for a spiral CT scan with contrast. Which of the options listed below would be a concern for the nurse when preparing the client for this test? (Select all that apply.) a. Paresthesia of the feet b. BUN is 40 mg/dL (14.28 mmol/L) c. History of asthma d. Positive pregnancy test e. Taking anticonvulsant medication

b. BUN is 40 mg/dL (14.28 mmol/L) c. History of asthma d. Positive pregnancy test People at higher risk for a reaction include those with past reactions to contrast media, asthma, and a history of heart, kidney and thyroid diseases. Individuals taking beta blockers or metformin are also at higher risk. Clients with poor renal function will not be able to clear the contrast agent from the kidneys (normal BUN is 7 - 20 mg/dL [2.5 - 7.1 mmol/L]). Pregnancy is typically a contraindication to a CT.

A respiratory therapist (RT) is collecting an arterial blood gas (ABG) sample. The RT must respond to an emergency and asks the nurse to manage the puncture site. Which actions should be completed? (Select all that apply.) a. Thoroughly wash the site with saline, then apply an antibacterial solution b. Check for distal capillary refill c. Apply snug gauze and secure with tape d. Remove dressing in one hour e. Apply pressure for 5 to 10 minutes

b. Check for distal capillary refill c. Apply snug gauze and secure with tape e. Apply pressure for 5 to 10 minutes Five to 10 minutes of pressure ensures adequate coagulation at the site. Checking capillary refill indicates if there are any changes to blood flow to the hand. The dressing can be removed prior to the next stick or within 24 hours.

A nurse is teaching adolescents about sexually transmitted diseases. What should the nurse emphasize is the most common infection? a. Herpes b. Chlamydia c. Gonorrhea d. Human immunodeficiency virus (HIV)

b. Chlamydia Chlamydia is the most frequently reported bacterial sexually transmitted disease in the United States. Prevention is similar to safe sex practices taught to prevent any sexually transmitted disease, such as abstinence, and the use of a condom and spermicide for protection during intercourse. This infection has subtle findings so the infected persons are less likely to pursue medical attention.

A 30 year-old client at 39-weeks gestation has just delivered and experienced a fetal demise. The client's partner is at the bedside. Which of the following nursing actions are appropriate at this time? (Select all that apply.) a. Place the infant on the maternal abdomen, skin-to-skin b. Clean and wrap the baby and offer it to the parents to view or hold when desired c. Stay with the parents and offer supportive care to both of them d. Ask the parents if there are any special religious or cultural rituals for neonatal death e. Offer the option of an autopsy to the parents at this time

b. Clean and wrap the baby and offer it to the parents to view or hold when desired Correct! c. Stay with the parents and offer supportive care to both of them Correct! d. Ask the parents if there are any special religious or cultural rituals for neonatal death Correct! Staying with the parents at this moment and offering physical and emotional support is appropriate. It is also appropriate to prepare the infant in a way that demonstrates care and respect for the baby and to offer everyone the opportunity to view and/or hold the infant as they desire. Placing a newborn on the mother's abdomen would be appropriate for a live birth, but inappropriate for this situation. The nurse must ask if there are cultural or religious rituals they would like for their infant. Although an autopsy should eventually be discussed, it would not be appropriate immediately after the birth.

The nurse is assessing a client who is two days post-surgery and notes new and sudden onset of confusion. There is an order to discharge the client to go home today. What would be the best action for the nurse to take? a. Teach a family member clean dressing change technique and address safety measures in the home b. Collaborate with the health care provider about the change of condition c. Make a clinic appointment with the primary health provider for follow-up care the next day d. Collaborate with the dietitian for increasing protein and calcium in the diet

b. Collaborate with the health care provider about the change of condition Although all the responses may be correct for a post-surgical client, a status change involving confusion must be reported, particularly if it is a new finding. As an advocate for the client, the nurse should protect the client from physical harm and collaborate with the health care provider about a change in the plan for discharge.

A community health clinic nurse is assessing a walk-in client who is experiencing lightheadedness. The client has a history of arthritis and takes naproxen (Aleve) and treats high cholesterol with fish oil and garlic. The assessment reveals that the client is pale, blood pressure is 88/40, pulse is 114, respiratory rate is 22, and temperature is 98.2 F (36.7 C). What specifically should the nurse ask this client about? (Select all that apply.) a. Tingling or numbness in the extremities b. Color of bowel movements c. Bruising d. Frequency and amount of naproxen used e. Photophobia

b. Color of bowel movements c. Bruising d. Frequency and amount of naproxen used NSAIDS (Aleve), fish oil, and garlic can all increase the risk for bleeding. The vital signs and pale skin color indicate possible hypovolemia (tachycardia and hypotension) secondary to blood loss. The nurse should inquire about other findings that may indicate bleeding, i.e., black tarry stools, bruising, and should determine the amount of NSAIDs taken daily.

A nurse is preparing a client for discharge following inpatient treatment for pulmonary tuberculosis (TB). Which of these instructions should be given to the client? a. Continue taking medications as prescribed b. Continue taking medications until findings are relieved c. Avoid contact with children, pregnant women or immunosupressed persons d. Take medication with aluminum hydroxide (Amphojel) if epigastric distress occurs

b. Continue taking medications until findings are relieved Early cessation of treatment may lead to development of drug resistant TB. Active TB is usually treated with a combination of four different antibiotics (isoniazid, rifampin, ethambutol and pyrazinamide) and can take anywhere from 6 to 12 months to completely kill the bacteria. As with any antibiotics, clients should continue to take medications even after they begin to feel better. There is no reason to avoid contact with children, pregnant women or immunosupressed persons once discharged from the hospital and adhering to medication schedule. Isoniazid should be taken on an empty stomach; ethambutol can be taken with food to avoid stomach upset. If taken with TB medications, Amphojel will interfere with absorption of these medications.

During a routine checkup, a client with a history of type 1 diabetes mellitus has the glycosolated hemoglobin (HbA1c) checked. The results indicate a level of 11%. Based on this result, what should the nurse emphasize during teaching? a. Rotation of injection sites b. Daily peripheral glucose monitoring c. Insulin mixing and preparation d. Review of diet and exercise recommendations

b. Daily peripheral glucose monitoring Normal results for Hg A1c (glycosolated hemoglobin) is 6% or less. Persons diagnosed with diabetes mellitus have guidelines designated by their health care provider; usually it's less than 7%. Hemoglobin A1c is an average serum glucose level for the prior three months. The peripheral stick for glucose is an approach to monitor daily fluctuations.

A young child is receiving treatment for lead poisoning. Which of the following is the most serious effect of long-term exposure to lead? a. Impaired kidney function b. Damage to the central nervous system c. Anemia and fatigue d. Lead colic and constipation

b. Damage to the central nervous system Lead toxicity can affect every organ system but it is especially dangerous for the brain. Lead can even alter the structure of the blood vessels in the brain and can lead to bleeding and brain swelling. In children, lead exposure is associated with lower IQ scores, learning disabilities, hyperactive behavior, and impaired hearing; higher levels of exposure can cause seizures and death. Neurological effects may persist into adulthood, despite treatment. Anemia (and fatigue), damage to the kidneys and abdominal pain (also called lead colic) are potentially reversible with treatment.

A client is admitted to the hospital following an automobile accident. Upon admission the client's blood alcohol concentration was 0.18%. Twelve hours after admission the client is diaphoretic, tremulous, and irritable; pulse and blood pressure measurements are elevated. The client states: "I have to get out of here." What is the most likely cause for these findings? a. Dissatisfaction with hospital care b. Early stage of alcohol withdrawal c. Anxiety related to being hospitalized d. Shock related to the injuries

b. Early stage of alcohol withdrawal This client's blood alcohol concentration is more than twice the legal limit in most states. After a period of heavy or prolonged alcohol use, people will experience alcohol withdrawal symptoms, such as insomnia, tremors, hyperactivity, hypertension, tachycardia and diaphoresis. The client must be treated immediately to prevent progression to more severe alcohol withdrawal symptoms, including seizures (which may begin 6-48 hours after cessation of alcohol intake) and delirium tremens (DTs).

The nurse is reviewing the medication administration record for a newly admitted client. The client is prescribed the beta blocker propranolol, but is not diagnosed with hypertension and does not have a history of heart disease. Which health issue might best explain the reason for prescribing propranolol? a. Raynaud's disease b. Essential tremors c. Parkinson's disease d. Schizophrenia

b. Essential tremors Propranolol is used to help control essential tremors. These tremors are the most common type of tremor; they are usually mild and not associated with any known pathology. Parkinson's tremor usually improves with dopaminergic and anticholinergic medications. Antipsychotic medications can cause tremors and they can be treated with benztropine. Beta blockers can aggravate symptoms of Raynaud's disease.

The nurse is assessing a client in the labor and delivery unit. Which of the following actions is correct when using palpation to assess the characteristics and pattern of uterine contractions? a. Place a hand on abdomen below the umbilicus and palpate uterine tone with fingertips b. Evaluate intensity by pressing fingertips into the uterine funds c. Determine frequency by timing the end of one contraction until the end of the next contraction d. Assess uterine contractions every 30 minutes throughout the first stage of labor

b. Evaluate intensity by pressing fingertips into the uterine funds To assess contractions for frequency, duration and intensity, the nurse will place one hand on the uterine fundus and use his or her fingers to feel the changes in the uterus as it contracts. The nurse can determine the frequency of the contractions by noting the time from the beginning of one contraction to the beginning of the next one. To determine the duration of the contraction, the nurse will note the time when tensing of the fundus is first felt (the beginning of one contraction) and again as relaxation occurs (end of contraction). It's best to time several consecutive contractions before charting frequency or duration.

The nurse and family members, who will be providing care at home, are discussing the client's continuing care needs after discharge to home. Which of these aspects of the discharge planning evaluation should receive priority consideration? a. Availability of community-based services b. Family's understanding of the client's health care needs c. Client's health insurance and prescription coverage d. Coordination of follow-up care with interdisciplinary team

b. Family's understanding of the client's health care needs Family members must be willing and able to provide the required care at the times needed and understand the client's health care needs before the client is discharged home. The discharge planning evaluation will take into account a wide variety of information, such as the home environment, and the availability of community-based services (such as support groups, hospice, or medical equipment and related supplies, etc.) Family members should understand the financial implications of discharge, including health insurance and prescription coverage.

The respiratory technician arrives to draw blood for arterial blood gas (ABG) analysis. What should the nurse understand about the procedure? a. Supplemental oxygen should be turned off 30 minutes prior to collecting the sample b. Firm pressure is applied over the puncture site for at least five minutes after the sample is drawn c. The blood sample must be kept at room temperature and delivered to the lab as soon as possible d. The femoral artery is the preferred sample site

b. Firm pressure is applied over the puncture site for at least five minutes after the sample is drawn The radial artery is preferred; the second choice is the brachial artery and then the femoral artery. If a client is receiving oxygen, it should not be turned off unless ordered. After drawing the sample, it's very important to press a gauze pad firmly over the puncture site until bleeding stops or at least five minutes. Do not ask the client to hold the pad because if insufficient pressure is used, a large painful hematoma may form. The sample of arterial blood must be kept cold, preferably on ice to minimize chemical reactions in the blood.

The lab results for a 70 year-old postoperative client indicate that the serum blood urea nitrogen (BUN), creatinine ratio, and hematocrit (HCT) levels are all elevated. Sodium, chloride and potassium lab results are slightly elevated. Based on these findings, which of the following issues may be the actual problem? a. Metabolic acidosis b. Fluid volume deficit c. Impaired gas exchange d. Renal insufficiency

b. Fluid volume deficit In fluid volume deficit, or hypovolemia, serum BUN and the creatinine ratio are elevated. HCT is also greater than normal because the RBCs are suspended in a decreased plasma volume. Urine specific gravity will also be increased. The age of the client and being NPO have contributed to the hypernatremia.

A 78yo reports having difficulty moving his bowels. What information is most important for the nurse to obtain during the assessment process? a. Trends in weight gain or loss b. Health history and patient's diet c. Elimination patterns over the past week d. Lab reports, including a CBC with differential

b. Health history and patient's diet The nurse should obtain the patient's health history, noting risk factors, comorbid conditions, and medications. The nurse should assess for patient's diet, including fiber intake. The nurse can then ask the patient to clarify what he means when he reports having difficulty moving his bowels (and determine if there are any misconception about bowel habits). Assessing the patient's health history and diet should uncover any eating or swallowing difficulties that could contribute to weight loss.

A client has received two units of whole blood today after an episode of gastrointestinal bleeding. Which laboratory report should the nurse be sure to monitor closely? a. White blood cells b. Hemoglobin and hematocrit c. Platelets d. Bleeding time

b. Hemoglobin and hematocrit The post-transfusion hematocrit provides immediate information about red cell replacement and if there is any continued blood loss; the follow-up hematocrit should be checked around 4 to 6 hours after the infusion is completed.

A hospitalized 8 month-old infant is receiving digoxin to treat tetralogy of Fallot. Prior to administering the next dose of medication, the parent reports that the baby vomited one time, just after breakfast. The heart rate is 72 BPM. What should be the initial response of the nurse? a. Give the dose after lunch b. Hold the medication c. Reduce the next dose by half d. Double the next dose

b. Hold the medication Toxic side effects of digoxin include bradycardia, dysrhythmia, nausea, vomiting, anorexia, dizziness, headache, weakness and fatigue. It isn't necessary to hold the medication for infants and children if there is only one episode of vomiting. However, it is appropriate to hold the medication and notify the health care provider for bradycardia. Normal resting heart rate for infants 1-11 months-old is 100-160 BPM.

During assessment of orthostatic vital signs on a client with cardiomyopathy, the nurse finds that the systolic blood pressure (BP) decreased from 145 to 110 mm Hg between the supine and upright positions while the heart rate (HR) rose from 72 to 96 beats per minute. In addition, the client reports feeling lightheaded when standing up. The nurse should implement which of the following actions? a. Increase fluids that are high in protein b. Instruct client to increase fluid intake for several hours c. Instruct the client to increase fluid intake for the next two days d. Restrict fluids for the next few hours

b. Instruct client to increase fluid intake for several hours This client is experiencing postural hypotension, a decrease in systolic blood pressure 15 mm Hg accompanied by an increase in heart rate 15 to 20 beats above the baseline with a change in position from supine to upright. This is often accompanied by lightheadedness. Fluid replacement is appropriate, but must be instituted very cautiously, as this client with cardiomyopathy will also be very sensitive to changes in fluid status and fluid overload may develop rapidly with aggressive rehydration. After the client increases fluid intake for one to two hours, the client should be reassessed for resolution of the postural hypotension.

A nurse in the emergency department suspects domestic violence as the cause of a client's injuries. What action should the nurse take first? a. Refer the patient to a victim advocate b. Interview the patient privately c. Photograph the specific injuries and include with documentation d. Ask the client if there are any old injuries

b. Interview the patient privately It is critical to separate the client from anyone who came in with the client, whether it be a partner or friend, and interview the client in privacy. With the use of the nursing process, the nurse's first action when a client is unstable or has potential problems is further assessment of the situation. The correct answer is the one most focused on gathering more information. During the private intake assessment the nurse would possibly institute the other actions in the remaining options.

The nurse is teaching the client with chronic renal failure (CRF) about medications. The client questions the purpose of taking aluminum hydroxide. What is the best explanation for the nurse to give the client about the therapeutic effects of this medication? a. Amphojel increases urine output b. It decreases serum phosphate c. The drug is taken to control gastric acid secretion d. It will reduce serum calcium

b. It decreases serum phosphate Aluminum binds phosphates that tend to accumulate in the client with chronic renal failure due to decreased filtration capacity of the kidney. Antacids such as Amphojel are commonly used to decrease serum phosphate.

The RN is making a presentation about Lyme disease to a group of volunteers who host hiking tours through grassy areas. Which statement made by one of the volunteers indicates that more teaching is needed? a. Lyme disease can spread to my brain if I don't seek treatment b. Lyme disease is caused by a virus because the symptoms are similar to the flu c. I should wear light-colored clothing and long pants when hiking d. I will call the doctor if I see a rash that looks like a bull's eye

b. Lyme disease is caused by a virus because the symptoms are similar to the flu Lyme disease is caused by bacteria called Borrelia burgdeorferi. It is transmitted by ticks that are passed it on from infected mice or deer. Because the ticks are so small, it is easier to see them on light-colored clothing; long pants and long-sleeved shirts help protect hikers. Symptoms of lyme disease are similar to influence and there may be a bull's eye rash at the site of the tick bite. Without antibiotics , the disease can spread to the brain, heart, and joints of the body.

An x-ray initially confirms the placement of a nasogastric (NG) feeding tube in the stomach. The nurse is now preparing to administer a medication through the tube. What action will the nurse take to verify tube placement? a. Place the end of the tube in water and observe for bubbling b. Measure the pH of aspirated gastric content. c. Auscultate for the sound of air produced by forcing air through the NG tube d. Assess for client coughing during administration of the medication

b. Measure the pH of aspirated gastric content. Bubbling or coughing would indicate the possibility of the tube being in the airway, but neither are used to determine placement in the stomach. Forcing air through the NG tube and auscultating the abdomen for the sound of the air is an unreliable method to determine tube placement. Measuring the pH of aspirated stomach contents confirms gastric placement.

The nurse is preparing to administer a blood transfusion for a client with anemia secondary to intraoperative blood loss. Which of the following interventions by the nurse will help reduce the risk of complications associated with the transfusion? (Select all that apply.) a. Warm blood in a microwave b. Monitor vital signs during blood administration c. Prior to infusion, check client identification against unit of blood to be transfused d. Prime the intravenous tubing with dextrose solution (D5W) e. Verify that a 20-gauge or larger catheter is used

b. Monitor vital signs during blood administration c. Prior to infusion, check client identification against unit of blood to be transfused e. Verify that a 20-gauge or larger catheter is used Typically a 20-gauge or larger needle/catheter is used if the client is going to receive packed red blood cells or whole blood; too small a bore needle may result in hemolysis during the transfusion. The nurse should closely monitor vital signs to identify a possible transfusion reaction. Prior to starting the transfusion, it's important to verify the client's identification and cross match it with the unit of blood to be transfused. The intravenous tubing should be primed only with normal saline; dextrose solutions may cause clumping of red blood cells. Special warmers may be used to prevent hypothermia but a microwave oven should never be used to warm the blood.

A nurse is anticipating providing guidance to parents of a toddler about readiness for toilet training. Which statement describes what the nurse should know in order to provide such guidance? a. Neuronal impulses are interrupted at the base of the ganglia b. Myelination of the spinal cord is completed by this age c. The child learns voluntary sphincter control through repetition d. The toddler can understand cause and effect

b. Myelination of the spinal cord is completed by this age Voluntary control of the sphincter muscles can be gradually achieved due to the complete myelination of the spinal cord. The time frame is between the ages of 18 to 24 months of age. If the child has not begun walking without holding on then myelination of sphincters is not complete and toilet training would not be successful.

A client asks the nurse to call the police and states: "I need to report that I am being abused by a nurse." The nurse should take which action? a. Assist with the report of the client's complaint to the police b. Obtain more details of the client's claim of abuse by a nurse c. Document the statement on the client's chart with a report to the manager d. Focus on reality orientation to time, place and person

b. Obtain more details of the client's claim of abuse by a nurse The advocacy role of the professional nurse, as well as the legal duty of the reasonable prudent nurse, requires the investigation of claims of abuse or violation of rights. The nurse is legally accountable for actions delegated to others. The application of the nursing process requires that the nurse gather more information, assessment before interventions and before documenting or reporting the complaint.

The nurse is assisting patients diagnosed with trigeminal neuralgia (tic douloureux) to meet their nutritional needs. Which approach should the nurse recommend? a. Encourage the patient to eat fish, liver, and chicken. b. Offer small meals consisting of high calories, soft foods. c. Provide additional servings of fruits and raw vegetables d. Assist the patient to sit in a chair for meals.

b. Offer small meals consisting of high calories, soft foods. If the patient is losing too much weight because of poor appetite due to pain in the jaw, the nurse needs to teach about foods that are high in calories and nutrients as well as food that require less chewing. The nurse should also suggest frequent, small meals that are eaten every two hours instead of 3 large meals daily.

The nurse is caring for a newly admitted 6 month-old infant diagnosed with nonorganic failure-to-thrive (NOFTT). What findings would the nurse expect to observe during the initial assessment? a. Dusky in color with poor skin turgor over abdomen b. Pale skin, thin arms and legs, and uninterested in surroundings c. Irritable and "colicky," making no attempts to turn or sit up d. Alert, laughing, playing with a rattle, and sitting with support

b. Pale skin, thin arms and legs, and uninterested in surroundings Diagnosis of NOFTT is weight consistently below the 3rd to 5th percentile for age and gender, progressive decrease in weight to below the 3rd to 5th percentile, or a decrease in the percentile rank of two major growth parameters in a short period of time. The nurse would expect to see a child who avoids eye contact, has pale skin, thin arms and legs, and is easily fatigued. NOFTT is due to psychosocial problems such as neglect, lack of knowledge about proper feeding or of the infant's needs. Many times the child engages in self-stimulatory behaviors (head banging or rocking) and is wary of close contact with people.

A nurse is caring for a 69 year-old client with end stage renal disease. What action should the nurse take to assess for patency in a fistula used for hemodialysis? a. Irrigate with 5 mL of 0.9% normal saline b. Palpate for a thrill over the fistula c. Observe for edema proximal to the site d. Feel for a bruit over the fistula

b. Palpate for a thrill over the fistula To assess for patency in a dialysis fistula or graft, the nurse auscultates for a bruit and palpates for a thrill. Remember, that you "feel" a "thrill." The other options are unrelated to evaluating patency.

A nurse is assigned to care for four clients. After listening to change-of-shift report, how would the nurse prioritize care for the following clients? (Drag the responses into the correct order.) a. The postop patient who has an order to be discharged home b. Patient with tracheostomy c. Patient with colonoscopy d. Patient in skeletal traction

b. Patient with tracheostomy c. Patient with colonoscopy d. Patient in skeletal traction a. The postop patient who has an order to be discharged home The nurse will check on the client with a tracheostomy (airway) first. The nurse would then check on the client who is to undergo a procedure (to ensure the prep was completed and the results of the bowel movements are clear). Next, the nurse would check on the client in skeletal traction, and finally the nurse would prepare the client who is ready for discharge.

The health care team is planning discharge for a 90 year-old client diagnosed with musculoskeletal weakness. Which intervention would be the priority to help prevent falls in the home? a. Begin therapy for muscle strengthening and balance b. Place night lights in the bedroom and bathroom c. Wear eyeglasses and hearing aid d. Take calcium and vitamin D supplements

b. Place night lights in the bedroom and bathroom Family members and the client should understand the simple actions they can take to help prevent falls in the home. More falls occur in the bedroom than in any other location; a simple environmental change would be to add night lights in the bedroom and bathroom. Muscle strengthening and balance exercises, taking calcium and wearing glasses may be all indicated for this client, but using night lights is an immediate and effective action to help prevent falls.

A nurse is caring for a patient diagnosed with an unstable spinal cord injury at the T7 level. Which intervention should take priority during the planning of care? a. Increase caloric intake to aid healing b. Place the patient on a pressure-reducing support surface c. Increase fluid intake to prevent dehydration d. Use skin care products designed for use with incontinence

b. Place the patient on a pressure-reducing support surface Patient is at the greatest risk for skin breakdown due to immobility and decreased sensation below the level of injury. The initial approach should be the selection and placement of the patient on the best support surface for the relief of pressure, shear, and friction forces.

The nurse is reviewing the lab results for a male client on a heparin infusion to treat a deep vein thrombosis (DVT) and cellulitis of the right lower leg. Which of the lab results would the nurse be most concerned about? a. White blood cells 15,100 per microliter (15.1 x 109/L) b. Platelet count 50,000 per microliter (500 x 109/L) c. Hemoglobin 14 g/dL (8.69 mmol/L or 140 g/L) d. Hematocrit 45% (0.45) e. Partial Thromboplastin Time 55 seconds

b. Platelet count Thrombocytopenia (abnormally low amount of platelets) and heparin-induced thrombocytopenia can occur in clients on heparin therapy. In an adult, a normal platelet count is about 150,000 to 450,000 platelets per microliter of blood (or 150-450 x 109/L). The PTT is within the therapeutic range. It is expected that the white blood cells would be slightly elevated in a client with an infection (cellulitis).

The client is admitted to an ambulatory surgery center and undergoes a right inguinal orchiectomy. Which option is the priority before the client can be discharged to home? a. Able to tolerate a regular diet b. Post-operative pain is managed c. Psychological counseling is scheduled d. Able to ambulate in the hallway with assistance

b. Post-operative pain is managed An orchiectomy is the surgical removal of one or both testicles. It is usually performed to treat cancer (testicular, prostate or cancer of the male breast), but it may also be performed to prevent cancer (with an undescended testicle.) Due to the location of the incision, pain management is the priority. Most men will be able to eat regularly when they get home; they should at least tolerate liquids before discharge. It's important that the client is able to get up and walk with assistance, but this is not the priority. Psychological counseling may be needed as part of long-term aftercare, but this is not an immediate priority.

The nurse is caring for a client with chronic renal failure. Which of the following orders written by the health care provider would the nurse question? a. Sodium polystyrene sulfonate (Kayexalate) 50 grams rectally today b. Potassium chloride (Micro-K) 20 mEq daily with breakfast c. Recombinant human erythropoietin (Epoetin alpha) 100 units/kg SubQ 1-3 times/week d. Furosemide (Lasix) 40 mg orally twice a day

b. Potassium chloride (Micro-K) 20 mEq daily with breakfast Hyperkalemia is the most common and life-threatening metabolic complication of renal failure. Consequently, additional potassium would be contraindicated. Recombinant human erythropoietin is prescribed to treat anemia. Diuretics such as furosemide are used to treat edema caused by chronic kidney failure. Kayexalate is a potassium-binding resin used to prevent or treat hyperkalemia.

The triage nurse identifies that a 16 yo patient is legally married ad has signed the consent form for treatment. What should be an appropriate action by the nurse? a. Withhold treatment until telephone consent can be obtained from the parter b. Proceed with the triage process in the same manner as any adult patient c. Refer the teenager to a community pediatric hospital emergency department d. Ask the teenager to wait until a parent or legal guardian can be contacted

b. Proceed with the triage process in the same manner as any adult patient Minors can become known as an emancipated minor through marriage, pregnancy, high school graduation, independent living, or service in the military. Therefore, this married patient has the legal capacity of an adult. Otherwise the page for legal signature is 18 years of age.

A nurse, during an assessment of a day-old newborn, notices that the breasts are enlarged bilaterally with a white, thin discharge. What action should the nurse perform next? a. Notify the health care provider within that shift b. Record the findings while thinking that they are "normal" c. Ask about medications taken during pregnancy d. Obtain fluid to check for glucose by Dextrostix

b. Record the findings while thinking that they are "normal" Newborn infants of both sexes may have engorged breasts and may secrete milk during the first few days to weeks after birth.

A newborn who is delivered at home and without a birth attendant is admitted to the hospital for observation. The initial temperature is 95 F (35 C) axillary. The nurse should recognize that cold stress may lead to what complication? a. Hyperglycemia b. Reduced partial pressure of oxygen in arterial blood (PaO2) c. Metabolic alkalosis d. Lowered basal metabolic rate

b. Reduced partial pressure of oxygen in arterial blood (PaO2) Hypothermia and cold stress cause a variety of physiologic stresses including increased oxygen consumption, metabolic acidosis, hypoglycemia, tachypnea and decreased cardiac output. The baby delivered in such circumstances needs careful monitoring. In this situation, the newborn must be warmed immediately to increase its temperature to at least 97 F (36 C). Normal core body temperature for newborns is 97.7 F to 99.3 F (36.5 C to 37.3 C).

The client has undergone a dilation and curettage (D & C) following a spontaneous abortion at 8 weeks. To promote an optimal recovery, what information should the nurse include in the discharge teaching? (Select all that apply.) a. Resume vaginal intercourse 6 weeks after the procedure b. Referral for grief counseling c. Expect heavy bleeding for at least a week d. Strenuous sport activities should be postponed until bleeding stops e. Use sanitary pads until vaginal bleeding has stopped

b. Referral for grief counseling d. Strenuous sport activities should be postponed until bleeding stops e. Use sanitary pads until vaginal bleeding has stopped`` Acknowledge that the client has experienced a loss and may want to attend a support group or have professional counseling. Most women experience some painful cramping initially, but it doesn't last long; bleeding can be expected for a few days to up to 2 weeks. The client should be told to avoid vaginal intercourse and not to use tampons for about 2 weeks. Most women can return to normal activities within a day or so but strenuous sport activities should be postponed until the bleeding stops.

A diabetic client asks the nurse: "Why did the health care provider order a glycosylated hemoglobin (HbA1c) measurement? My blood glucose reading was just done this morning." Which explanation would be best to help explain the purpose of the HbA1c? a. Provides a more precise blood glucose value than self-monitoring b. Reflects an average blood glucose for the prior several months c. Is performed to detect any complications of diabetes d. Measures the circulating levels of insulin

b. Reflects an average blood glucose for the prior several months The HbA1c is used to determine how well the client is managing the disease. The results reflect the average blood sugar level for the past 2 to 3 months; the more glucose in the blood, the more hemoglobin gets glycated (coated with glucose). The higher the HbA1c, the poorer the glucose management and the higher the risk of diabetic complications. For most diabetics, the goal is to keep the HbA1c at or below 6.5 - 7 %.

The nurse is assigned to a client newly diagnosed with active tuberculosis (TB) and a productive cough. Which of these interventions would be a priority for the nurse to implement? a. Place the client in a negative pressure private room and have disposable particulate respirators available for hospital employees b. Reinforce that everyone should wash their hands before and after entering the room c. Place client in a private room and implement droplet precautions d. Have the client cough into a tissue and dispose of the tissue in a separate bag

b. Reinforce that everyone should wash their hands before and after entering the room A client with active tuberculosis should be hospitalized in a negative pressure room to prevent cross contamination of the disease. The client would be placed on airborne precautions because this bacteria can be suspended in the air for long periods of time and may be carried for long distances on air currents. Any hospital employee entering the room would need to wear a disposable micron mask or disposable particulate respirator (N-95, for example). The Centers for Disease Control and Prevention (CDC) state that visitors can wear surgical masks.

The nurse is caring for a client in a home setting. Which action is most likely to ensure the safety of the nurse during a home visit? a. Carry a cell phone, pager and/or hand-held alarm b. Remain alert and leave if cues suggest the home is not safe c. Observe for evidence of weapons in the home d. Review documentation for previous entries about violence

b. Remain alert and leave if cues suggest the home is not safe Nurses need to assess and manage safety risks and have ongoing clinical supervision and support when making home visits. Proper safety should begin with a thorough assessment of the client's home to identify potential risks, such as pets (the most commonly assessed hazard), drug use and weapons. The nurse should also and develop a plan to mitigate (or eliminate) the risks and understand that there's always the option to end a visit early if the environment does not seem safe. Carrying a phone, using a buddy system, learning about the client prior to the visit can help mitigate risks.

The nurse, who is located in a large urban area, uses telecommunications to provide health care and education to clients in remote locations. What is the best reason for using telehealth? a. Empowers clients to take a greater interest in their illness b. Removes time and distance barriers from the delivery of care c. Reduces health care costs d. Standardizes electronic data sharing of health information

b. Removes time and distance barriers from the delivery of care Telehealth is the use of technology to deliver health care, health information, or health education at a distance. People in rural areas or homebound clients can communicate with providers via telephone, email or video consultation, thereby removing the barriers of time and distance for access to care. Although increased access to information and collaboration between the client and provider can be empowering, this is not the primary reason for using telecommunications/telehealth.

A nurse has been teaching adult clients about cardiac risks when they visit the hypertension clinic. Which evaluation data would be the best measure of learning? a. Responses to verbal questions b. Reported behavioral changes c. Completion of a mailed survey d. Performance on written tests

b. Reported behavioral changes If the clients alter behaviors such as smoking, drinking alcohol and stress management, these changed behaviors suggest that learning has occurred. Additionally, physical assessments, observed behaviors and lab data may confirm risk reduction.

The nurse observes a nursing assistant using aseptic hand rub and rubbing hands vigorously after leaving the room of a patient with C diff. Which action is most appropriate by the nurse? a. Ensure that visitors wash hands thoroughly before and after visiting. b. Require that the nursing assistant wash hands again using soap and water. c. Tell the patient to ask caregivers if they have all washed their hands d. Praise the nursing assistant for proper use of antiseptic hand rub

b. Require that the nursing assistant wash hands again using soap and water. Anyone who is hospitalized should be encouraged to ask caregivers if they washed their hands and to remind visitors to wash their hands. However, it is the nurse's responsibility to supervise the nursing assistant and to correct practice errors as needed. C diff is one of the few pathogens that require soap and water for cleansing the hands. Since antiseptic hand rub is ineffective against the hardy spores produced by the bacterium, the nurse should require the nursing assistant to wash his or her hands with soap and water, especially after providing care to this patient.

A mother asks about expected motor skill development for her 3 year-old child. Which activity is considered a typical motor skill for the 3 year-old? a. Tying shoelaces b. Riding a tricycle c. Jumping rope d. Playing hopscotch

b. Riding a tricycle Three year-old children are developing gross motor skills that require large muscle movement. While there will always be some variation between children, movement milestones typically include pedaling a tricycle, standing on one foot for a few seconds, walking backwards and jumping with both feet. The other activities listed require more coordination and are movement milestones for older children.

Behaviors of alcohol and drug abuse have outcomes of impaired judgment and increased risk-taking behavior. What nursing diagnosis best applies to this data? a. Risk for knowledge deficit b. Risk for injury c. Altered thought process d. Disturbance in self-esteem

b. Risk for injury Accidents increase as a result of intoxication of substances. Studies indicate alcohol is a factor in more than 50% of motor vehicle fatalities, in 53% of all deaths from accidental falls, in 64% of fatal fires, and in more than 80% of suicides.

The nurse is reinforcing teaching with a patient about compromised host precautions. The patient is receiving filgrastrim (Neupogen) for neutropenia. Which lunch selection suggests that the patient has learned abut the necessary dietary changes? a. BBQ beef, baked beans, and coleslaw from the deli b. Roast beef, mashed potatoes, sautéed green beans c. Peanut butter sandwiches, apple, cold-brewed iced tea d. Chicken sandwiches made with sprouts and farmer's cheese, skim milk

b. Roast beef, mashed potatoes, sautéed green beans Food should be cooked or canned. Avoid raw fruits and vegetables, undercooked meat and eggs, soft "moldy" cheeses, lunch meat, and salads from the deli. Should only consume pasteurized dairy products and fruit juices and avoid cold-brewed teas and drinks.

A client who is a victim of domestic violence states, "If I were better, I would not have been beaten." Which feeling best describes what the client may be experiencing at this time? a. Rejection b. Self-blame c. Helplessness d. Fear

b. Self-blame Victims of domestic violence may be immobilized by a variety of affective responses with one being self-blame. These clients often believe that a change in their behavior will cause the abuser to become nonviolent. They may even have been told this by their abuser. This is an untrue but not uncommon myth.

A nurse is planning care for a 2 year-old hospitalized child. Which issue will produce the most stress at this age? a. Fear of pain b. Separation anxiety c. Loss of control d. Bodily injury

b. Separation anxiety While a toddler will experience all of the stresses, separation from parents is the major stressor. Separation anxiety peaks in the toddler years.

The nurse assesses a full-term, 30 hour-old newborn and reviews its lab results. The nurse knows that the first-time mother is Rh negative and is breastfeeding exclusively. Which of these findings is a priority to report to the health care provider? a. Positive Coombs test b. Serum bilirubin of 11 mg/dL (188 mmol/L) c. Jaundice is observed d. Hematocrit 52%

b. Serum bilirubin of 11 mg/dL (188 mmol/L) Jaundice is a common condition in newborns. But for a full-term infant who is 30 hours-old, a total serum bilirubin level of 11 mg/dL (188 µmol/L) is high, which is why this is the priority finding to report to the health care provider. The concern about hyperbilirubinemia is increased because the mother is Rh negative (meaning there's a possible Rh incompatibility) and she is breastfeeding exclusively. The hematocrit is within normal limits for a newborn. The Coombs test results do not indicate if it's direct or indirect.

A client diagnosed with hypertension is started on atenolol. The nurse should instruct the client to immediately report which of these findings? a. Feeling tired b. Slow, irregular pulse c. Decreased sex drive d. Insomnia

b. Slow, irregular pulse Most of the side effects for the beta-blocker, atenolol (Tenormin) are transient or mild, such as decreased sex drive, low energy or feeling tired, depression or insomnia. However the client should understand that he needs to call the health care provider if he experiences any of these serious side effects: slow or uneven heartbeats, feeling short of breath, lightheadedness, fainting, or swelling of the feet or ankles.

Following a major burn to the lower extremities, a diet high in protein and carbohydrates is ordered for a 7yo child. What reason would the nurse give the family that would help explain these dietary requirements? a. Stimulate peristalsis for enhanced absorption of nutrients b. Spare protein catabolism to meet the child's metabolic needs c. Provide a well-balanced and nutritionally complete diet d. Strengthen the immune system to prevent infection

b. Spare protein catabolism to meet the child's metabolic needs Child's energy and protein requirement will be high due to catabolism of trauma, heat loss, and the demands of tissue regeneration. Good nutrition is important for wound healing and helps reduce the risk of infection, but a child who has been burned needs about 2 to 3 times more calories and proteins to help him/her heal and grow.

A nurse is speaking at a community meeting about personal responsibility for health when a participant asks about chiropractic treatment for illnesses. What should be the focus of the nurse's response? a. Exercise of joints b. Spinal column manipulation c. Electrical energy fields d. Mind-body balance

b. Spinal column manipulation The theory underlying chiropractic treatment is that interference with transmission of mental impulses between the brain and body organs produces diseases. Such interference is caused by misalignment of the vertebrae. Manipulation reduces the misalignment (subluxation).

The health care team is meeting to discuss discharge planning for a client following total hip replacement surgery. Which assessment finding is most important for the nurse to share? a. The client does not like the taste of the oral potassium supplement medication b. The home is a two-story and all bedrooms and bathroom are located upstairs c. The adult daughter will be responsible for shopping and driving the client after discharge d. The partner expresses some discomfort with the dressing change

b. The home is a two-story and all bedrooms and bathroom are located upstairs Nurses, because of the intimate work with clients, often discover important personal information that impacts discharge planning. It is important to share these insights and findings to assure client's needs are met when they go home. The client with major orthopedic surgery can expect some mobility impairment after discharge, so the physical characteristics of the home are critical to activities of daily living and safety. The other findings are helpful in planning but are not as critical to safety as the issue of stairs and access. Safety issues take priority.

Mass casualty survivors are brought to the emergency department (ED) after a disaster. The nurse is assigned to four clients who were triaged in the field and have just arrived in the ED. Which client will the nurse care for first? a. The person with multiple wounds and an open fracture b. The person with hypotension and a sucking chest wound c. The person with head trauma requiring mechanical ventilation d. The person with an undisplaced fracture of the radius

b. The person with hypotension and a sucking chest wound Typically, the tab colors used in triage are black, yellow, green and red. Red-tagged clients have immediate threats to life and require care right away; this would be the survivor with hypotension and a sucking check wound. Yellow-tagged clients have major injuries that need treatment within 30 minutes to 2 hours (the client with the open fracture), and green-tagged client have injuries that can be delayed more than 2 hours (the closed fracture). Black-tagged clients are treated last during a mass casualty situation because there is little chance for survival.

A parent asks the nurse about a Guthrie Bacterial Inhibition test that was ordered for her newborn. Which of the following points should the nurse discuss with the patient prior to this test? a. Routine screening of newborn infants is not mandatory in the US b. This test identifies an inherited disease c. This test will be delayed if the baby's weight is less than 5 pounds d. The urine test can be done after 6 weeks of age e. Positive tests require dietary control for prevention of brain damage f. Best results occur after the baby has been breast-feeding or drinking formula for 2 full days

b. This test identifies an inherited disease c. This test will be delayed if the baby's weight is less than 5 pounds d. The urine test can be done after 6 weeks of age e. Positive tests require dietary control for prevention of brain damage f. Best results occur after the baby has been breast-feeding or drinking formula for 2 full days Screening for PKU is mandated in all 50 states., though methods of screening vary. The Guthrie Bacterial Inhibition Assay (BIA) is one test used to diagnose phenylketonuria (PKU), a disease characterized by an enzyme deficiency. A blood sample is taken from the baby's heel shortly after birth, with a follow-up test 7 to 10 days later. Test results are more accurate if the baby weighs more than 5 pound and has been regularly drinking milk for more than 24 hours. A urine test is normally done after 6 weeks of age if a baby did not have the blood test.

A client is to receive three doses of potassium chloride 10 mEq in 100 mL of 0.9% normal saline to infuse over 30 minutes each. Which action is a priority assessment to perform before the nurse gives this medication? a. Oral fluid intake b. Urine output c. Bowel sounds d. Grip strength

b. Urine output Potassium chloride should only be administered after adequate urine output (greater than 20 mL/hour for two consecutive hours) has been established. For children the desired urine output is 1 mL/1 kg/1 hour. Impaired ability to excrete potassium via the kidneys can result in hyperkalemia.

A client of Hispanic heritage refuses emergency unit treatment until a curandero is called. What should the nurse understand about the practices of a curanderos? a. Herbal preparations will be used b. Uses holistic healing practices c. Offers spiritual advising d. Witchcraft

b. Uses holistic healing practices A curandero is a folk healer (or shaman) who uses a holistic approach that includes herbs, aromas and rituals, to treat the ills of the body, mind and spirit. Many times, the curandero works with traditional Western health care providers to restore health.

A woman in labor calls a nurse to assist her in the bathroom. The nurse notices a large amount of clear fluid on the bed linens. The nurse should act based on knowledge that fetal monitoring must now assess for what complication? a. Late accelerations b. Variable decelerations c. Early decelerations d. Periodic accelerations

b. Variable decelerations When the membranes rupture, there is increased risk initially of cord prolapse if the head is at a minus level. Fetal heart rate patterns may show variable decelerations, which require immediate nursing action to reposition the client, apply oxygen and notify the health care provider.

The oncology nurse is preparing to administer the initial dose of vincristine to a child diagnosed with acute lymphocytic leukemia (ALL). Which intervention is most appropriate to add to the plan of care? (Select all that apply.) a. Apply ice to the injection site if extravasation occurs b. Verify blood return before, during and after intravenous administration c. Monitor for numbness or tingling in the fingers and toes d. Select appropriate catheter for intrathecal administration e. Monitor liver enzyme tests

b. Verify blood return before, during and after intravenous administration c. Monitor for numbness or tingling in the fingers and toes e. Monitor liver enzyme tests ALL is the most common type of cancer in children and treatment protocols include vincristine. Vincristine (Oncovin) is for intravenous use only; intrathecal administration can be fatal. Vincristine is a vesicant that can cause significant local damage if extravasation occurs; treatment includes subcutaneous injection of an antidote and warm compresses (topical cooling may worsen the effect). Peripheral neuropathy is a major side effect associated with vincristine. The nurse should monitor for decreased hepatic functioning because vincristine is metabolized in the liver.

A client, admitted to the unit because of severe depression and suicidal threats, is placed on suicidal precautions. The nurse should be aware that the danger of the client committing suicide is greatest at what period of time? a. During the night shift when staffing is limited b. When the client's mood improves with an increase in energy level c. At the time of the client's greatest despair d. After a visit from the client's estranged partner

b. When the client's mood improves with an increase in energy level Suicide potential is often increased when there is an improvement in mood and energy level. At this time ambivalence is often decreased and a decision is made to commit suicide. The clients have the energy to carry through with the plan for suicide.

The school nurse is checking students for pediculosis capitis. Which manifestation observed by the nurse confirms the presence of pediculosis capitis? a. Scratching the head more than usual b. Whitish oval specks sticking to the hair shaft c. White flakes on the student's shoulders d. Oval patterns of occipital hair loss

b. Whitish oval specks sticking to the hair shaft Diagnosis of pediculosis capitis, or head lice, is made by observation of the white eggs (nits) firmly attached to the hair shafts. Treatment can include application of a medicated shampoo with lindane for children over 2 years-old and meticulous combing with a special comb for the removal of all nits.

A client who is a victim of domestic violence tells the batterer: "I need a little time away." How would the nurse expect that the batterer might respond? a. With a new commitment and an opportunity to seek counseling b. With fear of rejection, resulting in increased rage toward the client c. With relief, welcoming the separation as a means to have more personal time d. With acceptance, perceiving the comment as an indication that the relationship is in trouble

b. With fear of rejection, resulting in increased rage toward the client Those that batter others commonly react to such statements or actions with fear of rejection, abandonment and loss. These types of actions by the recipient of the battering only serve to increase the batterer's rage at the partner.

A client on warfarin therapy after coronary artery stent placement calls the clinic to ask: "Can I take Alka-Seltzer for an upset stomach?" What is the best response by the nurse? a. "Use about half the recommended dose of Alka-Seltzer." b. "Select another antacid that does not inactivate warfarin (Coumadin)." c. "Avoid Alka-Seltzer because it contains aspirin." d. "Take Alka-Seltzer at a different time of day than you take the warfarin (Coumadin)."

c. "Avoid Alka-Seltzer because it contains aspirin." Alka-Seltzer is an over-the-counter aspirin-antacid combination. Aspirin is an antiplatelet drug and taking this with warfarin will potentiate the anticoagulant effects of warfarin (Coumadin), which may increase the risk of bleeding.

The nurse is admitting a client who is newly diagnosed with a frontal lobe brain tumor. Which statement made by a spouse may provide important information about this diagnosis and should be communicated to the health care provider? a. "It seems our sex life is nonexistent over the past six months." b. "His breathing rate is usually below 12." c. "I find the mood swings and the change from being a calm person to being angry all the time hard to deal with." d. "In the morning and evening he complains that reading is next to impossible because the print is blurry."

c. "I find the mood swings and the change from being a calm person to being angry all the time hard to deal with." The frontal lobe of the brain controls affect, judgment and emotions. Dysfunction in this area results in findings such as emotional lability, changes in personality, inattentiveness, flat affect and inappropriate behavior.

A nurse is teaching a client with asthma about the correct use of the fluticasone inhaler. Which statement, if made by the client, would indicate that the teaching was effective? a. "The inhaler can be used whenever I feel short of breath." b. "If I forget a dose, I can double up on the next dose." c. "I should rinse my mouth after using the inhaler." d. "I should not use a spacer with my inhaler."

c. "I should rinse my mouth after using the inhaler." Fluticasone inhaled (Flovent Diskus) is an inhaled corticosteroid used to prevent asthma attacks. It is often used in conjunction with a bronchodilator. The client should be instructed to rinse the mouth after using the inhaler to wash away any steroid residue so as to reduce the risk of oral fungal infections.

The nurse is reviewing information about using a plastic thoracolumbosacral orthotic (TLSO) with a teen who was recently diagnosed with scoliosis. Which statement made by the client indicates the need for further education? a. "I will remove the brace when I shower or go swimming." b. "I should not lift objects over 10 pounds." c. "I should wear a sweatshirt under the orthotic to help protect my skin." d. "I should lie down in bed to remove the brace."

c. "I should wear a sweatshirt under the orthotic to help protect my skin." The TLSO is a custom molded brace prescribed to give support to the spinal column from the sixth thoracic vertebra to the sacrum. Clients are advised to wear only a tight fitting t-shirt under the brace to protect the skin and absorb sweat. Although the brace will not be damaged by water, it is typically removed when showering; the client will need to lie down to remove or put on the brace. The health care provider will instruct the client about how often and how long to wear the brace each day. The client is instructed not to attempt to bend or to lift objects weighing more than 10 pounds.

The nurse is assessing a 28 year-old female for risk factors contributing to osteoporosis. Which statement reported by the client should alert the nurse that additional teaching about this disease is indicated? (Select all that apply.) a. "I get sun exposure daily and always use sunblock protection." b. "I'm just started following the Mediterranean diet and already feel more energized." c. "I'm a professional dancer and train 8 to 10 hours a day." d. "I consume only skim milk, never whole milk." e. "I take 1000 mg OsCal (calcium carbonate) every morning with breakfast."

c. "I'm a professional dancer and train 8 to 10 hours a day." d. "I consume only skim milk, never whole milk." A nutritional plan that emphasizes fruits and vegetables, low-fat dairy and protein sources, and increased fiber (found in the Mediterranean diet) is beneficial to maintaining bone health and reducing the risk of heart disease. Adequate daily sun exposure is important for vitamin D synthesis, which is important for bone health. Although a calcium supplement is a good idea for this client and calcium carbonate should be taken with food for better absorption, calcium is absorbed more efficiently when it's taken in amounts of 500 to 600 mg at one time. Also, overtraining in women causes decreased estrogen levels and may increase the risk for osteoporosis.

A client is scheduled to receive an oral solution of radioactive iodine (131I). What information is the priority for the nurse to include when teaching the client about this treatment? a. "Your family can use the same bathroom as you are using, without any special precautions." b. "Drink plenty of water and empty your bladder often during the initial three days of therapy." c. "In the first 48 hours, you should avoid contact with children and pregnant women; be sure to flush the commode twice after urination or defecation." d. "Use disposable utensils for two days; if you feel nauseous within 12 hours of the first dose, please vomit in the toilet and flush it twice."

c. "In the first 48 hours, you should avoid contact with children and pregnant women; be sure to flush the commode twice after urination or defecation." The client's urine and saliva are radioactive for 24 hours after ingestion, and vomitus is radioactive for six to eight hours. The client should drink 3 to 4 liters of fluid a day for the initial 48 hours to help remove the (131I) from the body. To minimize exposure to radiation, nursing staff should plan to give care in the shortest time possible (less time equals less exposure), working as far away from the radiation source as possible. Each nurse should also wear a personal film badge or pocket dosimeter.

A client comes into the community health center upset and crying, stating: "I will die of cancer now that I have this disease." The client hands the nurse a piece of paper with the word "pheochromocytoma" written on it. What would be the best initial response by the nurse? a. "You probably have had episodes of sweating, heart pounding and headaches. Is that correct?" b. "This problem is diagnosed by blood and urine tests that reveal elevated levels of adrenaline and noradrenaline." c. "Pheochromocytomas are usually noncancerous, but they do need to be treated to avoid complications." d. "Computerized tomography (CT) or magnetic resonance imaging (MRI) are used to detect an adrenal tumor."

c. "Pheochromocytomas are usually noncancerous, but they do need to be treated to avoid complications." All of the options are correct information. The best response of the nurse is to address the issue presented by the client, which is "fear of cancer." Pheochromocytomas may release large amounts of adrenaline into the bloodstream after an injury or during surgery. For this reason, they can be life-threatening if unrecognized or untreated.

Two hours after receiving the first does of lithium, the client reports fine hand tremors. What is the nurse's best explanation for these findings? a. "Reducing dietary intake of sodium and fluids should minimize any side effects." b. "Taking lithium on an empty stomach should help minimize these symptoms." c. "These are common and expected side effects and should subside in a few days." d. "You are probably having an allergic reaction. The medication should be discontinued."

c. "These are common and expected side effects and should subside in a few days." Tremors are common side effects that usually subside quickly. Informing clients of these possible reactions can help them tolerate these initial difficulties, while continuing to take the drug and obtaining therapeutic effects.

A client calls the clinic and states to the triage nurse: "I had an upset stomach and took Pepto-Bismol and now my tongue looks black. What's happening to me?" What would be the nurse's best response? a. "Are your stools also black?" b. "How long have you had an upset stomach?" c. "This is a common and temporary side effect of this medication." d. "Come to the clinic so you can be seen by the health care provider."

c. "This is a common and temporary side effect of this medication." The best response would be to explain that a dark tint of the tongue is a common and temporary side effect of bismuth subsalicylate (Pepto-Bismol). Although it may also turn stools a darker color, do not confuse this with black, tarry stools, which is a sign of bleeding in the intestinal tract. After addressing the client's initial concern and the reason for the call, the nurse can ask about the upset stomach and then ask the client to come to the clinic if necessary.

The nurse is caring for a client with urinary incontinence. The client asks the nurse about the use of biofeedback to treat this condition. What is the most appropriate response by the nurse? a. "Surgery is generally needed in order to produce any real improvement." b. "Medications are the approved method of treating this type of problem." c. "This type of treatment has been used successfully to manage urinary incontinence." d. "Biofeedback has not been shown to be very helpful for urinary incontinence problems."

c. "This type of treatment has been used successfully to manage urinary incontinence." Biofeedback is a widely used and effective method to manage urinary incontinence, with rehabilitation of the pelvic floor muscles the treatment goal. The other statements are potential treatment options but are untrue statements. Depending on the cause of the incontinence, medications such as estrogens, alpha-adrenergic drugs, and the antimuscarinic drug tolterodine (Detrol) may be prescribed and/or surgery may be indicated.

An antibiotic is ordered to be administered to a 2 year-old child intramuscularly. The total volume of the injection equals 2 mL. What is the correct nursing action? a. Call to get a smaller volume ordered b. Check with pharmacy for a liquid form of the medication c. Administer the medication in two separate injections d. Give the medication in the dorsogluteal site

c. Administer the medication in two separate injections Intramuscular injections should not exceed a volume of 1 mL for infants and toddlers. Medication doses exceeding this volume should be split into two separate injections of 1 mL each. The nurse would insert the needle at a 90 degree angle into the anterolateral thigh muscle.

A nurse is assessing a newborn infant and observes low-set ears, short palpebral fissures, flat nasal bridge and indistinct philtrum. What priority focus in the maternal history should the nurse ask about? a. Use of vitamins and supplements during pregnancy b. Family genetic disorders c. Alcohol use during pregnancy d. Maternal and paternal ages

c. Alcohol use during pregnancy This cluster of facial characteristics is often linked to fetal alcohol syndrome (FAS). Lifelong developmental delays of varying severity can result. Recognizing this disorder is the first step in getting treatment, appropriate intervention and referrals.

An 8 year-old child is brought to the clinic by a parent who states: "This child was so sick last year with cold and the flu. I want her to have the influenza vaccine this year." The nurse assesses the child and reviews the child's history. What information would the nurse recognize as a contraindication for giving the child this vaccine? a. Family history of convulsions b. Recent exposure to an infectious disease c. Allergy to eggs d. Persistent, inconsolable crying after receiving other immunizations

c. Allergy to eggs An allergy to egg proteins is listed by the CDC as a contraindication for administering the influenza vaccine. Influenza vaccines are grown on egg embryos and may contain a small amount of egg protein.

Following an alert of an internal disaster and the need for beds, the charge nurse is asked to list the clients who can potentially be discharged. Which one of these clients should the charge nurse select? a. An older adult client with an implantable cardiac defibrillator (ICD) admitted yesterday after receiving multiple shocks b. A school-aged child admitted earlier today with a diagnosis of suspected bacterial meningitis c. An adult client, diagnosed with type 1 diabetes at age 10, admitted 36 hours ago with diabetic ketoacidosis d. An adolescent admitted the previous evening with Tylenol intoxication

c. An adult client, diagnosed with type 1 diabetes at age 10, admitted 36 hours ago with diabetic ketoacidosis The client with type 1 diabetes is the only one with a chronic condition who has been treated for more than a day and whose condition is the most stable. The other clients' conditions are either unstable and/or more acute. Tylenol intoxication requires at least three to four days of intensive observation for the risk of hepatic failure. Because acute bacterial meningitis can lead to permanent brain damage or death, treatment must be started as soon as possible. It is considered a medical emergency for someone with an ICD who experiences multiple shocks.

A patient is admitted with a diagnosis of renal caliculi. The patient reports oral temperature is 100.8 F. Which of these goals is the priority nursing focus for this patient? a. Control nausea b. Prevent infection c. Manage pain d. Maintain fluid balance

c. Manage pain An immediate goal of therapy is to alleviate pain, which can be quite severe.

A 65-year-old Hispanic-Latino client diagnosed with prostate cancer rates his pain as a six on a 0 to 10 scale. Other than Ibuprofen (Motrin), the client refuses all pain medication even though this does not relieve his pain. What should be the next action for the nurse to take? a. Report the situation to the primary care provider b. Talk with the client's family about the situation c. Ask the client about the refusal of certain pain medications d. Document the situation in the progress notes

c. Ask the client about the refusal of certain pain medications Belief regarding pain is one of the oldest culturally-related research areas in health care. Astute observations and careful assessments must be completed to determine the level of pain a person can tolerate. Nurses should investigate the meaning of pain to each client within a cultural explanatory framework. After this initial assessment is done the other options would most likely be implemented.

The nurse is providing discharge teaching to a client with asthma. The nurse should warn against the concurrent use of which over-the-counter medications? a. Cortisone ointments for skin rashes b. Cough medications containing guaifenesin c. Aspirin products for pain relief d. Histamine blockers for gastric distress

c. Aspirin products for pain relief Aspirin is known to induce asthma attacks. Aspirin-induced asthma is a distinct clinical syndrome affecting some asthmatic clients. It is characterized by the onset of an asthma attack within 30 minutes to 3 hours after the ingestion of aspirin. Although the name of the condition relates to aspirin, it is well established that affected clients are cross-sensitive to all nonsteroidal anti-inflammatory drugs (NSAIDs) that inhibit cyclo-oxygenase (COX) enzymes. Paracetamol (acetaminophen), however, is seldom associated with cross sensitivity in clients with aspirin induced asthma.

While giving care to a 2 yo patient, the nurse should remember that the toddler's tendency to say "no" to almost everything is an indication of what psychosocial skill? a. Frustration with adults b. Rejection of parents c. Assertion of control d. Stubborn behavior

c. Assertion of control Negativity is a normal behavior in toddlers. The nurse must be aware that this behavior is an important sign of the child's progress from dependency to autonomy and independence.

An 88 year-old client is admitted to the telemetry unit following a minor surgical procedure. The client's history includes insulin dependent diabetes and a previous myocardial infarction. The nurse responds to the client's ECG alarm and finds the client's rhythm shows asystole and the client obtunded but responsive. Prioritize the actions of the nurse (with 1 being the top priority). a. Initiate emergency response system if indicated b. Look at a different ECG lead to confirm rhythm c. Assess respirations and pulse d. Check a blood glucose level

c. Assess respirations and pulse a. Initiate emergency response system if indicated b. Look at a different ECG lead to confirm rhythm d. Check a blood glucose level After checking responsiveness, establishing a patent airway and then assessing breathing and circulation are the next priorities (ABCs). This assessment would provide information to decide whether the emergency response team is needed. Because the client is responsive, the monitor rhythm is not correct, as a client with asystole would be unresponsive. Asystole on a rhythm strip may simply be a loose lead; a quick way to check this is to select another lead. The client's obtunded state indicates that ion is needed, so assessment of a central pulse and blood pressure is indicated to determine whether cardiovascular compromise is responsible for this condition. If no evidence of an immediate cardiac event is present, the blood glucose should be checked. Stress and changes in food or fluid consumption secondary to surgery increase the risk of glucose imbalance in the person with diabetes.

The client is scheduled for coronary artery bypass. Based on principles of teaching and learning, what is the best initial approach by the nurse during pre-op teaching? a. Tour the coronary intensive unit b. Mail a videotape to the home c. Assess the client's learning style d. Administer a written pretest

c. Assess the client's learning style As with any anticipatory teaching, assessment of the client's level of knowledge and learning style should occur first. If possible, the three senses of hearing, seeing and touching should be used during any teaching to enhance recall.

The nurse manager informs the nursing staff that the clinical nurse specialist will be conducting a research study on staff attitudes toward client care, and all staff are invited to participate in the study if they wish. This affirms which ethical principle? a. Justice b. Beneficence c. Autonomy d. Anonymity

c. Autonomy Individuals must be free to make independent decisions about participation in research without coercion from others. Anonymity means the person's identity is not revealed. Beneficence is the state or quality of being kind, charitable, beneficial or a charitable act.

The nurse is administering the initial total parenteral nutrition (TPN) solution to a client. Which finding requires the nurse's immediate attention? a. Urine output of 300 mL in four hours b. Poor skin turgor c. Blood glucose of 350 mg/dL (19.4 mmol/L) d. Temperature of 99.5 F (37.5 C)

c. Blood glucose of 350 mg/dL (19.4 mmol/L) Total parenteral nutrition formulas contain dextrose in concentrations of 10% or greater to supply 20% to 50% of the total calories. Blood glucose levels should be checked every four to six hours. A sliding scale dose of insulin may be ordered to maintain the blood glucose level below 180 mg/dL (10 mmol/L).

A client with the diagnoses of anorexia nervosa, electrolyte imbalance, and cardiac dysrhythmias, is hospitalized on a medical unit. Which findings would the nurse expect during the admission process? a. Diarrhea, vomiting, dental erosion b. Hypertension, nervousness, moist skin that's warm to the touch c. Brittle hair, lanugo, amenorrhea d. Hyperthermia, tachycardia, increased metabolic rate

c. Brittle hair, lanugo, amenorrhea Brittle hair, lanugo and amenorrhea are findings associated with anorexia nervosa. The lanugo and amenorrhea are a result of the decreased levels of estrogen and dehydroepiandrosterone (DHEA). Other physical findings include reduced metabolic rate, constipation, dry skin, hypotension and bradycardia. Dental erosion, due to vomiting, is seen in both bulimia nervosa and anorexia nervosa; however, diarrhea is associated with laxative abuse and bulimina nervosa.

The client with a T-2 spinal cord injury reports having a "pounding" headache. Further assessment by the nurse reveals excessive sweating, rash, pilomotor erection, facial flushing, congested nasal passages and a heart rate of 50. What action should the nurse take next? a. Assist client with relaxation techniques b. Measure the client's respirations, blood pressure, temperature and pupillary responses c. Check the client for bladder distention and the urinary catheter for kinks d. Place the client into the bed and administer the ordered PRN analgesic

c. Check the client for bladder distention and the urinary catheter for kinks These are findings of autonomic dysreflexia, also called hyperreflexia. This response occurs in clients with a spinal cord injury above the T-6 level. It is typically initiated by any noxious stimulus below the level of injury such as a full bladder, an enema or bowel movement, fecal impaction, uterine contractions, changing of the catheter and vaginal or rectal examinations. The stimulus creates an exaggerated response of the sympathetic nervous system and can be a life-threatening event. The BP is typically extremely high. The priority action of the nurse is to identify and relieve the cause of the stimulus.

A client is diagnosed with protein-energy malnutrition secondary to colitis. Which findings would support this diagnosis? (Select all that apply.) a. High blood pressure b. Sodium 146 mEq/L (146 mmol/L) c. Cholesterol 110 mg/dL (2.85 mmol/L) d. Total lymphocyte count (LTC) 1000/mcL e. Hemoglobin 10.9 g/dL (6.76 mmol/L) f. Increased lean body mass

c. Cholesterol 110 mg/dL (2.85 mmol/L) d. Total lymphocyte count (LTC) 1000/mcL e. Hemoglobin 10.9 g/dL (6.76 mmol/L) TLC is used to assess immune function and is below 1500 when a client is malnourished. Hemoglobin may be low, secondary to low serum albumin, catabolism and anemia. A cholesterol level below 160 mg/dL (4.14 mmol/L) has been identified as a possible indicator of malnutrition. Sodium is not typically affected by protein-energy malnutrition (and the results are within normal limits in this scenario). With protein-energy malnutrition, there is a decrease in lean body mass, the heart rate is slower than normal, blood pressure is lower than normal and the person's temperature may be elevated.

The nurse is caring for a client who is experiencing a hypertensive crisis. The priority assessment in the first hour of care after admission to the critical care unit should focus on which factor? a.Heart rate b. Lung sounds c. Cognitive function d. Pedal pulses

c. Cognitive function The organ most susceptible to damage in hypertensive crisis is the brain, due to rupture of the cerebral blood vessels. Neurologic findings must be closely monitored.

The nurse is named in a lawsuit. Which of these factors will offer the best protection for the nurse in a court of law? a. Clinical specialty certification by an accredited organization b. Above-average performance reviews prepared by nurse manager c. Complete and accurate documentation of assessments and interventions d. Sworn statement that health care provider orders were followed

c. Complete and accurate documentation of assessments and interventions The medical record is a legal document. Documentation should include all steps of the nursing process; it must be complete, accurate, concise and in chronological order. Inaccurate or incomplete documentation will raise red flags and may indicate the nurse failed to meet the standards of care. The attorney will review the medical record with the nurse before giving a deposition (sworn pretrial testimony.) Above-average performance reviews could be considered supporting information. Certification is an "extra" based on the nurse's initiative; it is, however, unrelated to accurate charting.

A patient is in the 3rd month of her first pregnancy. During the interview, she tells a nurse that she has several sex partners and is unsure of the identity of her baby's father. Which of these nursing interventions is best at this time? a. Refer the patient to family planning clinic b. Discuss the risk for cervical cancer c. Counsel the woman to consent to HIV screening d. Perform tests for STDs

c. Counsel the woman to consent to HIV screening The patient's behavior places her at high risk for HIV. While it would be a good idea to draw blood to test for STDs, this can't be one without informed consent of the patient. Since the woman is already at a clinic seeking health care, it would be best to provide information (and possibly begin treatment) now, instead of simply referring her to another health care facility. The best response is for the nurse to provide information and counsel the woman to consent to HIV screening.

A nurse is caring for a postoperative client who develops evisceration of the abdominal incision. Which intervention should the nurse implement first to prevent additional complications? a. Place the client in dorsal recumbent position b. Medicate the client for pain with PRN order c. Cover the wound with a sterile saline-soaked dressing d. Call the health care provider within the hour

c. Cover the wound with a sterile saline-soaked dressing When evisceration occurs, the wound should first be quickly covered by sterile saline soaked dressings. This prevents tissue damage and drying of the area until a surgical repair can be done. The other interventions are also appropriate, though the call to the provider should occur immediately, as this is a medical emergency.

The parents of a 6 year-old child who normally enjoys school tells a nurse that the child has not been doing well since a grandmother died two months ago. Which statement most accurately describes thoughts on death and dying at this age? a. Death is perceived as being irreversible b. The child feels guilty for the grandmother's death c. Death is personified as the bogeyman or devil d. The child is worried that the child, too, might die

c. Death is personified as the bogeyman or devil Personification of death is typical of this developmental level. Recall that this age is at the end of the preschool period where magical thinking for the animation of inanimate objects is present.

The nurse is teaching effective stress management techniques to a client one hour before surgery. Which of these actions should the nurse recommend? a. Biofeedback b. Distraction c. Deep breathing d. Imagery

c. Deep breathing Deep breathing is a reliable and valid method for stress reduction and can be taught and reinforced in a short period of time preoperatively. The other approaches require more time and repetition over time for maximum effectiveness.

Parents are concerned that their 11yo child is a very picky eater. The nurse suggests which of these approaches be the best initial action? a. Provide fruit, vegetables, and protein snacks b. Encourage the child to keep a daily log of foods eaten c. Discuss the consequence of an unbalanced diet with the child d. Consider a liquid supplement to increase calories

c. Discuss the consequence of an unbalanced diet with the child A priority is to educate the preadolescent as to appropriate diet, and the outcomes that result if the diet is not adequate. Afterwards the other options are appropriate to implement

A postoperative client has a prescription for acetaminophen with codeine. What should a nurse recognizes as a primary effect of this combination? a. Increased onset of action b. Minimized side effects c. Enhanced pain relief d. Prevention of drug tolerance

c. Enhanced pain relief Combination of analgesics with different mechanisms of action can afford greater pain relief. Tylenol No. 3 is a combination of 300 mg of acetaminophen and 30 mg of codeine; Tylenol No. 4 is a combination of 300 mg of acetaminophen and 60 mg of codeine.

The nurse administers an intermittent intravenous medication through a client's peripherally inserted central catheter (PICC) and disconnects the infusion from the PICC site. To best maintain patency of the PICC site, which action does the nurse take next? a. Apply a negative pressure technique while flushing the catheter b. Apply a smooth, even flow when flushing the catheter c. Flush the catheter using a rapid push-pause technique d. Flush the catheter using a 3 - 5 mL sterile syringe

c. Flush the catheter using a rapid push-pause technique The rapid push-pause flush technique involves rapid instillation of 1-2 mL of flush with each push of the plunger. This technique creates turbulence, which decreases the adherence of fibrin and platelets to the lumen wall and prevents occlusion. To avoid excess pressure on the catheter, a syringe barrel of at least 10 mL should be used to flush a PICC. The plunger should not be fully depressed when it's removed from the PICC because this creates negative pressure, resulting in a reflux of blood into the catheter lumen and possible thrombus formation.

Several hours after a gastrectomy, the nasogastric tube (NGT) stops draining. After referring to the standing gastrectomy postoperative orders, what order will the nurse implement first? a. Notify the surgeon b. Increase the amount of suction c. Gently irrigate the tube with sterile normal saline d. Reposition the tube until it begins to drain

c. Gently irrigate the tube with sterile normal saline The nurse will assess the position and patency of the NGT, as well as the color and amount of gastric drainage. The nurse can gently irrigate the NG tube with sterile normal saline if it becomes clogged. But if that does not resolve the issue or repositioning the tube is needed, the nurse must call the surgeon. The NGT inserted in surgery should not be repositioned by the nurse because of the risk of disrupting any internal sutures. The NGT should be connected to low suction; it would be contraindicated to increase the suction.

A nurse has administered several blood transfusions over three days to a 12 year-old client with thalassemia. What lab value should the nurse monitor during this therapy? a. Reticulocyte count b. Platelet count c. Hemoglobin d. Red blood cell indices

c. Hemoglobin Children with beta thalassemia major will usually require blood transfusions about every three to four weeks throughout their life. Transfusions help maintain hemoglobin at a high enough concentration to provide oxygen to the body and prevent growth abnormalities and organ damage; therefore, the nurse should monitor hemoglobin following a transfusion. A reticulocyte count is used as a diagnostic tool (to help rule out iron-deficiency anemia). Monitoring platelets would be indicated following transfusion of platelets.

A pregnant client, at 34-weeks gestation, is diagnosed with a pulmonary embolism (PE). Which of these medications does the nurse anticipate the health care provider will initially order? a. Low dose aspirin therapy b. Warfarin (Coumadin) therapy every other day to maintain a PT at 1.5 to 2 times the control value c. Heparin infusion to maintain the aPTT at 1.5 to 2 times the control value d. Subcutaneous heparin 5000 units twice a day

c. Heparin infusion to maintain the aPTT at 1.5 to 2 times the control value Clients diagnosed with PE, whether pregnant or not, are initially treated with intravenous unfractionated heparin (UFH). The client's activated partial thromboplastic time (aPTT) should be monitored and kept in the therapeutic range of between 1.5 to 2 times the baseline value. Alternatively, low molecular weight heparins, such as enoxaparin (Lovenox), can be used to treat PE in women who are pregnant. Warfarin should never be given during pregnancy due to its teratogenic effects. Although aspirin has anticoagulant properties, low dose aspirin therapy (81 mg), with or without heparin, is more often used prophylactically to prevent the development of deep vein thrombosis.

A nurse is caring for a client who was successfully resuscitated from a pulseless arrhythmia. Which assessment is critical for the nurse to include in the plan of care? a. Blood glucose every four hours b. Temperature every two hours c. Hourly urine output d. White blood count trends

c. Hourly urine output Clients who have decreased glomerular perfusion from shock or other low blood pressure conditions are at risk for prerenal failure. Close observation of hourly urinary output is necessary for early detection of this condition in a client who has experienced a pulseless arrhythmia.

A client tells the nurse, "I have something very important to tell you if you promise not to tell." The nurse should respond with which statement? a. That depends on what you tell me b. I must document and report any information c. I can't make such a promise d. I must report everything to the treatment team

c. I can't make such a promise Secrets, inappropriate in therapeutic relationships, are counterproductive to the therapeutic efforts of the interdisciplinary team. Secrets may be related to risk for harm to self or others on the unit. The nurse should honor and help clients to understand the rights, limitations and boundaries regarding confidentiality and professional relationships

Nurse is doing preconception counseling with a woman who is planning a pregnancy. Which statement suggests that the patient understands the connection between alcohol consumption and fetal alcohol syndrome? a. Beer is not really hard alcohol, so I guess I can drink some b. I understand that a glass of wine with dinner is healthy c. If I drink, my baby may be harmed before I know I am pregnant d. Drinking with meals reduces the effects of alcohol

c. If I drink, my baby may be harmed before I know I am pregnant Alcohol has the greatest teratogenic effect during organogenesis in the first weeks of pregnancy. Therefore, women considering a pregnancy should not drink any alcoholic beverages.

A client with heart failure has digoxin ordered. What would the nurse expect to find when evaluating for the therapeutic effectiveness of this drug? a. Decreased chest pain with decreased blood pressure b. Increased heart rate with increased respirations c. Improved respiratory status with increased urinary output d. Diaphoresis with decreased urinary output

c. Improved respiratory status with increased urinary output Digoxin (Lanoxin), a cardiac glycoside, is used in clients with heart failure to slow and strengthen the heartbeat. As cardiac output is improved, renal perfusion is improved and urinary output increases. Clients can become toxic on this drug, indicated by findings of bradycardia or tachycardias above 120, arrhythmia, visual or gastrointestinal disturbances. Clients being treated with digoxin should have the apical pulse evaluated for one full minute prior to the administration of the drug.

The nurse is assessing a patient in her 3rd trimester. The patient is informed that the ultrasound suggests the baby is small for gestational age. An earlier ultrasound indicated normal birth. The nurse understands that this change is most likely due to what factor? a. Exposure to teratogens b. STIs c. Maternal hypertension d. Chromosomal abnormalities

c. Maternal hypertension Pregnancy-induced hypertension is a common cause of late pregnancy fetal growth restriction. Vasoconstriction reduces placental exchange of oxygen and nutrients. The other 3 conditions are associated with the first trimester time period.

A 4 year-old child is admitted with burns on the legs and lower abdomen. During the assessment of the child's hydration status, which finding indicates a less-than-adequate fluid replacement therapy? a. A fall in the hematocrit and decrease in blood urea nitrogen (BUN) b. Decreased heart rate and respiratory rate c. Increased hematocrit and decrease in the urine volume d. Decreased white blood cell count

c. Increased hematocrit and decrease in the urine volume Very aggressive fluid replacement is indicated in children who have been burned, because hypovolemia may quickly lead to shock. A rising hematocrit indicates a decrease in total blood volume, which is a finding consistent with dehydration. Because there is a fluid volume deficit, the kidneys will conserve water and urine output will decrease. The nurse should expect an increase in white blood cells due to tissue damage. With dehydration, both heart and respiratory rates will increase. BUN will increase due to both dehydration and the burns.

Which statement describes the advantage of using a decision grid to make decisions? a. It is the only truly objective way to make a decision in a group b. It is the fastest way for group decision making c. It is both a visual and a quantitative method of decision making d. It allows the data to be graphed for easy interpretation

c. It is both a visual and a quantitative method of decision making A decision grid allows the group to visually examine alternatives and evaluate them quantitatively with weighting.

A nurse enters a 2 year-old child's hospital room in order to administer an oral medication. When the nurse asks the child, "Are you ready to take your medicine?" the response is an immediate, "No!" What would be an appropriate next action by the nurse? a. Give the medication to the parent and ask the parent to give it b.Explain to the child that the medicine must be taken now c. Leave the room and return five minutes later and give the medicine d. Mix the medication with ice cream or applesauce

c. Leave the room and return five minutes later and give the medicine Because the nurse gave the child a choice about taking the medication, the nurse must comply with the child's response in order to build or maintain trust. Toddlers do not have an accurate sense of time, so leaving the room and coming back later is another episode to the toddler. Medications should not be mixed with food nor fluids because the amount taken, especially in children, cannot be controlled.

A nurse is providing a parenting class to individuals living in a community of older homes that were built prior to 1978. During a discussion about formula preparation, which statement is the most important by the nurse to tell the parents how to prevent lead poisoning? a. Boil the tap water for 10 minutes prior to preparing the formula b. Buy bottled water labeled "lead free" to mix with the formula c. Let tap water run for two minutes before adding to formula concentrate d. Use ready-to-feed commercial infant formula

c. Let tap water run for two minutes before adding to formula concentrate The use of lead-contaminated water to prepare infant formula is a major source of lead poisoning in infants who live in older houses. Drinking water may become contaminated by lead from old lead pipes or the lead solder used in sealing the water pipes in homes prior to 1978. Letting tap water run for several minutes will diminish the risk for lead contamination. These same houses have the risk of lead contamination from paint chips because prior to that time, paint and gasoline contained lead.

A client tells the nurse that the client is fearful of the planned surgery because of evil thoughts about a family member. What is the best initial response by the nurse? a. Deny the feelings b. Call a chaplain c. Listen to the client d. Cite recovery statistics

c. Listen to the client Therapeutic communications are based on attentive listening to expressed feelings. If the nurse is not familiar with the cultural beliefs of a client, the nurse's acceptance of feelings should be followed by questions about the client's beliefs.

The nurse is caring for a client in a violent relationship. The nurse should understand that immediately after an acute battering incident, the batterer may respond to the partner's injuries by taking which action? a. Seek medical help for the victim's injuries b. Be very remorseful and assist the victim to receive medical care c. Minimize the episode with an underestimation of the victim's injuries d. Contact a close friend and ask for help with the incident

c. Minimize the episode with an underestimation of the victim's injuries Many batterers lack an understanding of the effects of their behavior on the person who was battered. Batterers use excessive minimization and denial of the situation and their behaviors or intent.

The nurse witnesses a client who is exhibiting seizure activity. Which observation is the priority and will be used to help determine treatment? a. Identify the pattern of breathing b. Note the time from beginning to end c. Observe the sequence and types of movement d. Determine if loss of bowel or bladder control occurs

c. Observe the sequence and types of movement All behaviors observed during and after the seizure need to be reported and recorded. However, accurate descriptions of seizure activity and a system for recording and reporting the activity is essential to seizure management. For example, during the seizure event, the nurse needs to observe the client's facial expression, muscle tone, movements (jerking or twitching, for example), the part(s) of the body involved, and any automatic or repeated movement (lipsmacking, chewing, swallowing, for example.)

he client is observed falling out of bed when reaching for something on the overbed table. The client then states: "Don't just stand there. I feel fine - help me up." What is the correct order of actions the nurse should take? a. Call the health care provider b. Complete an incident report c. Obtain a complete set of vital signs d. Assist the client back to bed, with help from other staff

c. Obtain a complete set of vital signs d. Assist the client back to bed, with help from other staff a. Call the health care provider b. Complete an incident report The first step is always to assess the client for any obvious injuries and to obtain a complete set of vital signs (especially blood pressure) and neurologic assessments. If the client does not appear to be injured, staff members can assist the client back into bed. The nurse should then call the health care provider to report the incident. Finally, the nurse should complete the incident report. Of course, personal items should be placed close to the client so that s/he can reach them.

A client, who is receiving a blood transfusion, reports having a headache and low back pain. What are the nurse's actions? (Select all that apply.) a. Flush the line with saline b. Administer acetaminophen (Tylenol) 1000 mg c. Obtain first voided urine (within one hour of reaction) d. Establish a saline lock or patent IV e. Send the tubing and bag to the blood bank f. Stop the blood transfusion

c. Obtain first voided urine (within one hour of reaction) d. Establish a saline lock or patent IV e. Send the tubing and bag to the blood bank f. Stop the blood transfusion The transfusion should be stopped immediately to reduce the risk of further damage. A sample from the transfusion will be cross-matched with client samples from before and after transfusion to check for errors in cross-matching. If this is an acute hemolytic transfusion reaction, intravascular hemolysis causes hemoglobinuria with varying degrees of kidney failure and possibly even disseminated intravascular coagulation (DIC). A urinalysis would reveal any hemoglobinuria that occurred due to hemolysis. If it's a febrile nonhemolytic transfusion reaction (as determined by the lab tests), then the headache can be treated with a lower dose of acetaminophen.

An 8 yo child is admitted to the children's inpatient mental health unit. After the mother's departure, the patient cries and refuses to eat dinner. Which of the following nursing actions is most appropriate? a. Explain that the parent will be upset if the child does not cooperate b. Remind the child of the expectation to eat some or all of the dinner c. Offer to play with the child d. Tell the child that privileges will be denied for uncooperative behavior

c. Offer to play with the child Play can be used as a distraction and it also facilitates the development of a safe relationship. Play also helps children express their feelings more easily (through toys instead of words.) Setting limits is necessary, but this can be done later. Children must learn responsibility for their own behavior; it is not therapeutic to say their behavior will upset their parents.

The RN is working in a clinic where a client presents with a painful, blistering rash on the hip. The health care provider diagnoses shingles (herpes zoster). What is the priority nursing diagnosis? a. Risk for infection related to skin lesions b. Knowledge deficit related to disease process c. Pain related to nerve root inflammation and skin lesions d. Risk for impaired skin integrity related to skin lesions

c. Pain related to nerve root inflammation and skin lesions Shingles is a reactivation of the herpes zoster virus responsible for chickenpox. It is characterized by a vesicular rash in a unilateral dermatomal distribution. The first symptom of shingles is usually pain, tingling, or burning before the blisters form. The pain and burning may be severe, and can lead to long-term residual pain, known as postherpetic neuralgia. Early appropriate treatment with an antiviral medication such as acyclovir can reduce these long-term complications, as well as the duration and severity of the initial symptoms. Pain is the priority nursing diagnosis. It is important that the client keeps the sores clean and avoids contact with people who haven't gotten the herpes zoster vaccine or who haven't had chickenpox, as well as people with weakened immune systems, until the rash crusts over and heals.

A 3 year-old child who is diagnosed with celiac disease attends a day care center. Which of the following foods would be an appropriate snack? a. Vanilla cookies b. Cheese crackers c. Potato chips d. Peanut butter sandwich

c. Potato chips Children with celiac disease should eat a gluten-free diet. Gluten is found mainly in grains of wheat and rye and in smaller quantities in barley and oats. Corn, rice, soybeans and potatoes are digestible by persons diagnosed with celiac disease.

The nurse is performing a physical assessment on a client who just had an endotracheal tube (ET) inserted with a connection to a ventilator. Which finding should prompt the nurse to take immediate action to resolve the issue? a. Client is unable to speak b. Mist is visible in the T-Piece of the ventilator circuit c. Pulse oximetry of 86% saturation d. Breath sounds are heard bilaterally

c. Pulse oximetry of 86% saturation Pulse oximetry should not be lower than 90% saturation. Breath sounds are heard bilaterally so the placement of an ET is most likely in proper position. The ventilator settings will need to be rechecked. A client with an ET tube in place will not be able to talk when the ET tube balloon is inflated.

The client, who is 12-hours post gastric bypass surgery, is restless and reports increasing back and shoulder pain unrelieved by pain medication. What action should the nurse take first? a. Check the nasogastric (NG) tube for patency and reposition the tube b. Roll the client to side-lying position to ensure the epidural analgesia catheter is still in place c. Report the complaint to the surgeon immediately d. Place the client in Trendelenburg position

c. Report the complaint to the surgeon immediately Anastomotic leak is the most serious complication after bariatric surgery and the most common cause of death. Clients should be monitored for increased back, shoulder, or abdominal pain, restlessness, unexplained tachycardia, and oliguria; these findings should be immediately reported to the surgeon. A nasogastric tube should not be manipulated postoperatively because it could damage the surgical site. In order to reduce intra-abdominal pressure on the diaphragm and improve tidal volumes, the head of the bed should be elevated 30 to 45 degrees, preferably in reverse Trendelenburg position. Patient controlled analgesia is used for pain management; epidural analgesia is not often used because of the difficulty of locating exact spinal segments for proper insertion of a catheter.

An 80 year-old client admitted with a diagnosis of possible cerebral vascular accident (CVA) has had a blood pressure of 160/100 to 180/110 over the past two hours. The nurse has also noted increased lethargy since admission. Which of the following new findings should the nurse report immediately to the provider? a. Rapid pulse b. Incontinence c. Slurred speech d. Muscle weakness

c. Slurred speech Changes in speech patterns and level of conscious are indicators of potential continued intracranial bleeding or extensions of a stroke. Further diagnostic testing may be indicated. Recall the word "FAST" with stroke findings: "F" is for changes in the face such as drooping of corner of the eye or mouth, "A" is for a drifting down of one arm when the arms are raised to shoulder height, "S" is for slurred speech and "T" is to telephone 911.

A nurse is teaching a group of adults about modifiable cardiac risk factors. Which of the following should the nurse focus on first? a. Weight reduction b. Stress management c. Smoking cessation d. Physical exercise

c. Smoking cessation Smoking cessation is the priority for clients at risk for cardiac disease. Smoking's effects result in reduction of cell oxygenation and constriction of the blood vessels. All of the other factors should be addressed at some point in time.

A 2 year-old child is brought to the emergency department at 2:00 pm. The mother states: "My child has not had a wet diaper all day." The child is pale, with a heart rate of 132 beats per minute. What other assessment data would the nurse obtain next to help determine an admitting diagnosis? a. History of fluid intake b. Dietary patterns in the past 48 hours c. Status of the eyes and the tongue d. Description of play activity

c. Status of the eyes and the tongue Clinical findings of dehydration include sunken eyes, dry tongue, lethargy, irritability, dry skin, decreased play activity, and increased pulse. The normal pulse rate in this age child is 70-110 beats per minute. The other information would be collected afterwards.

A client has just returned to the medical-surgical unit postop for a segmental lung resection. After assessing the client, which is the first action the nurse should take? a. Apply the pulse oximeter and monitor oxygen saturation b. Administer the PRN pain medication c. Suction excessive tracheobronchial secretions d. Assist the client to turn, deep breathe and cough

c. Suction excessive tracheobronchial secretions This type of surgery involves removing a bronchovascular segment of a lobe. It is typically used to remove small, peripheral lung tumors. Surgical manipulation during this procedure, along with anesthesia, can increase mucus production and lead to airway obstruction, which is why the nurse may need to suction the client if there are excessive secretions. Since this client just returned from surgery, it is not the time to ask the client to cough and deep breathe. Vital signs and oxygen saturation would have been measured during the assessment. The client would probably be on a PCA, or have a intrathecal or intrapleural catheter for pain relief.

The patient is admitted to same day surgery for carpal tunnel release of the left wrist. Before the anesthetic is administered, what measures are used to prevent surgical errors? Select all that apply a. The pre-op nurse reviews all relevant documents b. The anesthesiologists asks if anyone has any concerns c. Surgical site is marked by surgeon d. The patient is asked to confirm correct surgical site e. The patient is asked to state name and DOB

c. Surgical site is marked by surgeon d. The patient is asked to confirm correct surgical site e. The patient is asked to state name and DOB Marking the correct site helps prevent wrong site operations. The patient must also verbally state name and DOB (and any other identifiers required by facility). Pre-op verification of all required document is done independently by at least 2 providers. When the patient is in the OR suite, a time out is called. This is the final safety check between the surgical, nursing, and anesthesia care teams immediately before the procedure. It is not enough for one person to ask if there are any other questions or concerns.

At the geriatric day care program, a client who has been diagnosed with a neurocognitive disorder is crying and repeatedly saying: "I want to go home. Call my daddy to come for me." The nurse should take which action? a. Give the client simple information about what the client will be doing that day b. Direct the client firmly to the assigned group activity c. Tell the client you will call someone to come get the client and suggest the client to join an exercise group while waiting d. Inform the client that the client must wait until the program ends at 5:00 pm to leave

c. Tell the client you will call someone to come get the client and suggest the client to join an exercise group while waiting Comfort and distraction are key approaches in validation therapy. They are the kindest and most effective actions for clients who have varying degrees of a neurocognitive disorder (formerly referred to as dementia.) The distressed, disoriented client should be gently oriented to reduce fear and increase the sense of safety and security. However, reorientation is often ineffective when the client is upset. Environmental changes provoke stress and fear, especially in clients diagnosed with any degree of Alzheimer's disease.

A client diagnosed with testicular cancer has undergone a unilateral orchiectomy. The client expresses fears about his prognosis prior to discharge. What information would the nurse want to include when helping the client better understand this type of cancer? a. With early intervention, the cure rate for teticular cancer is about 50% b. This surgery causes impotence and infertility c. Testicular cancer has a five-year survival rate of 95% with early diagnosis and treatment d. Intensive chemotherapy is the treatment of choice following surgery

c. Testicular cancer has a five-year survival rate of 95% with early diagnosis and treatment With aggressive treatment and early detection/diagnosis the cure rate is 90%. The other options are incorrect information. After unilateral orchiectomy, the remaining testicle can produce adequate sperm for fertility and impotence is unlikely. In bilateral orchiectomy, fertility is lost, so sperm banking prior to surgery is recommended. Dissection of lymph nodes for surgical cancer treatment may cause nerve injury, which would increase the risk of impotence.

The health care team consists of one licensed practical nurse (LPN), one unlicensed assistive person (UAP) and one LPN student. The charge nurse (an RN) has made the following assignments. Which assignment should be questioned by the nurse manager? a. Measuring vital signs and assisting with activities of daily living (ADLs) for the client admitted with myocardial infarction 4 days ago - UAP b. A child diagnosed with second-degree burns over 20% of the body, has IV packed red cells running and an order for IV albumin - charge nurse c. The admission at the change of shifts of a client diagnosed with atrial fibrillation and acute heart failure - LPN d. A client who was diagnosed with a major stroke 6 days ago - LPN student

c. The admission at the change of shifts of a client diagnosed with atrial fibrillation and acute heart failure - LPN LPNs can provide care for clients whose conditions are stable and there's a low likelihood of an emergency. Since it's a new admission, the client diagnosed with atrial fibrillation and heart failure should not be assigned to a student; the charge nurse (RN) should care for this client. A nurse can assign tasks or activities to UAP, as long as the care of the client is not too complex or variable and the client's condition is stable.

The nurse receives a report on a client being admitted with the diagnosis of cirrhosis of the liver and ascites. What should the nurse emphasize to the nursing assistant about providing care for this client? a. The client is to ambulate as tolerated and be positioned in semi-Fowler's position when in bed b. The client should remain on bed rest in a semi-Fowler's position c. The client should ambulate as tolerated, resting in bed with legs elevated between walks d. The client may ambulate and sit in a chair as tolerated

c. The client should ambulate as tolerated, resting in bed with legs elevated between walks Encourage alternating periods of ambulation and bed rest with legs elevated to mobilize edema and ascites. Encourage and assist the client to gradually increase the duration and frequency of walks.

The nurse is checking on clients in the unit. Which of these findings indicates that an infusion pump set to deliver a morphine drip basal rate of 10 mL per hour, plus PRN dosages for breakthrough pain, is not functioning correctly? a. The client states: "I just can't get relief from my pain." b. The level of the drug is 100 mL at 9 am and is 50 mL at 12 noon c. The level of the drug is 100 mL at 8 am and is 80 mL at 12 noon d. The client complains of discomfort at the IV insertion site

c. The level of the drug is 100 mL at 8 am and is 80 mL at 12 noon The minimal dose is 10 mL per hour, which would mean 40 mL is given in a four-hour period. If any PRN doses were given then less would be in the bag. Minimally, 60 mL should be left at 1200 (12 Noon). The pump is not functioning when more than expected medicine is left in the container.

A novice charge nurse is delegating duties. Which tasks, if delegated to an unlicensed assistive personnel (UAP), would require intervention by the nurse manager? a. To empty a urethral collection bag b. To feed a 2 year-old with a broken arm c. To bathe a woman with internal radon seeds and device d. To help an older adult client to the bathroom

c. To bathe a woman with internal radon seeds and device A client with internal radiation is complex care and is not suitable to be assigned to a UAP. Additionally, the client would not receive a complete bath because movement is minimized during this therapy to prevent the slippage of the device.

A client was admitted to the psychiatric unit after refusal to get out of the bed. Once admitted, the client is observed talking to unseen people and voiding on the floor. The nurse should handle the problem of voiding on the floor by which of these approaches? a. Require the client to mop the floor after each incident b. Restrict the client's fluids throughout the day c. Toilet the client more frequently with supervision d. Withhold privileges each time the voiding occurs

c. Toilet the client more frequently with supervision With a client that has altered thought processes, the appropriate nursing approach to change behaviors is to take an active role in attending to the physical needs of the client. The other options are incorrect approaches.

A client reports taking lithium as prescribed. Which of these findings indicate early signs of lithium toxicity? a. Electrolyte imbalance, tinnitus and cardiac arrhythmias b. Pruritus, rash and photosensitivity c. Vomiting, diarrhea and lethargy d. Ataxia, agnosia and course hand tremors

c. Vomiting, diarrhea and lethargy Serum lithium levels should be between 0.8 - 1.2 mEq/L (remember, the exact numbers may vary slightly depending on the lab). Diarrhea, vomiting, drowsiness, muscular weakness and lack of coordination may be early signs of lithium toxicity. Toxicity increases with increasing serum lithium levels, but clients may exhibit toxic finding at lithium levels below 2.0 mEq/L. Dehydration, other medications and other conditions can interfere with lithium levels.

A nurse is teaching a client with a diagnosis of metastatic bone disease about actions to prevent hypercalcemia. It would be important for the nurse to include which of these points? a. At least five servings of dairy products are to be eaten daily b. Early recognition of findings associated with tetany ê d. A restriction of fluid intake is to be less than one liter per day

c. Walking as much as possible keeps the calcium in the bone Mobility must be emphasized to prevent demineralization and breakdown of bones. Weight-bearing and resistance exercises will assist in this process.

A client with a central line catheter is being discharged. Which of the following methods is the most accurate way to evaluate the client's partner's ability to provide central line catheter dressing changes at home? a. Send the partner to a class about how to change dressings b. Provide pamphlets and pictures about the skill c. Watch the partner change the dressing d. Allow repeated practice on a lifelike mannequin

c. Watch the partner change the dressing Each of the teaching methods is appropriate for a manual skill such as a dressing change. However, the only one that actually demonstrates mastery is observing the partner perform the skill.

The mother of a 3 month old infant tells the nurse. "I want to change from formula to whole milk and add cereal and meats to my infant's diet." What should be emphasized as the nurse teaches about nutrition. a. Fluoridated tap water should be used to dilute milk b. Solid foods should be introduced at 3 to 4 months c. Whole milk is difficult for infants to digest d. Supplemental apple juice can be used between feedings.

c. Whole milk is difficult for infants to digest Cow's milk is not given to infants younger than a year old because the tough, hard curd that develops in the digestive tract is difficult to digest. In addition, it contains little iron and creates a high renal solute load. If infants drink milk with a minimal introduction to solid food they will have the tendency to develop anemia.

A 35 year-old female client talks to the nurse in her health care provider's office about her new diagnosis of uterine fibroids. What statement by the woman is incorrect and indicates that more teaching is needed? a. "Uterine fibroids are noncancerous tumors that grow slowly." b. "Fibroids occur more frequently in women my age but no one knows what causes them." c. "I sometimes experience pelvic pressure and pain, heavy menstrual bleeding, and I feel the need to urinate more often." d. "Even if the fibroids cause no problems, they will still need to be taken out."

d. "Even if the fibroids cause no problems, they will still need to be taken out." Fibroids that cause no findings may require only "watchful waiting." The client may just need pelvic exams or ultrasounds every once in a while to monitor the fibroid's growth. Treatment for the symptoms of fibroids (such as painful menses and heavy periods) may include oral contraceptives, IUDs, iron supplements to prevent or treat anemia (due to heavy periods), NSAIDs for cramps or pain or even short-term hormonal therapy to help shrink the fibroids. Surgical removal using myomectomy or hysterectomy is usually reserved as a final alternative after other treatment options have failed to provide adequate relief. In addition, concerns about loss of fertility with this diagnosis and its treatment may be important to this client who is still in her childbearing years.

At a well-child checkup, the nurse is assessing a 1 year-old who was born prematurely and is being evaluated for cerebral palsy (CP). Which information provided by the parents would support this diagnosis? a. "Our child isn't talking yet." b. "We think our child seems smaller than other babies this age." c. "Mealtime is so messy when he tries to feed himself." d. "He crawls by pushing off with one hand and leg while dragging the opposite hand and leg."

d. "He crawls by pushing off with one hand and leg while dragging the opposite hand and leg." Cerebral palsy refers to a group of conditions that affect movement, balance and posture. Prematurity, infections during pregnancy, and asphyxia during labor and delivery are risk factors for CP. Some children with CP may have delays in learning to roll over, sit, crawl or walk. Because this child was born prematurely, it would be expected that he would be smaller than other babies. At this age, most children can say a few words (like "mama"), but they are not talking, and mealtime can get pretty messy.

A male client admitted with a diagnosis of a spinal cord injury (SCI) at level C-5 asks the nurse how the injury will affect his sexual function. Which statement is the best response? a. "Normal sexual function is not possible." b. "Ejaculation will be normal." c. "Sexual functioning will not be impaired at all." d. "Sexual functioning may be possible."

d. "Sexual functioning may be possible." Even though sexual function is controlled by parts of the central nervous system, sexual function after a SCI will depend on many factors, such as the severity of the injury and even the age of the client. The client will need to get to know his body and learn how it reacts in certain situations. The client should also talk to his health care provider for options to achieve an erection.

A client calls the evening health clinic to state, "I know I have a severely low sugar since the Lantus insulin was given three hours ago and it peaks in two hours." What should be the nurse's initial response to the client? a. "Are you taking any other insulin or medication?" b. "Have you eaten anything today?" c. "What else do you know about this type of insulin?" d. "What are you feeling at this moment?"

d. "What are you feeling at this moment?" When a client's condition has changed from stable to unstable, the nurse's initial response should be to do further assessment of the client. Insulin glargine (Lantus) is a long-acting insulin (up to 24-hour duration) taken once daily; it is designed for flatter and longer action with no pronounced peak.

A nurse entering the room of a postpartum mother observes the baby lying at the edge of the bed while the mother sits in a nearby chair. The mother states, "This is not my baby, and I do not want it." After repositioning the child safely, the nurse should respond with which comment? a. "Many women have postpartum blues and need some time to love the baby." b. "What a beautiful baby! Her eyes are just like yours and so is her smile." c. "This is a common occurrence after birth, but you will come to accept the baby." d. "You seem upset. Tell me what the pregnancy and birth were like for you."

d. "You seem upset. Tell me what the pregnancy and birth were like for you." A nonjudgmental, open ended response facilitates dialogue between the client and the nurse. The other three options ignore the situation and the needs of the mother. Note that the correct answer is the only client-centered option that is directly associated with the given situation.

A client who has been experiencing influenza-like symptoms for the past 24 hours calls the health clinic and asks about the antiviral medication zanamivir (Relenza). How should the triage nurse respond? a. "Come in right away so we can treat you." b. "Call back tomorrow when you are sure you have the flu." c. "Do you have trouble swallowing big pills?" d. "Your chart states that you have asthma, so this product would not be recommended."

d. "Your chart states that you have asthma, so this product would not be recommended." Antiviral treatment should be initiated within 48 hours of illness onset. It won't cure the disease but it will shorten the time someone is sick and may reduce the severity of the illness. It is administered by oral inhalation. Zanamivir should not be used by people with chronic pulmonary disorders.

The parent of a toddler ask the nurse how long their child will have to sit in a car seat while in an automobile. What is the nurse's best response to the parents? a. "The child can use a regular seat belt when can sit still." b. "Your child must reach a height of 50 inches to sit in a seat belt." c. "The child must be 5 yo to use a regular seat belt." d. "Your child must use a car seat until he weighs at least 40 lbs."

d. "Your child must use a car seat until he weighs at least 40 lbs." The guidelines for car seats depend on the child's weight, height, age, and car type. Children should use car seats until they weight 40 lbs according tho the US National Highway Traffic Safety Administration.

The nurse is making patient room assignments. In order to minimize the risk of a hospital acquired infection, which of these children would be the most appropriate roommate for a 3-year-old child diagnosed with minimal change disease a. 3 year old with fracture, with a sibling that has Fifth disease b. 2 year old diagnosed with respiratory infection c. 6 year old with sickle cell disease experiencing vaso-occlusive crisis d. 4 year old with bilateral inguinal hernia repair

d. 4 year old with bilateral inguinal hernia repair Minimal change disease is a kidney disorder that can lead to nephrotic syndrome. Corticosteroids can cure the disease in most children but cytotoxic therapy and other drugs may be needed, but this treatment can reduce the child's ability to fight infection. The charge nurse must select a roommate who does not have an infection, which is the child child who had surgery. The sickle cell crisis may have triggered an infection. The child's sibling who has a viral disease has the potential to develop an infection.

The nurse has just listened to the change of shift report on an orthopedic unit. Which of the following clients should the nurse check first? a. A 20 year-old in skeletal traction for two weeks since a motorcycle accident b. A 75 year-old who is in skin traction of the left leg prior to a scheduled fractured hip repair surgery c. A 16 year-old who had an open reduction of a fractured wrist 10 hours ago d. A 72 year-old who returned from a right hip replacement surgery two hours ago

d. A 72 year-old who returned from a right hip replacement surgery two hours ago The nurse should compare clients to screen for one who has the most imminent risks and acute vulnerability for being unstable. The client who returned from surgery two hours ago is at risk for hemorrhage because the hip and femur are considered vascular areas and should be checked first. The 16 year-old is within the initial 24 hours post-op period and should be seen next. The 75 year-old is potentially vulnerable to age-related physical and cognitive impairments from being on bedrest and having a large bone fracture. The client who can safely be visited last is the 20 year-old who is two weeks post-injury.

An RN from a woman's wellness health clinic is temporarily reassigned to an adult medical unit. Which of these client assignments would be appropriate for this nurse? a. A newly diagnosed client with type 2 diabetes mellitus who is learning about foot care b. A client admitted for a barium swallow after a transient ischemic attack (TIA) c. A newly admitted client with a diagnosis of pancreatic cancer and severe dehydration d. A client from a motor vehicle accident with an external fixation device on the leg

d. A client from a motor vehicle accident with an external fixation device on the leg The nurse from the wellness clinic should be assigned to the client with the leg fracture. This client is the most stable and providing care for this client has predictable outcomes. The clues in the other options are: "newly diagnosed," "after a TIA," and "newly admitted... severe dehydration" - all of these clients have an health condition that's less stable than the client who is basically healthy (except for a fracture from an accident).

A client has started clozapine therapy. What information should the nurse emphasize to the client about this medication during discharge teaching? a. It may cause hypoglycemia b. The tablet should be swallowed with at least 8 ounces of water c. The client may experience dry skin d. A common side effect is extreme salivation

d. A common side effect is extreme salivation Extreme salivation (sialorrhea) is a common side effect of clozapine (Clozaril) therapy. Taking clozapine, or other atypical antipsychotics, may increase the risk of hyperglycemia, not hypoglycemia. In fact, clients diagnosed with schizophrenia are more likely to develop diabetes than people who do not have schizophrenia. The tablet dissolves quickly on the tongue and can be swallowed with saliva - no water is needed to swallow a disintegrating tablet. The client may also experience dry mouth, but dry skin is not a finding associated with this medication.

A nurse from the mental health unit is reassigned to the pediatrics unit and will be caring for a child with asthma. Which of these findings would the charge nurse emphasize as the first thing to indicate a worsening condition in the child? a. Coughing, especially if the cough is frequent and occurs in spasms b. An audible whistling or wheezing when the child exhales c. Increased need to use bronchodilators d. A downward trend in peak flow rates as measured by a peak flow meter

d. A downward trend in peak flow rates as measured by a peak flow meter If the child uses a peak flow meter, the reassigned nurse should know that a fall in peak flow rates is often the first sign of worsening asthma. Other early findings of worsening asthma in a child may include feeling tired, sneezing, experiencing a stomach ache, poor appetite and/or night-time cough. As the airways continue to narrow, the child will get less relief from the "quick relief" medication and will experience wheezing and a cough that won't go away.

Which of these clients should the nurse assess and monitor for Clostridium difficile (C. difficile) diarrhea? a. An older adult client living in a retirement center taking prednisone b. A young adult at home taking a prescribed amino glycoside c. An adolescent taking tetracycline for acne d. A hospitalized middle-aged client receiving IV cephalexin (Keflex)

d. A hospitalized middle-aged client receiving IV cephalexin (Keflex) Hospitalized clients, especially those receiving antibiotic therapy, are primary targets for C. difficile. Examples of antibiotics that frequently cause C. difficile are ampicillin, amoxicillin and cephalosporins, including cephalexin (Keflex). Antibiotics that occasionally cause C. difficile include penicillin, erythromycin, trimethoprim and quinolones, such as ciprofloxacin (Cipro).

A nurse is working with one licensed practical nurse (LPN) and a mental health tech (an unlicensed assistive personnel). Which newly admitted client would be appropriate to assign to the mental health tech? a. An adolescent diagnosed with dehydration and anorexia b. A young adult who reports to be a heroin addict and states, "I am in withdrawal and seeing spiders." c. A 76 year-old client diagnosed with severe depression d. A middle-aged client diagnosed with an obsessive compulsive disorder

d. A middle-aged client diagnosed with an obsessive compulsive disorder The mental health tech (a type of unlicensed assistive personnel or UAP) can be assigned to care for a client with a chronic condition after an initial assessment by the nurse. This client has minimal risk of instability of condition and has a situation of expected outcomes.

An external disaster has occurred in the town. The triage nurse from the emergency department is transported to the site and assigned to triage the injured. Which of these clients would the nurse tag as "to be seen last" by the providers at the scene? a. An older adult person with a open fracture of the left arm b. An infant with bilateral fractured lower legs with no active bleeding c. A teenager with small amount of bright red blood dripping out of the nose d. A middle-aged person with deep abrasions that are over 90% of the body

d. A middle-aged person with deep abrasions that are over 90% of the body The clients that are least likely to survive are to be tagged as the "last to be seen." Deep abrasions are usually treated as second or third degree burns because the fluid loss is great.

The clients listed below are all using patient-controlled analgesic (PCA) pump for pain control. Which of these clients is least appropriate to use a PCA pump? a. A young adult with a history of Down syndrome b. A teenager who reads at a 4th-grade level c. An older adult client with numerous arthritic nodules on the hands d. A preschooler with intermittent episodes of alertness

d. A preschooler with intermittent episodes of alertness A preschooler is the one client most likely to have difficulty with the use or understanding of a PCA pump. The preschooler also has a decreased level of consciousness and would not be able to fully benefit from the use of a PCA pump. School-age children, ages 6 and up, are better candidates for PCA electronic pumps.

An internal disaster has occurred at the hospital. The charge nurse is asked to review client acuity and determine which clients can and cannot be discharged. Which of these clients should not be discharged? a. An older adult client admitted two days ago with an acute exacerbation of ulcerative colitis b. An adolescent diagnosed with sepsis seven days ago and whose vital signs are maintained within low normal limits c. A middle-aged client known to have had an uncomplicated myocardial infarction four days ago d. A young adult in the second day of treatment for an overdose of acetaminophen

d. A young adult in the second day of treatment for an overdose of acetaminophen An overdose of acetaminophen requires close observation for several days. Also, the duration of the course of treatment for the oral antidote N-acetylcysteine (NAC) is approximately 72 hours. NAC will protect the liver if given within 8 hours after an acute ingestion, but you aren't given enough information to know how soon treatment was started after ingestion. When compared with the other clients, the client who overdosed on acetaminophen is the least stable and should not be discharged.

A patient tells the nurse "I'm in a lot of pain. As the nurse collects more information about the patient's pain, what should be the first step in pain assessment? a. Have the patient identify previous methods that relieved the pain b. Ask the patient to rate the pain on a scale of 1 to 10 c. Determine the location of the pain d. Accept the patient's report of pain

d. Accept the patient's report of pain Before anything else, the nurse must simply accept the fact that the patient is experiencing pain - pain is subjective. The nurse would then determine the intensity of pain, quality, location, onset, and pain relief.

A nurse is teaching an older adult client to use a metered-dose inhaler (MDI) and is concerned that the client is unable to coordinate the release of the medication with the inhalation phase. What should the nurse recommend to improve the delivery of the medication? a. Ask a family member to assist the client with the MDI b. Request a visiting nurse to follow the client at home c. Nebulized treatments for home care d. Add a spacer device to the MDI canister

d. Add a spacer device to the MDI canister Use of a spacer is especially useful with older adults because it allows more time to inhale and requires less eye-hand coordination. If the client is not using the MDI properly, the medication can get trapped in the upper airway with an outcome of a dry mouth and throat irritation. Using a spacer will allow more drug to be deposited in the lungs and less in the mouth.

The nurse is preparing a client for an intravenous pyelogram (IVP). The nurse should take which action to adequately prepare the client? a. Instruct the client to maintain a regular diet the day prior to the examination b. Restrict the client's fluid intake four hours prior to the examination c. Inform the client that only one x-ray of the abdomen is necessary d. Administer a laxative to the client the evening before and an enema the morning of the test

d. Administer a laxative to the client the evening before and an enema the morning of the test Bowel prep is important prior to this procedure because a cleaned out GI tract allows greater visualization of the bladder and ureters. The client is usually NPO 8 to 12 hours before the test. After the dye is injected intravenously, x-rays are taken of the kidneys, ureters, and bladder at several intervals (typically at 0, 5, 10 and 20 minutes).

The nurse is caring for a toddler who is diagnosed with an infection and whose temperature is 103 F (39.4 C). Which intervention would be most effective in lowering the client's temperature and promoting comfort? a. Immerse the child in a tub containing cool water b. Give a tepid sponge bath prior to giving an antipyretic medication c. Apply extra layers of clothing to prevent shivering d. Administer the prescribed antipyretic medication

d. Administer the prescribed antipyretic medication Fever is not the primary illness; it is a physiologic mechanism the body uses to fight an infection. Although tepid sponge baths can lower the body temperature, they can distress febrile children (as evidenced by crying, shivering and goosebumps.) Antipyretics can not only reduce the fever in the child, but they can also improve comfort and decrease irritability.

A newborn is diagnosed with hypothyroidism. In discussing the condition and treatment with the family, the nurse should emphasize which point? a. This rare problem is always hereditary b. They can expect the child will be mentally retarded c. Physical growth and development will be delayed d. Administration of thyroid hormone will prevent complications

d. Administration of thyroid hormone will prevent complications Early identification (ideally before 13 days-old) and continued treatment with levothyroxine (thyroid hormone replacement) corrects hypothyroidism in newborns, preventing problems. If undetected and untreated, hypothyroidism can result in poor growth and weight gain, slow heart rate, low blood pressure, and babies who are unusually quiet; the child will be at risk for permanent brain damage and intellectual disabilities. Approximately one in every 4000 babies is born with hypothyroidism.

The nurse is developing a teaching plan for parents on safety and risk-reduction in the home. Which of the following should the nurse give priority consideration to during teaching? a. Number of children in the home b. Age and knowledge level of the parents c. Proximity to emergency services d. Age of children in the home

d. Age of children in the home Age and developmental level of the child are the most important considerations in the provision of a framework for anticipatory guidance associated with safety, and should be given priority when teaching safety.

A woman, who delivered five days ago and who had been diagnosed with pregnancy induced hypertension (PIH), calls a hospital triage nurse hotline to ask for advice. She states, "I have had the worst headache for the past two days. It pounds and by the middle of the afternoon everything I look at looks wavy. Nothing I have taken helps." What should the nurse do next? a. Advise the client to have someone bring her to the emergency room as soon as possible b. Ask the client to explain what she has taken and how often, and then evaluate other specific complaints c. Advise the client that the swings in her hormones may be the problem; suggest that she call her health care provider d. Ask the client to stay on the line, get the address, and send an ambulance to the home

d. Ask the client to stay on the line, get the address, and send an ambulance to the home The woman is at risk for seizure activity. The ambulance needs to bring the woman to the hospital for evaluation and treatment. For at-risk clients, PIH may progress to preeclampsia and eclampsia prior to, during, or after delivery; this may occur up to 10 days after delivery.

A nurse is teaching a mother who will breast-feed for the first time. Which of these approaches is a priority? a. Show the mother films on the physiology of lactation b. Give the mother several illustrated pamphlets c. Give the mother privacy for the initial feeding d. Assist the mother to position the newborn at the breast

d. Assist the mother to position the newborn at the breast All of the approaches should be helpful in teaching. However, the priority is to place the infant to the breast as soon after birth as possible to establish contact and allow the newborn to begin to suck.

A postpartum Hispanic client refuses hospital food because it is "cold." What action should the nurse take initially? a. Have UAP reheat the food if the patient wishes b. Schedule the dietician to meet with patient c. Encourage the patient to eat for healing and strength d. Ask the patient what foods are acceptable or unacceptable

d. Ask the patient what foods are acceptable or unacceptable Many Hispanic clients subscribe to the rebalancing of "hot" and "cold" in the postpartum period. After giving birth, when a woman has lost blood, she is considered to be in a cold state; therefore, she needs to restore her humoral balance. What defines "cold" and "hot" can best be explained by the client and this needs to be incorporated into the plan of care. Note that the correct response allows for client feedback; this is the only client-centered option.

A client is admitted for first- and second-degree burns on the face, neck, anterior chest and hands. What should be the nurse's priority action? a. Administer pain medication b. Initiate intravenous therapy c. Cover the areas with dry sterile dressings d. Assess for dyspnea or stridor

d. Assess for dyspnea or stridor Due to the location of the burns, the client is at risk for the development of upper airway edema and subsequent respiratory distress. The other options are correct, but the priority is to assess breathing and manage the airway. The client with any signs of airway injury will be intubated and given 100% oxygen.

A nurse is caring for a client two hours after a right lower lobectomy. During the assessment of the chest drainage unit (CDU), the nurse notes bubbling in the water-seal chamber. What is the first action the nurse should take? a. Call the health care provider as soon as possible b. Check for any increase in the amount of drainage c. Reposition the client to improve the level of comfort d. Assess the chest tube dressing, tubing and drainage system

d. Assess the chest tube dressing, tubing and drainage system The first action the nurse should take is to thoroughly check the dressing, tubing and drainage system. Usually intermittent bubbling in the water-seal chamber right after surgery indicates an air leak from the pleural space; this is a common finding and should resolve as the lung re-expands. Continuous bubbling usually means a leak in the CDU, such as a loose connection or a leak around the insertion site. Other nursing actions will include assessing the color and amount of the drainage and assessing the lungs. After the initial post-operative period, the nurse will assist the client to change positions and cough and deep breath to help re-expand the lung and promote fluid drainage.

The nurse is providing care to a client in labor. The client has chosen natural childbirth with assistance from a doula, her mother and boyfriend. Which of the following nursing actions can help the client achieve her goal of an unmedicated labor and birth? a. Offer pain medication on a regular basis b. Limit the number of interactions with the doula c. Encourage the client to stay in bed in a side-lying position or semi-Fowler's position d. Assess the effectiveness of the labor support team and offer suggestions as needed

d. Assess the effectiveness of the labor support team and offer suggestions as needed The nurse's role involves clinical skills and administrative responsibilities that are not part of the doula's role. The RN is responsible for assessing both the mother and baby and remains an important part of the labor and birth in this scenario. The RN's expertise allows the RN to make helpful suggestions to the support persons and the client, such as encouraging the client to find comfortable positions, both in and out of bed. It is appropriate to let the client and her support persons know all of the pain control options, but it would be inappropriate to continually offer pain medication to someone who has chosen natural childbirth. Doulas use techniques such as imagery, massage, acupressure, and patterned breathing to reduce a woman's pain.

A nurse is caring for a client diagnosed with chronic obstructive pulmonary disease (COPD) and who becomes dyspneic. The nurse should take which action? a. Administer oxygen at six liters per minute via nasal cannula b. Place the client in a low Fowler's position c. Instruct the client to breathe into a paper bag d. Assist the client with pursed-lip breathing

d. Assist the client with pursed-lip breathing Pursed-lip breathing should be encouraged during periods of dyspnea in COPD to control rate and depth of respiration, to prevent alveolar collapse and to improve respiratory muscle coordination. Clients with COPD are usually on lower doses of oxygen, titrated to maintain an oxygen saturation of 88-91%. Semi-Fowler's position is usually most comfortable for someone with COPD, because this position allows the client's diaphragm to expand.

A client who is two days postop, has these vital signs: blood pressure of 120/70, heart rate of 110 BPM, respiratory rate of 26, and a temperature of 100.4 F (38 C). The client suddenly becomes profoundly short of breath (SOB) and the skin color becomes grayish in color. Which assessment should the a nurse do first based on the client's change in condition? a. Palpate the pulses for bounding and irregularity b. Check for orthostatic hypotension c. Assess the pupils for unequal responses to light d. Auscultate for diminished breath sounds

d. Auscultate for diminished breath sounds The findings suggest pulmonary embolus as a result of a piece of a clot in the legs that has broken off. Thus, the breath sound will most likely be diminished or absent in the lung where the embolus lodged.

A nurse is teaching a client to select foods rich in potassium to prevent digitalis toxicity. Which choice indicates the client understands this dietary requirement and recognizes which foods are highest in potassium? a. Naval orange b. Three apricots c. Small banana d. Baked potato

d. Baked potato A baked potato contains 610 milligrams of potassium. Apricots, oranges and bananas do have higher potassium content, but because of their size they are not the highest in potassium. A baked potato is the highest in potassium of the given options.

he nurse is caring for a client diagnosed with acute angina. The client is receiving an intravenous infusion of nitroglycerin. What is the priority assessment during this treatment? a. Heart rate b. Neurologic status c. Urine output d. Blood pressure

d. Blood pressure The vasodilatation that occurs as a result of this medication can cause profound hypotension. The client's blood pressure must be evaluated every 15 minutes until stable, and then every 30 minutes to every hour thereafter. Clients receiving IV nitroglycerin also require continuous ECG monitoring.

Two members of the interdisciplinary team are arguing about the plan of care for a client. Which action could any one of the members of the team use as a de-escalation strategy? a. Interrupt, apologize for interruption, and change the subject b. Adjourn the meeting and reschedule when everyone has calmed down c. Tell the violators they must calm down and be reasonable d. Bring the communication focus back to the client

d. Bring the communication focus back to the client Bringing the subject of the communication back to the client refocuses attention on the client's care, instead of the manner of communication. It is the most effective strategy because it is an example of collaboration. The other options are non-productive and may even make matters worse.

A child is treated with succimer for lead poisoning. Which of these assessments should the nurse perform first? a. Check serum potassium level b. Check blood calcium level c. Test deep tendon reflexes d. Check complete blood count (CBC) with differential

d. Check complete blood count (CBC) with differential Succimer (Chemet) is used in the management of lead or other heavy metal poisoning. Although it has generally well tolerated and has a relatively low toxicity, it may cause neutropenia. Therapy should be withheld or discontinued if the absolute neutrophil count (ANC) is below 1200/µ.

The client with cancer is being treated with a biological response modifier. Which of the following side effects does the nurse anticipate with biologic therapy? a. Constipation b. Hematuria c. Photophobia and sun sensitivity d. Chills and fever

d. Chills and fever Biological response modifier cancer therapy agents (for example, interferons and interleukins) are drugs that stimulate the body's own defense mechanisms to fight cancer cells. Flu-like findings such as chills, fever and nausea, are common side effects of this type of therapy. The other assessment findings are not what you would expect when the body is fighting pathogens.

The nurse assesses the use of coping mechanisms by an adolescent one week after the client had a motor vehicle accident resulting in multiple serious injuries. Which of these characteristics are most likely to be observed by the nurse? a. Identification, assimilation, withdrawal b. Intellectualization, rationalization, repression c. Ambivalence, dependence, demanding d. Denial, projection, regression

d. Denial, projection, regression Helplessness and hopelessness may contribute to regressive, dependent behavior, which often occurs at any age with hospitalization. Denial or minimization of the seriousness of the illness is used to avoid facing the worst situation. Recall that denial is the initial step in the process of working through any loss.

The 72 year-old client has an estimated blood loss of 600 mL during a gastric resection. The surgeon orders two units of packed cells (PC) to be administered in the post anesthesia care unit. During the administration of the second unit of PC, the nurse notes the following findings: hypertension, a bounding pulse, and increasing dyspnea. What is the probable cause of these findings? a. Bacterial transfusion reaction b. Transfusion-associated graft-versus-host disease c. Hemolytic transfusion reaction d. Circulatory overload

d. Circulatory overload Older clients are at risk for circulatory overload, especially when solutions are administered rapidly. Hypertension with a bounding pulse and dyspnea are key signs of fluid overload. The nurse should stop the infusion and contact the health care provider. The other options are related to blood transfusion reactions but are not related to findings of circulatory overload.

The nurse is preparing a speech to a local service organization about clinical trials in cancer care. Which of the following statements would be correct to include? (Select all that apply.) a. Clinical trials have led to improved cancer prevention and treatment b. There is a clinical trial protocol for all types of people with cancer c. A clinical trial is one of the first steps in the research process d. Clinical trials require approval of a human subjects review board

d. Clinical trials require approval of a human subjects review board Clinical trials involve the use and study of treatments on actual human subjects and have been important in making advances in cancer outcomes. After many stages of research have been completed, the clinical trial becomes one of the final steps of the process. Because they involve real people, human subjects review boards must approve the research. There is not a clinical trial for every type of cancer victim, as potential participants in clinical trials must meet specific protocols.

The client is 48 hours post-insertion of an abdominal catheter for peritoneal dialysis and is currently undergoing a fluid exchange. The nurse understands that which of these findings needs to be reported to the health care provider immediately? a. Slight pink-tinged drainage b. Abdominal discomfort c. Muscle weakness d. Cloudy drainage

d. Cloudy drainage Cloudy drainage is a sign of infection that may indicate peritonitis (inflammation of the peritoneum), a serious complication of peritoneal dialysis. The other options are expected during or after the procedure of catheter placement or peritoneal dialysis.

The client is a new admission diagnosed with Alzheimer's disease (AD). The nurse reviews all drugs (including complementary & integrated health therapies) routinely taken at home with a family member. Which of the following treatments would be a concern for the nurse? a. Donepezil (Aricept) b. Ginkgo biloba c. Omega-3 fatty acids d. Coconut oil

d. Coconut oil Donepezil, rivastigmine and galantamine are most commonly used in the treatment of AD. Complementary and integrative therapies used to treat AD include ginkgo biloba (a plant extract) and omega-3 fatty acids. While there really isn't enough research to support using these treatments, continued use won't necessarily be harmful. However, coconut oil, (which is a source of caprylic acid) is a concern. While there has been limited research on Katasyn (an experimental drug containing caprylic acid), there's no scientific evidence that coconut oil is safe and effective or prevents cognitive decline.

The nurse is caring for a child diagnosed with Kawasaki disease (mucocutaneous lymph node syndrome or infantile polyarteritis). The nursing care plan should be based on the knowledge that this child is at risk for developing what complication? a. Occlusions at the vessel bifurcations b. Pulmonary embolism c. Chronic vessel plaque formation d. Coronary artery aneurysm

d. Coronary artery aneurysm Kawasaki disease affects the mucus membranes, lymph nodes, walls of the blood vessels and the heart. It can cause inflammation of the arteries, especially the coronary arteries of the heart, which can lead to aneurysms and possible heart attack in the child.

An 80 yo patient has taken a benzodiazepine for insomnia for many years. The patient now reports experiencing anxiety and some confusion. What is most likely the reason for this? a. Decrease GI motility b. Poor rate of elimination by the kidneys c. Decreased liver function d. Decrease in lean body mass and increase in body fat

d. Decrease in lean body mass and increase in body fat Absorption, distribution and elimination of medications are al affected by age-related changes. Since drug distribution is most affected by the change in the body fat and lean body mass, this can lead to increased elimination half-life and prolonged effect of lipid soluble drugs such as benzodiazepines. Dosages that may have a therapeutic effect of a 65yo can produce significant side effects for older patients.

A nurse consistently ignores the call lights of clients who practice alternative lifestyles. The nurse's behavior is an example of what approach? a. Stereotyping b. Cultural insensitivity c. Prejudice d. Discrimination

d. Discrimination Discrimination is the differential treatment of individuals because they belong to a minority group. This generally refers to the limiting of opportunities, choices, or life experiences because of prejudices against individuals, cultures or social groups.

The nurse is instructing a 65 year-old female client diagnosed with osteoporosis. What is the most important instruction about exercise? a. Exercise to reduce weight over a few months b. Use exercise to strengthen muscles and protect bones c. Avoid exercise activities that increase the risk of fracture d. Do weight-bearing or resistance activities

d. Do weight-bearing or resistance activities Weight-bearing or resistance exercises are beneficial in the treatment of osteoporosis. Although loss of bone cannot be substantially reversed, further loss can be greatly reduced if the client includes these exercises. In addition, other approaches are estrogen replacement and calcium supplements in a treatment protocol.

The nurse notes that a prescription for captopril was changed to losartan, even though the captopril provided effective blood pressure control. What is the most likely reason for discontinuing the captopril? a. Sexual dysfunction b. Blurred vision c. Rash and itching d. Dry cough

d. Dry cough Captopril (Capoten) is an ACE inhibitor used to control blood pressure. Some common side effects include rash, itching and blurred vision. Like many antihypertensives, ACE inhibitors can cause impotence. But a chronic cough is one of the most common and disturbing problems for clients using ACE inhibitors, prompting a change in blood pressure medication. Even after discontinuing the ACE inhibitor, it may take weeks or months for the cough to go away. Angiotensin receptor blockers, such as losartan (Cozaar), are often prescribed when clients cannot tolerate an ACE inhibitor.

A new nursing assistant is instructed to weigh clients diagnosed with anorexia nervosa only if the clients wear a gown with underwear but no street clothing. What is the rationale for this intervention? a. Promotes feelings of success with gaining weight b. Allows the nursing assistant to better assess the client's skin c. Symbolically removes barriers between the client and staff d. Eliminates the risk of hiding objects in clothing or shoes

d. Eliminates the risk of hiding objects in clothing or shoes Some of the goals of treating anorexia nervosa are to restore clients to a healthy weight and to normalize eating patterns. Clients should be weighed in the morning, after they have voided. They are only to wear a (hospital) gown and underwear; wearing street clothing allows the client to hide objects in pockets (or shoes) that will add weight. Some therapists believe the client should initially be weighed "blind" (backing up onto the scale) so they can't see the numbers. Regardless of how it's done, being weighed is anxiety-provoking for the client. Recall that nursing assistants cannot assess clients.

A male client with benign prostatic hypertrophy is admitted with a distended bladder due to acute urinary retention. There is an order to insert an indwelling urinary catheter (IUC). What should the nurse understand about catheter insertion and care for this client? a.Remove the catheter after 24 hours b. Empty the bladder gradually and clamp after 1000 mL is collected c. Insert an 18-Fr, three-way irrigation catheter d. Empty the bladder quickly and completely

d. Empty the bladder quickly and completely With acute urinary retention, treatment begins with catheterization. It's best to use the smallest catheter size (14 to 16 Fr); a coude-tipped catheter is often used in men with an enlarged prostate or urethral stricture. Evidence-based practice supports complete and rapid emptying of the bladder. The client may need to have an IUC for several days. Bladder irrigation is performed to maintain patency of a retention catheter by removing sediment or clots, usually following a surgical procedure involving the urinary system.

A nurse is teaching home care to the parents of a child diagnosed with acute spasmodic croup. What type of care would be most important to emphasize? a. Sedation as needed to prevent exhaustion b. Antihistamines to decrease allergic responses c. Antibiotic therapy for 10 to 14 days d. Humidified air with an increase in oral fluids

d. Humidified air with an increase in oral fluids The most important aspects of home care for a child diagnosed with acute spasmodic croup are humidified air and increased oral fluids. Humidified air helps reduce vocal cord swelling. Taking the child out into the cool night air for 10 to 15 minutes can also reduce night time symptoms. Adequate systemic hydration aids mucociliary clearance by keeping secretions thin and easy to remove with minimal coughing effort.

An adolescent patient arrives at a clinic three weeks after the birth of her first baby. She tells the nurse she is very worried about not returning to her pre-pregnancy weight. What approach should the nurse take first? a. Review the patient's pattern of weight gain over the past year b. Give her several pamphlets on postpartum nutrition c. Ask the mother to record her diet for the next few weeks d. Encourage her to talk about her self-image

d. Encourage her to talk about her self-image Body image is very important to an adolescent. The nurse must acknowledge this and collect more information about the client's self-image before discussing nutritional needs, diet and/or exercise. Adolescents often need more support and information about what to expect after the birth of a child, especially since the postpartum period can be overwhelming for them. Nonjudgmental and developmentally appropriate interactions are needed to care for the physical and emotional needs of adolescents.

A nurse is teaching a class on human immunodeficiency virus (HIV) prevention. Which activity should be cautioned against since it is shown to increase the risk of HIV? a. Donation of blood to the state agencies b. Physical touch of a person with autoimmune deficiency syndrome (AIDS) c. Use of public bathrooms in any city d. Engaging in unprotected sexual encounters

d. Engaging in unprotected sexual encounters Because HIV is spread through exposure to bodily fluids, unprotected intercourse and shared drug paraphernalia remain the highest risks for this infection. The other actions are not at risk behaviors for HIV.

A 16 year-old client is admitted to a psychiatric unit with a diagnosis of attempted suicide. The nurse should understand that a more frequent cause for suicide in adolescents is which factor? a. Feelings of anger or hostility with others b. Reunion wish or fantasy of the supernatural c. Progressive failure to adapt socially d. Feelings of alienation or isolation from peers

d. Feelings of alienation or isolation from peers The isolation from peers may occur gradually to result in a loss of all meaningful social contacts. Isolation can be self-imposed or can occur as a result of the inability to express feelings to peers or family members. During adolescence, an important benchmark is to achieve a sense of identity and peer acceptance.

A nurse is providing care to a primigravida whose membranes spontaneously ruptured (ROM) four hours ago. At the time of rupture, maternal vital signs were within normal limits, she was dilated to 2 centimeters, and the baseline fetal heart rate (FHR) was 150 beats per minute (BPM). The nurse is now reassessing the client. Which of these assessment findings may be an early indication that the client is developing a complication of the labor process? a. Maternal temperature is 100 F (37.7 C) b. Cervical dilation of 4 centimeters c. Blood pressure is 138/88 mm Hg d. Fetal heart rate is 188 beats/minute

d. Fetal heart rate is 188 beats/minute Prolonged ruptured membranes may lead to maternal infection (as suggested by the slightly elevated temperature). But the primary concern is the fetal heart rate of 188; fetal heart rate is typically somewhere between 120 and 160 BPM. Fetal tachycardia may be an early sign of hypoxia. The nurse should contact the health care provider, assist the client to change positions, and administer oxygen and intravenous fluids.

A primigravida in the third trimester is hospitalized with a diagnosis of preeclampsia. The nurse determines that the client's blood pressure has a trend of increased readings. Which action should the nurse take first? a. Check the protein level in urine b. Check the client's deep tendon reflexes c. Take the temperature d. Have the client turn to the left side

d. Have the client turn to the left side The priority action in this situation is to turn the client to the left side to decrease pressure on the vena cava and promote adequate circulation to the placenta and kidneys. Urine protein level and output should be checked with each voiding. Temperature should be monitored every four hours or more often if indicated, but no data in the stem supports a check of temperature. The deep tendon reflexes are checked as needed especially when magnesium drips are being infused.

There is an order to administer intravenous gentamicin three times a day. What diagnostic finding indicates the client may be more likely to experience a toxic side effect of this medication? a. Low serum albumin b. Low serum blood urea nitrogen c. High gastric pH d. High serum creatinine

d. High serum creatinine Gentamicin is excreted unmodified by the kidneys. If there is any reduced renal function, toxicity can result. An elevated serum creatinine indicates reduced renal function and this puts the client at greater risk for toxicity. Reduced renal function will delay the excretion of many medications.

A group of nurses on a unit are discussing stoma care for clients who have had a stoma made for fecal diversion. Which stomal diversion poses the highest risk for skin breakdown? a. Ileal conduit b. Transverse colostomy c. Sigmoid colostomy d. Ileostomy

d. Ileostomy Ileostomy output, which is from the small intestine, is of continuous, liquid nature. This high pH, alkaline output contains gastric and enzymatic agents that when present on skin can denude skin in a few hours. Because of the caustic nature of this stoma output, adequate peristomal skin protection must be delivered to prevent skin breakdown. With a transverse colostomy the stool is of a somewhat mushy and soft nature. With a sigmoid colostomy the output is formed with an intermittent output. An ileal conduit is a urinary diversion with the ureters being brought out to the abdominal wall.

The father of an 8 month-old asks the nurse if the child's vocalizations are normal for his this age. Which sound should the nurse expect from a child at this age? a. Laughter b. Throaty sounds c. Cooing d. Imitation of sounds

d. Imitation of sounds Imitation of sounds such as "da-da" is expected at this time. Laughter occurs after the initial cooing.

The nursing team listens to a change-of-shift report and then the RN determines that the unlicensed assistive person (UAP) can measure vital signs for all clients except the 80-year-old female diagnosed with middle-stage Alzheimer's disease. What information mentioned in the report suggests the registered nurse should personally follow up and assess the client with Alzheimer's disease? a. Increase in heart rate by 10 beats per minute b. Change in the color and temperature of the fingers and toes c. A 10 mm Hg drop in systolic blood pressure d. Increased confusion, agitation and withdrawal

d. Increased confusion, agitation and withdrawal Infections and pain can quickly exacerbate common symptoms of Alzheimer's disease, including confusion, agitation or withdrawal. A urinary tract infection is one of the most common causes of sudden behavior changes in Alzheimer's victims. The nurse will need to conduct a thorough assessment of the client and cannot assign care of this client to a UAP.

A client requires rapid infusion of packed red cells. What is the benefit of using a blood warmer when rapidly infusing multiple units of packed red cells? a. Reduces the risk of acute allergic reaction b. Increases likelihood of serum hypokalemia c. Eliminates potential for lysis of the blood cells d. Increases peripheral dilatation and comfort

d. Increases peripheral dilatation and comfort When blood is received from the blood bank, it is cold. Rapidly infusing cold blood is uncomfortable and can cause hypothermia; warming the blood increases comfort and also dilates vasculature. The other options are factually incorrect. Allergic reactions are a result of antigens and characteristics of the blood. Lysis of cells is caused by physically breaking them if using too small a bore of needle. Blood warming is thought to actually facilitate movement of potassium.

A nurse is providing care to a 17 year-old client in the post anesthesia care unit (PACU) after an emergency appendectomy. Which finding is an early indication that there is diminished oxygenation? a. Abnormal breath sounds b. Cyanosis of the lips c. Pulse oximeter reading of 92% d. Increasing pulse rate

d. Increasing pulse rate The earliest sign of poor oxygenation is an increasing pulse rate, which is part of the body's compensatory mechanism. Abnormal breath sounds and cyanosis are late signs of poor oxygenation. A pulse oximetry reading of 92% is normal and less than 90% would be a concern.

The client is admitted with anemia, suspected to be caused by slowly bleeding esophageal varices. Which physician order should the nurse question? a. Administer ranitidine (Zantac) 150 mg tab twice a day by mouth b. Send three stool samples for occult blood c. Obtain complete blood count (CBC) d. Insert nasogastric (NG) tube to gravity

d. Insert nasogastric (NG) tube to gravity Esophageal varices are similar to varicose veins within the esophagus. A nurse would never insert an NG tube if this diagnosis is suspected because it might rupture the varices and cause an acute hemorrhage. The other orders make sense during a GI work-up for anemia. Checking CBC will provide hemoglobin and hematocrit values to quantify the degree of anemia. Ranitidine decreases stomach acid and may decrease loss of blood through a possible ulcer. Getting stool samples for occult blood can identify the presence of small amounts of blood in the stool that are not visible to the naked eye.

The health care provider has ordered the anti-infective tetracycline for a young woman. When teaching the client about the medication, what information would be necessary for the nurse to reinforce? a. It may cause hearing loss b. It should be taken with food or milk c. It may cause staining of the teeth d. It may decrease the effectiveness of some oral contraceptives

d. It may decrease the effectiveness of some oral contraceptives The most important point, for almost any anti-infective, is that it can decrease the effectiveness of some oral contraceptives; an alternative form of contraception should be used when taking the medication. Tetracycline should be taken on an empty stomach and never with milk. It is not given to children younger than 8 years old because it can stain developing teeth. Tetracycline is not known to cause hearing loss.

A 63 year-old client is diagnosed with severe pneumonia. Which intervention by the nurse promotes the client's comfort? a. Encourage visits from family b. Increase oral fluid intake c. Monitor vital signs frequently d. Keep conversations short

d. Keep conversations short Keeping conversations short will promote the client's comfort by decreasing the demands on a client's breathing and energy. Increased intake of fluids and monitoring vital signs are not related to comfort. While the presence of family is supportive, demands on the client to interact with the visitors may interfere with the client's rest.

In order to enhance a client's response to medication for chest pain from acute angina, the nurse should emphasize which approach? a. Eat smaller meals b. Limiting alcohol use c. Avoiding passive smoke d. Learning relaxation techniques

d. Learning relaxation techniques The only factor that can enhance the client's response to pain medication for angina is reduction of anxiety through relaxation methods. Anxiety may increase intensity to a point where pain medication outcomes are totally ineffective.

The nurse cares for a client who was admitted in status epilepticus and whose last seizure was four hours ago. What is the most important nursing assessment for this client? a. Respiratory status b. Injuries to the extremities c. Vital signs and oxygen saturation d. Level of consciousness

d. Level of consciousness Cerebral blood flow surges during seizure activity, depleting oxygen at the neuronal level. Cerebral anoxia may result in progressive brain tissue injury and destruction. The nurse should monitor the client's level of consciousness very closely.

A young adult seeks treatment in an outpatient mental health center. The client tells the nurse: "I am a government official being followed by spies." On further questioning, the client reveals: "My warnings must be heeded to prevent nuclear war." Which of the following actions should the nurse take? a. Confront the client's delusion b. Contact the government agency c. Ask for more information about the spies d. Listen quietly without comment

d. Listen quietly without comment The client's comments demonstrate grandiose ideas. The most therapeutic response is to listen but to also avoid being pulled into the client's delusional system. At some point validation of the present situation will need to be done. Confrontation at this time would be an inappropriate action and is not therapeutic.

A client who is thought to be homeless is brought to the emergency department (ED) by the police. The client is unkempt, has difficulty concentrating, is unable to sit still, and speaks in a loud tone of voice. Which of these actions is the appropriate nursing intervention for the client at this time? a. Allow the client to randomly move about the holding area until a hospital room is available b. Isolate the client in a secure room until control is regained by the client c. Engage the client in an activity that requires focus and individual effort d. Locate a room that features minimal stimulation during the admission process

d. Locate a room that features minimal stimulation during the admission process To be placed in a room with minimal stimulation allows the client with moderate anxiety or agitation to have human contact in an environment that does not exacerbate the condition. It also facilitates efficiency in the initial screening and admission process to the ED. By preventing behavioral escalation, this approach promotes safety for the client and staff.

A community health nurse has been caring for a 16 year-old who is 22-weeks pregnant with a history of morbid obesity, asthma and hypertension. Which of these lab reports need to be communicated to the health care provider as soon as possible? a. Hematocrit 33% (0.33) and platelets 200,000 μL b. Blood urea nitrogen 28 mg/dL (10 mmol/L) and glucose 225 mg/dL (12.5 mmol/L) c. Hemoglobin 11 g/dL (6.8 mmol/L) and calcium 6.7 mg/dL (1.67 mmol/L) d. Magnesium 0.8 mEq/L (0.33 mmol/L) and creatinine 3 mg/dL (265.26 μmol/L)

d. Magnesium 0.8 mEq/L (0.33 mmol/L) and creatinine 3 mg/dL (265.26 μmol/L) The magnesium is low and the creatinine is high, indicating acute renal failure - this is the highest priority. With the history of hypertension, the findings may indicate preeclampsia. The rest of client's lab values are all abnormal except for the platelets. The client needs to be referred for immediate follow-up with a health care provider.

The nurse is caring for a child diagnosed with seizures. While teaching the family and the child about the medication phenytoin, what information should the nurse emphasize? a. Omit the medication if the child is seizure-free b. Serve a diet that is high in iron c. Administer acetaminophen to promote sleep d. Maintain good oral hygiene and dental care

d. Maintain good oral hygiene and dental care Swollen and tender gums occur often with use of phenytoin. Good oral hygiene and regular visits to the dentist should be emphasized. The medication should never be stopped, even if the child is seizure-free; sudden discontinuation could result in status epilepticus. Acetaminophen is not a sleep-aid and iron interferes with phenytoin absorption.

The parents of a 5 month-old report that the infant has "vomited nine times in the past six hours." Based on this information, the nurse should observe for which fluid and electrolyte imbalance? a. Hemodilution effects b. Hemoconcentration effects c. Metabolic acidosis d. Metabolic alkalosis

d. Metabolic alkalosis Vomiting results in a loss of acid from the stomach. Prolonged vomiting results in excess loss of acid and leads to metabolic alkalosis. Findings include irritability, increased activity, hyperactive reflexes, muscle twitching and elevated pulse. Hemoconcentration due to fluid loss may occur, but is not the best answer because it does not answer the question about an imbalance.

A client is recovering from hip replacement and is taking acetaminophen with codeine (Tylenol No. 3) every three hours for pain. Which finding associated with opioid analgesics does the nurse anticipate when assessing the client? a. Itching and bruising at the incision site b. Elevated serum glucose c. Dry, unproductive cough d. No bowel movement for three days

d. No bowel movement for three days Side effects of opioid analgesic use include respiratory depression, sedation and constipation. The incision site may be bruised after surgery and it may itch, pull or feel numb, but this is unrelated to oral opioid use. Dry mouth is a possible side effect of acetaminophen with codeine, but not necessarily dry cough.

The new graduate nurse interviews for a position in a nursing department of a large health care agency that uses the approach of shared governance. Which of these statements best illustrates the shared governance model? a. Non-nurse managers supervise nursing staff in groups of units b. An appointed board oversees any administrative decisions c. Staff groups are appointed to discuss nursing practice and client education issues d. Nursing departments share responsibility for client outcomes

d. Nursing departments share responsibility for client outcomes Shared governance or self-governance is a method of organizational design. It promotes empowerment of nurses to give them responsibility for client care issues and outcomes with other divisions in the agency.

The nurse is caring for a patient who had a sigmoid colostomy and requests assistance with removing flatus from one-piece drainable ostomy pouch. Which should be the course intervention by the nurse? a. Pull the adhesive seal around the pouch to allow flatus to escape b. Pierce the plastic of the ostomy pouch with a pin to vent the flatus. c. Assist the patient to ambulate to reduce the flatus in the pouch. d. Open the bottom of the pouch to allow the flatus to be expelled.

d. Open the bottom of the pouch to allow the flatus to be expelled. The only correct way to prevent flatus from a one-piece drainable ostomy pouch is to instruct the patient to obtain privacy because the release of the flatus will cause odor. The patient should open the bottom of the pouch, allow the flatus to escape, and then close the bottom of the pouch.

The nurse works in an ambulatory care clinic where there are four children with gastrointestinal findings waiting to be seen by the health care provider. Which child is at greatest risk for developing metabolic acidosis? a. The child with alternating constipation and diarrhea b. The child with nausea and anorexia c. The child who has been vomiting for more than 48 hours d. The child with severe diarrhea for 24 hours

d. The child with severe diarrhea for 24 hours Severe diarrhea can lead to excessive loss of sodium bicarbonate from the body. If untreated, severe diarrhea can lead to metabolic acidosis. Prolonged vomiting, on the other hand, can result in metabolic alkalosis (due to acid loss.) Severe anorexia can also result in metabolic alkalosis.

A nurse is talking by telephone with a parent of a 4 year-old child who has chickenpox. Which approach demonstrates appropriate teaching by the nurse? a. The illness is only contagious when the lesions are present b. Chewable aspirin is the preferred analgesic c. Recommend an antiviral medication to relieve itching d. Papules, vesicles and crusts will be present at one time

d. Papules, vesicles and crusts will be present at one time All three stages of the chickenpox lesions will be present on the child's body at the same time. Children should not be medicated with aspirin due the possibility of developing Reye's syndrome. A person with chickenpox is contagious one to two days before their blisters appear and remain contagious until all the blisters have crusted over. Antiviral medications are not usually prescribed to otherwise healthy children. Over-the-counter hydrocortisone creams can help relieve itchy skin.

A client with a fractured femur has been in Russell's traction for 24 hours. Which nursing action is the priority? a. Check the skin around bony prominences b.Auscultate the lungs for bilateral atelectasis c. Inspect the pin sites for signs of drainage d. Perform a bilateral neurovascular check of lower extremities

d. Perform a bilateral neurovascular check of lower extremities Promotion of neurovascular integrity is the most important aspect of care for this client, which is why the nurse would want to regularly assess the pedal pulse, color, temperature, sensation, and capillary refill of the involved extremity. Russell's traction is Buck's traction with a sling under the knee. This is a skin traction and not skeletal traction with pins; therefore, the pin sites are an inappropriate focus.

The school nurse is providing information for teachers at a school where 10 yo child with epilepsy attends. What is the most important action a teacher can take when the child experiences a tonic-clonic seizure in the classroom. a. Clear the immediate are of anything that could harm the child b. provide privacy and reassure the other children c. note the sequence of movements with the time lapse of the event d. Place something soft and flat under the child's head

d. Place something soft and flat under the child's head During seizure activity, the priority would be to protect the child from physical injury. The teacher could place something soft and flat, like folded jacket under the child's head to help prevent head trauma. After protecting the head, the prioritized sequence would be to move furniture away from the child, note movements and time, and then provide privacy, if possible, while reassuring the other students.

The clinic nurse assists the health care provider with physical examinations and the collection of laboratory specimens. Which of these findings does the nurse recognize as being reportable to the public health department? a. Positive eye discharge confirming conjunctivitis b. Clinical findings of impetigo c. Skin scraping confirming the presence of ringworm d. Positive stool culture for shigella

d. Positive stool culture for shigella The Centers for Disease Control and Prevention (CDC) have a list of notifiable infectious diseases that is updated yearly. Shigellosis is the only reportable infection of those listed. Shigella are bacteria that can infect the digestive tract and cause (painful) diarrhea, cramping, vomiting, nausea; in severe cases it can cause seizures and kidney failure. Ringworm is a contagious fungal infection. Impetigo is a contagious, superficial bacterial skin infection. Conjunctivitis has many causes and is usually diagnosed from signs and symptoms and patient history.

The client, who is receiving chemotherapy through a central venous access device (CVAD) at home, is admitted to the intensive care unity (ICU) with a diagnosis of sepsis. Which of the following nursing interventions is the priority? a. Restrict contact with persons having known, or recent, infections b. Change the dressing over the site of the existing CVAD c. Insert an indwelling catheter d. Prepare the client for insertion of a new CVAD

d. Prepare the client for insertion of a new CVAD Many cases of sepsis occur in immunocompromised clients and clients with chronic and debilitating diseases. Since it's likely the existing CVAD is the source of the infection, it should be cultured and removed. A new central line (usually an internal jugular or subclavian) needs to be inserted since large amounts of IV fluids are needed to restore perfusion. The new central line will also allow venous access for labs, medications and measuring central venous pressure. Together with central venous pressure monitoring, an indwelling urinary catheter will help guide fluid volume replacement. Many hospitals have restrictions on visitors with known or recent infections to help protect all clients.

A nurse notes that a 2 year-old child recovering from a tonsillectomy has a temperature of 98.2 F (36.7 C) at 11:00 am. At 1:00 pm the child's parent reports that the child "feels very warm" to touch. What should the nurse do first? a. Administer the prescribed acetaminophen b. Reassure the parent that this is normal c. Offer the child cold oral fluids d. Reassess the child's temperature

d. Reassess the child's temperature The nurse should listen to and show respect for what the parent is saying, because the parent is more sensitive to the variations in the child's condition. However, the nurse knows that a low-grade fever (99-101 F or 37.2-38.3 C) is common after surgery, which is why the nurse should first reassess the temperature before implementing any intervention. Usually the surgeon is contacted if the temperature is higher than 101.5 F (38.6 C).

The 86 year-old client will be participating in a transitional care program after discharge from the hospital. What is the primary purpose of a transitional care program? a. Increase client understanding of discharge instructions b. Reduce insurance costs c. Increase satisfaction with nursing care d. Reduce readmissions to the hospital

d. Reduce readmissions to the hospital Older adults who complete a transitional care program after being discharged from the hospital are much less likely to be readmitted to the hospital. The Affordable Care Act mandates that each facility have a "quality assurance and performance improvement program", designed to help reduce unnecessary hospital readmissions.

The nurse observes a client with a diagnosis of obsessive-compulsive disorder on an inpatient unit. Which behavior is consistent with this medical diagnosis? a. Preference for consistent caregivers b. Repetitive, involuntary movements c. Verbalized suspicions about thefts on the unit d. Repeatedly checking that a door is locked

d. Repeatedly checking that a door is locked Obsessive-compulsive disorder is characterized by repetitive, unwanted, intrusive thoughts (obsessions) and irrational, excessive urges to perform certain actions (compulsions.) People know their thoughts and behaviors don't make sense, but they are often unable to stop them. Verbalized suspicions reflect a paranoid thought process. Repetitive, involuntary movements are characteristic of some antipsychotic medication side effects.

A newly admitted 78 year-old client is diagnosed with severe dehydration. When planning care for this client, the nurse should assign which task to an unlicensed assistive personnel (UAP)? a. Converse with the client to determine if the mucous membranes are impaired b. Check skin turgor every four hours along with the need to change the adult diaper c. Monitor client's ability for movement in the bed from side to side d. Report hourly outputs of less than 30 mL/hr within 15 minutes of the check

d. Report hourly outputs of less than 30 mL/hr within 15 minutes of the check When assigning a UAP, the nurse must communicate clearly about each delegated task with specific instructions on what must be reported and when. Because the RN is responsible for all care-related decisions, only routine tasks should be assigned to UAPs because such tasks do not require judgments and decisions.

The nurse is planning care for a client with a cerebral vascular accident (CVA). Which approach would be most effective in the prevention of skin breakdown? a. Place client in the wheelchair for four hours daily b. Pad the bony prominences c. Massage reddened bony prominence d. Reposition every two hours if in bed

d. Reposition every two hours if in bed Clients who are at risk for skin breakdown develop fewer pressure ulcers when turned every two hours. By relieving the pressure over bony prominences at frequent scheduled intervals, blood flow to areas of potential injury is maintained. If the client is in a wheelchair, a shift of the weight should be done every hour. Massage of reddened bony prominences is no longer recommended as a result of evidence-based research.

As a client is being discharged following resolution of a spontaneous pneumothorax, the client tells a nurse, "I'm going to Hawaii for a vacation next week." The nurse should warn the client to avoid which activity? a. Surfing b. Parasailing c. Swimming d. Scuba diving

d. Scuba diving The nurse would strongly emphasize the need for the client with a history of spontaneous pneumothorax problems to avoid high altitudes, flying in unpressurized aircraft and scuba diving. The negative pressure associated with diving could cause the lung to collapse again.

An 80 year-old client, who is experiencing unintentional weight loss, is admitted with a diagnosis of malnutrition. The nurse understands that which of these lab tests is the most sensitive measure of nutritional status? a. Serum calcium b. Urine creatinine c. Urine protein d. Serum albumin

d. Serum albumin Low serum albumin is a valuable indicator of protein deficiency and, later, nutritional status in adults. The normal range for serum albumin is 3.5-5.5 g/dL (35-55 g/L). When assessing older adults for malnutrition, you can use the S-C-A-L-E-S mnemonic screening tool. A problem with 3 or more of the following indicators increases an older adult's risk for malnutrition: S=sadness (depression); C=cholesterol (low); A=albumin (low); L=loss of weight; E=eating problems; S=shopping problems or inability to prepare a meal.

The nurse assesses a 70 year-old male's laboratory results during a routine clinic visit. Which result would indicate a need for information and education? a. RBC 5.0 million/mm3 ( 5 x 1012/L) b. Serum glucose 90 mg/dL (5 mmol/L) c. LDL Cholesterol 130 mg/dL (3.37 mmol/L) d. Serum albumin 2.5 g/dL (25 g/L)

d. Serum albumin 2.5 g/dL (25 g/L) Plasma protein concentrations can be affected by many diseases common in older adults. Hypoalbuminemia (less than 3 g/dL [or 30 g/L]) is an early indicator of increased risk of death and is usually caused by acute and chronic inflammatory responses; it can also be caused by nephrotic syndrome, hepatic cirrhosis, heart failure, and malnutrition. The health care provider needs to treat the underlying causes of the low albumin. The client also needs a dietary consult to learn about increasing his dietary intake of protein. Socioeconomic factors, especially financial factors, may need to be addressed to help the client comply with the recommendation.

The nurse working in the intensive care unit (ICU) is told that a client is being newly admitted with a diagnosis of hyperglycemic hyperosmolar nonketotic state (HHNS). The nurse would expect which of the following clinical findings in this client? (Select all that apply.) a. History of type 1 diabetes mellitus b. Ketonuria c. Metabolic acidosis d. Severe dehydration e. Blood glucose level of at least 600 mg/dL (33.33 mmol/L)

d. Severe dehydration e. Blood glucose level of at least 600 mg/dL (33.33 mmol/L) The typical client with HHNS will have a plasma glucose level of 600 mg/dL (33.33 mmol/L) or greater, high serum osmolality, profound dehydration, a serum pH greater than 7.3 and some alteration in consciousness. Unlike diabetic ketoacidosis, however, there is little to no ketosis. HHNS usually presents in older clients with type 2 diabetes mellitus who have some concomitant illness (usually an infection) that leads to reduced fluid intake, or who do not adhere to their diabetic medications and diet. All clients with HHNS require hospitalization and rapid treatment to correct the profound hypovolemia and hyperglycemia characteristic of this condition.

The nurse is caring for a client in the late stages of amyotrophic lateral sclerosis (ALS). Which finding would the nurse expect? a. Confusion b. Loss of half of visual field c. Tonic-clonic seizures d. Shallow respirations

d. Shallow respirations ALS is a chronic progressive neurodegenerative disease that affects nerve cells in the brain and spinal cord. In ALS, upper and lower motor neurons degenerate (die) and stop sending messages to muscles; all muscles under voluntary control eventually weaken and atrophy. People eventually lose their ability to speak, eat, move and breathe. However, ALS does not impair a person's mind or intelligence. ALS does not affect a person's ability to see, smell, taste, hear or recognize touch.

A nurse is working in an OB-GYN clinic. A 40 year-old woman in the first trimester of an unplanned pregnancy provides a health history to the nurse. Which information should receive priority attention? a. She has taken 800 mcg of folic acid daily for the past year b. Her father and brother have type 1 diabetes c. Her husband was treated for tuberculosis as a child d. She has been taking an ACE inhibitor for her blood pressure for the past two years.

d. She has been taking an ACE inhibitor for her blood pressure for the past two years. A report by the client that she has been taking medications in the first trimester of pregnancy should be followed up immediately. ACE inhibitors, commonly used to control high blood pressure, are pregnancy category X, as they can cause teratogenic effects on the developing fetus, increasing the risk of birth defects. Women who are taking medications and who are planning a pregnancy should be switched to medications that are not harmful to the developing fetus before they begin trying to get pregnant.

A client states, "I feel funny." The nurse uses electronic equipment to obtain vital signs and notes these findings: blood pressure 100/56 mm Hg, pulse 38, respirations 26. The client's previous reading: blood pressure 130/88 mm Hg, pulse 82, respirations 21. List the correct order of actions the nurse should now take (with 1 being the top priority). a. Notify the health care provider b. Assess for chest pain, dyspnea, low oxygen saturation, restlessness or other signs of respiratory or cardiac impairment c. Anticipate the need for ECG, oxygen administration, and emergency pacing d. Simultaneously check an apical and radial pulse manually

d. Simultaneously check an apical and radial pulse manually b. Assess for chest pain, dyspnea, low oxygen saturation, restlessness or other signs of respiratory or cardiac impairment a. Notify the health care provider c. Anticipate the need for ECG, oxygen administration, and emergency pacing The sudden drop in both blood pressure and pulse indicate an acute cardiovascular event requiring rapid assessment and intervention to prevent cardiac arrest. At this point, the patient is not in crisis but needs a quick targeted assessment. Verify the bradycardia manually; perfusion may not occur with some arrhythmias, such as premature ventricular contractions, so checking the apical rate while palpating a distal pulse provides a quick assessment. Assessment for cardiovascular and respiratory instability is next. With all the assessment data in hand, it's time to contact the health care provider, anticipating the need for ECG, oxygen as well as interventions to improve cardiac output such as atropine IV and cardiac pacing.

A nurse, who is assigned for five days to a client who has exhibited manipulative behaviors, becomes aware of feeling reluctance to interact with the client. The nurse should take what action next? a. Discuss the feelings of reluctance with an objective peer or supervisor within the next 24 hours b. Develop a behavior modification plan for the client that will promote more functional behavior within the next week c. Limit contacts with the client to avoid reinforcement of the manipulative behavior during the work times d. Talk with the client about the negative effects of manipulative behaviors on other clients and staff within the next few days

d. Talk with the client about the negative effects of manipulative behaviors on other clients and staff within the next few days The nurse who experiences stress in a therapeutic relationship can gain objectivity through discussion with other professionals. The nurse may wish to have a peer observe the nurse-client interactions with this client for a shift and then have a debriefing of positive and negative actions. The nurse must attempt to discover attitudes and feelings in the self that influence the nurse-client relationship in positive and negative ways.

A nurse has been assigned to four clients in the emergency department, with each client experiencing one of these conditions. Which client should the nurse check first? a. Viral pneumonia with atelectasis b. Spontaneous pneumothorax with a respiratory rate of 38 c. Acute asthma with episodes of bronchospasm d. Tension pneumothorax with slight tracheal deviation to the right

d. Tension pneumothorax with slight tracheal deviation to the right Tracheal deviation indicates a significant volume of air being trapped in the chest cavity with a mediastinal shift. In tension pneumothorax the tracheal deviation is away from the affected side. The affected side is the side where the air leak is in the lung. This situation also results in sudden air hunger, agitation, hypotension, pain in the affected side, and cyanosis with a high risk of cardiac tamponade and cardiac arrest.

A client who is HIV-positive is diagnosed with a herpes simplex type 1 (HSV-1) infection. The nurse understands that which issue is the most likely reason for the HSV-1 infection in this client? a. The client has experienced emotional stress b. The client has a history unprotected sexual activities c. The client had contact with saliva d. The client is immunosuppressed

d. The client is immunosuppressed HSV-1, or oral herpes, is spread through contact with saliva and the painful, fluid-filled blisters are often activated by stress. But the best reason why someone who is HIV-positive would also have oral herpes is because they are vulnerable to opportunistic infections that take advantage of a weakened immune system. Other common HIV-related, opportunistic infections and diseases include tuberculosis, bacterial pneumonia and candidiasis.

An 18 month-old child is on peritoneal dialysis in preparation for a renal transplant in the near future. When a nurse obtains the child's health history, the mother indicates that the child has not had the first measles, mumps, rubella (MMR) immunization. What should the nurse understand about the child's situation and administration of the immunization? a. Live vaccines are withheld in children with renal chronic illness b. An inactivated form of the vaccine can be given at any time c. The risk of the vaccine side effects precludes the administration of the vaccine d. The measles, mumps and rubella (MMR) vaccine should be given now, before the transplant

d. The measles, mumps and rubella (MMR) vaccine should be given now, before the transplant MMR is a live virus vaccine, and should be given at this time. Post-transplant, immunosuppressive drugs will be given and the administration of the live vaccine at that time would be contraindicated because of the compromised immune system.

The nurse, who is caring for a client with complex and unique health needs, describes the nature of the illness in an online social forum for nurses. Neither the client's real name nor any other personal identifiers are used. What, if any, consequence could result from posting this information online? a. here won't be any consequences because the information was posted on a website for nursing professionals b. There won't be any consequences because the client's real name was not used c. The nurse could be reprimanded for not clearing the information first with hospital administration d. The nurse could be fired for breach of confidentiality

d. The nurse could be fired for breach of confidentiality Even though the client was not identified by name, someone could probably figure out who the nurse was writing about. Many health care facilities have adopted a social media policy; it is important to understand that nurses can be fired for posting personal information about clients online, because this is an invasion of privacy. In addition to being a HIPAA violation, the Health Information Technology for Economic and Clinical Health Act (HITECH Act) gives states attorneys the right to pursue violations of patient privacy.

The nurse works with children who have chronic conditions requiring frequent hospitalization and activity limitations. Which statement best describes the effects of immobility in children? a. Children are more susceptible than adults to the multisystem effects of immobility b. Immobility promotes independence and self-reliance in children c. Immobilized children quickly develop confusion and mental status changes d. The physical effects of immobility are similar in both children and adults

d. The physical effects of immobility are similar in both children and adults The physical effects of immobility are similar for clients of almost any age. Care of the immobile child includes efforts to prevent complications of muscle atrophy, contractures, skin breakdown, constipation, bone demineralization, and cardiopulmonary complications. Immobility can negatively impact self-image and having to rely on others to meet their basic needs, especially in adolescents. Planning and providing nursing care in creative ways, and involving children in their care, and providing age-appropriate diversion can help reduce the effects of immobility. Older adults with chronic conditions are at greatest risk for developing confusion.

The nurse is talking with the parents of a child who has recently been diagnosed with Hemophilia A. What should the nurse understand about the offspring of an unaffected father and a carrier mother? a. All sons will have the disease b. There is a 50% probability that a son will have the disease c. All daughters will be carriers d. There is a 25% probability that a daughter will be a carrier

d. There is a 25% probability that a daughter will be a carrier Hemophilia A is a sex-linked recessive trait seen almost exclusively in males. When the carrier mother and the unaffected father are pregnant, there are four possible outcomes: 1) a 25% (one in four) chance of having a son without hemophilia 2) a 25% (one in four) chance of having a son with hemophilia 3) a 25% (one in four) chance of having a daughter who is a carrier 4) a 25% (one in four) chance of having a normal daughter who is not a carrier It may help to remember that the carrier woman only has one affected X chromosome, which is why there's only a 25% probability of a son having hemophilia. Notice that two of the response options have an absolute word in them ("all") - these types of statements are usually incorrect.

A nurse is assessing a client who is cachectic and has developed an enterocutaneous fistula following surgery to relieve a small bowel obstruction. The client's total protein level is reported as 4.5 g/dL. Which approach to therapy should the nurse anticipate? a. Serum lipase levels every 12 hours b. Blood for coagulation studies daily c. Additional potassium via IV administration d. Total parenteral nutrition (TPN) via central line

d. Total parenteral nutrition (TPN) via central line The client is not absorbing nutrients adequately as evidenced by the cachexia and low protein levels. A normal total serum protein level is 6.0-8.0 g/dL. Total parenteral nutrition is a mixture of dextrose, amino acids, lipids, electrolytes, vitamins and minerals that is administered intravenously to patients who cannot receive all of their nutritional needs via the enteral route. TPN will promote a positive nitrogen balance in this client who is unable to digest and absorb nutrients adequately, which is necessary for proper healing to occur.

A client is transported to the emergency department after a motor vehicle accident. When assessing the client 30 minutes after admission, the nurse notes several physical changes. Which finding would require the nurse's immediate attention? a. Increased restlessness b. Tachypnea c. Tachycardia d. Tracheal deviation

d. Tracheal deviation Tracheal deviation is a sign that a mediastinal shift has occurred, most likely due to a tension pneumothorax. Air escaping from the injured lung into the pleural cavity causes pressure to build, collapsing the lung and shifting the mediastinum to the opposite side. This obstructs venous return to the heart, leading to circulatory instability and may result in cardiac arrest. This is a medical emergency, requiring emergency placement of a chest tube to remove air from the pleural cavity relieving the pressure.

Which statement is the correct stage of cognitive development for Piaget's: Early Childhood - Preoperational stage a. Concepts are attached to concrete situations b. Analyzes situations and uses abstract logic and reasoning c. Uses sucking, grasping, listening, and looking to earn about the environment d. Uses magical thinking and imagination

d. Uses magical thinking and imagination

A client with hepatitis A (HAV) is newly admitted to the unit. Which action would be the priority to include in this client's plan of care within the initial 24 hours? a. Wear masks with shields if there is potential for fluid splash b. Use disposable utensils and plates for meals c. Provide soft easily digested food with frequent snacks d. Wear gown and gloves during client contact

d. Wear gown and gloves during client contact HAV is usually transmitted via the fecal-oral route, or when someone with the virus handles food without washing hands after using the bathroom. The virus can also be contracted by drinking contaminated water, eating raw shellfish from water polluted with sewage or by being in close contact with a person who is infected - even if that person has no findings. In fact, the disease is most contagious before findings ever appear. The nurse should recognize the importance of isolation precautions from the initial contact with the client on admission until the noncontagious convalescence period.

The nurse is setting up a patient's dinner tray. When the nurse turns her back to the patient, the patient grabs the nurse's buttocks and states that he is hungry for much more than dinner. Which of the following response by the nurse is indicated? a. ignore the behavior b. call the HCP c. quickly leave the room and ask UAP to assist the patient d. complete an incident report

d. complete an incident report To keep the therapeutic relationship intact, a nurse needs to set limits on appropriate behavior and not ignore bad behavior. Sexual harassment is a form of violence and is never part of the job. The nurse should report the incident to her supervisor and complete an incident report. The nurse has the right to ask not to be assigned to this patient.

The nurse receives an order for several medications for a client. Which combination of medications would require the nurse to contact the provider to discuss the orders? (Select all that apply.) a. Amlodipine (Norvasc) b. Insulin c. Verapamil (Calan, Covera, Isoptin, Verelan) d. Finasteride (Propecia, Proscar) e. Lithium (Eskalith, Lithobid) f. Furosemide (Lasix)

e. Lithium (Eskalith, Lithobid) f. Furosemide (Lasix) Lithium generally should not be given with diuretics. Furosemide may reduce excretion of lithium, which could result in lithium toxicity. Additionally, side effects of lithium are polyuria and polydipsia. The nurse should clarify the order before administering lithium and furosemide together.

A severely injured client is moved into an examination area of the emergency department. The family member who accompanied the client to the ED is screaming at the nurse, saying that someone better start doing something right away. What is the best response by the nurse? a. "I need you to go to the waiting area. You can come back when you're more in control." b. "I'm going to give you a few minutes alone so you can calm down." c. "I can't think when you are yelling at me. Talk to me in a normal voice." d. "I know you are upset. But please control yourself and sit down. Otherwise I will have to call security."

vd. "I know you are upset. But please control yourself and sit down. Otherwise I will have to call security." Most violent behavior is preceded by warning signs, such as yelling or swearing. The challenge for nurses is to apply interventions that de-escalate a person's response to stressful or traumatic events. The keys to effective limit setting are using commands to express the desired behavior and providing logical and enforceable consequences for noncompliance. Nurses should acknowledge the agitated person's feelings and be empathetic, reminding him or her that they are there to help.


Kaugnay na mga set ng pag-aaral

Hesi practice (fundamentals in nursing and nursing science)

View Set

Module 1.3 Financial Management/Managing Assets

View Set

Pharmacogenomics in Primary Care

View Set

Ch. 10: Taxation of Life Insurance Annuities- Premiums & Proceeds

View Set

Economics: Ch 8.2: How a Profit-Maximizing Monopoly Chooses Output and Price

View Set

Intro into Business: Midterm Exam

View Set